You are on page 1of 152

를 꿰뚫는 유

념과 원리 형훈
개 련서

유형 내신

수학〈하〉
ⅠV 집합과 명제 ⑤ -1ÉxÉ1인 정수 x는 -1, 0, 1이므로
x=-1일 때, 2x+4=2
x=0일 때, 2x+4=4
집합 x=1일 때, 2x+4=6
∴ {2x+4|-1ÉxÉ1, x는 정수}={2, 4, 6}
따라서 선지 중 집합 {2, 4, 6}을 조건제시법으로 바르게 나타낸
것은 ⑤이다.
001 ④

ㄱ. ‘큰 자연수의 모임’은 그 대상을 분명하게 결정할 수 없으므로


집합이 아니다. 004 19

ㄴ. ‘수학을 잘하는 학생들의 모임’은 그 대상을 분명하게 결정할 수


집합 A의 원소는 0, 1로 2개이므로 n(A)=2
없으므로 집합이 아니다.
집합 B에서 |x|É5, -5ÉxÉ5를 만족시키는 정수 x는
ㄷ. ‘가장 작은 자연수’는 1이므로 그 대상을 분명히 알 수 있다.
-5, -4, -3, -2, -1, 0, 1, 2, 3, 4, 5로 11개이므로
따라서 ‘가장 작은 자연수의 모임’은 집합이다.
n(B)=11
ㄹ. ‘100에 가까운 수의 모임’은 그 대상을 분명하게 결정할 수
집합 C에서 12의 양의 약수는 1, 2, 3, 4, 6, 12로 6개이므로
없으므로 집합이 아니다.
n(C)=6
ㅁ. ‘1보다 작은 소수’는 없으므로 그 대상을 분명히 알 수 있다.
∴ n(A)+n(B)+n(C)=2+11+6=19
따라서 ‘1보다 작은 소수의 모임’은 집합이다. yy TIP
따라서 집합인 것은 ㄷ, ㅁ이다.
TIP 005 ③

‘1보다 작은 소수의 모임’은 원소가 하나도 없으므로 공집합이다. ① 집합 {0}의 원소는 0으로 1개이므로
n({0})=1이다. (참)
② 공집합 ∆의 원소는 존재하지 않으므로 n(∆)=0이다. (참)
③ 집합 {∆}의 원소는 ∆으로 1개이므로 n({∆})=1+0이다.
002 ⑴ A={2, 3, 5, 7} ⑵ ², <, ², <
(거짓)
⑴ 10 이하의 소수는 2, 3, 5, 7이다. ④ 집합 {∆, {∆}}의 원소는 ∆, {∆}으로 2개이므로
∴ A={2, 3, 5, 7} n({∆, {∆}})=2이다. (참)
⑵ ⑴에 의하여 ⑤ 집합 {1, 2}의 원소는 1, 2로 2개이므로 n({1, 2})=2
1은 집합 A의 원소가 아니므로 1 ² A, 집합 {1, 2, 3}의 원소는 1, 2, 3으로 3개이므로
2는 집합 A의 원소이므로 2 < A, n({1, 2, 3})=3
4는 집합 A의 원소가 아니므로 4 ² A, ∴ n({1, 2})-n({1, 2, 3})=2-3=-1 (참)
따라서 선지 중 옳지 않은 것은 ③이다.
7은 집합 A의 원소이므로 7 < A
따라서 안에 알맞은 것은 순서대로
², <, ², <이다. 006 ④

집합 A의 모든 원소가 집합 B의 원소일 때, 집합 A는 집합 B의
부분집합이다.
003 ⑤
따라서 ① ∆, ② {2}, ③ {1, 3}, ⑤ {1, 2, 3}은
① 6의 양의 약수는 1, 2, 3, 6이므로 모두 집합 {1, 2, 3}의 부분집합이다.
{x|x는 6의 양의 약수}={1, 2, 3, 6} ④ {1, 4}에서 4²{1, 2, 3}이므로 집합 {1, 4}는 집합 {1, 2, 3}의
② 1<x<6인 짝수 x는 2, 4이므로 부분집합이 아니다.
{x|1<x<6, x는 짝수}={2, 4} 참고
③ 방정식 x(x-2)(x-4)(x-6)=0의 해는
집합 {1, 2, 3}의 모든 부분집합은
x=0 또는 x=2 또는 x=4 또는 x=6이므로
∆, {1}, {2}, {3}, {1, 2}, {2, 3}, {1, 3}, {1, 2, 3}이다.
{x|x(x-2)(x-4)(x-6)=0}={0, 2, 4, 6}
④ 6보다 작은 홀수 n은 1, 3, 5이므로
n=1일 때, 2n=2
n=3일 때, 2n=6 007 ⑴ A,B ⑵ A=B ⑶ B,A

n=5일 때, 2n=10 ⑴ A={x|xÛ`-1=0}={-1, 1}, B={-1, 0, 1}이므로


∴ {x|x=2n, n은 6보다 작은 홀수}={2, 6, 10} A,B

2 수학 <하>
⑵ A={2, 3, 5}, B={x|x는 5 이하의 소수}={2, 3, 5}이므로
A=B
012 ⑤
C
⑶ A={x|x는 2의 배수}, B={x|x는 6의 배수}에서 모든 6의 ① U =∆ (참)
C
배수는 2의 배수이므로 ② A;A =∆ (참)
B,A ③ 합집합에 대한 교환법칙이 성립하므로
A'B=B'A (참)
C
④ A'A =U (참)
ⅠV

008 ③ ⑤ 여집합과 차집합의 성질에 의하여
C


A-B=A;B (거짓)
ㄱ. 0은 ∆의 원소가 아니므로 0²∆ (거짓) 명
따라서 선지 중 항상 성립하는 것이 아닌 것은 ⑤이다.
ㄴ. 공집합은 모든 집합의 부분집합이므로 ∆,{0} (참) 제

ㄷ. 1은 집합 {1, 2, 3}의 원소이므로 1<{1, 2, 3} (거짓)


ㄹ. 모든 집합은 자기 자신의 부분집합이므로 {1},{1} (참)
따라서 옳은 것은 ㄴ, ㄹ이다. 013 ④

① A'B={x|x<A 또는 x<B} (거짓)


② A;B={x|x<A 그리고 x<B} (거짓)
009 ③ C
③ A ={x|x<U 그리고 x²A} (거짓)
① ∆은 집합 A의 원소이므로 ∆<A (참) ④ A-B={x|x<A 그리고 x²B} (참)
C C C
② ∆은 집합 A의 부분집합이므로 ∆,A (참) ⑤ (A;B) =A 'B ={x|x²A 또는 x²B} (거짓)
③ {1}은 집합 A의 원소가 아니므로 {1}²A (거짓) 따라서 선지 중 옳은 것은 ④이다.
④ {1, 2}는 집합 A의 원소이므로 {1, 2}<A (참)
⑤ {1, 2}는 집합 A의 부분집합이므로 {1, 2},A (참)
따라서 선지 중 옳지 않은 것은 ③이다. 014 ⑤

주어진 집합을 벤다이어그램으로 나타내면 다음과 같다.


U
010 ② A B 4
3 2
두 집합 A={1, 12, a}, B={1, 5, a+b}에 대하여 1 6
5 8
A=B이므로 5<B에서 5<A이어야 한다. 7
따라서 a=5이므로 B={1, 5, b+5}
또한 12<A에서 12<B이어야 하므로 ① A'B={1, 2, 3, 5, 8} (참)
b+5=12에서 b=7이다. ② A;B={3, 5} (참)
C
∴ ab=5_7=35 ③ B ={1, 4, 6, 7} (참)
④ A-B={1} (참)
C C C
⑤ A ;B =(A'B) ={4, 6, 7}+{2, 8} (거짓)

011 ④ 따라서 선지 중 옳지 않은 것은 ⑤이다.

집합 B의 원소는 다음과 같다.


+ -1 0 1
-1 -2 -1 0
015 ⑴ {5, 6} ⑵ {5, 6, 7, 9}

0 -1 0 1 A={1, 3, 5, 6}, B={1, 3, 7, 9}이므로


1 0 1 2 이를 벤다이어그램으로 나타내면 다음과 같다.
A B
즉, B={-2, -1, 0, 1, 2}
5 1 7
집합 C의 원소는 다음과 같다.
6 3 9
_ -1 0 1
-1 1 0 -1 C
⑴ A;(A;B) =A-(A;B)
0 0 0 0 ={1, 3, 5, 6}-{1, 3}
1 -1 0 1 ={5, 6}
즉, C={-1, 0, 1} ⑵ (A-B)'(B-A)={5, 6}'{7, 9} yy TIP
따라서 세 집합 A, B, C의 포함관계는 A=C,B이다. ={5, 6, 7, 9}

Ⅳ. 집합과 명제 3
TIP U
A B
<대칭차집합> U
두 집합 A, B에 대하여 집합 A B

(A-B)'(B-A)
를 ‘대칭차집합’이라 하고 ㄱ. A;B=∆ (참)
다음이 항상 성립한다. ㄴ. A'B+U (거짓)
(A-B)'(B-A)=(A'B)-(A;B) ㄷ. A-B=A (참)
ㄹ. 벤다이어그램에서
U U
A B A B

016 ③
A BC
A,B이므로 이를 벤다이어그램으로 나타내면 다음과 같다. C
U
이므로 A,B (참)
B C
ㅁ. A ,(B-A)에서 B-A=B이므로 A ,B
C

A 그런데 벤다이어그램에서
U U
A B A B

① A;B=A (거짓) ② A'B=B (거짓)


U U
B B
AC B
A A
C
이므로 A øB (거짓)
따라서 옳은 것은 ㄱ, ㄷ, ㄹ로 3개이다.
C TIP
③ A 'B=U (참) ④ B-A+∆ (거짓)
U U <‘두 집합 A, B는 서로소이다.’의 여러 가지 표현>
B B
A;B=∆, U
A A A B
A-B=A, B-A=B,
C C
A,B , B,A ,

C C
n(A;B)=0,
⑤ B ,A (거짓) n(A'B)=n(A)+n(B)
U
B
A

018 ④

따라서 선지 중 항상 옳은 것은 ③이다. A-(B;C)


TIP =A;(B;C) yy 여집합과 차집합의 성질
C

<‘집합 A는 집합 B의 부분집합이다.’의 여러 가지 표현> =A;( BC'CC ) yy 드 모르간의 법칙


C C
A,B, B ,A , U C C
B =(A;B )'(A;C ) yy 교집합에 대한 분배법칙
A;B=A, A'B=B,
C A =(A-B)'(A-C) yy 여집합과 차집합의 성질
A-B=∆, A;B =∆, C C
C ㈎`:`B 'C , ㈏`:`교집합에 대한 분배법칙이다.
두 집합 A와 B 은 서로소이다.
C C C
A 'B=U, B -A =∆

019 18

교집합에 대한 분배법칙을 이용하면


A;(B'C)=(A;B)'(A;C)
017 ③
={1, 2, 3, 5, 7}
두 집합 A, B가 서로소이므로 이를 벤다이어그램으로 나타내면 따라서 구하는 모든 원소의 합은
다음과 같다. 1+2+3+5+7=18이다.

4 수학 <하>
020 ④ 022 ③

각각 벤다이어그램으로 나타내면 다음과 같다. 6의 양의 약수는 모두 12의 양의 약수이다.


① A-(B;C)를 나타내는 부분은 즉, A6,A12이므로 A6'A12=A12
U U U ∴ p=12
A A A
12의 양의 약수이면서 18의 양의 약수인 수는
12와 18의 최대공약수인 6의 약수와 같으므로 yy TIP
ⅠV

B C B C B C A12;A18=A6, q=6 합

A - {BÖC} = A-{BÖC} ∴ p+q=12+6=18

② A;(B-C)를 나타내는 부분은 TIP 제
U U U 두 자연수 a, b에 대하여 a, b의 공약수는 a, b의 최대공약수의
A A A
약수이다.
따라서 12의 약수와 18의 약수 중 공통인 수는 12와 18의
B C B C B C 최대공약수인 6의 약수이다.
A Ö {B-C} = AÖ{B-C}
참고
③ A-(B-C)를 나타내는 부분은
U U U 자연수 n에 대하여 An={x|x는 n의 양의 약수}일 때,
A A A
세 자연수 p, q, r에 대하여
❶ q가 p의 배수이면 Ap,Aq이므로
Ap'Aq=Aq, Ap;Aq=Ap이다.
B C B C B C
❷ Ap;Aq=Ar일 때, r는 p와 q의 최대공약수이다.
A - {B-C} = A-{B-C}
④ (A-B);(A-C)를 나타내는 부분은
U U U
A A A

023 ②
B C B C B C
ㄱ. 4의 배수는 2의 배수이므로 A4,A2
{A-B} Ö {A-C} ={A-B}Ö{A-C}
C
따라서 A2'A4=A2 (참)
⑤ (A'C);B =(A'C)-B를 나타내는 부분은
ㄴ. 2의 배수와 3의 배수 중 공통인 수는
U U U
A A A 6의 배수이므로 yy TIP
A2;A3=A6 (참)
ㄷ. A2;(A3'A4)=(A2;A3)'(A2;A4)
B C B C B C
=A6'A4 (∵ A4,A2)
{AÕC} - B = {AÕC}-B =A4'A6
따라서 구하는 것은 ④이다. 이때, 2<A2'A6이지만 2²A4'A6이므로
A4'A6+A2'A6이다. (거짓)
따라서 옳은 것은 ㄱ, ㄴ이다.
TIP
021 ③ 두 자연수 a, b에 대하여 a, b의 공배수는 a, b의 최소공배수의
C C 배수이다.
A-B={1, 2, 5}, A ;B=B;A =B-A={6, 8},
C C C 따라서 2의 배수와 3의 배수 중 공통인 수는 2와 3의
A ;B =(A'B) ={4, 7, 10}이므로
최소공배수인 6의 배수이다.
A;B={3, 9}이다.
이를 벤다이어그램으로 나타내면 다음과 같다. 참고
U
A B 4 자연수 n에 대하여 An={x|x는 n의 배수인 자연수}일 때,
1 3 세 자연수 p, q, r에 대하여
6
7
2 ❶ q가 p의 배수이면 Aq,Ap이므로
9 8
5 10 Ap'Aq=Ap, Ap;Aq=Aq이다.
❷ Ap;Aq=Ar일 때, r는 p와 q의 최소공배수이다.
∴ A={1, 2, 3, 5, 9}

Ⅳ. 집합과 명제 5
024 ① 029 ⑤

n(A'B)=n(A)+n(B)-n(A;B)에서 A={x|x는 15 이하의 소수}


24=10+18-n(A;B)이므로 ={2, 3, 5, 7, 11, 13}
n(A;B)=4 이므로 집합 A의 모든 부분집합의 개수는 2ß`=64이다.
∴ n(A-B)=n(A)-n(A;B)
=10-4=6
030 15

025 11 집합 {1, 2, 3, {1, 2}}의 원소는 1, 2, 3, {1, 2}로 4개이므로 집합


{1, 2, 3, {1, 2}}의 진부분집합의 개수는 2Ý`-1=15이다.
두 집합 A, B가 서로소이므로 A;B=∆
즉, n(A;B)=0이다.
따라서 n(A'B)=n(A)+n(B)=13+26=39이므로
C C
n(A ;B )=n((A'B) )
C 031 ②

=n(U)-n(A'B) 4를 원소로 갖는 집합 A의 부분집합은


=50-39=11 집합 {1, 2, 3}의 부분집합에 원소 4를 포함시킨 것과 같으므로
구하는 부분집합의 개수는 집합 {1, 2, 3}의 부분집합의 개수와
같다.
026 ②
∴ 2Ü`=8(개)
C C C
n(A 'B )=n((A;B) )
=n(U)-n(A;B)
이므로 24=30-n(A;B)에서 032 ①

n(A;B)=6 집합 {a, c}와 서로소이기 위해서는


∴ n(A'B)=n(A)+n(B)-n(A;B) 두 원소 a, c를 포함하지 않아야 한다.
=10+15-6=19 집합 {a, b, c, d, e}의 부분집합 중
두 원소 a, c를 포함하지 않는 부분집합의 개수는
집합 {b, d, e}의 부분집합의 개수와 같다.
027 ③
따라서 구하는 집합의 개수는 2Ü`=8이다.
수학자 유클리드를 아는 학생의 집합을 A,
수학자 페르마를 아는 학생의 집합을 B라 하면
수학자 유클리드 또는 수학자 페르마를 아는 학생의 집합은 033 ③
A'B이고, 두 수학자를 모두 아는 학생의 집합은 A;B이다.
집합 A={-2, -1, 0, 1, 2, 3, 4}의 부분집합 중
이때, 주어진 조건에 의하여
두 원소 0, 4는 포함하고, -1은 포함하지 않는 것은
n(A)=18, n(B)=12, n(A'B)=23이므로
집합 {-2, 1, 2, 3}의 부분집합에
n(A'B)=n(A)+n(B)-n(A;B)에서
두 원소 0, 4를 포함시킨 것과 같다.
23=18+12-n(A;B)
따라서 구하는 부분집합의 개수는 집합 {-2, 1, 2, 3}의
∴ n(A;B)=7
부분집합의 개수와 같다.
따라서 두 수학자를 모두 아는 학생 수는 7이다.
∴ 2Ý`=16(개)

028 47

100 이하의 3의 배수는 3_1, 3_2, y, 3_33이므로


034 4

n(A)=33 B-A={2, 3, 8}이므로


100 이하의 5의 배수는 5_1, 5_2, y, 5_20이므로 (B-A),X,B를 만족시키는 집합 X의 개수는
n(B)=20 2, 3, 8을 모두 원소로 가지는 집합 B의 부분집합의 개수와 같다.
3과 5의 최소공배수는 15이므로 즉, 집합 X는 집합 B의 원소 중 2, 3, 8을 제외한 나머지 원소
100 이하의 15의 배수는 15_1, 15_2, y, 15_6이므로 4, 7을 포함할 수도 있고 포함하지 않을 수도 있다.
n(A;B)=6 따라서 구하는 집합 X의 개수는 집합 {4, 7}의 부분집합의 개수와
∴ n(A'B)=n(A)+n(B)-n(A;B) 같다.
=33+20-6=47 ∴ 2Û`=4(개)

6 수학 <하>
즉, aÛ`-3=1에서 aÛ`=4
035 48
∴ a=-2 또는 a=2
A={1, 2, 3, 6, 9, 18}이므로 Ú a=-2일 때, A={1, 6}, B={2, 1}이므로 A+B이다.
집합 A의 부분집합의 개수는 2ß`=64 Û a=2일 때, A={1, 2}, B={2, 1}이므로 A=B이다.
한편, 집합 A의 부분집합 중에서 Ú, Û에 의하여 a=2이다.
소수가 한 개도 포함되지 않은 집합의 개수는 참고
집합 {1, 6, 9, 18}의 부분집합의 개수와 같으므로 2Ý`=16
ⅠV
aÛ`-3=1 집
따라서 구하는 부분집합의 개수는 64-16=48이다. 결국 이 문제는 연립방정식 [ 의 해를 구하는 것이다.
aÛ`-a=2 합



036 7

집합 A={a|a=2nÛ`+1, nÉ3인 자연수}에서


040 풀이 참조

nÉ3인 자연수 n은 1, 2, 3이다. A={1, 2-a}, B={3, a-4, 2a-5}에서


n=1일 때, 2_1Û`+1=3이고 A,B이므로 1<A에서 1<B이어야 한다.
n=2일 때, 2_2Û`+1=9이고 Ú 1=a-4인 경우
n=3일 때, 2_3Û`+1=19이므로 a=5이므로 A={1, -3}, B={3, 1, 5}
A={3, 9, 19} 따라서 AøB이다.
집합 B={b|b는 a를 5로 나누었을 때의 나머지, a<A}에서 Û 1=2a-5인 경우
a=3일 때, 3을 5로 나누었을 때의 나머지는 3이고 a=3이므로 A={1, -1}, B={3, -1, 1}
a=9일 때, 9를 5로 나누었을 때의 나머지는 4이고 따라서 A,B이다.
a=19일 때, 19를 5로 나누었을 때의 나머지는 4이므로 Ú, Û에 의하여 a=3이다.
B={3, 4} 채점 요소 배점
따라서 집합 B의 모든 원소의 합은 3+4=7이다. a-4=1 또는 2a-5=1이어야 함을 찾기 30%
조건을 만족시키는 a의 값 구하기 70%

037 ④

'1Œ0 이하의 자연수는 1, 2, 3으로 3개이므로 n(A10)=3 041 ②


'2Œ0 이하의 자연수는 1, 2, 3, 4로 4개이므로 n(A20)=4
A={x|-1<x-aÉ0}에서 a-1<xÉa
n(Ak)=n(A10)+n(A20)
B={x|xÛ`-7x+6<0}에서 (x-1)(x-6)<0, 1<x<6이므로
=3+4=7
A,B이려면 집합 A의 모든 원소가 집합 B에 포함되어야 한다.
이므로 'k 이하의 자연수의 개수가 7이기 위해선
B
7É'k<8이어야 한다. A
따라서 49Ék<64를 만족시키는 자연수 k는 49, 50, y, 63으로 1 a-1 a 6 x
15개이다. 따라서 1Éa-1과 a<6을 만족시켜야 한다.
즉, 2Éa<6을 만족시키는 자연수 a는 2, 3, 4, 5이므로 그 합은
2+3+4+5=14이다.
038 131

일반성을 잃지 않고 a<b<c<d라 하면
가장 작은 두 수의 합과 곱이 각각 7, 12이므로 042 120

a+b=7, ab=12에서 a=3, b=4 C C


(A'B);(A 'B )=(A'B);(A;B)
C

또한 가장 큰 두 수의 합과 곱이 각각 14, 45이므로 =(A'B)-(A;B)


c+d=14, cd=45에서 c=5, d=9 ={1, 4, 5, 6}
∴ aÛ`+bÛ`+cÛ`+dÛ`=3Û`+4Û`+5Û`+9Û`=131 이므로 이를 벤다이어그램으로 나타내면 다음과 같다.
A B
1 2 4
039 ⑤
6 3 5
두 집합 A={1, aÛ`-a}, B={2, aÛ`-3}에 대하여
A,B이고 B,A이면 A=B이다. 따라서 B={2, 3, 4, 5}이므로
따라서 1<A이므로 1<B이어야 한다. 집합 B의 모든 원소의 곱은 2_3_4_5=120이다.

Ⅳ. 집합과 명제 7
9와 15의 최소공배수가 45이므로 A9;A15=A45이다.
043 ③
따라서 Am,A45에서 m은 45의 배수이어야 하므로
A;B={2, 5} yy㉠ m의 최솟값은 45이다.
이므로 B={b-1, b+2, 7}에서 (A18'A45),An이기 위해서
b-1=2, b+2=5이어야 한다. A18,An, A45,An이어야 하고,
즉, b=3이므로 B={2, 5, 7}이다. 이를 만족시키기 위해서 자연수 n은 18의 약수이면서
이때, A={2, a-1, a+1}에서 다음과 같이 경우를 나누어 생각할 45의 약수이어야 한다.
수 있다. 18과 45의 최대공약수가 9이므로 n의 최댓값은 9이다.
Ú a-1=5일 때, 즉 a=6이므로 A={2, 5, 7} 따라서 구하는 값은 45+9=54이다.
이때, A;B={2, 5, 7}이므로 ㉠을 만족시키지 않는다. 참고
Û a+1=5일 때, 즉 a=4이므로 A={2, 3, 5} 자연수 n에 대하여 An={x|x는 n의 배수인 자연수}
이때, A;B={2, 5}이므로 ㉠을 만족시킨다. 일 때, 세 자연수 p, q, r에 대하여
Ú, Û에 의하여 a=4 ❶ Ar,(Ap;Aq)일 때, r는 p와 q의 공배수이다.
∴ a+b=4+3=7 ❷ (Ap'Aq),Ar일 때, r는 p와 q의 공약수이다.

044 105

A-B=∆이므로 A,B이다.
A={1, 1-2i}, B={x|xÜ`+axÛ`+bx+c=0}에서
집합 A의 두 원소 1, 1-2i는 방정식
xÜ`+axÛ`+bx+c=0 yy㉠
047 ⑤

의 근이다. ㄱ. A'(A;B)를 벤다이어그램으로 나타내면


이때, a, b, c는 실수이므로 U U U
A B A B A B
1-2i의 켤레복소수 1+2i도 방정식 ㉠의 근이다.
즉, 방정식 ㉠의 세 근은 1, 1-2i, 1+2i이므로
삼차방정식의 근과 계수의 관계에 의하여
A Õ {AÖB} = A
-a=1+(1-2i)+(1+2i), a=-3
∴ A'(A;B)=A (참)
b=1_(1-2i)+(1-2i)(1+2i)+(1+2i)_1, b=7
ㄴ. A-(A-B)를 벤다이어그램으로 나타내면
-c=1_(1-2i)_(1+2i), c=-5
U U U
∴ abc=(-3)_7_(-5)=105 A B A B A B

045 ④
A - {A-B} = AÖB
두원 ∴ A-(A-B)=A;B (참)
xÛ`+yÛ`+2x-3y-11=0 yy ㉠ ㄷ. (A'B)-(A;B)를 벤다이어그램으로 나타내면
xÛ`+yÛ`-2x+2y-5=0 yy ㉡ U U U
A B A B A B
에 대하여 집합 A는 원 ㉠ 위의 점을 원소로 갖고,
집합 B는 원 ㉡ 위의 점을 원소로 갖는다.
그러므로 (A;B),C를 만족시키려면
{AÕB} - {AÖB} = {A-B}Õ{B-A}
직선 ax+by+6=0이 두 원 ㉠, ㉡의 교점을 모두 지나야 한다.
∴ (A'B)-(A;B)=(A-B)'(B-A) (참)
두 원 ㉠, ㉡의 교점을 지나는 직선의 방정식은
따라서 옳은 것은 ㄱ, ㄴ, ㄷ이다.
xÛ`+yÛ`+2x-3y-11-(xÛ`+yÛ`-2x+2y-5)=0
4x-5y-6=0에서 -4x+5y+6=0 다른 풀이

a=-4, b=5이므로 ㄱ. (A;B),A이므로


∴ a+b=1 A'(A;B)=A (참)
C C
ㄴ. A-(A-B)=A;(A;B )
C
=A;(A 'B)
046 54 C
=(A;A )'(A;B)
A9;A15는 9의 배수이면서 15의 배수인 자연수의 집합이므로 9와 =∆'(A;B)
15의 공배수의 집합이다. =A;B (참)

8 수학 <하>
ㄷ. (A'B)-(A;B)
=(A'B);(A;B)
C 050 23

C C
=(A'B);(A 'B ) A={x|xÛ`+x-12<0}에서
C C
={(A'B);A }'{(A'B);B } xÛ`+x-12<0, (x+4)(x-3)<0이므로
C C C C
={(A;A )'(B;A )}'{(A;B )'(B;B )} A={x|-4<x<3}
={∆'(B-A)}'{(A-B)'∆} A B
ⅠV
=(B-A)'(A-B) -4 3 5 x

=(A-B)'(B-A) (참) 합
이때, A;B=∆, A'B={x|-4<xÉ5}이므로 과
따라서 옳은 것은 ㄱ, ㄴ, ㄷ이다.
B={x|xÛ`+ax+bÉ0}={x|3ÉxÉ5}이어야 한다. 명
xÛ`+ax+b=(x-3)(x-5) 제

=xÛ`-8x+15
048 ⑤ 이므로 a=-8, b=15이다.
C C C ∴ b-a=23
(A'B);(A-B) =(A'B);(A;B )
C
=(A'B);(A 'B)
=(A;A )'B
C 051 ③
C C
=∆'B=B ① A△U=(A-U)'(U-A)=∆'A =A (참)
C C C C
이므로 B=A;B에서 B,A이다. ② A△A =(A-A )'(A -A)=A'A =U (참)
이를 벤다이어그램으로 나타내면 다음과 같다. ③ A△∆=(A-∆)'(∆-A)=A'∆=A (거짓)
U C C C
④ A △B =(A -B )'(B -A )
C C C
A
C C
=(A ;B)'(B ;A)
B
=(B-A)'(A-B)
=(A-B)'(B-A)=A△B (참)
⑤ A△B=A이면 (A-B)'(B-A)=A yy㉠
① AøB (거짓)
C 이때, (B-A)øA이므로 B-A=∆이어야 한다.
② A+B (거짓)
∴ B,A yy㉡
③ A'B=A (거짓)
㉠에서 (A-B)'∆=A, 즉 A-B=A이므로
④ A-B+∆ (거짓)
C C A;B=∆
⑤ A ;B=∆이므로 두 집합 A , B는 서로소이다. (참)
이때, ㉡에 의하여 A;B=B이므로 B=∆이다. (참)
따라서 선지 중 항상 옳은 것은 ⑤이다.
따라서 선지 중 옳지 않은 것은 ③이다.
다른 풀이
C
(A'B);(A-B) 을 다음과 같이 정리할 수도 있다.
(A'B);(A-B) =(A'B)-(A-B)
C
yy ㉠
052 풀이 참조

이를 벤다이어그램으로 나타내면 다음과 같다. A3;(A2'A5)=(A3;A2)'(A3;A5)


U U U =A6'A15
A B A B A B 이때, 6과 15의 최소공배수는 30이므로
n(A6'A15)=n(A6)+n(A15)-n(A6;A15)
=n(A6)+n(A15)-n(A30) yy㉠
{AÕB} - {A-B} = B 200 이하의 6의 배수는
즉, ㉠에서 (A'B)-(A-B)=B이다. 6_1, 6_2, y, 6_33이므로 n(A6)=33
이후는 본풀이와 동일하다. 200 이하의 15의 배수는
15_1, 15_2, y, 15_13이므로 n(A15)=13
200 이하의 30의 배수는

049 ④ 30_1, 30_2, y, 30_6이므로 n(A30)=6


따라서 집합 A3;(A2'A5)의 원소의 개수는 ㉠에서
C C
{(A'B);(A'B )};{(A'B) '(A-B)} n(A6)+n(A15)-n(A30)=33+13-6=40이다.
C C C C
={(A'B);(A'B )};{(A ;B )'(A;B )}
C C C 채점 요소 배점
={A'(B;B )};{(A 'A);B }
C A3;(A2'A5)를 A6'A15로 변형하기 30%
=(A'∆);(U;B )
C n(A6'A15)=n(A6)+n(A15)-n(A30)으로 식 세우기 30%
=A;B
n(A6'A15)의 값 구하기 40%
=A-B

Ⅳ. 집합과 명제 9
Û n(A;B)의 값이 최소인 경우 U
053 ③
A'B=U일 때이므로 A B
0

주어진 고등학교 1학년 학생 전체의 집합을 U, n(A;B)의 최솟값은 17 6 13


그 중 버스를 이용하여 통학하는 학생의 집합을 A, n(A)+n(B)-n(A'B)
지하철을 이용하여 통학하는 학생의 집합을 B라 하면 =23+19-36=6
버스와 지하철을 모두 이용하여 통학하는 학생의 집합은 A;B이고 따라서 구하는 최댓값과 최솟값의 합은 19+6=25이다.
버스와 지하철 중 어느 것도 이용하지 않고 통학하는 학생의 집합은 TIP
C C
A ;B 이다. 전체집합 U의 두 부분집합 A, B에 대하여
이때, 주어진 조건에 의하여 n(A;B)의 최댓값과 최솟값은 다음과 같이 정해진다.
n(U)=185, n(A)=65, n(B)=46, n(A;B)=21이므로 <n(A;B)의 최댓값>
n(A'B)=n(A)+n(B)-n(A;B) n(A;B)Én(A), n(A;B)Én(B)이므로
=65+46-21=90 n(A;B)의 최댓값은 n(A), n(B) 중 크지 않은 값이다.
따라서 버스와 지하철 중 어느 것도 이용하지 않고 통학하는 학생 ❶ n(A)Én(B)인 경우
C C
수는 n(A ;B )=n(U)-n(A'B)=185-90=95이다. n(A;B)의 최댓값은 n(A)이다.
이때, A,B이다.
❷ n(B)Én(A)인 경우
054 풀이 참조
n(A;B)의 최댓값은 n(B)이다.
주어진 학급의 학생 전체의 집합을 U, 이때, B,A이다.
그 중 축구를 좋아하는 학생의 집합을 A, U U
B A
농구를 좋아하는 학생의 집합을 B라 하면
A B
C C
축구와 농구를 모두 좋아하지 않는 학생의 집합은 A ;B 이므로
최대 최대
주어진 조건에 의하여
C C
n(U)=32, n(A)=20, n(B)=13, n(A ;B )=7이다.
❶ n(A)Én(B) ❷ n(B)Én(A)
⑴ 축구와 농구 중에서 적어도 하나를 좋아하는 학생의 집합은
A'B이므로
C
<n(A;B)의 최솟값>
n(A'B)=n(U)-n((A'B) )
C C
n(A'B)Én(U)이므로
=n(U)-n(A ;B )
n(A)+n(B)-n(A;B)Én(U)이다.
=32-7=25
n(A;B)¾n(A)+n(B)-n(U)에서
따라서 축구와 농구 중에서 적어도 하나를 좋아하는 학생 수는
❶ n(A)+n(B)¾n(U)인 경우
25이다.
n(A;B)의 최솟값은 n(A)+n(B)-n(U)이다.
⑵ 축구와 농구를 모두 좋아하는 학생의 집합은 A;B이므로
이때, A'B=U이다.
n(A'B)=n(A)+n(B)-n(A;B)에서
❷ n(A)+n(B)Én(U)인 경우
25=20+13-n(A;B), n(A;B)=8
n(A;B)의 최솟값은 0이다.
따라서 축구와 농구를 모두 좋아하는 학생 수는 8이다.
이때, A;B=∆이다.
⑶ 농구만 좋아하는 학생의 집합은 B-A이므로
U U
n(B-A)=n(B)-n(A;B)=13-8=5 A B 0 A B
따라서 농구만 좋아하는 학생 수는 5이다. 최

채점 요소 배점
n(A'B)의 값 구하기 30% ❶ n(A)+n(B)¾n(U) ❷ n(A)+n(B)Én(U)
n(A;B)의 값 구하기 30%
n(B-A)의 값 구하기 40%

055 ① 056 ⑴ 30 ⑵ 48

주어진 놀이동산 입장객의 전체의 집합을 U,


Ú n(A;B)의 값이 최대인 경우 U
A 13 그 중 회전목마를 이용한 입장객의 집합을 A,
B,A일 때이므로
B 4 롤러코스터를 이용한 입장객의 집합을 B라 하면
n(A;B)의 최댓값은
19 주어진 조건에 의하여
n(B)=19
n(U)=50, n(A)=26, n(B)=28이다.

10 수학 <하>
⑴ 두 놀이기구를 모두 이용한 입장객의 집합은 A;B이므로
❶ n(A)+n(B)¾n(U)인 경우
Ú n(A;B)의 값이 최대인 경우 U
B 22 n(A'B)의 최댓값은 n(U)이다.
A,B일 때이므로
A 2 이때, A'B=U이다.
n(A;B)의 최댓값은
26 ❷ n(A)+n(B)Én(U)인 경우
n(A)=26
n(A'B)의 최댓값은 n(A)+n(B)이다.
Û n(A;B)의 값이 최소인 경우 U 이때, A;B=∆이다. ⅠV
0
A'B=U일 때이므로 A B U U 집
A B 0 A 최대 B 합
n(A;B)의 최솟값은 22 4 24 과
최대
n(A)+n(B)-n(A'B) 명

=26+28-50=4
❶ n(A)+n(B)¾n(U) ❷ n(A)+n(B)Én(U)
따라서 구하는 값은 26+4=30이다.
⑵ 롤러코스터만 이용한 입장객의 집합은
<n(A'B)의 최솟값>
n(B-A)=n(B)-n(A;B)
n(A)Én(A'B), n(B)Én(A'B)이므로
=28-n(A;B) yy㉠
n(A'B)의 최솟값은 n(A), n(B) 중 작지 않은 값이다.
이때, ⑴에서 4Én(A;B)É26이므로
❶ n(A)Én(B)인 경우
㉠에서 2Én(B-A)É24이다.
n(A'B)의 최솟값은 n(B)이다.
따라서 n(B-A)의 최댓값은 24, 최솟값은 2이므로
이때, A,B이다.
구하는 값은 24_2=48이다.
❷ n(B)Én(A)인 경우
n(A'B)의 최솟값은 n(A)이다.
057 ⑤
이때, B,A이다.
U U
주어진 100명의 소비자 전체의 집합을 U, B A
그 중 음료수 A를 선호하는 사람의 집합을 A,
A B
음료수 B를 선호하는 사람의 집합을 B라 하면
최소 최소
주어진 조건에 의하여
n(U)=100, n(A)=56, n(B)=28이다. ❶ n(A)Én(B) ❷ n(B)Én(A)
이때, 두 음료수 중 어느 것도 선호하지 않는 사람의 집합은
C C C
A ;B =(A'B) 이므로
C C
n(A ;B )=n(U)-n(A'B)
=100-n(A'B) yy㉠
058 43

Ú n(A'B)의 값이 최소인 경우 U n(A)=5, n(B)=7이므로 n(A;B)의 값은


A A'B=U일 때 최솟값
B,A일 때이므로
B n(A)+n(B)-n(U)=5+7-10=2를 갖고
n(A'B)의 최솟값은
n(A)=56 A,B일 때 최댓값 n(A)=5를 갖는다.
∴ 2Én(A;B)É5
Û n(A'B)의 값이 최대인 경우 U
집합 U의 모든 원소가 양수이므로
A B
A;B=∆일 때이므로 A;B의 모든 원소의 합은
n(A'B)의 최댓값은 n(A;B)=5이고 집합 A;B(=A)의 원소 5개가
n(A)+n(B)=56+28=84이다. 6, 7, 8, 9, 10일 때 최댓값을 갖고,
Ú, Û에 의하여 56Én(A'B)É84이므로 n(A;B)=2이고 집합 A;B의 원소 2개가
㉠에서 M=100-56=44, m=100-84=16 1, 2일 때 최솟값을 갖는다.
∴ M-m=28 즉, M=6+7+8+9+10=40, m=1+2=3이므로
TIP M+m=43
전체집합 U의 두 부분집합 A, B에 대하여
n(A'B)의 최댓값과 최솟값은 다음과 같이 정해진다. 059 64
<n(A'B)의 최댓값>
n(A'B)=n(A)+n(B)-n(A;B)에서
n(A'B)=n(A)+n(B)-n(A;B)이므로
12=n(A)+n(B)-4이므로
n(A;B)가 최솟값을 가질 때,
n(A)+n(B)=16이다.
n(A'B)는 최댓값을 갖는다.
이때, n(A)=x라 하면 n(B)=16-x (4ÉxÉ12)

Ⅳ. 집합과 명제 11
집합 A_B={(a, b)|a<A, b<B}의 원소의 개수는 집합의 원소의 개수를 벤다이어그램으로 나타내면 다음과 같다.
n(A)_n(B)와 같으므로 A
n(A_B)=n(A)_n(B) 2
=x(16-x) 10 8
=-(x-8)Û`+64 0
1 x 2
따라서 n(A_B)는 x=8일 때, 최댓값 64를 갖는다. B C
참고
2명의 후보를 적은 투표용지는 30장이므로 B와 C가 동시에 적힌
산술평균과 기하평균의 관계를 학습한 이후
투표용지의 수를 x라 하면
다음과 같이 풀이할 수 있다.
10+8+x=30, x=12
n(A'B)=n(A)+n(B)-n(A;B)에서
따라서 n(A)=2+10+8=20, n(B)=1+10+12=23,
12=n(A)+n(B)-4이므로
n(C)=2+12+8=22이므로 표를 많이 얻은 후보부터 차례로
n(A)+n(B)=16이다.
나열하면 B, C, A이다.
n(A)>0, n(B)>0이므로
산술평균과 기하평균의 관계에 의하여
n(A)+n(B)¾2"Ãn(A)_n(B)
"Ãn(A)_n(B) É8, n(A)_n(B)É64
(단, 등호는 n(A)=n(B)=8일 때 성립한다.) 062 9
따라서 n(A)=n(B)=8일 때 n(A_B)의 최댓값은
방문한 45개의 음식점 전체의 집합을 U,
8_8=64이다.
기준 ㈎, ㈏, ㈐를 만족시키는 음식점의 집합을 각각 A, B, C라
하면 주어진 조건에 의하여
n(U)=45, n(A)=36, n(B)=25, n(C)=23이고,
060 ④ _표시를 받은 음식점의 집합은 (A'B'C) 이므로
C
C

n((A'B'C) )=4
첫째 날 봉사활동을 한 학생의 집합을 A,
집합의 원소의 개수를 벤다이어그램으로 나타내면 다음과 같다.
둘째 날 봉사활동을 한 학생의 집합을 B,
U
셋째 날 봉사활동을 한 학생의 집합을 C라 하면 A 4
주어진 조건에 의하여 n(A)=23, n(B)=15, n(C)=17,
n(A;B;C)=5, n(A'B'C)=40이다.
x
이때, 집합의 원소의 개수를 벤다이어그램으로 나타내면 다음과
B C
같다.
A 별점 2개를 받은 음식점의 수는 25이므로
a 색칠된 부분의 원소의 개수는 25이고,
d f 별점 3개를 받은 음식점의 수를 x라 하면 x=n(A;B;C)이므로
5
b c
n(A'B'C)=n(A)+n(B)+n(C)-25-2x yy TIP
e C
B C 이때, n(A'B'C)=n(U)-n((A'B'C) )=45-4=41
이므로 41=36+25+23-25-2x
n(A'B'C)=n(A)+n(B)+n(C)
∴ x=9
-n(A;B)-n(B;C)-n(C;A)
따라서 별점 3개를 받은 음식점의 개수는 9이다.
+n(A;B;C)
40=23+15+17-(d+5)-(e+5)-( f+5)+5 TIP
에서 d+e+ f=5 집합의 원소의 개수를 벤다이어그램에 U
A 4
따라서 그림과 같이 나타내면 별점 2개를 받은
a+b+c=n(A'B'C)-(d+e+ f)-5 음식점의 수는 a+b+c=25이다.
a c
=40-5-5=30 x=n(A;B;C)라 하면 x
이므로 하루만 봉사활동을 한 학생 수는 30이다. n(A'B'C) b
B C
=n(A)+n(B)+n(C)
-n(A;B)-n(B;C)-n(C;A)+n(A;B;C)
061 ④ =n(A)+n(B)+n(C)-(a+x)-(b+x)-(c+x)+x
=n(A)+n(B)+n(C)-(a+b+c)-2x
세 명의 회장 후보 A, B, C에게 투표한 학생의 투표용지의 집합을
가 성립한다.
각각 A, B, C라 하자.

12 수학 <하>
063 ① 066 ④

주어진 독서동아리 학생 전체의 집합을 U, ‘심청전’, ‘춘향전’, 주어진 벤다이어그램에 의하여


‘구운몽’을 읽은 학생의 집합을 각각 A, B, C라 하자. A'B={1, 2, 3, 4, 5, 6, 7}, A;B={2, 5}
주어진 조건에 의하여 n(U)=50, n(A)=33, n(B)=32, (A'B);X=X에서 X,(A'B)이고,
n(C)=27이고, (A;B)'X=X에서 (A;B),X이므로
집합의 원소의 개수를 벤다이어그램으로 나타내면 다음과 같다. (A;B),X,(A'B)를 만족시키는 집합 X는
ⅠV

U 집합 A'B의 부분집합 중에서 두 원소 2, 5를 모두 원소로 가지는 합
A

부분집합이다.

따라서 구하는 집합 X의 개수는 집합 {1, 3, 4, 6, 7}의 부분집합의

x 개수와 같다.
B C ∴ 2Þ`=32(개)

세 권 중 두 권만 읽은 학생이 30명이므로
색칠된 부분의 원소의 개수는 30이고,
세 권을 모두 읽은 학생 수를 x라 하면 x=n(A;B;C)이므로
067 8

n(A'B'C)=n(A)+n(B)+n(C)-30-2x A={1, 3, 5, 7, 9}, B={1, 3, 9}에 대하여


=33+32+27-30-2x A=B'C를 만족시키기 위해서
=62-2x 집합 C는 5, 7을 모두 원소로 포함해야 하고,
이때, n(A'B'C)Én(U)이므로 나머지 원소 1, 3, 9는 포함할 수도 있고 포함하지 않을 수도 있다.
62-2xÉ50에서 x¾6이다. 따라서 집합 C의 개수는 집합 {1, 3, 9}의 부분집합의 개수와 같다.
따라서 세 권을 모두 읽은 학생 수의 최솟값은 6이다. ∴ 2Ü`=8(개)
참고

n(A;B;C)의 값이 최소인 경우는 A'B'C=U일 때이다.


068 ③

{1, 2, 4};X+∆이므로 집합 X는 1, 2, 4 중 적어도 하나의


원소를 포함해야 한다.
064 ③
집합 U의 부분집합 중에서 1, 2, 4 중 한 개도 포함하지 않은 집합의
집합 A의 원소의 개수가 n일 때 개수는 집합 {3, 5}의 부분집합의 개수와 같으므로 2Û`=4
집합 A의 진부분집합의 개수는 2 -1이므로
n
따라서 구하는 집합 U의 진부분집합 X의 개수는
n n
조건 ㈎에서 2 -1=7, 2 =8=2Ü` ∴ n=3 (2Þ`-1)-4=27이다.
조건 ㈏에 의하여 집합 A의 원소의 최댓값은 15이므로
A={13, 14, 15}일 때, 집합 A의 모든 원소의 합이 최댓값
13+14+15=42를 갖는다. 069 64

A'C=B'C가 성립하기 위해서는


(A-B),C, (B-A),C가 성립해야 한다.
065 16 A={1, 3, 5, 7, 9}, B={3, 6, 9}에서
A-B={1, 5, 7}, B-A={6}이므로
A={1, 3, 5, 7, 9, 11, 13, 15}이고,
n 집합 C는 1, 5, 7과 6을 모두 원소로 포함해야 한다.
B={x|x는 7 의 일의 자리의 수, n은 자연수}에서
따라서 집합 C의 개수는 집합 {2, 3, 4, 8, 9, 10}의 부분집합의
n=1, 2, 3, 4, 5, y일 때,
n 개수와 같다.
7 의 일의 자리의 수는 7, 9, 3, 1이 이 순서대로 반복되므로
∴ 2ß`=64(개)
B={1, 3, 7, 9}이다.
이때, A'X=A에서 X,A이고, B;X=B에서 B,X이므로
B,X,A를 만족시키는 집합 X는
집합 A의 부분집합 중 집합 B의 네 원소 1, 3, 7, 9를 모두 원소로 070 풀이 참조

가지는 부분집합이다. 조건 ㈎에서 A,X이고


따라서 구하는 집합 X의 개수는 집합 {5, 11, 13, 15}의 부분집합의 조건 ㈏에서 {2, 5, 6};X={2, 5}이므로
개수와 같다. 집합 X는 {1, 2, 4, 5},X, 6²X를 만족시키는 집합 U의
∴ 2Ý`=16(개) 부분집합이다.

Ⅳ. 집합과 명제 13
따라서 집합 X는 집합 {3, 7}의 부분집합에 1, 2, 4, 5를 포함한 a, c, d는 포함하고 b는 포함하지 않거나
것과 같다. b, c, d는 포함하고 a는 포함하지 않아야 한다.
집합 {3, 7}의 부분집합은 ∆, {3}, {7}, {3, 7}이므로 따라서 집합 B는 집합 {e, f }의 부분집합에 a, b, c 또는 a, b, d
구하는 집합 X는 {1, 2, 4, 5}, {1, 2, 3, 4, 5}, {1, 2, 4, 5, 7}, 또는 a, c, d 또는 b, c, d를 포함한 것과 같으므로 구하는 집합 B의
{1, 2, 3, 4, 5, 7}이다. 개수는 4_2Û`=16이다.

채점 요소 배점
집합 X는 {1, 2, 4, 5},X, 6²X를 만족시키는 집합 U의 부분집
40%
074 5
합임을 구하기
1 1 1 1
집합 X 구하기 60% 집합 A의 원소의 크기를 비교하면 1> > > > 이다.
2 2Û` 2Ü` 2Ý`
Ú ak=1일 때
집합 Ak는 {1}로 1개뿐이다.
071 31 따라서 ak=1인 k의 개수는 1이다.
1
집합 A는 자연수 전체의 집합의 부분집합이므로 Û ak= 일 때
2
x가 집합 A의 원소이면 x는 자연수이고, 1 1 1 1
집합 Ak는 은 포함하고 , , 은 포함하지 않는
36 36 2 2Û` 2Ü` 2Ý`
도 집합 A의 원소이므로 도 자연수이다.
x x 부분집합이므로 집합 Ak의 개수는 집합 {1}의 부분집합의 개수와
따라서 x는 36의 약수이어야 하므로 같다.
1, 2, 3, 4, 6, 9, 12, 18, 36이 될 수 있다. 1
따라서 ak= 인 k의 개수는 2Ú`이다.
이때, 주어진 조건에 의하여 2
{1, 36},A 또는 {2, 18},A 또는 {3, 12},A 또는 {4, 9},A 1
Ü ak= 일때
또는 {6},A이므로 2Û`
1 1 1
1과 36, 2와 18, 3과 12, 4와 9를 각각 한 개의 원소로 생각하면 집합 Ak는 은 포함하고 , 은 포함하지 않는
2Û` 2Ü` 2Ý`
집합 A의 개수는 원소가 5개인 집합의 공집합이 아닌 부분집합의 1
부분집합이므로 집합 Ak의 개수는 집합 [1, ]의 부분집합의
개수와 같다. 2
따라서 집합 A의 개수는 2Þ`-1=31이다. 개수와 같다.
1
따라서 ak= 인 k의 개수는 2Û`이다.
2Û`
072 ②
Ý ak=
1
2Ü`
일때
집합 U의 부분집합의 개수는 2ß`=64 1 1
집합 Ak는 은 포함하고 은 포함하지 않는 부분집합이므로
조건 ㈎에 의하여 집합 X의 원소 중 적어도 하나는 짝수이어야 2Ü` 2Ý`
한다. 1 1
집합 Ak의 개수는 집합 [1, , ]의 부분집합의 개수와 같다.
2 2Û`
집합 U의 부분집합 중에서 짝수가 한 개도 포함되지 않은 집합의
1
개수는 집합 {1, 3, 5}의 부분집합의 개수와 같으므로 2Ü`=8 따라서 ak= 인 k의 개수는 2Ü`이다.
2Ü`
이때, 조건 ㈏에 의하여 n(X)>4인 것은 제외해야 한다. 1
Þ ak= 일 때
Ú n(X)=5일 때 2Ý`
{1, 2, 3, 4, 5}, {1, 2, 3, 4, 6}, {1, 2, 3, 5, 6}, 1
집합 Ak는 을 포함하는 부분집합이므로 집합 Ak의 개수는
{1, 2, 4, 5, 6}, {1, 3, 4, 5, 6}, {2, 3, 4, 5, 6}으로 6개 2Ý`
Û n(X)=6일 때 1 1 1
집합 [1, , , ]의 부분집합의 개수와 같다.
2 2Û` 2Ü`
{1, 2, 3, 4, 5, 6}으로 1개
1
따라서 구하는 집합 X의 개수는 따라서 ak= 인 k의 개수는 2Ý`이다.
2Ý`
64-8-6-1=49이다. Ú~Þ에 의하여
a1+a2+a3+y+a31
1 1 1 1
=(1_1)+{ _2}+{ _2Û`}+{ _2Ü`}+{ _2Ý`}=5
073 16 2 2Û` 2Ü` 2Ý`
참고
두 집합 U={a, b, c, d, e, f }, A={a, b, c, d}에 대하여
n(A;B)=3을 만족시키는 집합 B는 1 1 1 1
집합 A=[1,, , , ]의 공집합이 아닌 서로 다른
2 2Û` 2Ü` 2Ý`
a, b, c, d 중 3개만 원소로 가져야 하므로
부분집합의 개수는 2Þ`-1=31이므로 A1, A2, A3, y, A31과
a, b, c는 포함하고 d는 포함하지 않거나
같이 표기할 수 있다.
a, b, d는 포함하고 c는 포함하지 않거나

14 수학 <하>
집합 (ACB)CC와 AC(BCC)를
075 ③
벤다이어그램으로 나타내면
ㄱ. {∆}은 집합 A의 원소이므로 {∆}<A (참) U U U
A A A
ㄴ. 집합 A의 원소는 ∆, {∆}, {∆, {∆}}으로 총 3개이므로 집합
A의 진부분집합의 개수는 2Ü`-1=7이다. (참)
ㄷ. A'{A}={∆, {∆}, {∆, {∆}}, A}이므로 집합 A'{A}의 B C B C B C
원소는 ∆, {∆}, {∆, {∆}}, A로 총 4개이다.
ⅠV
{A *B} * C = {A * B} * C 집
따라서 n(A'{A})=4 (거짓) 합
U U U
따라서 옳은 것은 ㄱ, ㄴ이다. A A A 과

B C B C B C
076 ③
A * {B * C} = A * {B * C}
집합 P(A)={X|X,A}의 원소는 집합 A의 모든 부분집합이다. 이므로 (ACB)CC=AC(BCC) (참)
ㄱ. A,B이면 집합 A의 모든 부분집합은 집합 B의 부분집합이므로 ㄷ. ACA=(A;A)'(A'A)
C

P(A),P(B) (참) =A'A =U


C

ㄴ. 집합 A의 부분집합이면서 집합 B의 부분집합인 것은 A;B의 ACACA=UCA=(U;A)'(U'A)


C

부분집합과 같다. =A'U =A'∆=A


C

따라서 P(A;B)=P(A);P(B) (참) ACACACA=ACA=U


ㄷ. A={1, 2}, B={2, 3}이라 하면 ⋮
{1, 3}<P(A'B)이지만 {1, 3}²P(A), 이므로 자연수 n에 대하여
{1, 3}²P(B)이므로 P(A'B)+P(A)'P(B) (거짓) ACACyCA=A, ACACyCA=U이다.
따라서 옳은 것은 ㄱ, ㄴ이다.
[

[
A가 (2n-1)개 A가 2n개

그러므로 ACACyCA=A (참)

077 10
따라서 옳은 것은 ㄱ, ㄴ, ㄷ이다.
참고
[
A가 99개

n(Ap)=1에서 양의 약수의 개수가 1인 자연수는 1뿐이다.


C
n(Aq)=2에서 양의 약수의 개수가 2인 자연수는 소수이다. 문제에서 주어진 연산 ACB=(A;B)'(A'B) 에서
C C C
n(Ar)=3에서 양의 약수의 개수가 3인 자연수는 (소수)Û` 꼴이다. (ACB) ={(A;B)'(A'B) }
C
따라서 두 소수 x, y에 대하여 =(A;B) ;(A'B)
p=1, q=x, r=yÛ`이다. =(A'B)-(A;B)
Ú x=y일 때 이므로 ACB는 두 집합 A, B의 대칭차집합의 여집합을
pqr=xÜ`이고, xÜ`의 양의 약수의 개수는 4이므로 n(Apqr)=4 의미한다.
Û x+y일 때
pqr=xyÛ`이고, xyÛ`의 양의 약수의 개수는 2_3=6이므로
n(Apqr)=6
Ú, Û에 의하여 집합 Apqr의 원소의 개수는 4 또는 6이므로 그 합은 079 7

4+6=10이다. Z;{1, 3, 5, 6}={1, 5}이므로 1<Z, 5<Z, 3²Z, 6²Z


Y☆{3, 4, 5, 6}=Z에서
Y
078 ⑤
1
3
4 5
ㄱ. ACB=(A;B)'(A'B)
C 6
C
=(B;A)'(B'A)
집합 Z는 벤다이어그램에서 색칠된 부분이므로
=BCA (참)
{3, 6},Y;{3, 4, 5, 6}, 5<{3, 4, 5, 6}-Y,
ㄴ. 집합 ACB를 벤다이어그램으로 나타내면 다음과 같다.
1<Y-{3, 4, 5, 6}이다.
U
A B 즉, {1, 3, 6},Y이어야 한다.
따라서 {2, 3, 4, 7}'X=Y이려면
집합 X는 적어도 두 원소 1, 6을 가져야 하므로
s의 최솟값은 1+6=7이다.

Ⅳ. 집합과 명제 15
S(A)S(B)=x(52-x)
080 ④
=-(x-26)Û`+676
집합 (A B) C를 벤다이어그램으로 나타내면 다음과 같다. 따라서 S(A)S(B)는 x=26일 때 최댓값 676을 갖는다.
U U U 참고
A A A
산술평균과 기하평균의 관계를 학습한 이후
다음과 같이 풀이할 수 있다.
B C B C B C S(A'B)=S(A)+S(B)-S(A;B)에서
{A B} C = {A B} C 45=S(A)+S(B)-7이므로
n(A)는 3의 배수인 100 이하의 자연수의 개수이므로 S(A)+S(B)=52이다.
n(A)=33 이때, 두 집합 A, B의 원소는 모두 양수이다.
n(B)는 4의 배수인 100 이하의 자연수의 개수이므로 따라서 S(A)>0, S(B)>0이므로
n(B)=25 산술평균과 기하평균의 관계에 의하여
n(C)는 5의 배수인 100 이하의 자연수의 개수이므로 S(A)+S(B)¾2"ÃS(A)S(B)
n(C)=20 "ÃS(A)S(B)É26, S(A)S(B)É676
n(A;B)는 12의 배수인 100 이하의 자연수의 개수이므로 (단, 등호는 S(A)=S(B)=26일 때 성립한다.)
n(A;B)=8 따라서 S(A)S(B)의 최댓값은 676이다.
n(B;C)는 20의 배수인 100 이하의 자연수의 개수이므로
n(B;C)=5
n(C;A)는 15의 배수인 100 이하의 자연수의 개수이므로
n(C;A)=6 082 15

n(A;B;C)는 60의 배수인 100 이하의 자연수의 개수이므로 집합 X에 대하여 집합 X의 모든 원소의 합을 S(X)라 하자.
n(A;B;C)=1이다. S(A)=x1+x2+x3+x4+x5+x6=52이므로
집합의 원소의 개수를 벤다이어그램으로 나타내면 다음과 같다. S(B)=(x1+x2+x3+x4+x5+x6)+6k
U
A =52+6k
40
20 S(A'B)=S(A)+S(B)-S(A;B)
7 5 96=52+(52+6k)-(8+11+13)
1
13 10
6k+72=96, k=4
4
B C (A;B),A이므로 집합 A는 8, 11, 13을 포함하고
집합 B는 8+4=12, 11+4=15, 13+4=17
따라서 구하는 집합 (A B) C의 원소의 개수는
즉, 12, 15, 17을 원소로 갖는다.
20+13+10+1=44이다.
또한 (A;B),B이므로 B={8, 11, 12, 13, 15, 17}
참고
따라서 집합 B의 원소 중 두 번째로 큰 수는 15이다.
U U U
A A A

B C B C B C
083 ④

A {B C} = A {B C} ㄱ. A={1, 2}, B={6}이면 S(A)<S(B)이지만 A,B가


따라서 (A B) C=A (B C)가 성립한다. 성립하지 않는다. (거짓)
C
ㄴ. A =B이면 A;B=∆이고 A'B=U이므로
S(A)+S(B)=S(U)=21이다. (참)
ㄷ. A={1, 2, 4}일 때, S(A)=7이므로
S(A)ÉS(B)에서 7ÉS(B)를 만족시켜야 한다.
081 676
이때, ㄴ에 의하여 0ÉS(B)É21이므로 구하는 집합 B의
S(A'B)=S(A)+S(B)-S(A;B)에서 개수는 전체집합 U의 모든 부분집합의 개수 2ß`=64에서
A'B=U, A;B={3, 4}이므로 0ÉS(B)<7인 집합 B의 개수를 빼면 되므로 집합 B의 원소의
S(U)=S(A)+S(B)-S({3, 4}) 개수를 기준으로 생각해 보면 다음과 같다.
45=S(A)+S(B)-7 Ú n(B)=0인 경우
S(A)+S(B)=52 가능한 집합 B는 ∆으로 1개
S(A)=x (7ÉxÉ45)라 하면 Û n(B)=1인 경우
S(B)=52-x이므로 가능한 집합 B는 {1}, {2}, {3}, {4}, {5}, {6}으로 6개

16 수학 <하>
Ü n(B)=2인 경우 집합의 원소의 개수를 벤다이어그램으로 나타내면 다음과 같다.
가능한 집합 B는 {1, 2}, {1, 3}, {1, 4}, {1, 5}, {2, 3}, A
{2, 4}로 6개
Ý n(B)=3인 경우 a 12
가능한 집합 B는 {1, 2, 3}으로 1개 4
b
Þ n(B)¾4인 경우 B C
ⅠV
S(B)의 최솟값은 B={1, 2, 3, 4}일 때 10이므로 가능한
노래 동아리에만 가입한 학생의 집합은 색칠한 부분과 같고, 집
집합 B가 존재하지 않는다. 합
n(B)=15이므로 색칠한 부분에 속하는 학생 수는 과
그러므로 구하는 집합 B의 개수는
a+b가 최대일 때 최소이고, a+b가 최소일 때 최대이다. 명
64-(1+6+6+1)=50이다. (참) 제
a+4+12Én(A)=18에서 aÉ2이고,
따라서 옳은 것은 ㄴ, ㄷ이다.
b+4+12Én(C)=21에서 bÉ5이므로
0ÉaÉ2, 0ÉbÉ5이다.
084 ①
Ú a=0, b=0일 때, a+b=0으로 최소이므로
주어진 학교 전교생 전체의 집합을 U, 구하는 최댓값은 n(B)-4-0=11이다.
국어, 영어, 수학을 보충수업으로 신청한 학생의 집합을 각각 Û a=2, b=5일 때, a+b=7로 최대이므로
A, B, C라 하자. 구하는 최솟값은 n(B)-4-7=4이다.
전체 학생 수를 x라 하면 주어진 조건에 의하여 Ú, Û에서 구하는 최댓값과 최솟값의 차는
n(U)=x, n(A)=0.6x, n(B)=0.3x, n(C)=0.2x, 11-4=7이다.
n(A;B;C)=0.1x이다.
집합의 원소의 개수를 벤다이어그램으로 나타내면 다음과 같다. 086 풀이 참조
U
A ⑴ 집합 U={1, 2, x, 4, 5}의 부분집합 중 원소의 개수가 2인
0.1x 부분집합은
{1, 2}, {1, x}, {1, 4}, {1, 5},
{2, x}, {2, 4}, {2, 5},
B C {x, 4}, {x, 5},
{4, 5}로 10개이다.
이때, 오직 한 과목만 신청한 학생은 전교생의 50`%이므로
∴ n=10
벤다이어그램에서 색칠한 부분의 원소의 개수는 0.5x이고,
⑵ 집합 U의 부분집합 중 1을 포함하고 원소의 개수가 2인
두 과목만 신청한 학생 수를 y라 하면
부분집합의 개수는 4이다.
벤다이어그램에서 빗금 친 부분의 원소의 개수가 y이다.
마찬가지로 2, x, 4, 5를 각각 포함하는 원소의 개수가 2인
0.5x=n(A)+n(B)+n(C)-2y-3_n(A;B;C)
부분집합의 개수도 4이므로
=0.6x+0.3x+0.2x-2y-3_0.1x
s1+s2+s3+y+sn=4_(1+2+x+4+5)
y=0.15x yy ㉠
4(x+12)=120
n(A'B'C)=n(A)+n(B)+n(C)-y-2_n(A;B;C)
∴ x=18
=0.6x+0.3x+0.2x-0.15x-0.2x (∵ ㉠)
=0.75x 채점 요소 배점

이때, 어느 과목도 신청하지 않은 학생이 140명이므로 원소의 개수가 2인 부분집합을 일일이 세서 n의 값 구하기 30%
C
n((A'B'C) )=n(U)-n(A'B'C) 1, 2, x, 4, 5를 각각 포함하는 원소의 개수가 2인 부분집합의 개수가
40%
=x-0.75x 4임을 구하기

=0.25x=140 x의 값 구하기 30%


에서 x=560이다.
따라서 두 과목만 신청한 학생 수는
y=0.15_560=84이다. (∵ ㉠)
087 114

Ú 최대인 원소가 1인 경우
원소의 개수가 2 이상인 부분집합은 존재하지 않는다.
085 7
Û 최대인 원소가 2인 경우
연극, 노래, 춤 동아리에 가입한 학생들의 집합을 각각 A, B, C라 2는 포함하고, 3, 4, 5는 포함하지 않는 부분집합 중 원소의
하면 주어진 조건에 의하여 개수가 2 이상인 집합은 {1, 2}로 1개뿐이다.
n(A)=18, n(B)=15, n(C)=21, n(A;B;C)=4, Ü 최대인 원소가 3인 경우
n(A;C)=16 3은 포함하고, 4, 5는 포함하지 않는 부분집합 중 원소의 개수가

Ⅳ. 집합과 명제 17
2 이상인 집합의 개수는 집합 {1, 2}의 부분집합 중 공집합이 n((A-B)'(B-A))=k이면
아닌 집합의 개수와 같다. 집합 C는 (A-B)'(B-A)의 k개의 원소를 모두 포함하는 U의
8-k
따라서 2Û`-1=3(개) 부분집합이고, 집합 C의 개수는 2 이므로
8-k
Ý 최대인 원소가 4인 경우 2 =16=2Ý`에서 k=4
4는 포함하고, 5는 포함하지 않는 부분집합 중 원소의 개수가 2 이때, n(A;B)=2이고, 1<(A;B)이므로
이상인 집합의 개수는 집합 {1, 2, 3}의 부분집합 중 공집합이 a, b 중 하나는 2, 4, 6, 8 중 하나의 값을 갖고 다른 하나는 3, 5, 7
아닌 집합의 개수와 같다. 중 하나의 값을 가져야 하고, 선택된 값 중에서 작은 값이 a, 큰 값이
따라서 2Ü`-1=7(개) b이다. (∵ a<b)
Þ 최대인 원소가 5인 경우 따라서 순서쌍 (a, b)의 개수는 4_3=12이다.
5를 포함한 부분집합 중 원소의 개수가 2 이상인 집합의 개수는
집합 {1, 2, 3, 4}의 부분집합 중 공집합이 아닌 집합의 개수와
같다.
090 ④

따라서 2Ý`-1=15(개) 주어진 조건에 의하여


Ú~Þ에 의하여 각 집합의 최대인 원소를 모두 더한 값은 Ú ∆<X이면 S-∆=S<X
1_0+2_1+3_3+4_7+5_15=114 Û {a}<X이면 {b, c}<X이고 {a}'{b, c}=S<X이므로
참고 ∆<X
집합 U={1, 2, 3, 4, 5}의 부분집합 중 원소의 개수가 2 이상인 Ü {b}<X이면 {a, c}<X이고 S<X이므로 ∆<X
집합 X의 개수는 다음과 같이 구할 수 있다. Ý {c}<X이면 {a, b}<X이고 S<X이므로 ∆<X
집합 U의 모든 부분집합은 2Þ`개, Þ {a}<X, {b}<X이면 {a, b}<X이므로 {c}<X
집합 U의 원소의 개수가 1인 부분집합은 따라서 집합 S의 모든 부분집합이 X의 원소이다.
{1}, {2}, {3}, {4}, {5}로 5개, 마찬가지로 {b}<X, {c}<X일 때와 {c}<X, {a}<X일
집합 U의 원소의 개수가 0인 부분집합은 ∆으로 1개이므로 집합 때도 집합 S의 모든 부분집합이 X의 원소이다.
X의 개수는 2Þ`-5-1=26이다. Ú~Þ에 의하여 조건을 만족시키는 집합 X는
{∆, S}, {∆, {a}, {b, c}, S},
{∆, {b}, {a, c}, S}, {∆, {c}, {a, b}, S},
{∆, {a}, {b}, {c}, {a, b}, {b, c}, {a, c}, S}로 5개이다.
088 ①
참고

A-B=∆이므로 A,B이다. 집합 S의 임의의 부분집합 A에 대하여 A<X이면


Ú n(A)=1인 경우 조건 ㈎에 의하여 (S-A)<X이고
가능한 집합 A의 개수는 4이다. 이때, 각각 가능한 집합 B의 조건 ㈏에 의하여 A'(S-A)=S<X이다.
개수는 2Ü`-1=7이므로 순서쌍 (A, B)의 개수는 4_7=28 따라서 다시 조건 ㈎에 의하여 S-S=∆<X이다.
Û n(A)=2인 경우 즉, 집합 X는 두 집합 ∆, S를 항상 원소로 갖는다.
가능한 집합 A의 개수는 6이다. 이때, 각각 가능한 집합 B의
개수는 2Û`-1=3이므로 순서쌍 (A, B)의 개수는 6_3=18
Ü n(A)=3인 경우 091 ⑤
가능한 집합 A의 개수는 4이다. 이때, 각각 가능한 집합 B의
1 m n
개수는 1이므로 순서쌍 (A, B)의 개수는 4_1=4 =-i이므로 X={(-i) +i |m<A, n<B}이다.
i
Ú~Ü에 의하여 구하는 순서쌍 (A, B)의 개수는 ㄱ. A={2, 3}, B={1, 5}이면
28+18+4=50이다. 1
(-i)Û`+i =(-i)Û`+i Þ`=-1+i,
1
참고 (-i)Ü`+i =(-i)Ü`+i Þ`=2i
특정 원소를 포함하는 부분집합의 개수와 특정 원소를 포함하지 이므로 X={-1+i, 2i}이다. (참)
않는 부분집합의 개수는 같으므로 A-B=∆을 만족시키는 두 ㄴ. (-i)Ú`=(-i)Þ`=-i, (-i)Û`=-1, (-i)Ü`=i,
집합 A, B의 순서쌍 (A, B)의 개수와 A;B=∆을 (-i)Ý`=1이고, i Ú`=i Þ`=i, i Û`=-1, i Ü`=-i, i Ý`=1이다.
m n
만족시키는 두 집합 A, B의 순서쌍 (A, B)의 개수는 같다. (-i) +i 으로 가능한 모든 값을 원소로 가질 때
X={-2i, 2i, -2, 2, 0, -1+i, 1+i, -1-i, 1-i}
yy TIP
이므로 n(X)의 최댓값은 9이다. (참)
089 ③
ㄷ. 2<X이려면 (-i) +i =2에서
m n

A'C=B'C를 만족시키려면 m=n=4일 때이므로 두 집합 A와 B는 모두 4를 포함해야


(A-B),C, (B-A),C를 만족시켜야 한다. 한다.

18 수학 <하>
이를 만족시키는 두 집합 A, B의 개수는 각각 2Ý`이므로 순서쌍 Û x=3k-1일 때
(A, B)의 개수는 2Ý`_2Ý`=256이다. (참) xÛ`+x+1=(3k-1)Û`+(3k-1)+1
따라서 옳은 것은 ㄱ, ㄴ, ㄷ이다. =9kÛ`-3k+1=3(3kÛ`-k)+1
TIP 은 3의 배수가 아니다.
m n
(-i) +i 으로 가능한 모든 값은 다음과 같이 구할 수 있다. Ü x=3k일 때
xÛ`+x+1=(3k)Û`+3k+1
+ i -1 -i 1 ⅠV
=9kÛ`+3k+1=3(3kÛ`+k)+1
i 2i -1+i 0 1+i 집
은 3의 배수가 아니다. 합
-1 -1+i -2 -1-i 0 과
Ú~Ü에서 x=3k-2 (1ÉkÉ34, k는 자연수)일 때,
-i 0 -1-i -2i 1-i 명
xÛ`+x+1이 3의 배수이므로 제
1 1+i 0 1-i 2
Z={1Û`+1+1, 4Û`+4+1, 7Û`+7+1, 10Û`+10+1, y,
100Û`+100+1}
={3, 21, 57, 111, y, 10101}
092 39 이때, 집합 X의 원소는 100 이하의 자연수이므로
X;Z={3, 21, 57}
A2n=An;A2에서 n과 2의 최소공배수가 2n이므로 2와 n은
따라서 구하는 집합 X-Z의 원소의 개수는
서로소이다. 즉, n은 홀수이다.
n(X-Z)=n(X)-n(X;Z)=100-3=97이다.
한편, 90<(A2-An)이므로 90은 n의 배수가 아니다.
즉, n은 90의 양의 약수가 아니다.
90 이하의 홀수는 45개이고,
이 중 90의 약수는 1, 3, 5, 9, 15, 45로 6개이므로 095 ②
구하는 자연수 n의 개수는 45-6=39이다.
[kx]는 자연수이므로 [kx]=kx이기 위해선 kx의 값이 자연수이어야
한다.
093 45
즉, 자연수 a에 대하여 kx=a에서 x=
a
꼴이다.
k
100 이하의 자연수는 7k-6, 7k-5, 7k-4, 7k-3, 7k-2,
a
7k-1, 7k (k는 자연수) 꼴 중에 하나로 나타낼 수 있다. ∴ Ak=[ |a는 자연수]
k
이때, 각 꼴로 나타내어지는 100 이하의 자연수의 개수는 각각 1 2 3 k+1 2k+3
=[ , , , y, 1, , y, 2, , y]
15, 15, 14, 14, 14, 14, 14이다. k k k k k
집합 A의 서로 다른 두 원소의 합이 7의 배수가 아니려면 ㄱ. 모든 자연수는 두 집합 Am, An의 원소이므로
7k 꼴의 자연수를 2개 이상 원소로 포함할 수 없고, Am;An+∆ (거짓)
7k-6 꼴, 7k-1 꼴의 자연수를 동시에 원소로 포함할 수 없고, ㄴ. m이 n의 약수이면 어떤 자연수 h에 대하여 n=mh가
7k-5 꼴, 7k-2 꼴의 자연수를 동시에 원소로 포함할 수 없고, 성립하므로
7k-4 꼴, 7k-3 꼴의 자연수를 동시에 원소로 포함할 수 없다. 1 2 3 h 2h 3h
Am=[ , , , y]=[ , , , y],
m m m mh mh mh
따라서 n(A)가 최대이기 위해서는 집합 A가
1 2 3 1 2 3
7k 꼴의 자연수 중 1개를 원소로 포함하고, An=[ , , , y]=[ , , , y]
n n n mh mh mh
동시에 포함할 수 없는 두 꼴 중에서 원소의 개수가 더 많은 꼴, 따라서 Am,An이므로 집합 Am은 집합 An의
즉 7k-6 꼴, 7k-5 꼴의 자연수 각각 15개를 모두 원소로 부분집합이다. (참)
포함하고, 7k-4 꼴 또는 7k-3 꼴의 자연수 중에 하나의 1 3 1 2 4 5
ㄷ. A2=[ , 1, , 2, y], A3=[ , , 1, , , 2, y],
꼴(14개)을 모두 원소로 포함해야 한다. 2 2 3 3 3 3
따라서 n(A)의 최댓값은 1+15+15+14=45이다. 1 1 1 2 5
A6=[ , , , , , 1, y]
6 3 2 3 6
1 1
이때, ²A2, ²A3이므로
094 97 6 6
A2'A3+A6이다. (거짓) yy 참고
집합 Z의 원소는 집합 Y의 원소 중 3의 배수인 수이다.
따라서 옳은 것은 ㄴ이다.
100 이하의 자연수 x는 3k-2, 3k-1, 3k (k는 자연수) 꼴 중에
참고
하나로 나타낼 수 있다.
m과 n의 최소공배수가 l이면 ㄴ에 의하여
Ú x=3k-2일 때
Am,Al, An,Al이므로 (Am'An),Al이 항상 성립한다.
xÛ`+x+1=(3k-2)Û`+(3k-2)+1
=9kÛ`-9k+3=3(3kÛ`-3k+1)
은 3의 배수이다.

Ⅳ. 집합과 명제 19
명제 099 15

명제 p` Ú q가 거짓임을 보이려면
집합 P의 원소 중에서 집합 Q의 원소가 아닌 것을 찾으면 된다.
096 ③ U
P Q
명제는 참, 거짓을 명확히 판별할 수 있는 문장이나 식이고 조건은 1 3 4
2
변수의 값에 따라 참, 거짓을 결정할 수 있는 문장이나 식이다. 5
8 6 7
ㄱ. ‘1+2<5’는 참이므로 명제이다. 9
ㄴ. ‘3x-2=1’은 x의 값에 따라 참, 거짓을 결정할 수 있으므로 C
즉, P;Q =P-Q={1, 5, 9}이므로
조건이다. 따라서 명제가 아니다.
구하는 모든 원소의 합은 1+5+9=15이다.
ㄷ. ‘x=1이면 2x+1=4이다.’는 거짓이므로 명제이다.
ㄹ. ‘한국에서 가장 아름다운 곳은 제주도이다.’에서 아름답다는 그
기준이 명확하지 않으므로 참, 거짓을 결정할 수 없다. 따라서 100 ④

명제가 아니다. 명제 p` Ú ~`q가 참이면 P,Q 이다.


C

ㅁ. ‘삼각형의 세 내각의 크기의 합은 360ù이다.’는 거짓이므로 U


명제이다. QC

따라서 명제인 것은 ㄱ, ㄷ, ㅁ으로 3개이다. P

097 풀이 참조
① P'Q+P, ② P-Q=P, ③ Q-P=Q, ④ P;Q=∆,
C C
⑴ P={1, 2, 3, 6}, Q={1, 2, 3, 4, 6, 12} ⑤ P'Q =Q
⑵ ⑴에서 P,Q가 성립하므로 명제 ‘p` Ú q’는 참이다. 따라서 항상 옳은 것은 ④이다.
즉, 명제 ‘6의 약수는 12의 약수이다.’는 참이다.

101 ③
098 ③
p`:`-1<xÉ3, q`:`x<a에서
① (반례) 3은 3의 배수이지만 6의 배수가 아니다. (거짓) 두 조건 p, q의 진리집합을 각각 P, Q라 하면
② (반례) x=-1일 때 xÛ`=1이지만 x+1이다. (거짓) 명제 ‘p이면 q이다.’가 참이기 위해선 P,Q이어야 한다.
③ 조건 x-2>0에서 x>2, 조건 2x+1>3에서 x>1이므로 Q
P
x-2>0이면 2x+1>3이다. (참)
-1 3 a x
④ (반례) x=1, y=1일 때 xy=1로 홀수이지만 x+y=2로
짝수이다. (거짓) yy TIP 1 따라서 a>3이어야 하므로
⑤ (반례) 한 쌍의 대변만 평행한 사다리꼴은 평행사변형이 아니다. 구하는 정수 a의 최솟값은 4이다.
(거짓) yy TIP 2
따라서 참인 명제는 ③이다. 102 ④
TIP 1
두 조건 p`:`1<xÉ6, q`:`2Éx<5의 진리집합을 각각 P, Q라
<홀수, 짝수의 연산>
하자.
(홀수)+(홀수)=(짝수) (홀수)_(홀수)=(홀수) 조건 ‘~`p 또는 q’의 부정은 ‘p 그리고 ~`q’이고 그 진리집합은
C
(홀수)+(짝수)=(홀수) (홀수)_(짝수)=(짝수) P;Q 이다.
C
(짝수)+(짝수)=(짝수) (짝수)_(짝수)=(짝수) 이때, ~`q`:`x<2 또는 x¾5이므로 P;Q 을 수직선 위에 나타내면
다음과 같다.
따라서 xy가 홀수이면 x, y가 모두 홀수이므로 x+y는 짝수이다.
C C
Q Q
TIP 2 P
1 2 5 6 x
<사각형의 포함 관계>
사각형 따라서 조건 ‘~`p 또는 q’의 부정은 ‘1<x<2 또는 5ÉxÉ6’이다.
사다리꼴
평행사변형

마름모 정사 직사각형 103 ④


각형
주어진 벤다이어그램에서
A={1, 2, 3, 5}, B={2, 4}이다.

20 수학 <하>
① 2<A이므로 집합 A의 어떤 원소는 짝수이다. (참)
② 1<A이므로 집합 A의 어떤 원소는 홀수이다. (참)
106 ②

③ B={2, 4}이므로 집합 B의 모든 원소는 짝수이다. (참) 주어진 명제 ‘우리 반의 모든 학생의 수학 점수는 80점 미만이다.’의
④ B={2, 4}이므로 집합 B에 홀수인 원소는 존재하지 않는다. 부정은 ‘우리 반의 어떤 학생의 수학 점수는 80점 이상이다.’이므로
(거짓) 이와 같은 것은 선지 중 ‘② 우리 반에 수학 점수가 80점 이상인
⑤ 2<B이므로 집합 B의 어떤 원소는 짝수이다. (참) 학생이 있다.’이다.
ⅠV
따라서 선지 중 거짓인 것은 ④이다. 참고

TIP ‘①’과 ‘③’, ‘주어진 명제’와 ‘⑤’는 각각 서로 동일한 명제이고, 합

❶ ‘모든 x에 대하여 p’ ‘④’의 부정은 ‘①’과 ‘③’이다.

이는 ‘전체집합의 모든 원소가 조건 p를 만족시킨다.’는 제
것과 같다. 따라서 ‘모든 x에 대하여 p’가 거짓임을 보이기
위하여 조건 p를 거짓이 되게 하는 적어도 하나의 원소 x가 107 ③
전체집합에 속한다는 것을 보이면 된다.
주어진 명제의 부정 ‘어떤 실수 x에 대하여 xÛ`+8x+kÉ0이다.’가
❷ ‘어떤 x에 대하여 p’
참이 되기 위해선
이는 ‘p인 x가 존재한다.’는 것과 같다. 따라서 ‘어떤 x에
대하여 p’가 거짓임을 보이기 위해 전체집합의 모든 원소가
y=x@+8x+k
조건 p를 거짓이 되게 한다는 것을 보이면 된다.
x

이차방정식 xÛ`+8x+k=0의 판별식을 D라 할 때


104 ①
D
=16-k¾0이어야 한다.
4
ㄱ. ‘모든 x에 대하여 p(x)이다.’가 참이면 P=U이다. (참)
즉, kÉ16이므로 k의 최댓값은 16이다.
ㄴ. ‘어떤 x에 대하여 p(x)이다.’가 거짓이면 P=∆이다. (거짓)
ㄷ. ‘모든 x에 대하여 ~`p(x)이다.’의 부정은 ‘어떤 x에 대하여
p(x)이다.’이고, 이 명제가 참이면 P+∆이다. (거짓) 108 풀이 참조
따라서 옳은 것은 ㄱ이다.
⑴ 역 : 이등변삼각형은 정삼각형이다. (거짓)
대우 : 이등변삼각형이 아니면 정삼각형이 아니다. (참)
105 ③ ⑵ 역 : 두 실수 a, b에 대하여 a=0이고 b=0이면 a+b=0이다.
(참)
ㄱ. (반례) x=0일 때, xÛ`=0이다. (거짓)
대우 : 두 실수 a, b에 대하여 a+0 또는 b+0이면 a+b+0이다.
ㄴ. xÛ`-4x+4=(x-2)Û`É0에서 x=2이다.
(거짓)
즉, x=2일 때, xÛ`-4x+4É0이다. (참)
(반례) a=-1, b=1이면 a+b=(-1)+1=0이다.
ㄷ. 실수는 x>0 또는 x=0 또는 x<0이다.
⑶ 역 : 두 실수 a, b에 대하여 a+0이고 b+0이면 ab+0이다. (참)
x>0일 때, |x|Û`=xÛ`
대우 : 두 실수 a, b에 대하여 a=0 또는 b=0이면 ab=0이다.
x=0일 때, |0|Û`=0Û`
(참)
x<0일 때, |x|Û`=(-x)Û`=xÛ`
TIP
이므로 모든 실수 x에 대하여 |x|Û`=xÛ`이다. (참)
1 3 ❶ 명제와 그 역의 참, 거짓은 항상 일치하지 않는다.
ㄹ. xÛ`+x+1={x+ }Û`+ >0에서
2 4 ❷ 명제와 그 대우의 참, 거짓은 항상 일치한다.
모든 실수 x에 대하여 xÛ`+x+1>0이므로 따라서 주어진 명제와 그 대우 중 참, 거짓 판별이 더 간단한
xÛ`+x+1=0을 만족시키는 실수 x는 존재하지 않는다. (거짓) 것의 참, 거짓을 판별하자.
ㅁ. xÛ`+2x+3=(x+1)Û`+2>0이므로
모든 실수 x에 대하여 xÛ`+2x+3>0이다. (참)
따라서 참인 명제는 ㄴ, ㄷ, ㅁ으로 3개이다.
참고
109 ③

주어진 명제의 역과 참, 거짓은 다음과 같다.


<보기>의 ㄱ~ㅁ에서 ‘모든 실수 x에 대하여’, ‘어떤 실수 x에
① 역 : xÛ`=yÛ`이면 x=y이다. (거짓)
대하여’가 없다면 명제가 아닌 조건이다.
(반례) x=-1, y=1일 때, xÛ`=yÛ`이지만 x+y이다.
일반적으로 조건 p는 명제가 아니지만 전체집합에 대하여 조건 p
② 역 : xÛ`=9이면 xÛ`-6x+9=0이다. (거짓)
앞에 ‘모든’이나 ‘어떤’이 있으면 참, 거짓을 판별할 수 있는
(반례) x=-3일 때, (-3)Û`=9이지만
명제가 된다.
(-3)Û`-6_(-3)+9+0이다.

Ⅳ. 집합과 명제 21
③ 역 : xÛ`É1이면 xÉ1이다. (참) Q라 하자.
(증명) xÛ`É1이면 xÛ`-1=(x+1)(x-1)É0에서 ⑴ p`:`x>0, q`:`|x|=x HjK x¾0
-1ÉxÉ1이므로 xÉ1이다. 이므로 P,Q
④ 역 : x+y>0이면 x>0, y>0이다. (거짓) 따라서 p는 q이기 위한 충분 조건이다.
(반례) x=2, y=-1일 때, x+y=2+(-1)>0이지만 ⑵ p`:`|x+3|=2, q`:`x=-1
y<0이다. 이때, |x+3|=2에서 x+3=2 또는 x+3=-2
⑤ 역 : -2ÉxÉ4이면 -1Éx<2이다. (거짓) 즉, x=-1 또는 x=-5이므로 Q,P
(반례) x=-2
따라서 p는 q이기 위한 필요 조건이다.
따라서 역이 참인 명제는 ③이다.
⑶ p`:`x+yi=0, q`:`x=0이고 y=0
에서 P=Q
110 ② 따라서 p는 q이기 위한 필요충분 조건이다.
⑷ p`:`|x|=|y| HjK x=y 또는 x=-y,
명제 ‘~`p` Ú q’의 역인 ‘q` Ú ~`p’가 참이므로
q`:`xÛ`=yÛ` HjK x=y 또는 x=-y
그 대우인 ‘p` Ú ~`q’도 참이다.
이므로 P=Q
따라서 반드시 참인 명제는 ②이다.
따라서 p는 q이기 위한 필요충분 조건이다.
⑸ p`:`xÛ`>1 HjK x<-1 또는 x>1,
111 ④
q`:`x>1
두 명제 p` Ú ~`q, ~`p` Ú r가 참이므로 에서 Q,P이므로 p는 q이기 위한 필요 조건이다.
그 대우인 q` Ú ~`p, ~`r` Ú p도 각각 참이다. ⑹ p`:`∠A=∠B,
즉, 두 명제 q` Ú ~`p와 ~`p` Ú r가 참이므로 q`:`삼각형 ABC가 이등변삼각형
q` Ú r와 그 대우인 ~`r` Ú ~`q도 참이다. HjK ∠A=∠B 또는 ∠B=∠C 또는 ∠C=∠A
ㄱ. 명제 p` Ú q는 항상 참인지 알 수 없다. 에서 P,Q이므로 p는 q이기 위한 충분 조건이다.
ㄴ. 명제 q` Ú ~`p는 항상 참이다.
ㄷ. 명제 p` Ú ~`r는 항상 참인지 알 수 없다.
ㄹ. 명제 ~`r` Ú ~`q는 항상 참이다.
따라서 항상 참인 것은 ㄴ, ㄹ이다.
113 ④

두 조건 p, q의 진리집합을 각각 P, Q라 하자.
TIP
이때, p는 q이기 위한 충분조건이지만 필요조건이 아니기 위해선
<삼단논법 : ‘p` Ú q, q` Ú r가 참이면 p` Ú r가 참이다.’>
P,Q이지만 P+Q이어야 한다.
조건 p, q, r의 진리집합을 각각 P, Q, R라 할 때,
① p`:`xz=yz HjK x=y 또는 z=0이므로 Q,P
명제 p` Ú q가 참이면 P,Q,
즉, p는 q이기 위한 필요조건이다.
명제 q` Ú r가 참이면 Q,R
② P={1, 2, 3, 4, 6, 12}, Q={1, 2, 3, 6}이므로 Q,P
즉, P,Q,R에서 P,R이다.
즉, p는 q이기 위한 필요조건이다.
따라서 명제 p` Ú r가 참이다.
③ q`:`xÛ`=x HjK x=0 또는 x=1이므로 P=Q
이와 같은 방법으로 주어진 참인 명제에서 새로운 참인 명제를
즉, p는 q이기 위한 필요충분조건이다.
이끌어 낼 수 있다.
④ x와 y가 모두 유리수이면 xy는 유리수이다.
참고 하지만 xy가 유리수일 때, x와 y가 모두 유리수가 아닐 수도
있다.
전체집합 U에 대하여 문제에서 주어진 세 조건 p, q, r의
(반례) x='2, y='2일 때, xy='2_'2=2
진리집합을 각각 P, Q, R라 하면 그 포함 관계는 다음과 같다.
U
즉, P,Q, P+Q이므로 p는 q이기 위한 충분조건이지만
QC 필요조건이 아니다.
P
RC ⑤ p`:`|x|É3 HjK -3ÉxÉ3이므로 Q,P
즉, p는 q이기 위한 필요조건이다.
따라서 선지 중 구하는 것은 ④이다.
이를 이용하여 주어진 명제들의 참, 거짓을 파악할 수 있다.

114 ④

112 풀이 참조
ㄱ. P+Q이므로 p는 q이기 위한 필요충분조건이 아니다. (거짓)
C
다음과 같이 주어진 조건을 p, q라 하고, 그 진리집합을 각각 P, ㄴ. R,P 이므로 r는 ~`p이기 위한 충분조건이다. (참)

22 수학 <하>
C C
ㄷ. R,Q에서 Q ,R 이므로 ~ r는 ~`q이기 위한 필요조건이다.
(참)
118 ③

따라서 옳은 것은 ㄴ, ㄷ이다. 주어진 명제를 대우 를 이용해서 증명하자.


x, y가 모두 홀수이면
x=2a+1, y=2b+1 (a, b는 음이 아닌 정수)
115 ④ 로 나타낼 수 있으므로 ⅠV
C xy=(2a+1)(2b+1) 집
p는 ~`q이기 위한 충분조건이므로 P,Q
C C =2(2ab+a+b)+1 합
q는 ~`r이기 위한 필요조건이므로 R ,Q에서 Q ,R 과
C 이때, 2ab+a+b가 0 또는 자연수 이므로
즉, P,Q ,R가 성립한다. 명

이를 벤다이어그램으로 나타내면 다음과 같다. xy는 홀수 이다. yy TIP
U 따라서 주어진 명제의 대우 가 참이므로 주어진 명제도 참이다.
R
QC ㈎ : 대우, ㈏ : 0 또는 자연수, ㈐ : 홀수
P
TIP
Ú a=b=0일 때, 2ab+a+b=0이므로
① P,R (참) 2(2ab+a+b)+1=0+1=1은 홀수이다.
② P;Q=∆ (참) Û a=0, b+0 또는 a+0, b=0 또는 a+0, b+0일 때,
C
③ P;Q =P (참) 2ab+a+b는 자연수이므로
C C
④ R-Q=R;Q =Q (거짓) 2(2ab+a+b)는 짝수이고, 2(2ab+a+b)+1은 홀수이다.
⑤ Q'R=U (참)
따라서 선지 중 옳지 않은 것은 ④이다.

119 풀이 참조

116 ① ⑴ 주어진 명제의 대우는


‘자연수 n에 대하여 n이 홀수이면 nÛ`은 홀수이다.’이다.
ㄱ. A=B이면 A;C=B;C이다. (참) ⑵ 주어진 명제의 대우인
ㄴ. A={1}, B={2}, C={1, 2}일 때, ‘자연수 n에 대하여 n이 홀수이면 nÛ`은 홀수이다.’를 증명해보자.
A'C=B'C이지만 A+B이므로 n이 홀수일 때,
A=B는 A'C=B'C이기 위한 필요조건이 아니다. (거짓) n=2k-1(k는 자연수)라 하면
ㄷ. A=B이면 A-C=B-C이므로 nÛ`=(2k-1)Û`=4kÛ`-4k+1=2(2kÛ`-2k)+1
A=B는 A-C=B-C이기 위한 충분조건이지만 에서 2kÛ`-2k는 0 또는 자연수이므로
A={1}, B={2}, C={1, 2}일 때, 2(2kÛ`-2k)+1은 홀수이다. 즉, nÛ`은 홀수이다.
A-C=B-C이지만 A+B이므로 A=B는 따라서 주어진 명제의 대우가 참이므로 주어진 명제도 참이다.
A-C=B-C이기 위한 필요조건이 아니다. (거짓) 참고
따라서 옳은 것은 ㄱ이다.
⑵에서 n=2k+1 (k는 음이 아닌 정수)로 놓으면 다음과 같이
참고
증명할 수 있다.
ㄱ에서 A={1, 2}, B={1, 3}, C={1, 4}일 때, nÛ`=(2k+1)Û`=4kÛ`+4k+1=2(2kÛ`+2k)+1
A;C=B;C={1}이지만 A+B이므로 에서 2kÛ`+2k는 0 또는 자연수이므로
A=B는 A;C=B;C이기 위한 필요조건이 아닌 2(2kÛ`+2k)+1은 홀수이다. 즉, nÛ`은 홀수이다.
충분조건이다.
ㄱ, ㄴ, ㄷ에서 A=B는 모두 필요조건이 아닌 충분조건이다. 채점 요소 배점
주어진 명제의 대우를 구하기 20%
대우를 이용하여 주어진 명제를 증명하기 80%

117 ④

명제와 그 대우는 참, 거짓이 일치한다.


따라서 주어진 명제 대신 그 대우인
120 ④

④ ‘두 자연수 m, n에 대하여 m, n이 모두 짝수이면 m+n이 주어진 명제의 대우는


짝수이다.’ ‘세 자연수 a, b, c에 대하여 a, b, c가 모두 홀수 이면
를 증명하면 된다. aÛ`+bÛ`+cÛ`이다.’

Ⅳ. 집합과 명제 23
이때, 세 자연수 a, b, c가 모두 홀수 이면 a+b a+b-2'¶ab
-'¶ab =
2 2
aÛ`, bÛ`, cÛ`도 모두 홀수이다.
'a -2'a 'b+'b Û`
`
Û
aÛ`+bÛ`은 짝수 , cÛ`은 홀수 이므로 aÛ`+bÛ`+cÛ`이다. =
2
따라서 주어진 명제의 대우가 참이므로 주어진 명제도 참이다. ('a -'b )Û`
= ¾0 yy㉠
㈎ : 모두 홀수, ㈏ : 짝수, ㈐ : 홀수 2
참고 a+b
이므로 -'¶ab ¾0
2
자연수 a, b, c가 모두 홀수이면
a+b
a=2x-1, b=2y-1, c=2z-1 (x, y, z는 자연수) 따라서 ¾ '¶ab 이다.
2
(2x-1)Û`+(2y-1)Û`=(2z-1)Û`에서 (단, 등호는 ㉠에서 'a='b , 즉 a=b 일 때 성립한다.)
좌변은 (2x-1)Û`+(2y-1)Û`=2(2xÛ`+2yÛ`-2x-2y+1)이므로
㈎ : ¾, ㈏ : ('a -'b )Û`, ㈐ : a=b
짝수이고,
우변은 (2z-1)Û`=2(2zÛ`-2z)+1이므로 홀수이다.
따라서 aÛ`+bÛ`=cÛ`을 만족시키는 124 ⑴ 4 ⑵ 12
자연수 a, b, c가 존재하지 않는다.
⑴ x>0, y>0이므로 산술평균과 기하평균의 관계에 의하여
2'¶xyÉx+y=4, '¶xyÉ2, xyÉ4
(단, 등호는 x=y일 때 성립한다.)
121 ② 따라서 xy의 최댓값은 4이다.
⑵ x>0, y>0이므로 산술평균과 기하평균의 관계에 의하여
a+b'2=k (k는 유리수 )라 하면 3x+4y¾2'Ä3x_4y=2'3Œ6=12 (∵ xy=3)
k-a (단, 등호는 3x=4y일 때 성립한다.)
b'2=k-a, '2=
b
따라서 3x+4y의 최솟값은 12이다.
k-a
이때, 는 유리수 이므로 yy TIP
b
'2가 무리수 라는 사실에 모순이다.
125 ③
따라서 a+b'2 는 무리수이다.
x>0, y>0이므로 xy>0이다.
㈎ : 유리수, ㈏ : 유리수, ㈐ : 무리수
산술평균과 기하평균의 관계에 의하여
TIP
3 3 9
두 유리수 x, y에 대하여 {2x+ }{2y+ }=4xy+6+6+
y x xy
y 9
x+y, x-y, xy, (x+0)도 유리수이다. =4xy+ +12
x xy
k-a 9
따라서 a, b, k가 유리수이므로 도 유리수이다. ¾2¾Ð4xy_ +12=24
b xy
9 3
(단, 등호는 4xy=
xy
, 즉 xy= 일 때 성립한다.)
2 yy TIP
3 3
따라서 {2x+ }{2y+ }의 최솟값은 24이다.
122 ④ y x
TIP
ㄱ. (반례) a=0, b=0일 때, |a|+|b|=0이므로 |a|+|b|>0은
9 3
절대부등식이 아니다. 4xy= 에서 (2xy)Û`=3Û`이므로 xy= 이다. (∵ xy>0)
xy 2
1 3
ㄴ. 모든 실수 a에 대하여 aÛ`-a+1={a- }Û`+ >0이 항상
2 4
성립하므로 절대부등식이다.
1 3
ㄷ. 모든 실수 a, b에 대하여 aÛ`-ab+bÛ`={a- b}Û`+ bÛ`¾0
126 ⑤
2 4
a>0, b>0이므로 산술평균과 기하평균의 관계에 의하여
(단, 등호는 a=b=0일 때 성립한다.)
1 4 4a b
즉, aÛ`+bÛ`¾ab가 항상 성립하므로 절대부등식이다. (a+b){ + }=1+ + +4
a b b a
따라서 절대부등식은 ㄴ, ㄷ이다. 4a b
¾5+2¾Ð _ =9
b a
4a b
(단, 등호는
b
= , 즉 2a=b일 때 성립한다.)
a yy TIP
123 ④
1 4
따라서 (a+b){ + }의 최솟값은 9이다.
a>0, b>0이므로 a=('a )Û`, b=('b )Û`, '¶ab='a 'b이다. a b

24 수학 <하>
TIP 따라서 a+2b의 최댓값은 M=5, 최솟값은 m=-5이므로
4a b M-m=5-(-5)=10
= 에서 4aÛ`=bÛ`이므로 b=2a 또는 b=-2a이다.
b a TIP
이때, a>0, b>0이므로 2a=b이다. 양수 k에 대하여 x에 대한 부등식 xÛ`ÉkÛ`의 해는
xÛ`-kÛ`=(x+k)(x-k)É0에서 -kÉxÉk이다.
따라서 (a+2b)Û`É5Û`에서 -5Éa+2bÉ5이다. ⅠV

127 ② 합

주어진 삼각형은 빗변의 길이가 6인 직각삼각형이므로 명
밑변의 길이와 높이를 x, y (x>0, y>0)라 하면 제

130 20
x
y 연못의 가로의 길이와 세로의 길이를 각각
a, b (a>0, b>0)라 하면
6
지름의 길이가 5'2이므로 aÛ`+bÛ`=50,
피타고라스 정리에 의하여 xÛ`+yÛ`=36 yy㉠ 연못의 둘레의 길이는 2a+2b이다.
1 코시-슈바르츠의 부등식에 의하여
이고, 직각삼각형의 넓이는 xy이다.
2
(2a+2b)Û`É(2Û`+2Û`)(aÛ`+bÛ`)=8_50=400
산술평균과 기하평균의 관계에 의하여
(단, 등호는 a=b일 때 성립한다.)
xÛ`+yÛ`¾2"ÃxÛ`yÛ`=2xy (∵ x>0, y>0)
0<2a+2bÉ20 yy TIP
(단, 등호는 xÛ`=yÛ`, 즉 x=y일 때 성립한다.)
따라서 구하는 최댓값은 20이다.
1
즉, 36¾2xy이므로 양변을 4로 나누면 9¾ xy TIP
2
따라서 이 삼각형의 넓이의 최댓값은 9이다. (2a+2b)Û`É20Û`에서 -20É2a+2bÉ20이다.
참고 이때, a>0, b>0이므로 0<2a+2bÉ20이다.
등호는 x=y일 때 성립하므로 ㉠에 대입하면
2xÛ`=36, x=3'2
따라서 x=y=3'2일 때, 이 삼각형의 넓이는 최댓값 9를
갖는다.
131 ③

조건 ‘집합 X의 원소 중 10보다 작은 수는 많아야 4개이다.’의


의미는 집합 X의 원소 중 10보다 작은 수가 0개~4개일 수
128 ⑤
있다는 것이므로 그 부정은 집합 X의 원소 중 10보다 작은 수가 5개
(aÛ`+bÛ`)(xÛ`+yÛ`)-(ax+by)Û` 이상임을 의미한다.
=(aÛ`xÛ`+aÛ`yÛ`+bÛ`xÛ`+bÛ`yÛ`)-(aÛ`xÛ`+2abxy+bÛ`yÛ`) 즉, 주어진 조건의 부정은
=aÛ`yÛ`-2abxy+bÛ`xÛ` ‘③ 집합 X의 원소 중 적어도 5개는 10보다 작다.’이다.
= (ay-bx)Û`
이때, a, b, x, y는 실수이므로
(ay-bx)Û`¾0 yy㉠
따라서 (aÛ`+bÛ`)(xÛ`+yÛ`) ¾ (ax+by)Û`이다. 132 ②

(단, 등호는 ㉠에서 ay-bx=0, 즉 ay=bx 일 때 성립한다.)


1 1 yy TIP
ㄱ. x<y<0이면 > 이다. (참)
㈎ : ¾, ㈏ : (ay-bx)Û`, ㈐ : ay=bx x y
ㄴ. (반례) x=-2, y=1일 때, (-2)Û`>1Û`이지만 -2<1이다.
(거짓)
ㄷ. xÛ`+yÛ`=0이면 x=0이고 y=0이므로 |x|+|y|=0이다. (참)
129 ③
ㄹ. 주어진 명제의 대우는 ‘xÛ`-5x+6=0이면 x=3이다.’이다.
aÛ`+bÛ`=5이므로 코시-슈바르츠의 부등식에 의하여 이때, xÛ`-5x+6=(x-2)(x-3)=0에서
(1Û`+2Û`)(aÛ`+bÛ`)¾(a+2b)Û` x=2 또는 x=3이므로 x=2가 반례이다. (거짓)
5_5¾(a+2b)Û` (단, 등호는 2a=b일 때 성립한다.) ㅁ. (반례) x=0, y=2일 때, x+y¾2이지만 x<1이다. (거짓)
-5Éa+2bÉ5 yy TIP 따라서 참인 명제는 ㄱ, ㄷ이다.

Ⅳ. 집합과 명제 25
TIP P={x|xÉ2 또는 x¾5}, Q={x|-3ÉxÉk}이다.
x, y가 실수일 때 명제 ~`q` Ú p의 반례가 될 수 있는 값은
C
1 1 Q 의 원소이면서 집합 P의 원소가 아니어야 하므로
❶ 0<x<y이면 > 이다. C C C
x y Q -P=Q ;P 의 원소이다.
C C
1 1 집합 Q ={x|x<-3 또는 x>k}이고, P ={x|2<x<5}이므로
❷ x<y<0이면 > 이다.
x y
QC QC
1 1
❸ x<0<y이면 < 이다. PC
x y
-3 2 k 5 x
C C
집합 Q ;P 에 정수인 원소가 x=4뿐이려면 3Ék<4이어야 한다.

133 ㄴ, ㄷ, ㅁ

ㄱ. n(A)=0이면 집합 A의 원소가 존재하지 않으므로 A=∆이다. 137 ⑤


(참) C C
P;(Q 'R)=(P;Q )'(P;R)
ㄴ. (반례) A={1}, B={2}일 때,
=(P-Q)'(P;R)=∆
n(A)=n(B)이지만 A+B이다. (거짓)
이므로 P-Q=∆, P;R=∆이어야 한다.
ㄷ. A,B인 경우는 다음과 같다.
즉, P,Q이고 두 집합 P와 R는 서로소이다.
Ú A,B이지만 A+B인 경우 n(A)<n(B)이다.
이를 벤다이어그램으로 나타내면 다음과 같다.
Û A=B인 경우 n(A)=n(B)이다.
U
Ú, Û에 의하여 n(A)Én(B)이다. (거짓) Q R
ㄹ. A,B이면 ㄷ에서 n(A)Én(B)이므로 P
n(A)Én(B)이고 n(A)¾n(B)이면 n(A)=n(B)이다.
ㄷ의 Û에 의하여 A=B이다. (참)
ㅁ. n(A-B)=0이면 A-B=∆에서 A,B이므로 ㄱ. 주어진 정보로 두 집합 Q와 R의 포함 관계는 알 수 없다. (거짓)
n(A)Én(B)이다. (거짓) ㄴ. 두 집합 P와 R는 서로소이므로 R,P 이다.
C

따라서 거짓인 명제는 ㄴ, ㄷ, ㅁ이다. 즉, 명제 r` Ú ~`p는 참이다. (참)


C
ㄷ. P;R =P,Q이므로 명제 (p이고 ~`r)` Ú q는 참이다. (참)
따라서 참인 것은 ㄴ, ㄷ이다.
134 ②
다른 풀이
P={2, 3, 5, 7}이고 명제 ~`p` Ú q가 참이 되려면
C
C C
P ,Q이므로 P ={1, 4, 6, 8, 9, 10}에서 P;(Q 'R)=∆이므로
C
집합 Q는 6개의 원소를 반드시 포함해야 하므로 두 집합 P와 Q 'R는 서로소이다.
가능한 집합 Q의 개수는 2Ý`=16이다. 이를 벤다이어그램으로 나타내면 다음과 같다.
U

P QC R
135 ④

두 조건 p, q의 진리집합을 각각 P, Q라 하면
C
명제 q` Ú ~`p가 참이 되기 위해선 Q,P 이어야 한다. 이후 풀이는 본풀이와 같다.
이때 p`:`a+1<xÉ2a에서 ~`p`:`xÉa+1 또는 x>2a이다. 참고
C
PC PC P;(Q 'R)=∆이 성립할 때, 참인 명제는
C C
Q Q P,Q에서 p` Ú q, Q ,P 에서 ~`q` Ú ~`p ,
0 a+1 2a 6 x C
P,R 에서 p` Ú ~`r, R,P 에서 r` Ú ~`p이다.
C

0Éa+1에서 a¾-1이고
2aÉ6에서 aÉ3이어야 하므로
조건을 만족시키는 a의 값의 범위는 -1ÉaÉ3이다.
따라서 구하는 정수 a는 -1, 0, 1, 2, 3이므로 138 ③

그 합은 (-1)+0+1+2+3=5이다. 명제 p` Ú q가 참이므로 P,Q


C
명제 ~`p` Ú q가 참이므로 P ,Q
C
명제 ~`r` Ú p가 참이므로 R ,P이다.
136 ④ C
이때, P,Q, P ,Q에서 (P'P )=U,Q
C
∴ Q=U
두 조건 p, q의 진리집합을 각각 P, Q라 하면 이를 벤다이어그램으로 나타내면 다음과 같다.

26 수학 <하>
Q=U 영훈이와 민지의 말은 거짓이므로 영훈이와 인수는 농구장에
P 가지 않았고, 민지는 극장에 갔다.
C
R
이때, 농구장에 간 사람이 아무도 없어 모순이다.
따라서 인수의 말은 거짓이다.
Ü 민지의 말이 참인 경우
C
ㄱ. P ,Q (참)
C
독서실 농구장 극장 ⅠV
ㄴ. R-P =R;P+∆ (거짓)
C C
영훈 × × ○ 집
ㄷ. (R 'P ),U=Q (참) 합
인수 × ○ ×
따라서 옳은 것은 ㄱ, ㄷ이다. 과
민지 ○ × × 명

영훈이와 인수의 말은 거짓이므로 영훈이는 농구장에 가지
139 ②
않았고, 인수는 농구장에 갔고, 민지는 극장에 가지 않았다.
명제 ‘~`r이면 ~`p이고 ~`q이다.’가 거짓임을 보이는 원소는 집합 이때, 민지는 독서실, 영훈이는 극장에 간 것이 되며,
C C C
R 에 속하면서 P ;Q 에 속하지 않아야 하므로 모순이 없으므로 민지의 말이 참이다.
C C C C C
R ;(P ;Q ) =R ;(P'Q) Ú~Ü에 의하여 모순이 없는 민지의 말이 참이어야 하므로 독서실,
C
=(P'Q);R 농구장, 극장에 간 사람을 차례대로 나타내면 민지, 인수, 영훈이다.
=(P'Q)-R
따라서 주어진 명제가 거짓임을 보이는 원소는
집합 ② (P'Q)-R의 원소이다. 141 ④

다른 풀이 주어진 명제의 부정은


‘x<U, y<U인 모든 x, y에 대하여 |x-y|É2이다.’이고
명제 ‘~`r이면 ~`p이고 ~`q이다.’의 대우는
‘p 또는 q이면 r이다.’이므로 이 명제가 거짓임을 보이는 원소는 |x-y| -1 0 1
P'Q에 속하면서 R에 속하지 않아야 한다. -1 0 1 2
따라서 주어진 명제가 거짓임을 보이는 원소는 0 1 0 1
C
집합 (P'Q);R =(P'Q)-R의 원소이다. 1 2 1 0
이므로 주어진 명제의 부정은 참이다.
140 ④

문제의 상황을 표로 나타내면 다음과 같다. 142 1ÉtÉ3

독서실 농구장 극장 명제 ‘p`:`x>1인 모든 실수 x에 대하여 x>2-t이다.’가


영훈 ○ 참이기 위해선 2-tÉ1, t¾1이어야 하고
인수 × 명제 ‘q`:`xÉ0인 어떤 실수 x에 대하여 x¾t-3이다.’가
민지 × 참이기 위해선 t-3É0, tÉ3이어야 한다.
따라서 구하는 실수 t의 값의 범위는 1ÉtÉ3이다.
주어진 상황에 모순이 없으려면 각 행(가로)과 열(세로)마다 ○가
1개, ×가 2개씩 표시되어야 한다.
Ú 영훈이의 말이 참인 경우 143 ③

독서실 농구장 극장 p`:`nÉxÉn+3, q`:`-2<xÉ3이라 하고


영훈 ○ 두 조건 p, q의 진리집합을 각각 P, Q라 하면
인수 ○ 주어진 명제가 참이기 위해선 P;Q+∆이어야 한다.
민지 ○ Q
P P
인수와 민지의 말은 거짓이므로 영훈이와 인수는 농구장에 갔고,
n -2 n+3 n 3 n+3 x
민지는 극장에 갔다.
따라서 -2<n+3이고 nÉ3이어야 하므로
이때, 농구장에 간 사람이 두 명이므로 모순이다.
-5<nÉ3이어야 한다.
따라서 영훈이의 말은 거짓이다.
이를 만족시키는 정수 n은 -4, -3, -2, -1, 0, 1, 2, 3으로
Û 인수의 말의 참인 경우
8개이다.
독서실 농구장 극장
영훈 ×
인수 × 144 ②

민지 × ○ 명제 ‘모든 실수 x에 대하여 xÛ`+2ax+5>0이다.’가 거짓이기

Ⅳ. 집합과 명제 27
위해서는 그 부정인 ‘어떤 실수 x에 대하여 xÛ`+2ax+5É0이다.’가 역 `:`a+0 또는 b+0이면 aÛ`+bÛ`>0이다.(참)
참이어야 하므로 (증명) 대우는 ‘aÛ`+bÛ`É0이면 a=0, b=0이다.’이고,
이차방정식 xÛ`+2ax+5=0의 판별식을 DÁ이라 하면 대우가 참이므로 명제도 참이다.
DÁ ㄹ. 명제`:`ab=0이면 aÛ`+ab+2bÛ`=0이다. (거짓)
=aÛ`-5¾0, (a+'5)(a-'5)¾0에서
4 (반례) a=1, b=0일 때, ab=1_0=0이지만
aÉ-'5 또는 a¾'5 yy ㉠ aÛ`+ab+2bÛ`=1+0+0+0이므로 거짓이다.
또한 명제 ‘어떤 실수 x에 대하여 xÛ`-ax+2aÉ0이다.’가 거짓이기 역 `:`aÛ`+ab+2bÛ`=0이면 ab=0이다. (참)
위해서는 그 부정인 ‘모든 실수 x에 대하여 xÛ`-ax+2a>0이다.’가 b 7
(증명) aÛ`+ab+2bÛ`={a+ }Û`+ bÛ`=0에서
참이어야 하므로 2 4
이차방정식 xÛ`-ax+2a=0의 판별식을 Dª라 하면 b
a+ =0, b=0이므로 a=b=0 ∴ ab=0
Dª=aÛ`-8a<0, a(a-8)<0에서 2
0<a<8 yy ㉡ ㅁ. 명제`:`ab+1>a+b>2이면 a>1, b>1이다. (참)
㉠, ㉡에서 '5Éa<8을 만족시키는 정수 a는 3, 4, 5, 6, 7로 (증명) ab+1>a+b에서
5개이다. ab-a-b+1=(a-1)(b-1)>0
a>1, b>1 또는 a<1, b<1
이때, a+b>2이므로 a>1, b>1

145 ③ 역 `:`a>1, b>1이면 ab+1>a+b>2이다. (참)


(증명) a>1, b>1이면 a+b>2이고,
명제 ㈎에서 집합 A의 원소이지만 집합 B의 원소는 아닌 것이 ab+1-(a+b)=(a-1)(b-1)>0
존재하므로 A-B+∆, 즉 AøB이다. ∴ ab+1>a+b>2
또한 명제 ㈏에 의하여 집합 B의 원소 중 집합 C의 원소가 존재하지 따라서 역과 대우가 모두 참인 것은 ㄱ, ㄴ, ㄷ, ㅁ으로 4개이다.
않으므로 B;C=∆, 즉 두 집합 B와 C는 서로소이다. 참고
따라서 선지의 벤다이어그램 중 두 명제 ㈎와 ㈏가 항상 참이 되도록
ㄱ에서 a가 실수라는 조건이 없으면
하는 것은
방정식 aÛ`-a+1=0의 두 허근이 반례가 되어
③ U
명제 ‘aÜ`+1=0이면 a+1=0이다.’는 거짓이다.
A C
B

이다. yy TIP 147 ②

TIP
명제 ‘모음이 적힌 카드의 뒷면에는 짝수가 적혀 있다.’가 참임을
②는 조건 ㈎를 만족시키지 않고, 확인하기 위해선
①, ④, ⑤는 조건 ㈏를 만족시키지 않는다. 모음이 적힌 카드 뒷면에 짝수가 적혀 있는지,
그 대우인 홀수가 적힌 카드 뒷면에 자음이 적혀 있는지
확인해야 한다.
따라서 a , 1 을 확인해야 한다.
146 ④

명제와 그 대우의 참, 거짓은 일치하므로 주어진 명제와 그 역이


모두 참인 것을 구하면 된다.
ㄱ. 명제`:`a+1=0이면 aÜ`+1=0이다. (참) 148 ③
(증명) a=-1이면 aÜ`+1=(-1)Ü`+1=0
㈎에 의하여 A는 범인이 아니다.
역 `:`aÜ`+1=0이면 a+1=0이다. (참)
㈏에 의하여 B가 범인이면 C도 범인이고
(증명) aÜ`+1=(a+1)(aÛ`-a+1)=0
㈏의 대우에 의하여 공범이 없으면 B는 범인이 아니다.
a는 실수이므로 a+1=0 yy 참고
㈐에 의하여 A, B, C 중 한 사람은 범인이므로 C가 범인이다.
ㄴ. 명제`:`a+b>0, ab>0이면 a>0, b>0이다. (참) 이를 범인이면 ○, 범인이 아니면 ×로 표기하여 정리하면 다음과
(증명) ab>0에서 a>0, b>0 또는 a<0, b<0이고, 같다.
이때 a+b>0이므로 a>0, b>0이다.
A B C
역 `:`a>0, b>0이면 a+b>0, ab>0이다.(참)
× ○ ○
ㄷ. 명제`:`aÛ`+bÛ`>0이면 a+0 또는 b+0이다. (참)
× × ○
(증명) 대우는 ‘a=0, b=0이면 aÛ`+bÛ`É0이다.’이고
대우가 참이므로 명제도 참이다. 따라서 C는 반드시 범인이다.

28 수학 <하>
이를 수직선 위에 나타내면 다음과 같다.
149 ③
P
Q
네 조건 p, q, r, s를 각각 다음과 같이 정의하자.
-4 a-3 a+3 7 x
p`:`제품이 10대, 20대에게 선호도가 높다.
q`:`제품의 가격이 싸다. 조건을 만족시키기 위해선
r`:`제품의 기능이 많다. -4<a-3이고 a+3É7이어야 하므로
s`:`제품의 판매량이 많다. -1<a이고 aÉ4에서 -1<aÉ4이어야 한다. ⅠV

㈎, ㈏, ㈐에 의하여 알 수 있는 참인 명제는 다음과 같다. 따라서 이를 만족시키는 정수 a는 0, 1, 2, 3, 4로 5개이다. 합

㈎ p` Ú s, ㈏ q` Ú s, ㈐ r` Ú p

각 선지가 나타내는 명제는 다음과 같다.

① r` Ú ~`q, ② ~`q` Ú ~`s, ③ ~s` Ú ~`r, ④ p` Ú r, ⑤ p` Ú ~`q 153 ②
㈎와 ㈐에 의하여 r` Ú s가 참이므로 그 대우인 ~`s` Ú ~`r도 참이다.
세 조건 p, q, r의 진리집합을 각각 P, Q, R라 하자.
따라서 선지 중 옳은 것은 ③이다.
p가 q이기 위한 충분조건이기 위해선 P,Q이어야 하고,
p가 r이기 위한 필요조건이기 위해선 R,P이어야 하므로
150 ④ 이를 수직선에 나타내면 다음과 같다.
Q
두 조건 p, q의 진리집합을 각각 P, Q라 하면 P P
R
p가 q이기 위한 충분조건이므로 P,Q이다. x
C C a -1 2 5 b
따라서 Q ,P 이다.
C C 조건을 만족시키기 위해선
Q ={1}, P ={x|xÛ`+ax-5=0}이므로
a<-1이고 b¾5이어야 하므로
xÛ`+ax-5=0의 양변에 x=1을 대입하면
정수 a의 최댓값은 -2이고 정수 b의 최솟값은 5이다.
1+a-5=0 ∴ a=4
따라서 구하는 값은 -2+5=3이다.

151 ④

두 조건 p, q의 진리집합을 각각 P, Q라 할 때 154 ⑤

p가 q이기 위한 필요충분조건이기 위해선 P=Q이어야 한다. ㄱ. A;B=A는 A,B이기 위한 필요충분조건이고


ㄱ. p`:`|x-y|=|x+y|에서 A'B=B는 A,B이기 위한 필요충분조건이다.
|x-y|=|x+y|의 양변을 제곱하면 그러므로 p는 q이기 위한 필요충분조건이다.
(x-y)Û`=(x+y)Û`, xÛ`-2xy+yÛ`=xÛ`+2xy+yÛ`, xy=0 C
ㄴ. A=B 이면 A'B=B 'B=U이다.
C

즉, P=Q이므로 p는 q이기 위한 필요충분조건이다. A'B=U일 때 A;B+∆인 경우도 있으므로 A+B 이다.


C

ㄴ. p`:`|x|+|y|=|x+y|에서 그러므로 p는 q이기 위한 충분조건이지만 필요조건은 아니다.


|x|+|y|=|x+y|의 양변을 제곱하면 ㄷ. A,B 또는 A,C이면 A,(B'C)이다.
|x|Û`+2|x||y|+|y|Û`=(x+y)Û` 한편, A,(B'C)이어도 다음 벤다이어그램처럼 A,B 또는
xÛ`+2|xy|+yÛ`=xÛ`+2xy+yÛ` A,C가 아닐 수도 있다.
|xy|=xy이므로 xy¾0 U
즉, Q,P이므로 p는 q이기 위한 필요조건이다. B C

ㄷ. p`:`|x+y|<|x-y|에서 A
|x+y|¾0, |x-y|¾0이므로
|x+y|, |x-y|와 |x+y|Û`, |x-y|Û`의 대소 관계는 같다.
|x+y|Û`-|x-y|Û`=(x+y)Û`-(x-y)Û`=4xy<0이므로 그러므로 p는 q이기 위한 충분조건이지만 필요조건은 아니다.
xy<0 따라서 구하는 것은 ㄴ, ㄷ이다.
즉, P=Q이므로 p는 q이기 위한 필요충분조건이다.
따라서 p가 q이기 위한 필요충분조건인 것은 ㄱ, ㄷ이다.

155 ⑤

152 ③
Ú 명제 p` Ú q (참)
두 조건 p, q의 진리집합을 각각 P, Q라 하면 (증명) X,(A;B)이면 X,(A;B),(A'B)이므로
p가 q이기 위한 필요조건이기 위해선 Q,P가 성립해야 한다. X,(A'B)이다.
q`:`|x-a|É3에서 -3Éx-aÉ3, a-3ÉxÉa+3 명제 q` Ú p (거짓)

Ⅳ. 집합과 명제 29
(반례) U 따라서 필요충분조건은 ㄴ이다.
A B
X

157 ③

n
'2 가 유리수 라 가정하면 '2= yy ㉠
따라서 조건 p는 조건 q이기 위한 충분조건이므로 m
<p, q>=1 을 만족시키는 서로소 인 두 자연수 m, n이 존재한다.
Û 명제 q` Ú r (거짓) ㉠의 양변을 각각 제곱하여 정리하면 nÛ`=2mÛ` yy㉡
(반례) U 이때, 2mÛ`은 2의 배수이다.
A B
X
nÛ`이 2의 배수이므로 n은 2 의 배수이다. yy TIP
n=2k (k는 자연수)라 하면 ㉡에서
4kÛ`=2mÛ`, mÛ`= 2kÛ`

명제 r` Ú q (참) 이때, mÛ`이 2의 배수이므로 m은 2 의 배수이다.


X,A 또는 X,B이면 X,(A'B)이다. 즉, m, n이 모두 2 의 배수이므로
따라서 조건 q는 조건 r이기 위한 필요조건이므로
m, n이 서로소 라는 조건에 모순 이다.
<q, r>=-1
따라서 '2 는 유리수가 아니다.
Ü 명제 r` Ú p (거짓)
㈎`:`유리수, ㈏`:`서로소, ㈐`:`2, ㈑`:`2kÛ`, ㈒`:`모순
(반례) U U
A B A B TIP
X X
명제 ‘자연수 n에 대하여 nÛ`이 2의 배수이면 n은 2의
배수이다.’는 문제 119에서 증명한 명제이다.

명제 p` Ú r (참)
X,(A;B)이면 X,A 또는 X,B이다.
따라서 조건 r는 조건 p이기 위한 필요조건이므로
158 ②

<r, p>=-1 m, n 중 적어도 하나가 홀수라 가정하면


Ú~Ü에 의하여 Ú m=2a, n=2b-1(a, b는 자연수)일 때,
<p, q>-2<q, r>-3<r, p> mÛ`+nÛ`=(2a)Û`+(2b-1)Û`=4aÛ`+(4bÛ`-4b+1)
=1-2_(-1)-3_(-1)=6 =4(aÛ`+bÛ`-b)+1
이므로 mÛ`+nÛ`을 4로 나누었을 때의 나머지는 1 이다.

156 ② Û m=2a-1, n=2b`(a, b는 자연수)일 때,


mÛ`+nÛ`=(2a-1)Û`+(2b)Û`=(4aÛ`-4a+1)+4bÛ`
조건 ‘p`:`x, y, z 중 적어도 하나는 2 이하이다.’ =4(aÛ`+bÛ`-a)+1
를 만족시키는 경우는 다음과 같다.
이므로 mÛ`+nÛ`을 4로 나누었을 때의 나머지는 1 이다.
Ú 2 이하인 수가 1개, 2보다 큰 수가 2개
Ü m=2a-1, n=2b-1`(a, b는 자연수)일 때,
Û 2 이하인 수가 2개, 2보다 큰 수가 1개
mÛ`+nÛ`=(2a-1)Û`+(2b-1)Û`
Ü 2 이하인 수가 3개
=(4aÛ`-4a+1)+(4bÛ`-4b+1)
ㄱ. (x-2)(y-2)(z-2)É0
=4(aÛ`+bÛ`-a-b)+2
HjK x-2, y-2, z-2 중 1개만 0 이하
이므로 mÛ`+nÛ`을 4로 나누었을 때의 나머지는 2 이다.
또는 x-2, y-2, z-2 모두 0 이하
HjK Ú 또는 Ü Ú ~ Ü에 의하여 mÛ`+nÛ`은 4의 배수가 아니다. 이것은 mÛ`+nÛ`이
즉, 조건 ㄱ은 p이기 위한 충분조건이다. 4의 배수라는 조건에 모순이다.
ㄴ. x, y, z 중 최솟값은 2 이하이다. 따라서 두 자연수 m, n에 대하여 mÛ`+nÛ`이 4의 배수이면 m, n은
HjK x, y, z 중 2 이하의 값이 존재한다. 모두 짝수이다.
HjK x, y, z 중 적어도 하나는 2 이하이다. ㈎`:`1, ㈏`:`1, ㈐`:`2
즉, 조건 ㄴ은 p이기 위한 필요충분조건이다.
ㄷ. x, y, z 중 최댓값은 2 이하이다.
HjK x, y, z가 모두 2 이하이다.
159 ④

HjK Ü f(a)+ f(b)라 가정하면


즉, 조건 ㄷ은 p이기 위한 충분조건이다. 일반성을 잃지 않고, 음이 아닌 두 정수 kÁ, kª에 대하여

30 수학 <하>
a=3kÁ+1, b= 3kª+2 이다. TIP

aÛ`+ab+bÛ`=(3kÁ+1)Û`+(3kÁ+1)( 3kª+2 )+( 3kª+2 )Û` 두 양수 x, y에 대하여


x¾y이면 xÛ`¾yÛ`이고, xÛ`¾yÛ`이면 x¾y이다.
=3{3(kÁ)Û`+3(kª)Û`+3kÁkª+4kÁ+5kª+2}+ 1
x<y이면 xÛ`<yÛ`이고, xÛ`<yÛ`이면 x<y이다.
이므로 f(aÛ`+ab+bÛ`)=0 이라는 조건에 모순이다.
따라서 x, y가 양수일 때,
따라서 명제 ‘f(aÛ`+ab+bÛ`)=0이면 f(a)= f(b)이다.’가 참이다. x, y의 대소 관계는 xÛ`, yÛ`의 대소 관계와 같다.
㈎`:`3kª+2, ㈏`:`1, ㈐`:` f(aÛ`+ab+bÛ`)=0
ⅠV



160 풀이 참조

⑴ 주어진 명제의 대우는 ‘자연수 n에 대하여 n이 3의 배수가 아니면 162 풀이 참조 제

nÛ`은 3의 배수가 아니다.’이다. ('x+'y)Û`-('Äx+y)Û`


n이 3의 배수가 아니면 ='x Û`+2'x 'y+'y Û`-'Äx+y Û`
자연수 k에 대하여 n=3k-1 또는 n=3k-2이다. =x+2'¶xy+y-x-y (∵ x¾0, y¾0)
Ú n=3k-1일 때, =2'¶xy¾0 yy ㉠
nÛ`=(3k-1)Û`=9kÛ`-6k+1 이므로 ('x+'y )Û`¾'Äx+y Û`이다.
=3(3kÛ`-2k)+1 이때, 'x+'y ¾0, 'Äx+y ¾0이므로
이므로 nÛ`은 3의 배수가 아니다. 'x+'y ¾'Äx+y이다.
Û n=3k-2일 때, ㉠에서 등호는 xy=0, 즉 x=0 또는 y=0일 때 성립한다.
nÛ`=(3k-2)Û`=9kÛ`-12k+4
채점 요소 배점
=3(3kÛ`-4k+1)+1
('x+'y )Û`, 'Äx+y Û` 식 사용하여 증명하기 80%
이므로 nÛ`은 3의 배수가 아니다.
등호가 성립할 때의 x, y의 값 구하기 20%
Ú, Û에 의하여 nÛ`은 3의 배수가 아니다.
따라서 주어진 명제의 대우가 참이므로 주어진 명제도 참이다.
⑵ '3 을 유리수라 가정하면
n
'3= (m, n은 서로소인 자연수)이다.
m 163 ⑤
이 식의 양변을 제곱하여 정리하면
ㄱ. (반례) a=2, b=-3일 때,
nÛ`=3mÛ` yy ㉠
|a+b|=|2+(-3)|=1,
즉, nÛ`은 3의 배수이므로 n은 3의 배수이다.
|a-b|=|2-(-3)|=5
n=3k (k는 자연수)라 하면 ㉠에서
에서 1<5이다. (거짓) yy 참고 1
9kÛ`=3mÛ`, mÛ`=3kÛ`
즉, mÛ`이 3의 배수이므로 m도 3의 배수이다. ㄴ. |a-b|¾0, |a|+|b|¾0이므로

m, n이 모두 3의 배수이므로 서로소라는 조건에 모순이다. |a-b|Û`과 (|a|+|b|)Û`의 크기를 비교하면 된다.

따라서 '3은 유리수가 아니므로 무리수이다. |a-b|Û`-(|a|+|b|)Û`


=(aÛ`-2ab+bÛ`)-(aÛ`+2|a||b|+bÛ`)
채점 요소 배점
=-2(ab+|ab|)É0
대우를 이용하여 ⑴의 명제를 증명하기 50%
(단, 등호는 |ab|=-ab, 즉 abÉ0일 때 성립한다.)
⑴의 명제와 귀류법을 이용하여 ⑵의 명제 증명하기 50%
∴ |a-b|É|a|+|b| (참)
ㄷ. a>b>0에서 'Äa-b >0, 'a-'b >0이므로
'Äa-b Û`과 ('a-'b )Û`의 크기를 비교하면 된다.
161 ② 'Äa-b Û`-('a-'b )Û`

(|a|+|b|)Û`-|a+b|Û`=|a|Û`+2|a||b|+|b|Û`-|a+b|Û` =(a-b)-(a-2'a 'b+b)

=aÛ`+2|ab|+bÛ`-(aÛ`+2ab+bÛ`) =2'b('a-'b)>0
∴ 'Äa-b >'a-'b (참)
= 2(|ab|-ab)
ㄹ. Ú |a|<|b|일 때,
이때, |ab|¾ab 이므로 2(|ab|-ab) ¾0
|a|-|b|<0, |a-b|>0이므로
따라서 (|a|+|b|)Û`¾|a+b|Û`이고, |a|-|b|<|a-b|가 성립한다.
|a|+|b|¾0, |a+b|¾0이므로 |a|+|b|¾|a+b|이다. Û |a|¾|b|일 때,
(단, 등호는 |ab|=ab, 즉 ab¾0 일 때 성립한다.) |a|-|b|¾0, |a-b|¾0이므로
㈎ : 2(|ab|-ab), ㈏ : |ab|¾ab, ㈐ : ab¾0 (|a|-|b|)Û` 과 |a-b|Û`의 크기를 비교하면 된다.

Ⅳ. 집합과 명제 31
|a-b|Û`-(|a|-|b|)Û` D
=(-'5)Û`-k<0이므로 k>5이다.
=(a-b)Û`-(|a|Û`-2|a||b|+|b|Û`) 4
=aÛ`-2ab+bÛ`-(aÛ`-2|ab|+bÛ`) k-5>0이므로 산술평균과 기하평균의 관계에 의하여
=2(|ab|-ab) 9 9
k+5+ =k-5+ +10
k-5 k-5
이때, |ab|¾ab이므로 2(|ab|-ab)¾0
9
∴ |a|-|b|É|a-b| ¾2¾Ð(k-5)_ +10=16
k-5
(단, 등호는 |a|¾|b|이고 |ab|=ab일 때, 즉 9
이때, 등호는 k-5= 일 때 성립하므로
aÉbÉ0 또는 a¾b¾0일 때 성립한다.) k-5
Ú, Û에 의하여 |a|-|b|É|a-b| (참) (k-5)Û`=9에서 k=8 (∵ k>5)
따라서 항상 성립하는 것은 ㄴ, ㄷ, ㄹ이다. 따라서 이차방정식 xÛ`-2'5x+k=0이 허근을 가질 때,
참고 1 9
k+5+ 는 k=8일 때 최솟값 16을 가지므로
k-5
|a+b|Û`-|a-b|Û`=(aÛ`+2ab+bÛ`)-(aÛ`-2ab+bÛ`)=4ab
a=8, b=16이다.
이므로
∴ a+b=24
ab¾0일 때 |a+b|¾|a-b|,
ab<0일 때 |a+b|<|a-b|가 성립한다.

참고 2

절대부등식 ‘|a|-|b|É|a+b|’도 종종 출제된다.


166 풀이 참조

이는 ‘ㄹ’과 유사한 방법으로 다음과 같이 증명한다. 4 4


4x+ =4(x-3)+ +12
x-3 x-3
Ú |a|<|b|인 경우
x-3>0이므로 (∵ x>3)
|a|-|b|<0, |a+b|¾0이므로
산술평균과 기하평균의 관계에 의하여
|a|-|b|<|a+b|
4 4
Û |a|¾|b|인 경우 4(x-3)+ +12¾2¾Ð4(x-3)× +12 yy ㉠
x-3 x-3
|a|-|b|¾0, |a+b|¾0이므로 =2×4+12=20
(|a|-|b|)Û`과 |a+b|Û`의 크기를 비교하면 된다. 4
따라서 4x+ 의 최솟값은 20이다.
(|a|-|b|)Û`-|a+b|Û` x-3
=(|a|Û`-2|a||b|+|b|Û`)-(a+b)Û` 4
㉠에서 4(x-3)= 일 때 등호가 성립하므로
=-2(|ab|+ab)É0 x-3
(단, 등호는 |ab|=-ab, 즉 abÉ0일 때 성립한다.) (x-3)Û`=1, x=4 (∵ x>3)
∴ |a|-|b|É|a+b| 4
즉, 4x+ 는 x=4일 때 최솟값 20을 갖는다.
x-3
Ú, Û에 의하여 |a|-|b|É|a+b|이다.
TIP
산술평균과 기하평균의 관계를 이용하여
164 풀이 참조
식 f(x)+
k
(k는 상수)의 최솟값을 구할 때,
g(x)
aÛ`+bÛ`-2(a-b-1) 역수의 곱을 이용할 수 있도록 식을 변형한다.
=(aÛ`-2a+1)+(bÛ`+2b+1) 이때, 변형하기 어려운 분모 g(x)를 기준으로
=(a-1)Û`+(b+1)Û` yy ㉠ 변형하기 쉬운 f(x) 부분을 g(x) 꼴로 바꾸는 것이 중요하다.
이때, 두 실수 a, b에 대하여 만약 x>-1일 때,
(a-1)Û`¾0, (b+1)Û`¾0이므로 2 x
+ 의 최솟값을 구해야 하는 경우
aÛ`+bÛ`-2(a-b-1)¾0 x+1 8
즉, aÛ`+bÛ`¾2(a-b-1)이 성립한다. 2
의 분모 x+1을 기준으로
이때, 등호는 ㉠에서 (a-1)Û`+(b+1)Û`=0일 때이므로 a-1=0, x+1
b+1=0, 즉 a=1, b=-1에서 성립한다. x x+1 1
를 - 로 바꾸면
8 8 8
채점 요소 배점 산술평균과 기하평균의 관계에 의하여
주어진 부등식이 성립함을 증명하기 80% 2 x 2 x+1 1
+ = + -
등호가 성립할 때의 a, b의 값 구하기 20% x+1 8 x+1 8 8
2 x+1 1 1 7
¾2¾Ð _ - =1- =
x+1 8 8 8 8
165 ④ (단, 등호는 x=3일 때 성립한다.)
로 최솟값을 구할 수 있다.
이차방정식 xÛ`-2'5x+k=0의 판별식을 D라 하면

32 수학 <하>
채점 요소 배점 5 3 15a 15b
(5a+3b){ + }=25+{ + }+9
a b b a
4
식 4x+ 변형하기 20%
x-3 a b
=34+15{ + }
최솟값 구하기 50%
b a
a b
최솟값을 가질 때 x의 값 구하기 30% ¾34+15_2¾Ð _
b a
=34+30=64
ⅠV
(단, 등호는 a=b일 때 성립한다.) 집
이때, 5a+3b=4이므로 합
167 풀이 참조
5 3 5 3

4{ + }¾64, + ¾16 명
⑴ 잘못된 부분: ④ a b a b

①의 등호가 성립할 때는 2x=y이고 따라서 구하는 최솟값은 16이다.
2 1
②의 등호가 성립할 때는 = 에서 x=2y이다.
x y
2x=y이면서 x=2y를 동시에 만족시키는 두 양수 x, y는 169 ①
존재하지 않으므로 ③에서 등호는 성립하지 않는다.
두 양수 a, b에 대하여 2a+1>0, 3b+1>0이므로
따라서 최솟값은 8이 될 수 없다.
산술평균과 기하평균의 관계에 의하여
2 1 2x 2y
⑵ (2x+y){ + }=4+ + +1 1 1
x y y x {(2a+1)+(3b+1)}{ + }
2a+1 3b+1
x y
=2{ + }+5 2a+1 3b+1
y x =1+{ + }+1
3b+1 2a+1
x y
¾2_2¾Ð _ +5=9 2a+1 3b+1
y x ¾2+2¾Ð _ =4
3b+1 2a+1
2 1
따라서 (2x+y){ + }의 최솟값은 9이고 (단, 등호는 (2a+1)Û`=(3b+1)Û`, 즉 2a=3b일 때 성립한다.)
x y
x y 1 1 1
등호는 = , 즉 xÛ`=yÛ`에서 x=y일 때 성립한다. 이때, + = 이므로
y x 2a+1 3b+1 5
(∵ x>0, y>0) 1 1
{(2a+1)+(3b+1)}{ + }¾4에서
2a+1 3b+1
TIP
1
(2a+3b+2)_ ¾4, 2a+3b¾18
2 1 5
③에서 두 양수 x, y에 대하여 부등식 (2x+y){ + }¾8이
x y 따라서 2a+3b의 최솟값은 18이다.
2 1
항상 성립하는 것은 맞지만 (2x+y){ + }=8을
x y
만족시키는 두 실수 x, y가 존재하는 것은 보장하지 않는다.
두 양수 a, b에 대하여 산술평균과 기하평균의 관계인
170 ②

a+b¾2'¶ab (단, 등호는 a=b일 때 성립)를 이용하여 x y


a>0, b>0이므로 직선 + =1과 x축, y축으로 둘러싸인
a b
최솟값 또는 최댓값을 구할 때 다음을 확인해야 한다.
1 yy TIP
❶ a+b의 최솟값을 구할 때 삼각형의 넓이는 ab이다.
2
ab가 상수인지, a=b가 가능한지 확인한다. y
❷ ab의 최댓값을 구할 때
a+b가 상수인지, a=b가 가능한지 확인한다. b x y
+ =1
a b

채점 요소 배점
처음으로 잘못된 부분이 ④임을 찾기 20% O a x
2x=y, x=2y를 만족시키는 x, y가 존재하지 않음을 설명하기 30%
x y
올바른 최솟값 구하기 30% 이때, 직선 + =1이 점 (3, 5)를 지나므로
a b
최솟값을 가질 때 x, y의 값 구하기 20% 3 5
+ =1
a b
산술평균과 기하평균의 관계에 의하여
3 5 3 5 15
1= + ¾2¾Ð _ =2¾Ð
168 ③ a b a b
3 5
ab

(단, 등호는 = 일 때 성립한다.)


a>0, b>0이므로 a b
산술평균과 기하평균의 관계에 의하여 '¶ab ¾2'1Œ5이므로 ab¾60

Ⅳ. 집합과 명제 33
1 즉, 0<'a+'¶3bÉ2'6 (∵ a>0, b>0)
따라서 ab¾30이므로 구하는 최솟값은 30이다.
2 이므로 최댓값은 2'6이다.
TIP 이때, 등호는 '¶3a='b, 즉 3a=b일 때 성립하므로
x y 3 9
직선 + =1의 x절편은 a, y절편은 b이다. ㉠에 대입하면 4a=6에서 a= , b=
a b 2 2
3 9
따라서 'a+'¶3b 는 a= , b= 일 때, 최댓값 2'6 을 갖는다.
2 2

7
171 x=-1 또는 x=1일 때, 최솟값
3
을 갖는다.

;;Á9¤;;
xÛ`+
16
9xÛ`+3
=xÛ`+ 174 ②
xÛ`+;3!;
3x+4y=5 yy ㉠
1 16 1 1 이므로 코시-슈바르츠의 부등식에 의하여
={xÛ`+ }+ _ -
3 9 3
xÛ`+;3!; (1Û`+2Û`){(3x)Û`+(2y)Û`}¾(1_3x+2_2y)Û`=25
(단, 등호는 6x=2y, 즉 3x=y일 때 성립한다.) yy ㉡
1 16 1 1 7
¾2¾Ð{xÛ`+ }_{ _ }- 3 = 3 즉, 9xÛ`+4yÛ`¾5에서 9xÛ`+4yÛ`의 최솟값은 m=5
3 9
xÛ`+;3!;
1
㉡에서 3x=y이므로 ㉠에 대입하면 y=1, x=
1 16 1 3
등호는 xÛ`+ = _ 에서
3 9 1
xÛ`+;3!; 따라서 a= , b=1이므로
3
1 16 1 4 m 5
{xÛ`+ }Û`= , xÛ`+ = , xÛ`=1 = =15
3 9 3 3 ab
;3!;_1
즉, x=-1 또는 x=1일 때 성립한다.
7
따라서 주어진 식은 x=-1 또는 x=1일 때, 최솟값 을 갖는다.
3

175 ③
172 ③
ㄱ. x>0, y>0이고 x+y=4이므로
x-1 1 1 1 산술평균과 기하평균의 관계에 의하여
= = =
xÛ`-x+1 xÛ`-x+1 x(x-1)+1 1
x+ x+y¾2'¶xy (단, 등호는 x=y일 때 성립한다.)
x-1 x-1 x-1
∴ 4¾2'¶xy, '¶xyÉ2 (참)
1
= yy ㉠ ㄴ. 코시-슈바르츠의 부등식에 의하여
1
x-1+ +1
x-1 ('x+'y )Û`É(1Û`+1Û`)('x Û`+'y Û`)
x-1>0이므로 산술평균과 기하평균의 관계에 의하여 =2(x+y)=8 (∵ x+y=4)
1 1 (단, 등호는 x=y일 때 성립한다.)
x-1+ +1¾2¾Ð(x-1)_ +1=3
x-1 x-1 ∴ 'x+'y É2'2 (참)
(단, 등호는 x=2일 때 성립한다.) ㄷ. (반례) x=1, y=3이면 x+y=4이지만
㉠에서 1 1 1
+ =1+ >'2 (거짓) yy TIP
1 1 'x 'y '3
É
1 3 따라서 옳은 것은 ㄱ, ㄴ이다.
x-1+ +1
x-1
다른 풀이
1
따라서 주어진 식은 x=2일 때, 최댓값 을 갖는다.
3 ㄴ. ('x+'y )Û`=x+y+2'¶xy=4+2'¶xy yy ㉠
ㄱ에서 '¶xyÉ2이므로 ㉠에서
('x+'y )Û`É4+2_2=8
3 9 이때, 'x+'y¾0이므로 'x+'y É2'2 (참)
173 a= , b= 일 때, 최댓값 2'6을 갖는다.
2 2
TIP
a+b=6 yy㉠
이므로 코시-슈바르츠의 부등식에 의하여 1 2'3-2
a=1+ , b='2라 하면 aÛ`-bÛ`= >0이므로
'3 3
('a+'¶3b)Û`É(1Û`+'3 Û`)('a Û`+'b Û`)
aÛ`>bÛ`에서 a>b이다.
É4(a+b)=24 (∵ ㉠)

34 수학 <하>
176 ③ 179 ⑴6 ⑵9

aÛ`>0, bÛ`>0, cÛ`>0, dÛ`>0이므로 ⑴ 산술평균과 기하평균의 관계에 의하여


산술평균과 기하평균의 관계에 의하여 y+z z+x x+y
+ +
aÛ`+bÛ` x y z
"ÃaÛ`bÛ` É =1, |ab|É1, -1ÉabÉ1
2 y z z x x y
={ + }+{ + }+{ + }
"ÃcÛ`dÛ` É
cÛ`+dÛ`
=9, |cd|É9, -9ÉcdÉ9
x x y y z z ⅠV
2 y x z y x z 집
={ + }+{ + }+{ + }
(단, 등호는 |a|=|b|=1, |c|=|d|=3일 때 성립한다.) x y y z z x 합

(-1)+(-9)Éab+cdÉ1+9 y x z y x z
¾2¾Ð _ +2¾Ð _ +2¾Ð _ 명
x y y z z x
-10Éab+cdÉ10이므로 MÁ=10 제
=2+2+2=6 (단, 등호는 x=y=z일 때 성립한다.)
코시-슈바르츠의 부등식에 의하여
y+z z+x x+y
(ac+bd)Û`É(aÛ`+bÛ`)(cÛ`+dÛ`) 따라서 + + 의 최솟값은 6이다.
x y z
(ac+bd)Û`É2_18=6Û` (단, 등호는 ad=bc일 때 성립한다.) 1 1 1
⑵ (x+y+z){ + + }
-6Éac+bdÉ6이므로 Mª=6 x y z
∴ MÁ+Mª=10+6=16 x x y y z z
=1+ + + +1+ + + +1
y z x z x y
y x z y x z
=3+{ + }+{ + }+{ + }
x y y z z x
y x z y x z
177 ② ¾3+2¾Ð _ +2¾Ð _ +2¾Ð _
x y y z z x
원의 중심이 선분 BD 위에 있으면 선분 BD가 이 원의 지름이므로 =3+6=9 (단, 등호는 x=y=z일 때 성립한다.)
∠A=∠C=90ù이고, 두 삼각형 BAD, BCD는 각각 1 1 1
따라서 (x+y+z){ + + }의 최솟값은 9이다.
x y z
직각삼각형이다.
다른 풀이
ABÓ=1, ADÓ=7이므로
BDÓ Û`=1Û`+7Û`=50 yy㉠ ⑵ 코시-슈바르츠의 부등식에 의하여
BCÓ=x, CDÓ=y라 하면 ㉠에 의하여 1 1 1
(x+y+z){ + + }
x y z
xÛ`+yÛ`=50 yy㉡
1 Û` 1 Û` 1 Û`
따라서 코시-슈바르츠의 부등식에 의하여 =('x Û`+'y Û`+'z Û`)[{
} +{ } +{ }]
'x 'y 'z
(1Û`+1Û`)(xÛ`+yÛ`)¾(x+y)Û`
1 1 1 Û`
(x+y)Û`É100 (∵ ㉡) ¾{'x_ +'y_ +'z_ }
'x 'y 'z
∴ 0<x+yÉ10 (∵ x+y>0) =(1+1+1)Û`=9 (단, 등호는 x=y=z일 때 성립한다.)
(단, 등호는 x=y=5일 때 성립한다.) 1 1 1
따라서 (x+y+z){ + + }의 최솟값은 9이다.
따라서 사각형 ABCD의 둘레의 길이는 x+y+8É18이므로 x y z
구하는 사각형 ABCD의 둘레의 길이의 최댓값은 18이다.

178 ④ 180 ③

b+c=x, c+a=y, a+b=z라 하자.


코시-슈바르츠의 부등식에 의하여
세 식의 양변을 각각 더하면
(1Û`+1Û`+1Û`)(aÛ`+bÛ`+cÛ`)¾(a+b+c)Û`=36
2(a+b+c)=x+y+z에서
aÛ`+bÛ`+cÛ`¾12
따라서 aÛ`+bÛ`+cÛ`의 최솟값은 12이다. a+b+c= ;2!; (x+y+z)이므로
TIP
1 1 1
a= (y+z-x), b= (z+x-y), c= (x+y-z)
<코시-슈바르츠의 부등식의 일반화> 2 2 2
자연수 n에 대하여 이다. 그러므로
aÁ, aª, y, an, xÁ, xª, y, xn이 실수일 때, a b c
+ +
b+c c+a a+b
{(aÁ)Û`+(aª)Û`+y+(an)Û`}{(xÁ)Û`+(xª)Û`+y+(xn)Û`}
1 y+z-x z+x-y x+y-z
¾(aÁxÁ+aªxª+y+anxn)Û` = { + + }
2 x y z
xÁ xª x
(단, 등호는 = =y= n 일 때 성립한다.) 1 y z z x x y
aÁ aª an = { + + + + + }+{ -;2#; }
2 x x y y z z

Ⅳ. 집합과 명제 35
1 y x z y x z U
¾ {2¾Ð _ +2¾Ð _ +2¾Ð _ }+{ -;2#; } Q
2 x y y z z x P
R
y x z y x z
= 1 {¾Ð _ +¾Ð _ +¾Ð _ }+{ -;2#; }
x y y z z x
3
= (단, 등호는 x=y=z일 때 성립한다.)
2 ㄱ. 명제 q` Ú p의 역은 p` Ú q이고 P,Q이므로 명제 p` Ú q는
따라서 세 양수 a, b, c에 대하여 주어진 부등식이 성립한다. 참이다.
1 3 ㄴ. 명제 ~`r` Ú ~`q의 역은 ~`q` Ú ~`r이고 R,Q에서
㈎`:` , ㈏`:`- , ㈐`:`1
2 2 C C
Q ,R 이므로 명제 ~`q` Ú ~`r는 참이다.
ㄷ. 명제 ~`p` Ú r의 역은 r` Ú ~`p이고 R,P이므로 명제
r` Ú ~`p는 거짓이다.
따라서 역이 참이 되는 명제는 ㄱ, ㄴ이다.
181 ⑤

조건 q가 조건 p이기 위한 충분조건이지만 필요조건은 아니고,


조건 r이기 위한 필요조건이지만 충분조건이 아니므로
m+24, n+24이고, 두 명제 q` Ú p, r` Ú q가 참이다. 184 20

세 조건 p, q, r의 진리집합이 각각 Am, A24, An이므로 ‘직선 l 위의 어떤 점 P에 대하여 ∠APB=90ù이다.’가 참이 되기


A24,Am, An,A24 yy ㉠ 위해선 점 P가 선분 AB를 지름으로 하는 원 위에 존재해야 한다.
㉠에서 m은 24의 배수이고, n은 24의 양의 약수이므로 즉, 직선 l`:`x-3y=k와 선분 AB를 지름으로 하는 원의 교점이
m의 최솟값은 48이고, n의 최댓값은 12이다. 존재하면 된다. yy ㉠
따라서 구하는 값은 48+12=60이다. y
B{1,`4}
x-3y=k
P P

182 ②
O x
24
조건 ‘p`:` 가 자연수이다.’에서 조건 p가 참이 되도록 하는 A{-1,`-2}
x
자연수 x는 24의 양의 약수이므로 A(-1, -2), B(1, 4)이므로 선분 AB를 지름으로 하는
P={1, 2, 3, 4, 6, 8, 12, 24}이다. 원의 중심의 좌표는 (0, 1)이고, 반지름의 길이는 '1Œ0이다.
조건 ‘p이고 ~`q’의 진리집합이 P이므로 |-3-k|
㉠이 성립하려면 É'1Œ0
C
P;Q =P에서 P;Q=∆이다. "Ã1Û`+(-3)Û`
|k+3|É10, -13ÉkÉ7이므로 M=7, m=-13
자연수 a로 가능한 값은
∴ M-m=20
5, 7, 9, 10, 11과 13ÉaÉ23인 자연수, 25Éa<40인 자연수이다.
따라서 구하는 자연수 a의 개수는
5+11+15=31이다.
185 ①

세 학생 A, B, C가 대답한 내용을 표로 나타내면 다음과 같다.


183 ③ 1등 2등 3등 4등

세 조건 p, q, r의 진리집합을 각각 P, Q, R라 하면 우석 A C
1 1 상섭 B C
Ú 조건 p에서 + =1의 양변에 각각 xy를 곱하면
x y 기훈 B
x+y=xy이고, x+0, y+0이므로 종현 A
P={(x, y)|x+y=xy, xy+0}
A의 대답에 따라 경우를 나누어보면
Û 조건 q에서 (x-1)(y-1)=1, xy-x-y+1=1
Ú 우석이가 1등을 한 경우
xy=x+y이므로 Q={(x, y)|x+y=xy}
1등 2등 3등 4등
Ü 조건 r에서 2(x+y)=2xy
우석 ○ ×
x+y=xy이고, x, y는 길이이므로 x>0, y>0
상섭 ○ ○
R={(x, y)|x+y=xy, x>0, y>0}
Ú~Ü에서 R,P,Q이므로 전체집합을 U라 하고 기훈 ×
벤다이어그램으로 나타내면 다음과 같다. 종현 ×

36 수학 <하>
종현이는 2등이 아니며, C의 대답에 따라 상섭이는 4등을 했다.
한편, B의 대답에서 기훈이가 4등을 했다는 대답은 거짓이
188 8

되므로 상섭이는 2등을 했고, 상섭이가 2등과 4등을 동시에 할 수 ㄱ. p`:`(x-y)Û`+(y-z)Û`+(z-x)Û`=0


없으므로 이는 모순이다. HjK x=y=z
Û 종현이가 2등을 한 경우 q`:`|x|=|y|=|z|
명제 p` Ú q는 참이다.
1등 2등 3등 4등
명제 q` Ú p는 거짓이다.
ⅠV
우석 × ○ 집
(반례) x=-1, y=1, z=1 합
상섭 ○ × ×

그러므로 p는 q이기 위한 충분조건이지만 필요조건은 아니다.
기훈 ○ 명
ㄴ. p`:`x=y=z
종현 ○ 제
q`:`(x-y)(y-z)(z-x)=0
우석이는 1등이 아니며, B의 대답에서 상섭이가 2등을 했다는 HjK x=y 또는 y=z 또는 z=x
대답은 거짓이 되므로 기훈이는 4등을 했고, C의 대답에서 그러므로 p는 q이기 위한 충분조건이지만 필요조건은 아니다.
상섭이는 4등이 아니므로 우석이가 3등이 된다. ㄷ. p`:`|xy|+|yz|+|zx|=0
Ú, Û에 의하여 모순이 없는 상황을 결과로 정리하면 HjK xy=0, yz=0, zx=0
1등은 상섭, 2등은 종현, 3등은 우석, 4등은 기훈이다. HjK x=0, y=0 또는 y=0, z=0 또는 z=0, x=0
q`:`x, y, z 중 적어도 2개는 0이다.
그러므로 p는 q이기 위한 필요충분조건이다.
186 ②
ㄹ. 명제 p` Ú q는 참이다.
조건 ㈏에서 n(P-Q)=2, n(Q-P)=4이므로 (증명) x>y>z이면 x-y>0, y-z>0, z-x<0이므로
두 집합 P, Q의 원소의 개수를 표시하면 다음과 같다. (x-y)(y-z)(z-x)<0이다.
P Q 명제 q` Ú p는 거짓이다.
(반례) x-y<0, y-z>0, z-x>0이면
2 3 4
(x-y)(y-z)(z-x)<0이지만 y>z>x이다.
yy TIP 1
조건 ㈎, ㈐에 의하여 그러므로 p는 q이기 위한 충분조건이지만 필요조건은 아니다.
집합 R는 집합 P'Q의 부분집합이지만 ㅁ. p`:`xy+yz+zx¾0
집합 P의 부분집합이 아니고, 집합 Q의 부분집합도 아니다. q`:`|x+y+z|=|x|+|y|+|z|
P'Q의 부분집합의 개수는 2á` HjK xÛ`+yÛ`+zÛ`+2(xy+yz+zx)
집합 P의 부분집합의 개수는 2Þ` =xÛ`+yÛ`+zÛ`+2(|xy|+|yz|+|zx|)
집합 Q의 부분집합의 개수는 2à` HjK xy+yz+zx=|xy|+|yz|+|zx|
P;Q의 부분집합의 개수는 2Ü` HjK xy¾0, yz¾0, zx¾0
따라서 구하는 집합 R의 개수는 HjK x¾0, y¾0, z¾0 또는 xÉ0, yÉ0, zÉ0
2á`-(2Þ`+2à`-2Ü`)=360 명제 q` Ú p는 참이다.
명제 p` Ú q는 거짓이다.
(반례) x=2, y=3, z=-1일 때,
187 207 xy+yz+zx=6-3-2=1¾0이지만
|x+y+z|=4, |x|+|y|+|z|=6이므로
8 이하의 자연수 a에 대하여 aÛ`이 8 이하가 되도록 하는 a의 값은
|x+y+z|+|x|+|y|+|z|
1, 2뿐이다.
그러므로 p는 q이기 위한 필요조건이지만 충분조건은 아니다.
따라서 명제 ‘a<A인 어떤 a에 대하여 aÛ`<U이다.’가 참이 되려면
따라서 a=3, b=1, c=1이므로 a+2b+3c=8이다.
A;{1, 2}+∆이어야 한다.
즉, 집합 A는 1, 2 중 적어도 하나를 원소로 가지므로 TIP 1
집합 A의 개수는 m=2¡`-2ß`=192이다.
q`:`(x-y)(y-z)(z-x)<0
한편, 부등식 bÛ`-4b-1<0을 만족시키는 8 이하의 자연수 b는
HjK x-y, y-z, z-x 중 하나만 음수 또는 모두 음수
1, 2, 3, 4이다.
x-y만 음수이면 y>z>x이다.
따라서 명제 ‘b<B인 모든 b에 대하여 bÛ`-4b-1<0이다.’가 참이
y-z만 음수이면 z>x>y이다.
되려면 집합 B는 집합 {1, 2, 3, 4}의 공집합이 아닌
z-x만 음수이면 x>y>z이다.
부분집합이어야 한다.
x-y, y-z, z-x가 모두 음수이면 x<y, y<z, z<x이어야
따라서 집합 B의 개수는 n=2Ý`-1=15이다.
하지만 이를 모두 만족시키는 경우는 존재하지 않는다.
∴ m+n=192+15=207

Ⅳ. 집합과 명제 37
TIP 2
x=-2, y=-3, z=1일 때,
191 ③

xy+yz+zx=6-3-2=1¾0이지만 "ÃnÛ`-1이 유리수라 가정하면


q
|x+y+z|=4, |x|+|y|+|z|=6이므로 "ÃnÛ`-1= p (p, q는 서로소인 자연수)로 놓을 수 있다.
|x+y+z|+|x|+|y|+|z|이다.
이 식의 양변을 제곱해서 정리하면
pÛ`(nÛ`-1)=qÛ`이다.
p는 qÛ`의 약수이고, p, q는 서로소인 자연수이므로
189 9
p=1이다. 따라서 nÛ`-1=qÛ`이므로
두 조건 p, q의 진리집합을 각각 P, Q라 하면 nÛ`= qÛ`+1 이다.
p가 q이기 위한 필요충분조건이므로 P=Q이다.
C C
자연수 k에 대하여
따라서 P =Q
C
Ú q=2k일 때
이때, p`:`xÛ`-4+0에서 P ={-2, 2}이므로
C (2k)Û`<nÛ`< (2k+1)Û` 인 자연수 n이 존재하지 않는다.
Q ={-2, 2}이다.
Û q=2k+1일 때
즉, x에 대한 방정식 xÝ`+(k-5)xÛ`-4k+4=0의
실근은 -2 또는 2뿐이어야 한다. (2k+1)Û` <nÛ`<(2k+2)Û`인 자연수 n이 존재하지 않는다.
xÝ`+(k-5)xÛ`-4k+4=0에서 q
Ú, Û에 의하여 "ÃnÛ`-1= (p, q는 서로소인 자연수)를
p
(xÛ`-4)(xÛ`+k-1)=0
만족시키는 자연수 n은 존재하지 않는다.
xÛ`=4 또는 xÛ`=1-k
따라서 "ÃnÛ`-1은 무리수이다.
1-k=4에서 k=-3이거나
㈎`:` f(q)=qÛ`+1, ㈏`:`g(k)=(2k+1)Û`
1-k<0에서 k>1이어야 한다. yy TIP
∴ f(2)+g(3)=5+49=54
따라서 이를 만족시키는 10보다 작은 정수 k는
-3, 2, 3, 4, 5, 6, 7, 8, 9로 9개이다.
TIP
xÛ`=1-k가 실근을 가지면 x=Ñ2뿐이어야 하므로 실근을 가질
192 풀이 참조

때는 1-k=4이어야 하고, 주어진 명제의 대우


아니면 허근을 가져야 하므로 1-k<0이어야 한다. ‘m>n을 만족시키는 두 자연수 m, n에 대하여 m과 n이 서로소가
아니면 m-n과 m+n은 서로소가 아니다.’를
증명해 보자.
190 ⑤ m과 n이 서로소가 아니므로 m과 n의 최대공약수를 k라 할 때, k는
1이 아닌 자연수이고,
ㄱ. xÛ`+yÛ`+zÛ`-xy-yz-zx
m=k_m0, n=k_n0이다.
1
= {(x-y)Û`+(y-z)Û`+(z-x)Û`}¾0 (단, m0, n0은 서로소인 두 자연수이고, m0>n0이다.)
2
(단, 등호는 x=y=z일 때 성립한다.) 즉, m-n=k(m0-n0), m+n=k(m0+n0)이므로
∴ xÛ`+yÛ`+zÛ`¾xy+yz+zx (거짓) m-n과 m+n은 모두 k의 배수이다.
ㄴ. xÜ`+yÜ`+zÜ`-3xyz 따라서 m-n과 m+n은 서로소가 아니다.
1 주어진 명제의 대우가 참이므로 주어진 명제가 참이다.
= (x+y+z){(x-y)Û`+(y-z)Û`+(z-x)Û`}
2
채점 요소 배점
¾0 (∵ x+y+z>0) (단, 등호는 x=y=z일 때 성립한다.)
주어진 명제의 대우 구하기 20%
∴ xÜ`+yÜ`+zÜ`¾3xyz (참)
대우에서 m과 n의 최대공약수를 이용하여 증명하기 80%
ㄷ. 산술평균과 기하평균의 관계에 의하여
x+y¾2'¶xy, y+z¾2'¶yz, z+x¾2'¶zx 이므로
(x+y)(y+z)(z+x)¾2'¶xy_2'¶yz_2'¶zx
=8xyz 193 풀이 참조
(단, 등호는 x=y=z일 때 성립한다.)
yÛ`-3x=2를 만족시키는 정수 x, y가 존재한다고 가정하자.
∴ (x+y)(y+z)(z+x)¾8xyz (참)
3x=yÛ`-2이고, y가 정수이므로 정수 k에 대하여
따라서 항상 성립하는 것은 ㄴ, ㄷ이다.
y=3k 또는 y=3k+1 또는 y=3k+2로 나타낼 수 있다.
참고
Ú y=3k일 때
ㄱ에서 부등식 xÛ`+yÛ`+zÛ`¾xy+yz+zx는
3x=(3k)Û`-2=9kÛ`-2=3(3kÛ`-1)+1
모든 실수 x, y, z에 대하여 항상 성립한다.
이므로 등식을 만족시키는 정수 x가 존재하지 않는다.

38 수학 <하>
Û y=3k+1일 때 직사각형 DEFG의 넓이가 xy이고, x>0, y>0이므로
3x=(3k+1)Û`-2=9kÛ`+6k-1 산술평균과 기하평균의 관계에 의하여
=3(3kÛ`+2k-1)+2 25 25 xy
10= x+y¾2¾Ð xy=5¾Ð
이므로 등식을 만족시키는 정수 x가 존재하지 않는다. 12 12 3
Ü y=3k+2일 때 25
(단, 등호는 x=y일 때 성립한다.) yy ㉡
12
3x=(3k+2)Û`-2=9kÛ`+12k+2
'¶xyÉ2'3, xyÉ12이므로 ⅠV
=3(3kÛ`+4k)+2
직사각형 DEFG의 넓이의 최댓값은 S=12이고 집
이므로 등식을 만족시키는 정수 x가 존재하지 않는다. 합
12 과
Ú~Ü에서 조건을 만족시키는 정수 x가 존재하지 않으므로 ㉠, ㉡을 연립하여 풀면 x= , y=5이므로
5

모순이다. 12 제
그때의 둘레의 길이는 l=2{ +5}
따라서 yÛ`-3x=2를 만족시키는 정수 x, y는 존재하지 않는다. 5
24 14
채점 요소 배점 ∴ l-S={ +10}-12=
5 5
주어진 명제의 결론 부정하기 20%
y=3k, y=3k+1, y=3k+2인 경우로 나누어 이때 정수 x가
80%
존재하지 않는 모순을 찾아 증명하기
196 ④

x, y는 실수이므로 xÛ`¾0, yÛ`¾0이다.


따라서 xÛ`+yÛ`+1>0이므로 산술평균과 기하평균의 관계에 의하여

194 ④ 2xÛ`+yÛ`-4x+
25
xÛ`+yÛ`+1
ADÓ=x, ABÓ=y라 하면 xy=1200 yy ㉠ 25
=xÛ`+yÛ`+1+ +xÛ`-4x-1
3 xÛ`+yÛ`+1
철수가 지불해야 하는 총비용은 x+2y에 비례한다.
2 25
¾2¾Ð(xÛ`+yÛ`+1)_ +xÛ`-4x-1 yy ㉠
산술평균과 기하평균의 관계에 의하여 xÛ`+yÛ`+1
3 3 =xÛ`-4x+9
x+2y¾2¾Ð x_2y=2'¶3xy=120
2 2 =(x-2)Û`+5¾5 yy ㉡
3 3 25
이때, 등호는 x=2y에서 y= x일 때 성립하므로 ㉠에서 등호는 xÛ`+yÛ`+1= 에서 xÛ`+yÛ`=4일 때
2 4 xÛ`+yÛ`+1
3 성립하고
㉠에 대입하면 xÛ`=1200, x=40
4
㉡에서 이 값은 x=2일 때 최솟값 5를 갖는다.
따라서 철수가 지불해야 하는 총비용이 최소일 때,
따라서 x=2일 때 xÛ`+yÛ`=4에서 y=0이므로
B지점에서 C지점까지 설치할 울타리의 길이는 40이다.
a=2, b=0, m=5이다.
∴ a+b+m=2+0+5=7

195 ①

그림과 같이 삼각형 ABC에 내접하는 직사각형을 DEFG라 하고


197 ②

DEÓ=x, DGÓ=y라 하자. 직육면체의 세 모서리의 길이를 각각


A a, b, c (a>0, b>0, c>0)라 하면
D 직육면체의 대각선의 길이는 "ÃaÛ`+bÛ`+cÛ` =6이므로
y
x G
aÛ`+bÛ`+cÛ`=36 yy ㉠
E 이 직육면체의 겉넓이는 2(ab+bc+ca)이다.
B C 이때,
F
1
세 삼각형 ABC, ADE, FGC가 서로 닮음이므로 aÛ`+bÛ`+cÛ`-(ab+bc+ca)= {(a-b)Û`+(b-c)Û`+(c-a)Û`}¾0
2
6`:`8=ADÓ`:`x=x`:`CGÓ에서 (단, 등호는 a=b=c일 때 성립한다.)
3 4 즉, ab+bc+caÉaÛ`+bÛ`+cÛ`=36
ADÓ= x, CGÓ= x이고
4 3
2(ab+bc+ca)É72이므로 M=72이다.
삼각형 ABC가 직각삼각형이므로 ACÓ=10에서
등호는 a=b=c일 때 성립하므로
3 4
ACÓ=ADÓ+DGÓ+CGÓ= x+y+ x=10 ㉠에 대입하면 3aÛ`=36, aÛ`=12, a=2'3
4 3
25 이때, 직육면체의 모든 모서리의 길이의 합은
∴ x+y=10 yy ㉠
12 k=4(a+b+c)=12a=24'3이다.

Ⅳ. 집합과 명제 39
M 72

k
=
24'3
='3 199 ③

다른 풀이 a+b=2이므로 b=2-a
직육면체의 세 모서리의 길이를 각각 이때, a, b는 음이 아닌 실수이므로 0ÉaÉ2이다.
a, b, c (a>0, b>0, c>0)라 하면 산술평균과 기하평균의 관계에 의하여
직육면체의 대각선의 길이는 "ÃaÛ`+bÛ`+cÛ` =6이므로 1 a 1 a
+ = +
a+2 b+2 a+2 (2-a)+2
aÛ`+bÛ`+cÛ`=36 yy ㉠
(4-a)+a(a+2)
이 직육면체의 겉넓이는 2(ab+bc+ca)이다. =
(a+2)(4-a)
코시-슈바르츠의 부등식에 의하여 aÛ`+a+4
=
(ab+bc+ca)Û`É(aÛ`+bÛ`+cÛ`)(bÛ`+cÛ`+aÛ`) -(a+2)(a-4)
=(aÛ`+bÛ`+cÛ`)Û`=36Û` (aÛ`-2a-8)+3a+12
=
-(aÛ`-2a-8)
0<ab+bc+caÉ36이므로 M=72이다.
3(a+4)
(단, 등호는 a=b=c일 때 성립한다.) =-1-
aÛ`-2a-8
이후 풀이는 본풀이와 같다. 3
=-1-
(a+4)(a-6)+16
a+4
3
198 ⑤ =-1-
a+4+
16
-10
a+4
삼각형 ABC의 넓이는 세 삼각형 PAB, PBC, PAC의 넓이의 합과
같으므로 3 1
¾-1- =
2_4-10 2
1 1
_6_8= (8a+6b+10c)에서 (단, 등호는 (a+4)Û`=16, 즉 a=0일 때 성립한다.) yy 참고
2 2
4a+3b+5c=24 yy ㉠ 1 a 1
따라서 + 의 최솟값은 이다.
a+2 b+2 2
코시-슈바르츠의 부등식에 의하여
참고
4 Û` 3 Û` 5 `Û
('4Œa Û`+'3Œb Û`+'5Œc Û`){¾ +¾ +¾ }
a b c b=2-a이므로 등호가 성립할 때 b=2이다.
¾(4+3+5)Û`
(단, 등호는 a=b=c일 때 성립한다.)
4 3 5
24_{ + + }¾12Û` (∵ ㉠)
a b c 200 4
4 3 5
+ + ¾6 a+b+c=5에서 a+b=5-c
a b c
4 3 5 aÛ`+bÛ`+cÛ`=11에서 aÛ`+bÛ`=11-cÛ`
따라서 구하는 + + 의 최솟값은 6이다.
a b c 코시-슈바르츠의 부등식에 의하여
다른 풀이 (1Û`+1Û`)(aÛ`+bÛ`)¾(a+b)Û`
2(11-cÛ`)¾(5-c)Û`
삼각형 ABC의 넓이는 세 삼각형 PAB, PBC, PAC의 넓이의 합과
3cÛ`-10c+3É0
같으므로
(3c-1)(c-3)É0
1 1
_6_8= (8a+6b+10c)에서 1
2 2 ÉcÉ3
3
4a+3b+5c=24 yy ㉠
1
산술평균과 기하평균의 관계에 의하여 따라서 M=3, m= 이므로
3
4 3 5
(4a+3b+5c){ + + } 1
a b c M+3m=3+3_ =4
3
12a 20a 12b 15b 20c 15c
=16+ + + +9+ + + +25 다른 풀이
b c a c a b
a b b c a c a+b+c=5 yy ㉠
=50+12{ + }+15{ + }+20{ + }
b a c b c a
aÛ`+bÛ`+cÛ`=11 yy ㉡
¾50+24+30+40=144 (∵ a>0, b>0, c>0)
(a+b+c)Û`=aÛ`+bÛ`+cÛ`+2(ab+bc+ca)
(단, 등호는 a=b=c일 때 성립한다.)
5Û`=11+2(ab+bc+ca)
4 3 5
이때, ㉠에 의하여 24_{ + + }¾144에서 ab+bc+ca=7 yy ㉢
a b c
4 3 5 aÛ`¾0, bÛ`¾0이므로
구하는 + + 의 최솟값은 6이다.
a b c 산술평균과 기하평균의 관계에 의하여

40 수학 <하>
aÛ`+bÛ`¾2|ab|
11-cÛ`¾2|7-bc-ca| (∵ ㉡, ㉢)
=2|7-c(a+b)|
=2|7-c(5-c)| (∵ ㉠)
=2|cÛ`-5c+7|
=2(cÛ`-5c+7) yy TIP
ⅠV
즉, 11-cÛ`¾2(cÛ`-5c+7)에서

3cÛ`-10c+3É0 합

1
(3c-1)(c-3)É0, ÉcÉ3
3 명

1
따라서 M=3, m= 이므로
3
1
M+3m=3+3_ =4
3
TIP
c에 대한 이차방정식 cÛ`-5c+7=0의 판별식을 D라 하면
D=(-5)Û`-4_1_7<0이므로
모든 실수 c에 대하여 항상 cÛ`-5c+7>0이다.
따라서 2|cÛ`-5c+7|=2(cÛ`-5c+7)이다.

참고

1
c뿐만 아니라 a, b의 최댓값과 최솟값도 각각 3, 이다.
3

Ⅳ. 집합과 명제 41
Ⅴ 함수와 그래프 따라서 선지 중 집합 X에서 집합 Y로의 함수가 아닌 것은 ⑤이다.

함수
204 ②

주어진 함수 f 의 그래프는
{(1, 3), (2, 1), (3, 1), (4, 5), (5, 3)}을 나타낸다.
함수 f 의 정의역은 X={1, 2, 3, 4, 5}, 치역은 {1, 3, 5}이므로
201 ②
a=1+2+3+4+5, b=1+3+5
집합 X에서 집합 Y로의 대응이 함수이기 위해선 ∴ a-b=2+4=6
집합 X의 각 원소에 집합 Y의 원소가 오직 하나씩 대응해야 한다.
ㄱ. 집합 X의 원소 3은 집합 Y의 원소에 대응하지 않으므로
함수가 아니다.
ㄴ. 1, 2는 c에 대응되고, 3, 4는 a에 대응되어
집합 X의 각 원소에 집합 Y의 원소가 오직 하나씩 대응하므로
함수이다.
205 ③

ㄷ. 집합 X의 원소 2는 집합 Y의 원소 a와 c에 대응되므로 함수가 집합 X의 각 원소에 집합 Y의 원소가 오직 하나씩 대응할 때,


아니다. 이 대응은 집합 X에서 집합 Y로의 함수이다.
따라서 함수인 것은 ㄴ이다. x<X, y<Y라 하면
① y y

3
202 ⑤ 2
1 O a x
① 집합 X의 각 원소에 집합 Y의 원소가 오직 하나씩 대응하므로 O 1 2 3 4 x
이 대응은 집합 X에서 집합 Y로의 함수이다. (참)
x=2일y때, 대응되는 y의 값이 1과
y 3으로 2개이므로 함수가 아니다.
② 집합 X의 원소 1에 집합 Y의 원소 0이 대응하므로 f(1)=0이다.
y y

(참)
3
③ 함수 f `:`X`2Ú`Y의 정의역은 X이다. (참) 2 y O x -a Oy a x
④ 함수 f `:`X`2Ú`Y의 공역은 Y이다. (참) 1 O a x
3
⑤ f(1)=0, f(2)=2, f(3)=0이므로 O 1 2 3 4 x 2
1 y O a x
함수 f 의 함숫값 전체의 집합인 함수 f 의 치역은 {0, 2}이다.
y 주어진 그래프를 x=a라 하면 x=a일 때 대응되는 y의 값이
1 y2 3 4 x
O
(거짓)
2개 이상이므로 함수가 아니다.
따라서 선지 중 옳지 않은 것은 ⑤이다.
③ Oy x y
y O x -a Oy a x

3
y x -a y a x
203 2
1 y
⑤ O
O a x
O
3
① f f f② f 1 2 3 4 x 2
X Y X X Y Y X Y O
a x
각1 원소yx에 대응되는 y의 값이 O오직 하나씩이므로 함수이다.
-1 0 -1 -1 0 0 -1 0
Oy x ④ O 1 2y 3 4 x
0 1 0 0 1 1 0 1
1 2 1 1 2 2 1 2 y y
O x
O x -a O a x

③ f f f④ f
X Y X X Y Y X Y x -a a x
O O
-1 0 -1 -1 0 0 -1 0 -a<x<a일 때 대응되는 y의 값이 각각 2개씩이므로 함수가
y
0 1 0 0 1 1 0 1 아니다.
1 2 1 1 2 2 1 2 ⑤ y
O x

⑤ f Y X f
X Y O x
-1 0 -1 0
0 1 0 1
x>0일 때 대응되는 y의 값이 각각 2개씩이므로 함수가 아니다.
1 2 1 2
따라서 선지 중 함수의 그래프가 될 수 있는 것은 ③이다.

42 수학 <하>
TIP ⑵ 공역이 R이므로 치역이 R인 그래프는 ㄱ, ㄴ, ㄹ, ㅂ이다.
함수이려면 정의역의 모든 원소에 공역의 원소가 하나씩 ⑶ 함수 f `:`R`2Ú`R가 일대일대응이려면 일대일함수이고 치역과
대응되어야 하므로 함수의 그래프가 되려면 정의역의 모든 원소 공역이 같아야 한다. yy TIP3
k에 대하여 그래프가 직선 x=k와 항상 단 한 개의 점에서만 따라서 ⑴, ⑵에 의하여 일대일대응은
만나야 한다. 따라서 y축과 평행한 직선이 그래프와 두 개 ㄱ, ㄹ, ㅂ이다.
이상의 점에서 만나는 경우가 있으면 그 그래프는 함수의 ⑷ 함수 f `:`R`2Ú`R가 항등함수이려면 x<R에 대하여
그래프가 아니다. f(x)=x이므로 ㄱ이다.
⑸ 함수 f `:`R`2Ú`R가 상수함수이려면 x<R에 대하여
f(x)=c (c는 상수)이므로 ㅁ이다.
206 ④
TIP1
정의역이 {-1, 0, 1}인 두 함수 f, g 가 서로 같은 함수이려면 명제 ‘xÁ+xª이면 f(xÁ)+ f(xª)’의 대우는
f(-1)=g(-1), f(0)=g(0), f(1)=g(1)을 ‘f(xÁ)= f(xª)이면 xÁ=xª’이다.
모두 만족시켜야 한다.
ㄱ. f(-1)=g(-1)=1, f(0)=g(0)=0, f(1)=g(1)=1이므로 TIP2
f=g이다. ‘f(xÁ)= f(xª)이면 xÁ=xª’를 만족시키려면 함수 f(x)가
ㄴ. f(1)=1, g(1)=3에서 f(1)+g(1)이므로 f+g이다. 일대일함수이어야 한다. 일대일함수의 그래프가 되려면 치역의

ㄷ. f(-1)=g(-1)=2, f(0)=g(0)=1, f(1)=g(1)=0이므로 함
모든 원소 k에 대하여 x축과 평행한 직선 y=k와 함수 수
f=g이다. y= f(x)의 그래프가 오직 한 점에서만 만나야 한다. 와
따라서 두 함수 f, g 가 서로 같은 함수인 것은 ㄱ, ㄷ이다. 그

TIP3 프

207 ④ 일대일대응이려면 일대일함수이면서 치역이 공역과 같아야


하므로 공역의 모든 원소에 대응되는 정의역의 원소가 단
정의역이 {1, 2}인 두 함수 f(x)=ax-1, g(x)=xÛ`+b가
하나씩 존재해야 한다.
서로 같으려면 f(1)=g(1), f(2)=g(2)이어야 한다.
따라서 일대일대응의 그래프는 공역의 모든 원소 k에 대하여
f(1)=a-1, g(1)=1+b이므로
그래프가 직선 y=k와 오직 한 점에서만 만난다.
a-1=1+b에서 a-b=2 yy ㉠
f(2)=2a-1, g(2)=4+b이므로
2a-1=4+b에서 2a-b=5 yy ㉡
㉠, ㉡을 연립하여 풀면 a=3, b=1 210 ②

∴ a+b=3+1=4
함수의 그래프를 그려서 x축과 평행한 직선과 만나는 교점의 개수가
오직 1인 그래프를 찾으면 된다.
208 ② ① y y y② y

치역이 {2, 4, 6}이어야 하므로 정의역의 4개의 원소가 y=-2x+3


y=-2x+3
1 1 y=1 y=1
2개/1개/1개로 나뉘어서 2, 4, 6에 하나씩 대응되면 된다.
따라서 f(1)+ f(2)+ f(3)의 값이 최대이려면 1, 2, 3 중 2개가 6에 O O x x O O x x

대응되고, 나머지 하나는 4에 대응되고, 4는 2에 대응되면 되므로 y y y y


③ ④
f(1)+ f(2)+ f(3)의 최댓값은 6+6+4=16이다. y=|x-4|
y=|x-4|

y=x@-2x
y=x@-2x
209 풀이 참조
O 2O 2x x O O 4 4x x

⑴ 함수 f `:`R`2Ú`R가 일대일함수이려면 ⑤ y
y=[x]
정의역 R의 임의의 두 원소 xÁ, xª에 대하여 3
‘xÁ+xª이면 f(xÁ)+ f(xª)’ 2
1
즉, ‘f(xÁ)= f(xª)이면 xÁ=xª’ yy TIP1 -3-2 -1
O 1 2 3 4 x
를 만족시켜야 한다. -1
따라서 x축에 평행한 직선과 주어진 함수의 그래프의 교점의 -2
-3
개수가 항상 1인 것을 찾으면 yy TIP2
ㄱ, ㄷ, ㄹ, ㅂ이다. 따라서 선지 중 일대일함수인 것은 ②이다.

Ⅴ. 함수와 그래프 43
다른 풀이 일대일함수이고, ㉠에 의하여 일대일대응이다. (참) yy TIP
실수 전체의 집합 R에 대하여 함수 f `:`R`2Ú`R가 따라서 선지 중 옳지 않은 것은 ④이다.
일대일함수이려면 정의역 R의 임의의 두 원소 xÁ, xª에 대하여
TIP
xÁ+xª이면 f(xÁ)+ f(xª), 즉 f(xÁ)= f(xª)이면 xÁ=xª를
유한집합 X에 대하여 함수 f `:`X`2Ú`Y의 치역을 Z라 하면
만족시켜야 한다.
n(Z)Én(X)이다.
① f(x)=1에서 1+2이지만 f(1)= f(2)=1이므로
이때, 함수 f 가 일대일함수이면 n(Z)=n(X)이다.
일대일함수가 아니다.
② f(x)=-2x+3에서 f(xÁ)= f(xª)이면
참고
f(xÁ)- f(xª)=(-2xÁ+3)-(-2xª+3)
=-2(xÁ-xª)=0 명제 ‘일대일함수이면 일대일대응이다.’는 거짓이고,

에서 xÁ=xª이므로 일대일함수이다. 그 역인 ‘일대일대응이면 일대일함수이다.’는 참이다.

③ f(x)=xÛ`-2x에서 0+2이지만 f(0)= f(2)=0이므로 즉, 일대일함수는 일대일대응이기 위한 필요조건이지만

일대일함수가 아니다. 충분조건은 아니다.


일대일함수 (치역)=(공역)
④ f(x)=|x-4|에서 3+5이지만 f(3)= f(5)=1이므로
일대일함수가 아니다.
1 1
⑤ f(x)=[x]에서 0+ 이지만 f(0)= f { }=0이므로
2 2
일대일대응
일대일함수가 아니다.
따라서 선지 중 일대일함수인 것은 ②이다.

213 ④

211 4
함수 f `:`R`2Ú`R가 일대일대응이려면 일대일함수이고 공역과
f(1)- f(3)=4를 만족시키려면 치역이 같아야 한다.
f(1)=5, f(3)=1 또는 f(1)=7, f(3)=3이어야 한다. á 2x (x>0)
함수 f(x)=Ò 에서
이때, f(2)=5이고 f 가 일대일대응이므로 f(1)+5이다. » (aÛ`-5a)x (xÉ0)
따라서 f(1)=7, f(3)=3이고, 이때 f(4)=1이다. x>0일 때, 함수 y=2x의 그래프가 다음과 같고,
f xÉ0일 때, 함수 y=(aÛ`-5a)x의 그래프는 원점을 지나는 직선이다.
X Y
y
1 1

2 3

3 5

4 7 x
O

∴ f(3)+ f(4)=3+1=4

이때, 함수 f(x)가 일대일대응이 되려면 xÉ0에서 직선


y=(aÛ`-5a)x의 기울기가 양수이어야 하므로 yy TIP
212 ④
aÛ`-5a=a(a-5)>0
① 함수 f 가 일대일함수일 때, 공역과 치역이 같으면 ∴ a<0 또는 a>5
함수 f 는 일대일대응이다. (참)
TIP
② 함수 f 가 상수함수이면 치역 { f(x)|x<X}의 원소의
직선 y=(aÛ`-5a)x의 기울기가 0인 경우(❶)와 기울기가 음수
개수는 1이다. (참)
인 경우(❷)에 함수 y= f(x)의 그래프가 각각 다음과 같으므로
③ 함수 f 가 항등함수이면 f(x)=x이므로 함수 f의
일대일함수가 아니다. 따라서 일대일대응도 아니다.
치역은 정의역과 같다. (참)
❶ y y ❷ y y
④ 함수 f 의 정의역의 임의의 두 원소 xÁ, xª에 대하여
f(xÁ)= f(xª)일 때 xÁ=xª이면 함수 f 는 일대일함수이다.
이때, 함수 f 의 치역과 공역이 같지 않으면 함수 f 는
일대일대응이 아니다. (거짓) yy 참고
⑤ n(X)=n(Y)이고, (공역)=(치역) yy ㉠ O O x x O O x x

이면 정의역과 치역의 원소의 개수가 같으므로 함수 f 는

44 수학 <하>
정의역의 서로 다른 원소에 공역의 서로 다른 원소가 대응되어야
214 ⑤
한다.
함수 y=ax+b의 그래프는 a<0이므로 기울기가 음수인 직선이다. 조건 ㈏에 의하여 f(1)+3, f(3)=2이므로
함수 f(x)=ax+b가 X에서 Y로의 일대일대응이려면 치역이 공역 f(1)의 값은 1, 2, 3, 4, 5 중 2, 3을 제외한 3가지,
Y={y|-6ÉyÉ10}과 같아야 하므로 함수 y= f(x)의 그래프는 f(2)의 값은 1, 2, 3, 4, 5 중 f(1)의 값과 2를 제외한 3가지,
다음과 같다. f(4)의 값은 1, 2, 3, 4, 5 중 f(1), f(2)의 값과 2를 제외한 2가지
y 따라서 함수 f의 개수는 3_3_2=18이다.
10

-3 O
5
x 217 ③

f(2)=5이고, g(5)=3이므로
-6
(g ç f)(2)=g( f(2))=g(5)=3
즉, f(-3)=10, f(5)=-6이면 되므로 g(5)=3이고, f(3)=2이므로
-3a+b=10, 5a+b=-6 ( f ç g)(5)= f(g(5))= f(3)=2
두 식을 연립하여 풀면 a=-2, b=4 ∴ (g ç f)(2)+( f ç g)(5)=3+2=5 Ⅴ
∴ a+b=2 함


215 ⑴ 64 ⑵ 24 ⑶ 4 ⑷ 1 ⑸ 6 218 ② 래

á 2-x (x¾0)
⑴ 함수가 되려면 모든 정의역의 원소에 치역의 원소가 하나씩 f(x)=Ò 에서
» xÛ`+1 (x<0)
대응되어야 한다. X의 원소 1, 2, 3에 하나씩 대응될 수 있는
f(4)=2-4=-2, f(-2)=(-2)Û`+1=5,
Y의 원소가 각각 4가지이므로
f(5)=2-5=-3이므로
구하는 X에서 Y로의 함수의 개수는
( f ç f ç f)(4)= f( f( f(4)))
4_4_4=64이다.
= f( f(-2))= f(5)=-3
⑵ 일대일함수이려면 정의역의 서로 다른 원소에 공역의 서로 다른
원소가 대응되어야 하므로
1에 대응될 수 있는 수는 4가지,
2에 대응될 수 있는 수는 1에 대응되는 수를 제외한 3가지,
3에 대응될 수 있는 수는 1, 2에 대응되는 수를 제외한 2가지이다.
219 ⑴ 22 ⑵ 2 ⑶ 10

따라서 구하는 X에서 Y로의 일대일함수의 개수는 ⑴ f(2)=5이고, g(5)=22이므로


4_3_2=24이다. (g ç f)(2)=g( f(2))=g(5)=22
⑶ X에서 Y로의 상수함수를 f 라 할 때, 가능한 함수 f 는 ⑵ g(2)=1이고, f(1)=2이므로
f(x)=1 또는 f(x)=2 또는 f(x)=3 또는 f(x)=4로 ( f ç g)(2)= f(g(2))= f(1)=2
4개이다. ⑶ h(-1)=3, g(3)=6, f(6)=17이므로
⑷ X에서 X로의 함수 f 가 항등함수일 때, f(x)=x로 함수 f 는 (( f ç g) ç h)(-1)=( f ç g)(h(-1))= f(g(h(-1)))
1개이다. = f(g(3))= f(6)=17
⑸ X에서 X로의 함수가 일대일대응이려면 정의역의 서로 다른 h(1)=1, g(1)=-2, f(-2)=-7이므로
원소에 공역의 서로 다른 원소가 하나씩 대응되어야 하므로 ( f ç (g ç h))(1)= f((g ç h)(1))= f(g(h(1)))
1에 대응될 수 있는 수는 3가지, = f(g(1))= f(-2)=-7
2에 대응될 수 있는 수는 1에 대응되는 수를 제외한 2가지, ∴ (( f ç g) ç h)(-1)+( f ç (g ç h))(1)
3에 대응될 수 있는 수는 1, 2에 대응되는 수를 제외한 1가지이다. =17+(-7)=10
따라서 구하는 X에서 X로의 일대일대응의 개수는
참고
3_2_1=6
❶ 일반적으로 두 함수 f, g 에 대하여 g ç f+ f ç g 이다.
즉, 함수의 합성에 대하여 교환법칙이 성립하지 않는다.
❷ 세 함수 f, g, h에 대하여 ( f ç g) ç h= f ç (g ç h)이다.
216 ② 즉, 함수의 합성에 대하여 결합법칙이 성립하므로
f ç g ç h로 나타낼 수 있다.
조건 ㈎에 의하여 함수 f 는 일대일함수이므로

Ⅴ. 함수와 그래프 45
g f
220 ② X X X

( f ç g)(x)= f(g(x)) 1 1 1

=3(-x+k)+2=-3x+3k+2 2 2 2
(g ç f)(x)=g( f(x)) 3 3 3
=-(3x+2)+k=-3x-2+k
( f ç g)(x)=(g ç f)(x)에서 ( f ç g)(2)= f(g(2))= f(2)=3, f(3)=1이므로
-3x+3k+2=-3x-2+k ( f ç g)(2)+ f(3)=3+1=4
3k+2=-2+k, 2k=-4
∴ k=-2
다른 풀이
223 ⑤
f ç g=g ç f이면 모든 실수 a에 대하여
어떤 함수의 역함수가 존재하기 위한 필요충분조건은 그 함수가
( f ç g)(a)=(g ç f)(a)이다.
일대일대응인 것이다.
따라서 a=0일 때, ( f ç g)(0)=(g ç f)(0)임을 이용하면
ㄱ. f(x)=|x|+1
( f ç g)(0)= f(g(0))= f(k)=3k+2
y
(g ç f)(0)=g( f(0))=g(2)=-2+k
에서 3k+2=-2+k, 2k=-4 y=f{x}
∴ k=-2
1
O x

함수 f(x)는 일대일대응이 아니므로 역함수가 존재하지 않는다.


221 ④
á 1-x (x¾0)
( f ç g)(5)=8에서 f(g(5))=8이고, ㄴ. g(x)=Ò
» 1-2x (x<0)
함수 f 가 일대일대응이고, f(6)=8이므로 y
g(5)=6이다. yy TIP
한편, ( f ç g)(3)= f(g(3))= f(4)=9이다.
∴ f(4)+g(5)=9+6=15
1
O x
TIP y=g{x}
함수 f 가 일대일대응이면 함수 f 는 일대일함수이므로
함수 g(x)는 일대일대응이므로 역함수가 존재한다.
f(a)= f(b)이면 a=b이다.
á 1+x (x>0)
즉, f(a)=k, f(b)=k이면 a=b이다. ㄷ. h(x)=Ò
» 1-xÛ` (xÉ0)
따라서 f(g(5))=8, f(6)=8에서 g(5)=6이다.
y

y=h{x}
1

222 ③ O x

f(2)=3, g(3)=1을 만족시키는 두 함수 f, g 에 대하여


함수 f ç g 를 나타내면 다음과 같다. 함수 h(x)는 일대일대응이므로 역함수가 존재한다.
g f 따라서 역함수가 존재하는 것은 ㄴ, ㄷ이다.
X X X

1 1 1

2 2 2
224 ①
3 3 3
f(1)=5이므로 f ÑÚ`(5)=1이다. yy TIP
이때, ( f ç g)(1)=1을 만족시키기 위해서 f(3)=4에서 f ÑÚ`(4)=3이므로
g(1)=1이면 g 가 일대일대응을 만족시키지 않게 되고, (gÑÚ` ç f ÑÚ`)(4)=gÑÚ`( f ÑÚ`(4))=gÑÚ`(3)이고,
g(1)=2이면 ( f ç g)(1)=3이 되므로 g(4)=3에서 gÑÚ`(3)=4이므로
g(1)=3, f(3)=1이어야 한다. (gÑÚ` ç f ÑÚ`)(4)=4이다.
따라서 조건을 만족시키는 함수 f ç g 는 다음과 같다. ∴ f ÑÚ`(5)+(gÑÚ` ç f ÑÚ`)(4)=1+4=5

46 수학 <하>
TIP TIP
주어진 두 함수 f, g 는 모두 일대일대응이므로 모두 역함수가 <역함수의 성질>
존재한다. 역함수가 존재하는 두 함수 f `:`X`2Ú`Y, g`:`Y`2Ú`Z와
함수 f `:`X`2Ú`Y에 대하여 f(a)=b일 때, 역함수 두 항등함수 Iþ`:`X`2Ú`X, Iç`:`Y`2Ú`Y에 대하여
f ÑÚ``:`Y`2Ú`X에서 f ÑÚ`(b)=a이므로 주어진 함수 f에서 ❶ ( f ÑÚ`)ÑÚ`= f
화살표를 반대 방향으로 보내면 f ÑÚ`가 된다. ❷ f ÑÚ` ç f=Iþ, f ç f ÑÚ`=Iç
따라서 주어진 함수 f 의 역함수 f ÑÚ`는 다음과 같다. X=Y이면 f ç f ÑÚ`= f ÑÚ` ç f=Iþ이고,
f —! X+Y이면 f ç f ÑÚ`+ f ÑÚ` ç f 임에 주의하자.
X Y
❸ (g ç f)ÑÚ`= f ÑÚ` ç gÑÚ`
1 4
일반적으로 (g ç f)ÑÚ`+gÑÚ` ç f ÑÚ`임에 주의하자.
2 5

3 6

226 ④

Ú f ÑÚ`(-2)=k라 하면 f(k)=-2이다.
주어진 그래프에 의하여 f(0)=-2이므로 k=0
∴ f ÑÚ`(-2)=0


Û ( f ç gÑÚ`)ÑÚ`(0)=((gÑÚ`)ÑÚ` ç f ÑÚ`)(0) 수

225 ⑴ 2 ⑵ 4 ⑶ 3 ⑷ 2 ⑸ -8 =(g ç f ÑÚ`)(0)

=g( f ÑÚ`(0)) yy ㉠ 래
⑴ f ÑÚ`(4)=a라 하면 f(a)=4에서 프
f ÑÚ`(0)=m이라 하면 f(m)=0이다.
3a-2=4, a=2
주어진 그래프에 의하여 f(-1)=0이므로 m=-1
∴ f ÑÚ`(4)=2
㉠에서 g( f ÑÚ`(0))=g(-1)=1
⑵ ( f ÑÚ`)ÑÚ`= f이므로
∴ f ÑÚ`(-2)+( f ç gÑÚ`)ÑÚ`(0)=0+1=1
( f ÑÚ`)ÑÚ`(2)= f(2)=4
⑶ f ç f ÑÚ`=I (I는 항등함수)이므로
( f ç f ÑÚ`)(3)=I(3)=3
⑷ ( f ç g)ÑÚ`=gÑÚ` ç f ÑÚ`이므로
227 ①

( f ç g)ÑÚ`(1)=(gÑÚ` ç f ÑÚ`)(1) ( f ÑÚ` ç g)ÑÚ` ç h= f에서


=gÑÚ`( f ÑÚ`(1)) yy ㉠ ( f ÑÚ` ç g) ç ( f ÑÚ` ç g)ÑÚ` ç h=( f ÑÚ` ç g) ç f
f ÑÚ`(1)=k라 하면 f(k)=1에서 I ç h= f ÑÚ` ç g ç f (I는 항등함수)
3k-2=1, k=1이므로 ∴ h= f ÑÚ` ç g ç f
㉠에서 gÑÚ`( f ÑÚ`(1))=gÑÚ`(1)=m이라 하면 g(m)=1에서 h(-2)=( f ÑÚ` ç g ç f)(-2)= f ÑÚ`(g( f(-2)))
-m+3=1, m=2 = f ÑÚ`(g(5))= f ÑÚ`(11)
∴ ( f ç g)ÑÚ`(1)=2 f ÑÚ`(11)=k라 하면 f(k)=11에서
⑸ ( f ÑÚ` ç g)ÑÚ`=gÑÚ` ç ( f ÑÚ`)ÑÚ`=gÑÚ` ç f이므로 -2k+1=11, k=-5
( f ç ( f ÑÚ` ç g)ÑÚ` ç f ÑÚ`)(5) ∴ h(-2)=-5
=( f ç gÑÚ` ç f ç f ÑÚ`)(5) 다른 풀이

=( f ç gÑÚ`)(5) (∵ f ç f ÑÚ`=I, I는 항등함수) ( f ÑÚ` ç g)ÑÚ` ç h= f에서


= f(gÑÚ`(5)) yy ㉡ (( f ÑÚ` ç g)ÑÚ` ç h)(-2)= f(-2)
gÑÚ`(5)=t라 하면 g(t)=5에서 ( f ÑÚ` ç g)ÑÚ`(h(-2))=5
-t+3=5, t=-2 ( f ÑÚ` ç g)(5)=h(-2)
㉡에서 f(gÑÚ`(5))= f(-2)=-8 ( f ÑÚ` ç g)(5)= f ÑÚ`(g(5))= f ÑÚ`(11)
∴ ( f ç ( f ÑÚ` ç g)ÑÚ` ç f ÑÚ`)(5)=-8 이때, f ÑÚ`(11)=k라 하면 f(k)=11에서
다른 풀이 -2k+1=11, k=-5
⑷ ( f ç g)(x)= f(-x+3) ∴ h(-2)=-5
=3(-x+3)-2=-3x+7
이므로 ( f ç g)ÑÚ`(1)=k라 하면 ( f ç g)(k)=1에서
-3k+7=1, k=2 228 ③

∴ ( f ç g)ÑÚ`(1)=2 함수 y=g(x)의 그래프가 다음과 같다.

Ⅴ. 함수와 그래프 47
y 다른 풀이 1
y=g{x}
x-5
함수 y= 의 역함수가 y=ax+b이다.
5
2
x-5
함수 y= 의 역함수를 구하면 다음과 같다.
3 2
x-5
3 y= 를 x에 대하여 풀면 x=2y+5이고,
- 2
2
O 1 x x와 y를 서로 바꾸면 y=2x+5이다.
Ú gÑÚ`(2)=k라 하면 역함수가 y=2x+5이므로 a=2, b=5
∴ ab=10
g(k)=2, k<1이므로 yy TIP
1 다른 풀이 2
2k+3=2, k=-
2
x-5
1 f(x)= 라 하면 f ÑÚ`(x)=ax+b이다.
f(gÑÚ`(2))= f {- }=7 2
2
f(5)=0, f(1)=-2이므로 f ÑÚ`(0)=5, f ÑÚ`(-2)=1이다.
Û f ÑÚ`(g(2))= f ÑÚ`(8)=m이라 하면
f ÑÚ`(0)=b=5, f ÑÚ`(-2)=-2a+b=1에서 a=2
f(m)=8이므로 -2m+6=8, m=-1
∴ ab=10
∴ f(gÑÚ`(2))+ f ÑÚ`(g(2))=7+(-1)=6

TIP
함수 y=g(x)의 그래프에서 g(x)É5일 때 xÉ1이고, 231 ⑴ h(x)=-2x+5 ⑵ h(x)=-2x+1

g(x)>5일 때 x>1이다. ⑴ ( f ç h)(x)= f(h(x))=-h(x)+2이므로


( f ç h)(x)=g(x)에서
-h(x)+2=2x-3
∴ h(x)=-2x+5
229 7
⑵ (h ç f)(x)=h( f(x))=h(-x+2)이므로
f ÑÚ`(5)=k라 하면 f(k)=5 (h ç f)(x)=g(x)에서
4x+1 h(-x+2)=2x-3
f{ }=-2x+15에서
3
-x+2=t라 하면 x=2-t이므로
4_5+1
-2x+15=5일 때, x=5이므로 k= =7 h(t)=2(2-t)-3=-2t+1
3
∴ f ÑÚ`(5)=7 ∴ h(x)=-2x+1

다른 풀이 다른 풀이

4x+1 3t-1 함수 f(x)는 역함수가 존재하므로 f(x)=-x+2의 역함수를


=t로 치환하면 x= 이므로
3 4 구하면 다음과 같다.
3t-1 3 31 y=-x+2를 x에 대하여 풀면 x=-y+2
f(t)=-2_ +15=- t+
4 2 2 x와 y를 서로 바꾸면 y=-x+2이므로
f ÑÚ`(5)=k라 하면 f(k)=5이므로 f ÑÚ`(x)=-x+2 yy ㉠
3 31 ⑴ f ç h=g에서 h= f ÑÚ` ç g 이다.
- k+ =5, k=7
2 2
∴ h(x)=( f ÑÚ` ç g)(x)= f ÑÚ`(g(x))
∴ f ÑÚ`(5)=7
= f ÑÚ`(2x-3)=-(2x-3)+2 (∵ ㉠)
=-2x+5
⑵ h ç f=g에서 h=g ç f ÑÚ`이다.
230 ⑤
∴ h(x)=(g ç f ÑÚ`)(x)=g( f ÑÚ`(x))
함수 y=ax+b의 역함수를 구하면 다음과 같다. =g(-x+2)=2(-x+2)-3 (∵ ㉠)
y-b =-2x+1
y=ax+b를 x에 대하여 풀면 x= 이고,
a
x-b
x와 y를 서로 바꾸면 y= 이므로
a

역함수는 y=
x-b
이다.
232 hÑÚ`(x)=3x-13
a
두 함수 f(x), g(x)는 역함수가 존재하므로
x-5
이 함수가 y= 와 같으므로 a=2, b=5 g=h ç f 에서 h=g ç f ÑÚ`이고,
2
∴ ab=10 hÑÚ`=(g ç f ÑÚ`)ÑÚ`= f ç gÑÚ`이다.

48 수학 <하>
함수 g(x)=x+4의 역함수를 구하면 다음과 같다.
y=x+4를 x에 대하여 풀면 x=y-4이고,
235 ⑤

x와 y를 서로 바꾸면 y=x-4이므로 g= f ÑÚ`이므로


gÑÚ`(x)=x-4이다. ( f ÑÚ` ç g)(c)+( f ç g)ÑÚ`(b)
∴ hÑÚ`(x)=( f ç gÑÚ`)(x)= f(gÑÚ`(x)) =( f ÑÚ` ç f ÑÚ`)(c)+( f ç f ÑÚ`)ÑÚ`(b)
= f(x-4)=3(x-4)-1 이때, f ç f ÑÚ`=I (I는 항등함수)이므로
=3x-13 ( f ç f ÑÚ`)ÑÚ`(b)=IÑÚ`(b)=I(b)=b이다.
다른 풀이 한편, 주어진 그래프는 다음과 같다.
y y=x
g(x)=(h ç f)(x)에서 x+4=h(3x-1) y=f{x}

t+1
3x-1=t로 치환하면 x= 이므로 d
3
t+1 t+13 c
h(t)= +4= 이다.
3 3
b
x+13 a
h(x)= 의 역함수를 구하면 다음과 같다.
3 x
O a b c d e
x+13
y= 을 x에 대하여 풀면 x=3y-13이고,
3 Ⅴ
f(d)=c이므로 f ÑÚ`(c)=d이고,
x와 y를 서로 바꾸면 y=3x-13 함
f(e)=d이므로 f ÑÚ`(d)=e이다.

∴ hÑÚ`(x)=3x-13 따라서 ( f ÑÚ` ç f ÑÚ`)(c)= f ÑÚ`( f ÑÚ`(c))= f ÑÚ`(d)=e이다. 와
∴ ( f ÑÚ` ç g)(c)+( f ç g)ÑÚ`(b)=e+b 그

233 ② 236 ④

gÑÚ` ç f ÑÚ`=( f ç g)ÑÚ`이므로 ( f ç g)ÑÚ`(bx-6)=x에서 3


함수 y= x+4의 그래프와 그 역함수의 그래프는 직선 y=x에
2
( f ç g)(x)=bx-6이다.
대하여 대칭이고, 이때 두 함수의 교점은 직선 y=x 위에
한편, f(x)=2x-a, g(x)=-x+2a에서
존재한다. yy 참고
( f ç g)(x)= f(g(x))= f(-x+2a)
3
=2(-x+2a)-a=-2x+3a 즉, 구하는 교점의 좌표는 직선 y= x+4와 직선 y=x의 교점의
2
이므로 -2x+3a=bx-6 3
좌표와 같고, 방정식 x+4=x에서 x=-8이므로 구하는 교점의
a=-2, b=-2 2
∴ a+b=-4 좌표는 (-8, -8)이다.
a=-8, b=-8
∴ a+b=-16

234 ③ 참고

함수와 그 역함수의 그래프의 교점이 항상 직선 y=x 위에 있는


함수 f(x)의 역함수가 존재하지 않으려면 함수 f(x)가
것은 아니다.
일대일대응이 아니어야 한다.
함수 y= f(x)의 그래프가 직선 y=x에 대하여 대칭인 두 점
á (k+3)x+3 (x<0)
f(x)=Ò 에서 (a, b), (b, a)`(a+b)를 지나면 f(a)=b, f(b)=a에서
» (2k-7)x+3 (x¾0)
f ÑÚ`(b)=a, f ÑÚ`(a)=b이므로 역함수 y= f ÑÚ`(x)의 그래프도
y=(k+3)x+3은 점 (0, 3)을 지나고 기울기가 k+3인 직선이고,
두 점 (a, b), (b, a)를 지난다.
y=(2k-7)x+3은 점 (0, 3)을 지나고 기울기가 2k-7인
즉, 두 함수 y= f(x), y= f ÑÚ`(x)의 그래프가 직선 y=x 위에
직선이다.
있지 않은 교점을 갖는다.
함수 f(x)가 일대일대응이려면 두 직선의 기울기의 부호가 모두
단, 함수와 그 역함수의 그래프가 직선 y=x 위에 있지 않은
양수이거나 모두 음수이어야 하므로
교점을 갖는 경우 함수 y= f(x)가 일대일대응이면서 직선
두 직선의 기울기의 곱이 양수이어야 한다.
y=x에 대하여 대칭인 두 점 (a, b), (b, a)(a+b)를 모두
7
즉, (k+3)(2k-7)>0이므로 k<-3 또는 k> 지나므로 함수 f(x)가 x의 값이 증가할 때 y의 값이 증가하면
2
따라서 함수 f(x)가 일대일대응이 아니도록 하는 k의 값의 범위가 이러한 경우는 발생하지 않는다.

7 따라서 함수의 그래프가 x의 값이 증가할 때 y의 값도 증가하는


-3ÉkÉ 이므로 구하는 모든 정수 k의 값의 합은 형태이면 그 역함수의 그래프와의 교점은 함수의 그래프와
2
(-3)+(-2)+(-1)+0+1+2+3=0 직선 y=x의 교점과 같다.

Ⅴ. 함수와 그래프 49
하지만 함수의 그래프가 x의 값이 증가할 때 y의 값도 증가하는 함수 f(x)의 치역이 {y|yÉ4}이므로 함수 g(x)의 정의역은
형태가 아닐 경우 그 역함수의 그래프와의 교점은 반드시 {x|xÉ4}이다.
직선 y=x 위에만 있다고 할 수 없음에 주의해야 한다. 따라서 함수 g(x)의 정의역의 원소 중 자연수는
1, 2, 3, 4로 4개이다.

참고

일반적으로 함수의 공역은 실수 전체의 집합이고 역함수가


237 ④
정의되기 위해서는 일대일대응이어야 하므로 치역이 실수 전체의
일차함수 f(x)=ax+b (a, b는 실수)라 하면 함수 y= f(x)의 집합이 되어야 역함수가 존재한다고 생각할 수 있다.
그래프가 점 (-2, 1)을 지나므로 하지만 공역은 반드시 실수 전체의 집합이어야 하는 것이
f(-2)=1에서 -2a+b=1 yy ㉠ 아니므로 일대일함수인 경우 공역을 치역과 일치시켜 역함수를
함수 y= f ÑÚ`(x)의 그래프가 점 (-2, 1)을 지나므로 정의할 수 있다.
f ÑÚ`(-2)=1에서 f(1)=-2이므로 a+b=-2 yy ㉡
㉠, ㉡을 연립하여 풀면 a=-1, b=-1이므로
f(x)=-x-1 239 ①
∴ f(3)=-4
함수 y= f(x)의 그래프와 그 역함수 y=g(x)의 그래프를 나타내면
다른 풀이 다음과 같다.
함수 y= f(x)의 그래프가 점 (-2, 1)을 지난다. y
y=f{x}
또한 함수 y= f ÑÚ`(x)의 그래프가 점 (-2, 1)을 지나므로 y=x

이와 직선 y=x에 대하여 대칭인 함수 y= f(x)의 그래프는


점 (1, -2)를 지난다. 4 y=g{x}
3
따라서 함수 y= f(x)의 그래프는 두 점 (-2, 1), (1, -2)를
지나는 직선이다.
-2-1 O 3 4 x
즉, f(x)= {x-(-2)}+1=-x-1
1-(-2)
함수 y= f(x)의 그래프가 증가하는 형태이므로 역함수 y=g(x)의
∴ f(3)=-4
그래프와의 교점은 직선 y=x 위에 존재한다.
참고 따라서 두 함수 y= f(x), y=g(x)의 그래프의 교점은 함수
일차함수 y= f(x)의 그래프와 그 역함수 y= f ÑÚ`(x)의 y= f(x)의 그래프와 직선 y=x의 교점과 같다.
그래프가 교점 (a, b)를 가지면 그 교점이 직선 y=x 위에 xÛ`-6x+12=x에서
존재하거나(a=b일 때) 함수 y= f(x)의 그래프가 기울기가 xÛ`-7x+12=(x-3)(x-4)=0
-1인 직선이다.(a+b일 때) ∴ x=3 또는 x=4
이 문제에서 교점 (-2, 1)이 직선 y=x 위의 점이 아니므로 따라서 두 교점의 좌표는 (3, 3), (4, 4)이므로
함수 y=f(x)의 그래프는 기울기가 -1인 직선이고, 점 (-2, 1)을 두 교점 사이의 거리는 ¿¹1Û`+1Û`='2이다.
지나므로 f(x)=-x-1임을 알 수 있다.

240 ③

주어진 집합을 X={-2, -1, 0, 1, 2},


238 ① Y={-3, -2, -1, 0, 1, 2, 3}이라 하자.
ㄱ. f(-2)= f(2)=0, f(-1)= f(1)=-3, f(0)=-4이다.
함수 f(x)의 역함수의 정의역은 함수 f(x)의 치역과 같다.
이때, f(0)=-4는 공역의 원소가 아니므로
함수 y=-(x-2)Û`+8`(xÉ0)의 그래프가 다음과 같으므로
f 는 X에서 Y로의 함수가 아니다.
f(x)É4이다.
ㄴ. g(x)=¿·xÛ`+1=|x|+1이다.
y
8 g(-2)=g(2)=3, g(-1)=g(1)=2, g(0)=1이므로
y=-{x-2}@+8 g 는 X에서 Y로의 함수이다.
16
ㄷ. h(-2)=h(2)=[- ]=-4,
4
5
4
h(-1)=h(1)=[- ]=-1, h(0)=[0]=0
5
이때, h(-2)=h(2)=-4는 공역의 원소가 아니므로
O 2 x
h는 X에서 Y로의 함수가 아니다.

50 수학 <하>
ㄹ. i(-2)=-3, i(-1)=-1, i(0)=1, i(1)=-3, 따라서 f(240)= f(2Ý`_3_5)=4_ f(2)+ f(3)+ f(5)-5이고,
i(2)=-2이므로 i는 X에서 Y로의 함수이다. 조건 ㈎에 의하여 f(2)=2, f(3)=3, f(5)=5이므로
ㅁ. j(-2)=3, j(-1)=2, j(0)=1, j(1)=0, j(2)=1이므로 f(240)=4_2+3+5-5=11
j는 X에서 Y로의 함수이다.
따라서 <보기> 중 X에서 Y로의 함수인 것은 ㄴ, ㄹ, ㅁ으로 3개이다.
244 ③

241 ⑤ 주어진 함수를 범위를 나누어 구하면 다음과 같다.


Ú x<-3일 때
정의역이 X인 두 함수 f(x)=|4x|, g(x)=xÛ`이 서로 같으려면
y=-(x-2)-(x+1)-(x+3)=-3x-2
정의역 X의 모든 원소 x에 대하여 f(x)=g(x)가 성립해야 한다.
Û -3Éx<-1일 때
방정식 f(x)=g(x)를 만족시키는 x의 값을 구하면
y=-(x-2)-(x+1)+(x+3)=-x+4
á 4x (x¾0)
f(x)=|4x|=Ò 이므로 Ü -1Éx<2일 때
» -4x (x<0)
y=-(x-2)+(x+1)+(x+3)=x+6
Ú x≥0일 때
Ý x¾2일 때
4x=xÛ`에서 x=0 또는 x=4
y=(x-2)+(x+1)+(x+3)=3x+2
Û x<0일 때
-4x=xÛ`에서 x=-4
Ú ~ Ý에서 주어진 함수의 그래프는 다음과 같다. Ⅴ
y 함
Ú, Û에 의하여 정의역 X의 원소로 가능한 것은 -4, 0, 4이다. 수
따라서 정의역 X는 집합 {-4, 0, 4}의 부분집합 중 공집합이 아닌 8 와
7 그
것이므로 구하는 집합 X의 개수는 2Ü`-1=7이다. 래
5 프

242 11

t에 대한 이차방정식 tÛ`-2t f(x)-3 f(x)=0의 실근이 존재하지 -3 -1 O 2 x


않으려면 이차방정식 tÛ`-2t f(x)-3 f(x)=0의 판별식을 D라
따라서 x=-1일 때 최솟값 5를 가지므로 a=-1, b=5
할 때,
∴ a+b=4
D
={- f(x)}Û`+3 f(x)= f(x){ f(x)+3}<0에서
4
-3< f(x)<0
주어진 그래프에서 부등식 -3< f(x)<0을 만족시키는 x의 값은 245 ⑤

0, 2, 4, 5이다. f(-x)= f(x)를 만족시키려면


따라서 구하는 값은 0+2+4+5=11이다. f(-2)= f(2), f(-1)= f(1)을 만족시켜야 한다.
f(-2)= f(2), f(-1)= f(1), f(0)이 될 수 있는 값이 각각
-2, -1, 0, 1, 2로 5개씩이므로 조건을 만족시키는 함수 f 의
243 ②
개수는 5_5_5=125이다.
240=2Ý`_3_5=16_15이고,
조건 ㈎에 의하여 f(2)=2, f(3)=3, f(5)=5이다.
조건 ㈏에 의하여 246 148
f(15)= f(3)+ f(5)-1=7
X에서 X로의 모든 함수의 개수는 4Ý`이므로
f(4)= f(2)+ f(2)-1=3이므로
구하는 함수의 개수는 4Ý`에서
f(16)= f(4)+ f(4)-1=5
{ f(1)-3}{ f(2)-2}{ f(3)-2}+0을 만족시키는 함수 f의
∴ f(240)= f(15_16)= f(15)+ f(16)-1
개수를 빼주면 된다.
=7+5-1=11
f(1)+3, f(2)+2, f(3)+2인 함수의 개수는
다른 풀이
f(1)로 가능한 값은 1, 2, 4 중 하나이므로 3가지,
조건 ㈏에 의하여 2 이상의 자연수 aÁ, aª, y, aÇ에 대하여 f(2)로 가능한 값은 1, 3, 4 중 하나이므로 3가지,
f(aÁ_aª_y_aÇ) f(3)으로 가능한 값은 1, 3, 4 중 하나이므로 3가지,
= f(aÁ_aª_y_aÇÐÁ)+ f(aÇ)-1 f(4)로 가능한 값은 4가지이므로
= f(aÁ_aª_y_aÇЪ)+ f(aÇÐÁ)+ f(aÇ)-2 함수의 개수는 4_3Ü`이다.
⋮ 따라서 구하는 함수의 개수는
= f(aÁ)+ f(aª)+y+ f(aÇ)-(n-1) 4Ý`-4_3Ü`=4(4Ü`-3Ü`)=4_37=148

Ⅴ. 함수와 그래프 51
다른 풀이 1
f(x)+xÜ` f {- }=-6xÜ` yy ㉠
x
{ f(1)-3}{ f(2)-2}{ f(3)-2}=0을 만족시키려면
1
f(1)=3 또는 f(2)=2 또는 f(3)=2를 만족시켜야 한다. 주어진 등식의 양변에 x 대신 을 대입하면
x
f(1)=3을 만족시키는 함수의 집합을 A, 1 1 6
xÛ` f {- }- f(x)=
f(2)=2를 만족시키는 함수의 집합을 B, x x x
f(3)=2를 만족시키는 함수의 집합을 C라 하면 양변에 -x를 각각 곱하면
구하는 함수 f의 개수는 n(A'B'C)이다. 1
f(x)-xÜ` f {- }=-6 yy ㉡
x
n(A)=n(B)=n(C)=4Ü`
㉠과 ㉡의 양변을 각각 더하면
n(A;B)=n(B;C)=n(C;A)=4Û`
2 f(x)=-6xÜ`-6, f(x)=-3xÜ`-3
n(A;B;C)=4이므로
∴ f(2)=-24-3=-27
n(A'B'C)
=n(A)+n(B)+n(C)-n(A;B)-n(B;C)-n(C;A)
+n(A;B;C) 249 ④
=3_4Ü`-3_4Û`+4=148
[2x-1]=[2x]-1이므로 f(x)=2x-[2x]+1이다. yy TIP1
1
- Éx<0에서 [2x]=-1이므로 f(x)=2x+2
247 132 2
1
f(x-y)= f(x)- f(y) yy ㉠ 0Éx< 에서 [2x]=0이므로 f(x)=2x+1
2
㉠에 x=0, y=0을 대입하면 f(0)= f(0)- f(0)=0 1
Éx<1에서 [2x]=1이므로 f(x)=2x
㉠에 x=0, y 대신 -y를 대입하면 f(y)= f(0)- f(-y)에서 2
f(-y)=- f(y)이다. yy ㉡ 3
1Éx< 에서 [2x]=2이므로 f(x)=2x-1
2
㉠에 y 대신 -y를 대입하면

f(x+y)= f(x)- f(-y)= f(x)+ f(y) (∵ ㉡)
따라서 함수 y= f(x)의 그래프가 다음과 같다. yy TIP2
이므로
y
f(2)= f(1+1)= f(1)+ f(1)=2 f(1)
2
f(3)= f(2+1)= f(2)+ f(1)=3 f(1)
y=f{x}
f(4)= f(3+1)= f(3)+ f(1)=4 f(1)
⋮ 1
따라서 f(n)=n f(1)`(n은 자연수)이다.
-2 3 -1 1 O 1 1 3 2 x
∴ f(33)=33 f(1)=33_4=132 - -
2 2 2 2

ㄱ. f(-2.1)=-4.2-[-5.2]=-4.2+6=1.8 (거짓)
248 ②
ㄴ. f(x-1)=2(x-1)-[2(x-1)-1]
주어진 등식의 양변에 x=-2를 대입하면 =2x-2-[2x-3]
1 1 =2x-2-([2x-1]-2)
f(2)+2 f {- }=-12
4 2 =2x-[2x-1]
1 ∴ f(x)= f(x-1) (참)
f(2)+8 f {- }=-48 yy ㉠
2 1
ㄷ. xÛ`+1=2에서 xÛ`=2, x=Ñ'2이므로 이차함수
1 2
주어진 등식의 양변에 x= 을 대입하면
2 1
y= xÛ`+1의 그래프는 두 점 (-'2, 2), ('2, 2)를 지나고,
1 1 2
4 f {- }- f(2)=3
2 2 꼭짓점의 좌표가 (0, 1)이므로 다음과 같다.
1 1
8 f {- }- f(2)=6 yy ㉡ y y= x@+1
2 2
2
㉠에서 ㉡의 양변을 각각 빼면 2 f(2)=-54
y=f{x}
∴ f(2)=-27
다른 풀이 1

주어진 등식의 양변에 x 대신 -x를 대입하면 -2 3 -1 1 O 1 1 3 2 x


- -
1 1 2 2 2 2
f(x)+x f {- }=-6x
xÛ` x 1
함수 y= xÛ`+1의 그래프와 함수 y= f(x)의 그래프가 위와
양변에 xÛ`을 각각 곱하면 2

52 수학 <하>
1 TIP3
같이 5개의 점에서 만나므로 방정식 f(x)= xÛ`+1의 실근의
2
모든 실수 x에 대하여 f(x)= f(x+1)이 성립할 때,
개수는 5이다. (참)
x+1=t라 하면 x=t-1이므로 f(t-1)= f(t)이다.
따라서 옳은 것은 ㄴ, ㄷ이다.
즉, 모든 실수 x에 대하여 f(x)= f(x-1)이 성립한다.
다른 풀이

1
ㄴ. 함수 y= f(x)의 그래프가 간격으로 모양이 반복되므로
2 250 ②
1 간격으로도 모양이 반복된다.
함수 f(x)=[x]-3x를 x의 범위를 나누어 구하면 다음과 같다.
즉, 모든 실수 x에 대하여 f(x)= f(x+1)이 성립한다. 따라서
yy TIP
모든 실수 x에 대하여 f(x)= f(x-1)이다. (참) yy TIP3
Ú -1Éx<0일 때 f(x)=-1-3x
TIP1 Û 0Éx<1일 때 f(x)=-3x
<가우스 기호의 성질> Ü 1Éx<2일 때 f(x)=1-3x
[x]는 x보다 크지 않은 최대의 정수이다. Ý 2Éx<3일 때 f(x)=2-3x
(즉, [x]는 x의 정수부분이고, x-[x]는 x의 소수부분이다.) Þ x=3일 때 f(3)=3-9=-6
❶ [x]Éx<[x]+1 Ú ~ Þ에서 함수 y= f(x)의 그래프는 다음과 같다.
❷ [x+n]=[x]+n (n은 정수) y Ⅴ
n 2

❸ [ ]=(n을 k로 나눈 몫) (n, k는 자연수)
k -1 O1 2 3 수
x 와
-1

TIP2 -2 래
-3 프
<가우스 기호를 포함한 함수의 그래프 그리기> -4
-5
❶ y=[ f(x)] -6
가우스 기호 안의 식의 값이 정수일 때를 기준으로 x의 값의 -7

범위를 나눈다. 정수 n에 대하여 nÉ f(x)<n+1일 때 따라서 치역은 {y|-7<yÉ2}이다.


y=n임을 이용하여 구간별로 그래프를 그리면 된다.
TIP
y=[2x]
y y nÉx<n+1(n은 정수)일 때, [x]=n이므로
y=2x
y=[2x] [x]의 값에 따라 x의 범위를 나누려면 x의 범위를
2 2
nÉx<n+1과 같이 나누어야 한다.
1 1
1 1 따라서 -1ÉxÉ3인 x의 범위를 -1Éx<0, 0Éx<1,
- -
2 2
-1 O1 1 3 x -1 O1 1 3 x y, 2Éx<3, x=3과 같이 나누어서 함수식을 구한다.
2 2 2 2
-1 -1
-2
-2
251 ①
❷ y= f(x)-[ f(x)]
조건 ㈎에 의하여 g(x)=x이고, h(x)=k (k<X)이다.
가우스 기호 안의 식의 값이 정수일 때를 기준으로 x의 값의
조건 ㈏에서 g(2)=2이므로 f(1)=2, h(3)=k=2
범위를 나눈다.
따라서 h(x)=2이다.
정수 n에 대하여 nÉ f(x)<n+1일 때 y= f(x)-n이므로
조건 ㈐에서 2 f(6)= f(3)이고, 이때 조건 ㈎에 의하여 함수 f는
nÉ f(x)<n+1인 x의 범위에서 함수 y= f(x)의 그래프를
일대일대응이므로 f(6)=3, f(3)=6이고, f(2)=1이다.
함숫값이 0 이상 1 미만이 되도록 y축의 방향으로 -n만큼
∴ f(2)+g(3)+h(6)=1+3+2=6
평행이동하면 된다.
y=2x-[2x]
y y=2x y
252 ②

3 ㄱ. f(x)=5-x라 하면 다음과 같다.


2 f
y=2x-[2x] X X
1 1
1 1
-1 O1 1 3 x -1 1 O 1 1 3 x 2 2
2 2 - 2 2
2
1 -1 3 3
-
2
-2 4 4

Ⅴ. 함수와 그래프 53
따라서 일대일함수이고, 일대일대응이다.
ㄴ. g(x)=(x를 5로 나눈 나머지)라 하면 다음과 같다.
253 ①

g 함수 f(x)=xÜ`-xÛ`-4x-3이 항등함수이려면 집합 X의 각 원소
X X
x에 대하여 f(x)=x이어야 한다.
1 1
xÜ`-xÛ`-4x-3=x, xÜ`-xÛ`-5x-3=0, (x+1)Û`(x-3)=0
2 2 x=-1 또는 x=3이므로
3 3 정의역 X의 원소로 가능한 것은 -1, 3이다.
4 4 따라서 정의역 X는 집합 {-1, 3}의 부분집합 중 공집합이 아닌
것이므로 구하는 집합 X의 개수는 2Û`-1=3이다.
따라서 항등함수이고, 일대일함수, 일대일대응이다.
ㄷ. h(x)=|x|+|x-6|-3이라 하면
x<X에 대하여 x>0, x-6<0이므로
y=|x|+|x-6|-3=x-(x-6)-3=3이다.
254 ②

h ㄱ. 두 함수 y= f(x)와 y=g(x)의 그래프의 교점의 좌표를


X X
(a, b)라 하면 f(a)=b, g(a)=b이다.
1 1 1 2 b 2b
h(a)= f(a)+ g(a)= + =b이므로 함수 y=h(x)의
2 2 3 3 3 3
3 3
그래프도 점 (a, b)를 지난다. (참)
ㄴ. 두 함수 y= f(x)와 y=g(x)의 그래프가 모두 y축에 대하여
4 4
대칭이면 모든 실수 x에 대하여 f(-x)= f(x),
따라서 상수함수이다. g(-x)=g(x)가 성립한다. 이때,
1 2 1 2
x-1 h(-x)= f(-x)+ g(-x)= f(x)+ g(x)=h(x)
ㄹ. i(x)=[ ]+1이라 하면 3 3 3 3
4
즉, h(-x)=h(x)이므로 함수 y=h(x)의 그래프도 y축에
x-1
x<X에 대하여 0É <1이므로 대하여 대칭이다. (참)
4
x-1 ㄷ. (반례) f(x)=2x, g(x)=-x이면 y= f(x)와 y=g(x)는
y=[ ]+1=0+1=1이다.
4 모두 일대일대응이지만
i 1 2 2x 2x
X X h(x)= f(x)+ g(x)= - =0이므로
3 3 3 3
1 1
y=h(x)는 일대일대응이 아니다. (거짓)
2 2
따라서 옳은 것은 ㄱ, ㄴ이다.
3 3
참고
4 4
1_ f(x)+2_g(x)
h(x)= 이므로 함수 y=h(x)의 그래프는
2+1
따라서 상수함수이다.
각각 두 함수 y= f(x), y=g(x)의 그래프 위의 점인
á x+1 (x는 홀수)
ㅁ. j(x)=Ò 라 하면 다음과 같다. P(a, f(a)), Q(a, g(a))에 대하여 선분 PQ를 2`:`1로
» x (x는 짝수)
내분하는 점의 자취이다.
j
X X

1 1

2 2

3 3
255 ④

4 4 2x-1¾0일 때, f(x)=(2x-1)-kx+1=(2-k)x
2x-1<0일 때, f(x)=-(2x-1)-kx+1=-(k+2)x+2
따라서 상수함수, 항등함수, 일대일함수, 일대일대응 어떤 ( 1
(2-k)x {x¾ }
것에도 해당되지 않는다. 2
즉, f(x)={
따라서 <보기> 중에서 1
-(k+2)x+2 {x< }
9 2
상수함수는 ㄷ, ㄹ이므로 p=2,
함수 f(x)가 일대일대응이 되려면 두 직선 y=(2-k)x와
항등함수는 ㄴ이므로 q=1,
y=-(k+2)x+2의 기울기가 모두 양수이거나 모두 음수이어야
일대일함수는 ㄱ, ㄴ이므로 r=2,
하므로 -(2-k)(k+2)>0, (k+2)(k-2)>0
일대일대응은 ㄱ, ㄴ이므로 s=2이다.
∴ k<-2 또는 k>2
∴ p+q+r+s=7

54 수학 <하>
TIP a>0 yy ㉡
1 또한 치역이 공역과 같아야 하므로 치역이 실수 전체의 집합이 되기
x= 일 때, y=(2-k)x와 y=-(k+2)x+2의 함숫값이
2 위해서 x=1일 때 두 함수 y=xÛ`-ax+b와 y=ax+2의 함숫값이
1 같아야 한다.
1- k로 서로 같으므로 두 직선의 기울기가 모두 양수이거나
2
즉, 1-a+b=a+2이므로 b=2a+1 yy ㉢
모두 음수이면 함수 f(x)의 치역이 실수 전체의 집합이 되므로
㉠, ㉡, ㉢에 의하여 점 (a, b)의 자취의 방정식은
함수 f(x)는 일대일대응이 된다.
b=2a+1`(0<aÉ2)이고, 좌표평면 위에 자취를 나타내면 다음과
같다.
b
256 ②
5
á (2a-1)x-b+1 (x¾2)
함수 f(x)=Ò 에서
» xÛ`-4x+3b (x<2)
x<2일 때 포물선 y=xÛ`-4x+3b의 축은 x=2이고, 아래로
1
볼록한 모양이므로
O 2 a
x<2에서 함수 y= f(x)의 그래프는 다음과 같다.

채점 요소 배점 Ⅴ
일대일함수일 조건 구하기 40% 함

‘(치역)=(공역)’일 조건 구하기 40% 와
자취의 방정식을 구하고, 자취 나타내기 20% 그
2 x 래

함수 f(x)가 일대일대응이기 위해서 일대일함수이어야 하므로
x¾2에서 직선 y=(2a-1)x-b+1의 기울기는 음수이어야 한다.
1
258 ⑴ k¾1 ⑵ kÉ1 ⑶ k=1
2a-1<0에서 a< yy ㉠
2 f(x)=xÛ`-2x+4=(x-1)Û`+3에서
또한 치역이 공역과 같아야 하므로 치역이 실수 전체의 집합이 되기 함수 y=(x-1)Û`+3의 그래프가 다음과 같다.
위해서 x=2일 때 두 함수 y=xÛ`-4x+3b와 y
y=(2a-1)x-b+1의 함숫값이 같아야 한다.
즉, -4+3b=4a-b-1이므로
3
4a-4b+3=0, b=a+
4
3
3 1 3 5
㉠에 의해서 b=a+ < + =
4 2 4 4
따라서 정수 b의 최댓값은 1이다. O 1 x

함수의 최솟값이 3이므로 치역은 항상 Y={y|y¾3}의


257 풀이 참조 부분집합이다. 따라서 함수 f `:`X`2Ú`Y는 모든 실수 k에 대하여
정의된다.
á xÛ`-ax+b (x¾1)
함수 f(x)=Ò 에서 ⑴ 함수 f 가 일대일함수이려면 이차함수 그래프의 축이 x=1이므로
» ax+2 (x<1)
k¾1이어야 한다.
a Û aÛ`
x¾1일 때 함수 y=xÛ`-ax+b={x- }`+b- 이 ⑵ kÉ1이면 함수 f의 치역은 {y|y¾3}이다.
2 4
일대일함수이기 위해서 꼭짓점의 x좌표는 1보다 작거나 같아야 k>1이면 함수 f의 치역은 {y|y¾ f(k)}이고, 이때
a f(k)>3이므로 치역이 집합 Y와 같지 않다.
하므로 É1에서 aÉ2 yy ㉠
2 따라서 함수 f의 공역과 치역이 같으려면 kÉ1이어야 한다.
이때, x¾1에서 함수 y= f(x)의 그래프는 다음과 같다. ⑶ 함수 f 가 일대일대응이려면 ⑴, ⑵를 모두 만족시켜야 하므로
k=1이다.

259 ⑴1 ⑵2

1 x ⑴ 정의역이 X={x|xÉk}인 함수 f(x)=-xÛ`+4x-1이


함수 f(x)가 일대일함수이려면 x<1에서 직선 y=ax+2의 일대일함수이려면 이차함수 y=-xÛ`+4x-1=-(x-2)Û`+3의
기울기가 양수이어야 하므로 그래프의 축이 x=2이므로 kÉ2이어야 한다. yy ㉠

Ⅴ. 함수와 그래프 55
y 구하는 함수 f 의 개수는 96_3이다.
Ú, Û에서 구하는 함수 f 의 개수는
f{k} y=f{x} 96+96_3=96_4=384

O x
k 2
261 ②

조건 ㈎에서 x+y에 대하여 f(x)+ f(y)이므로


이때, 치역은 {y|yÉ f(k)}이므로 치역이 공역 Y={y|yÉ2}와 함수 f 는 일대일함수이다.
같으려면 f(k)=2이어야 한다. 이때, 조건 ㈏를 만족시키는 함수 f 를 구하면 다음과 같다.
-kÛ`+4k-1=2에서 Ú x=0이고, y=1 또는 y=-1일 때
kÛ`-4k+3=0, (k-1)(k-3)=0 f(0_y)= f(0) f(y)에서 f(0)= f(0) f(y)
∴ k=1 (∵ ㉠) f(0){ f(y)-1}=0이므로
⑵ 정의역이 X={x|x¾k}인 함수 f(x)=xÛ`+2x-6이 f(0)=0 또는 f(y)=1이다.
일대일함수이려면 이차함수 y=xÛ`+2x-6=(x+1)Û`-7의 f(0)+0이면 f(1)=1, f(-1)=1이어야 하므로 함수 f 가
그래프의 축이 x=-1이므로 k¾-1이어야 한다. yy ㉡ 일대일함수라는 조건에 모순이다.
y 즉, f(0)=0이다.
y=f{x}
Û x=-1, y=1일 때
f{k} f(-1_1)= f(-1) f(1)에서 f(-1)= f(-1) f(1)
-1 f(-1){ f(1)-1}=0이므로
O k x
f(-1)=0 또는 f(1)=1이다.
Ú에서 f(0)=0이고, f 가 일대일함수이므로 f(-1)+0이다.
즉, f(1)=1이다.
Ú, Û에서 f(0)=0, f(1)=1이고,
이때, 치역은 {y|y¾ f(k)}이므로 치역이 공역 X={x|x¾k}와
f(-1)로 가능한 값은 -4, -3, -2, y, 4의 9개 중 0, 1을
같으려면 f(k)=k이어야 한다.
제외한 7개이다.
kÛ`+2k-6=k에서
따라서 구하는 함수 f 의 개수는 7이다.
kÛ`+k-6=0, (k-2)(k+3)=0
∴ k=2 (∵ ㉡)

262 ②

260 384 명제 ‘a+b인 어떤 a<X, b<X에 대하여 f(a)= f(b)이다.’의


부정 ‘a+b인 모든 a<X, b<X에 대하여 f(a)+ f(b)이다.’를
조건 ㈏에 의하여 함수 f 는 일대일함수이다.
만족시키는 함수 f 는 일대일함수이다.
조건 ㈎에서 f(1)+ f(2)+ f(3)이 홀수인 경우는 다음과 같다.
따라서 구하는 함수의 개수는 X에서 X로의 함수의 개수에서
Ú f(1), f(2), f(3)이 모두 홀수인 경우
X에서 X로의 일대일함수의 개수를 뺀 것과 같다.
f(1)로 가능한 값은 1, 3, 5, 7로 4개이고,
X에서 X로의 함수의 개수는 4Ý`이고, X에서 X로의 일대일함수의
f(2)로 가능한 값은 1, 3, 5, 7 중 f(1)을 제외한 3개이고,
개수는 4_3_2_1이므로
f(3)으로 가능한 값은 1, 3, 5, 7 중 f(1), f(2)를 제외한
구하는 함수 f의 개수는 4Ý`-4_3_2_1=232이다.
2개이고,
f(4)로 가능한 값은 1, 2, 3, y, 7 중 f(1), f(2), f(3)을
제외한 4개이므로
함수 f 의 개수는 4_3_2_4=96이다.
263 ①

Û f(1), f(2), f(3) 중 하나만 홀수인 경우 조건 ㈏에서 f(3)=6이다.


f(1)이 홀수인 경우를 구해보면 다음과 같다. f
X Y
f(1)로 가능한 값은 1, 3, 5, 7로 4개이고, 1
1
f(2)로 가능한 값은 2, 4, 6으로 3개이고, 2
2
f(3)으로 가능한 값은 2, 4, 6 중 f(2)를 제외한 2개이고, 3
3
f(4)로 가능한 값은 1, 2, 3, y, 7 중 f(1), f(2), f(3)을 4
4
5
제외한 4개이므로 5 6
함수 f 의 개수는 4_3_2_4=96이다.
f(2), f(3)이 홀수인 경우도 마찬가지이므로 조건 ㈎에 의하여 함수 f 는 일대일함수이다.

56 수학 <하>
f(4)=1 또는 f(4)=2이면 조건 ㈐를 만족시키지 못하므로 ④ {y|0ÉyÉ3}ø{x|-1ÉxÉ2}이므로
yy TIP 합성함수 g ç h가 정의되지 않는다.
f(4)¾3에서 f(4)=3 또는 f(4)=4 또는 f(4)=5이다. ⑤ {y|-1ÉyÉ2},{x|-2ÉxÉ2}이므로
f(4)의 값에 따라 함수 f의 개수를 구해 보면 다음과 같다. 합성함수 h ç f 가 정의된다.
Ú f(4)=3일 때 따라서 선지 중 합성함수가 정의되지 않는 것은 ④이다.
f(1)로 가능한 값은 1, 2로 2가지
f(2)로 가능한 값은 1, 2 중 f(1)을 제외한 1가지
f(5)로 가능한 값은 1, 2, 4, 5 중 f(1), f(2)를 제외한
2가지이므로 265 ①

함수의 개수는 2_1_2=4 합성함수 f ç g 가 정의되기 위해서 함수 g 의 치역이 함수 f 의


Û f(4)=4일 때 정의역의 부분집합이어야 한다.
f(1)로 가능한 값은 1, 2, 3으로 3가지 g(0)=-2a+b, g(1)=-a이므로 함수 g 의 치역이 함수 f 의
f(2)로 가능한 값은 1, 2, 3 중 f(1)을 제외한 2가지 정의역 {0, 1}의 부분집합이 되기 위해서 가능한 경우는 다음과
f(5)로 가능한 값은 1, 2, 3, 5 중 f(1), f(2)를 제외한 같다.
2가지이므로 Ú g(0)=g(1)=0일 때
함수의 개수는 3_2_2=12 -2a+b=0, -a=0에서 a=0, b=0이므로 a+b=0 Ⅴ
Ü f(4)=5일 때 Û g(0)=g(1)=1일 때 함
f(1)로 가능한 값은 1, 2, 3, 4로 4가지 -2a+b=1, -a=1에서 a=-1, b=-1이므로 a+b=-2 수

f(2)로 가능한 값은 1, 2, 3, 4 중 f(1)을 제외한 3가지 Ü g(0)=0, g(1)=1일 때 그
f(5)로 가능한 값은 1, 2, 3, 4 중 f(1), f(2)를 제외한 -2a+b=0, -a=1에서 a=-1, b=-2이므로 a+b=-3 래

2가지이므로 Ý g(0)=1, g(1)=0일 때
함수의 개수는 4_3_2=24 -2a+b=1, -a=0에서 a=0, b=1이므로 a+b=1
Ú ~ Ü에서 구하는 함수 f 의 개수는 Ú ~ Ý에서 모든 a+b의 값의 합은
4+12+24=40 0+(-2)+(-3)+1=-4

TIP
함수 f 가 일대일함수이므로 함숫값이 각각 서로 달라야 한다.
조건 ㈐에서 f(1)< f(4), f(2)< f(4)이려면 f(4)의 값보다
작은 서로 다른 두 값 f(1), f(2)가 존재해야 하므로 공역의
266 ④

원소 1, 2, 3, 4, 5, 6에서 f(4)의 값은 1과 2는 될 수 없다. 7Ç` (n은 자연수)의 일의 자리의 수는 7, 9, 3, 1의 네 수가 차례로


따라서 f(4)¾3이어야 한다. 반복되므로
f(1)=7, f(2)=9, f(3)=3, f(4)=1, f(5)=7, y이다.
9Ç` (n은 자연수)의 일의 자리의 수는 9, 1의 두 수가 차례로

264 ④ 반복되므로
n이 홀수일 때 g(n)=9이고, n이 짝수일 때 g(n)=1이다.
두 함수 i, j에 대하여 합성함수 i ç j가 정의되려면 함수 j의 치역이 즉, ( f ç g)(25)= f(g(25))= f(9)= f(2_4+1)= f(1)=7
함수 i의 정의역의 부분집합이어야 한다. (h ç f)(75)=h( f(75))
주어진 함수 f, g, h의 정의역과 치역을 구하면 다음과 같다. =h( f(4_18+3))=h( f(3))
0ÉxÉ3에서 -1Éx-1É2이므로 =h(3)
함수 f의 정의역은 {x|0ÉxÉ3}, 치역은 {y|-1ÉyÉ2}이다. =(3ß`을 5로 나눈 나머지)+1
1 이때, 3Ç` (n은 자연수)의 일의 자리의 수는 3, 9, 7, 1의 네 수가
-1ÉxÉ2에서 0ÉxÛ`É4, 0É xÛ`É2이므로
2
차례로 반복되므로 3ß`의 일의 자리의 수는 9이다.
함수 g의 정의역은 {x|-1ÉxÉ2}, 치역은 {y|0ÉyÉ2}이다.
-2ÉxÉ2에서 -1Éx+1É3, 0É|x+1|É3이므로 따라서 3ß`을 5로 나눈 나머지는 4이므로 yy TIP
함수 h의 정의역은 {x|-2ÉxÉ2}, 치역은 {y|0ÉyÉ3}이다. (h ç f)(75)=4+1=5이다.
① {y|0ÉyÉ2},{x|0ÉxÉ3}이므로 ∴ ( f ç g)(25)+(h ç f)(75)=7+5=12
합성함수 f ç g가 정의된다. TIP
② {y|0ÉyÉ3},{x|0ÉxÉ3}이므로
어떤 자연수를 5로 나누면 십의 자리 이상은 5로
합성함수 f ç h가 정의된다.
나누어떨어지므로 어떤 자연수를 5로 나눈 나머지는 일의 자리의
③ {y|-1ÉyÉ2},{x|-1ÉxÉ2}이므로
수를 5로 나눈 나머지와 같다.
합성함수 g ç f 가 정의된다.

Ⅴ. 함수와 그래프 57
따라서 f(4)=1, f(2)=1이 되어 함수 f 가 일대일대응이라는
267 ③
조건을 만족시키지 않는다.
f(1)=(4의 약수의 개수)=3, Û f(4)=2이면
f(2)=1, (g ç f)(4)=g(2)=4가 되어 (g ç f)(4)=2를 만족시키지 않는다.
f(3)=(6의 약수의 개수)=4, Ü f(4)=3이면
f(4)=(7의 약수의 개수)=2 f(1)=3이므로 함수 f 가 일대일대응이라는 조건을
이므로 함수 f 가 다음과 같다. 만족시키지 않는다.
f Ý f(4)=4이면
X X
(g ç f)(4)=2에서 g(4)=2이다.
1 1
f g
2 2 X X X X

3 3 1 1 1 1

4 4 2 2 2 2

3 3 3 3
Ú g(1)=4이므로 ( f ç g)(1)= f(g(1))= f(4)=2이고,
4 4 4 4
(g ç f)(1)=g( f(1))=g(3)이다.
( f ç g)(1)=(g ç f)(1)이므로 g(3)=2이다. ( f ç g)(1)=1에서 g(1)=1이면 ( f ç g)(1)=3이 되어
Û g(3)=2이므로 ( f ç g)(3)= f(g(3))= f(2)=1이고, 조건을 만족시키지 않으므로 g(1)=3이고, ( f ç g)(1)=1에서
(g ç f)(3)=g( f(3))=g(4)이다. f(3)=1이다.
( f ç g)(3)=(g ç f)(3)이므로 g(4)=1이다. 따라서 두 함수 f, g 는 다음과 같다.
Ü g(4)=1이므로 ( f ç g)(4)= f(g(4))= f(1)=3이고, f g
X X X X
(g ç f)(4)=g( f(4))=g(2)이다.
1 1 1 1
( f ç g)(4)=(g ç f)(4)이므로 g(2)=3이다.
2 2 2 2
Ý g(2)=3이므로 ( f ç g)(2)= f(g(2))= f(3)=4이고,
(g ç f)(2)=g( f(2))=g(1)이다. 3 3 3 3

( f ç g)(2)=(g ç f)(2)이므로 g(1)=4이다. 4 4 4 4

Ú ~ Ý에서 함수 g 는 다음과 같다.


Ú ~ Ý에서 g(3)+(g ç f)(2)=1+g(2)=1+4=5
g
X X

1 1

2 2

3 3 269 ③

4 4 ㄱ. f(100)= f(10_10+0)= f(10)+0


= f(10_1+0)= f(1)+0
∴ (g ç g)(3)+g(4)=g(2)+1=3+1=4
= f(10_0+1)
= f(0)+1=1 (참)
ㄴ. f(999)= f(10_99+9)= f(99)+9
268 ② = f(10_9+9)+9= f(9)+9+9
= f(10_0+9)+9+9= f(0)+9+9+9
f(1)=3, g(2)=4이므로 다음과 같다.
=9+9+9=27
f g
X X X X ( f ç f)(999)= f( f(999))= f(27)
1 1 1 1 = f(10_2+7)= f(2)+7
2 2 2 2 = f(10_0+2)+7= f(0)+2+7
3 3 3 3 =2+7=9 (참)
4 4 4 4 ㄷ. (반례) n=15일 때,
f(15)= f(10_1+5)= f(1)+5
이때, ( f ç g)(1)=1, (g ç f)(4)=2를 만족시키는 경우를 f(4)의 = f(10_0+1)+5= f(0)+1+5
값에 따라 살펴보면 다음과 같다. =1+5=6
Ú f(4)=1이면 이때, f(15)=6은 6의 배수이지만 15는 6의 배수가 아니다.
(g ç f)(4)=2에서 g(1)=2이므로 (거짓)
( f ç g)(1)=1에서 f(2)=1이다. 따라서 옳은 것은 ㄱ, ㄴ이다.

58 수학 <하>
참고

음이 아닌 정수 n에 대하여
271 ⑤

n=aµ_10µ``+y+aª_10Û`+aÁ_10+a¼ f Ú`(1)= f(1)=3,


일 때, (a¼, aÁ, y, aµ은 0 또는 한 자리 자연수) f Û`(1)=( f ç f)(1)= f( f(1))= f(3)=2,
f(aµ_10µ``+y+aª_10Û`+aÁ_10+a¼) f Ü`(1)=( f ç f Û`)(1)= f( f Û`(1))= f(2)=1,
= f(aµ_10
m-1
+y+aª_10+aÁ)+a¼ f Ý`(1)=( f ç f Ü`)(1)= f( f Ü`(1))= f(1)=3, y
= f(aµ_10
m-2
+y+aª)+aÁ+a¼ 따라서 f Ç` (1)의 값은 3, 2, 1의 세 수가 차례로 반복되므로
3_33+1
⋮ f Ú` â` â`(1)= f (1)= f(1)=3이다.
=aµ+y+aª+aÁ+a¼ 마찬가지 방법에 의하여
∴ f(n)=aµ+y+aª+aÁ+a¼ f Ú`(3)= f(3)=2,
따라서 f(n)은 n의 각 자리의 숫자를 모두 더한 값을 f Û`(3)=( f ç f)(3)= f( f(3))= f(2)=1,
함숫값으로 가지는 함수이다. f Ü`(3)=( f ç f Û`)(3)= f( f Û`(3))= f(1)=3,
f Ý`(3)=( f ç f Ü`)(3)= f( f Ü`(3))= f(3)=2, y
따라서 f Ç` (3)의 값은 2, 1, 3의 세 수가 차례로 반복되므로
3_66+2
f Û` â` â`(3)= f (3)= f Û`(3)=1이다.
∴ f Ú` â` â`(1)- f Û` â` â`(3)=3-1=2

270 ② 다른 풀이

f(1)=3, f Û`(1)=2, f Ü`(1)=1 수
f(x)=ax+b`(a, b는 실수)라 하면 와
f Û`(x)= f( f(x))= f(ax+b) f(2)=1, f Û`(2)=3, f Ü`(2)=2 그
f(3)=2, f Û`(3)=1, f Ü`(3)=3이므로 래
=a(ax+b)+b 프
=aÛ`x+ab+b f Ü`=I (I는 항등함수)이다. yy TIP
3_33+1
조건 ㈎에 의하여 f Ú` â` â`(1)= f (1)= f(1)=3
3_66+2
f Û`(2)=2aÛ`+ab+b=5 yy ㉠ f Û` â` â`(3)= f (3)= f Û`(3)=1
f Ü`(x)= f( f Û`(x))= f(aÛ`x+ab+b) ∴ f Ú` â` â`(1)- f Û` â` â`(3)=2
=aÜ`x+aÛ`b+ab+b
TIP
마찬가지 방법에 의하여 n+1
‘f Ú`= f 이고, 자연수 n에 대하여 f = f ç f Ç` 이다’로 정의된
f Þ`(x)=aÞ`x+aÝ`b+aÜ`b+aÛ`b+ab+b
함수 f Ç` 을 포함하는 문제는 합성함수에서 자주 출제된다.
조건 ㈏에서
이때, 정의역의 한 원소에서 시작하여 차례로 대응되는 함숫값을
f Þ`(4)- f Þ`(8)=4aÞ`-8aÞ`=-4aÞ`=128
찾으면 반복되는 규칙을 쉽게 찾을 수 있다.
이므로 aÞ`=-32, a=-2
이 문제에서 1 → 3 → 2 → 1 → y이므로
㉠에 대입하면
2 → 1 → 3 → 2 → y, 3 → 2 → 1 → 3 → y임을 알 수 있다.
8-2b+b=5, b=3
즉, f Ü`(1)=1, f Ü`(2)=2, f Ü`(3)=3이므로
따라서 f(x)=-2x+3이므로 f(6)=-9이다.
f Ü`=I (I는 항등함수)이다.
다른 풀이
이를 이용하면 IÇ` =I이므로
3_33+1
일차함수는 합성을 해도 일차함수이고, 이때 일차함수를 합성할 f Ú` â` â`= f =( f Ü`)Ü` Ü` ç f=IÜ` Ü` ç f=I ç f= f 임을 알 수 있다.
때마다 일차항의 계수가 거듭제곱된다.
주어진 함수를 f(x)=ax+b`(a, b는 실수)라 하면
f Þ`(x)는 일차항의 계수가 aÞ`이므로
f Þ`(x)=aÞ`x+c`(c는 실수)라 하면 조건 ㈏에서 272 ③
f Þ`(4)- f Þ`(8)=(4aÞ`+c)-(8aÞ`+c)
주어진 함수 y= f(x)의 그래프에서
=-4aÞ`=128
( 1
이므로 aÞ`=-32, a=-2 2x {0ÉxÉ }
2
∴ f(x)=-2x+b f(x)={ 이다.
1
-2x+2 { <xÉ1}
f Û`(x)= f( f(x))= f(-2x+b) 9 2
=-2(-2x+b)+b 5 5 4
f { }=-2_ +2=
7 7 7
=4x-b
5 4 4 6
이므로 조건 ㈎에 의하여 f Û` { }= f { }=-2_ +2=
7 7 7 7
f Û`(2)=8-b=5, b=3
5 6 6 2
따라서 f(x)=-2x+3이므로 f(6)=-9이다. f Ü` { }= f { }=-2_ +2=
7 7 7 7

Ⅴ. 함수와 그래프 59
5 2 2 4 ㄷ. f(xÁ)+ f(xª)이므로 xÁ, xª 중 하나는 유리수이고, 하나는
f Ý` { }= f { }=2_ =
7 7 7 7 무리수이다.
⋮ 이 중 xÁ을 유리수라 할 때,
5 4 6 2 xÁ+0이면 xÁxª는 무리수이므로 f(xÁxª)=1이지만
이므로 f Ç` { }의 값은 , , 가 차례로 반복된다.
7 7 7 7
xÁ=0이면 xÁxª=0이므로 f(xÁxª)='2이다. (거짓)
16=3_5+1이므로
따라서 옳은 것은 ㄱ이다.
5 5 5 5
f { }+ f Û` { }+ f Ü` { }+y+ f Ú`ß` { }
7 7 7 7 참고
4 6 2 4
=5_{ + + }+ f ç f ç f= f 라 해서 f ç f=I (I는 항등함수)가 아님에
7 7 7 7
64 주의하자.
=
7 함수 f 의 역함수가 존재할 경우 f ç f ç f= f 에서 f ç f=I 가
성립하지만, 함수 f 의 역함수가 존재하지 않을 경우 f ç f=I 라
할 수 없다.
273 5

f(1)=3, f Û`(1)=2, f Ü`(1)=4, f Ý`(1)=1


f(2)=4, f Û`(2)=1, f Ü`(2)=3, f Ý`(2)=2 276 풀이 참조
f(3)=2, f Û`(3)=4, f Ü`(3)=1, f Ý`(3)=3
⑴ x가 유리수일 때, f(x)=x이므로
f(4)=1, f Û`(4)=3, f Ü`(4)=2, f Ý`(4)=4
( f ç f)(x)= f(x)=x이다.
에서 f Ý`=I (I는 항등함수)이다.
4_8+1 4_24+3 x가 무리수일 때, f(x)=1-x이고,
f Ü` Ü`(a)+ f á` á`(b)= f (a)+ f (b)
x가 무리수이면 1-x도 무리수이므로
= f(a)+ f Ü`(b)
( f ç f)(x)= f(1-x)=1-(1-x)=x이다.
이 값이 최대가 되려면 f(a)= f Ü`(b)=4이어야 하므로
∴ ( f ç f)(x)=x
a=2, b=1
⑵ ( f ç f)(x)=x에서 f ç f=I (I는 항등함수)이므로
∴ 2a+b=5
( f ç f ç f)(x)=(I ç f)(x)= f(x)이다.
따라서 ( f ç f ç f)(a)+( f ç f)('3a)= f(a)+'3a
274 ②
Ú a가 유리수일 때
f Ú`(50)=49, f Û`(50)=48, f Ü`(50)=47, y, f Þ` â`(50)=0, f(a)+'3a=a+'3a=(1+'3)a이므로
4+'3
f Þ` Ú`(50)=-1, f Þ` Û`(50)=1, f Þ` Ü`(50)=0, y (1+'3)a=4+'3, a=
1+'3
이므로 f Ç` (50)의 값은 n¾51부터 -1, 1, 0의 세 수가 차례로
이는 유리수가 아니므로 모순이다.
반복된다.
Û a가 무리수일 때
300=50+250=50+3_83+1이므로
f(a)+'3a=1-a+'3a=1+('3-1)a이므로
f Ü` â` â`(50)= f Þ` Ú`(50)=-1
1+('3-1)a=4+'3
'3+3 (3+'3)(1+'3)
a= = =3+2'3
275 ① '3-1 2
Ú, Û에 의하여 a=3+2'3이다.
ㄱ. Ú x<Q일 때, f(x)='2이고
( f ç f ç f)(x)= f( f( f(x)))= f( f('2)) 채점 요소 배점

= f(1)='2 ⑴ x가 각각 유리수, 무리수일 때 함수 ( f ç f)(x) 구하기 40%

Û x²Q일 때, f(x)=1이고 ⑵ x가 각각 유리수, 무리수인 경우로 나누어 조건을 만족시키는 a의


60%
( f ç f ç f)(x)= f( f( f(x)))= f( f(1)) 값 구하기

= f('2)=1
Ú, Û에 의하여 ( f ç f ç f)(x)= f(x)이다. (참) yy 참고
ㄴ. Ú x<Q일 때, f(x)='2이므로 277 풀이 참조

x+ f(x)=x+'2 는 무리수이다. ⑴ f(1)=2, f Û`(1)=1


따라서 f(x+ f(x))=1이다. f(2)=1, f Û`(2)=2이므로
Û x²Q일 때, f(x)=1이므로 x=1 또는 x=2일 때 f Û`(x)=x이다.
x+ f(x)=x+1은 무리수이다. f(3)=4, f Û`(3)=5, f Ü`(3)=3
따라서 f(x+ f(x))=1이다. f(4)=5, f Û`(4)=3, f Ü`(4)=4
Ú, Û에 의하여 모든 x<R에 대하여 f(x+ f(x))=1이므로 f(5)=3, f Û`(5)=4, f Ü`(5)=5이므로
함수 g(x)의 치역은 {1}로 원소의 개수가 1이다. (거짓) x=3 또는 x=4 또는 x=5일 때 f Ü`(x)=x이다.

60 수학 <하>
따라서 f µ``이 항등함수가 되도록 하는 자연수 m의 최솟값은 f f f
X X X X X X
2와 3의 최소공배수인 6이다.
1 1 1 1 1 1
⑵ f ß`=I`(I는 항등함수)이므로
6_166+4 6_333+2 2 2 2 2 2 2
f Ú`â` â` â`= f = f Ý`,` f Û`â` â` â`= f = f Û`이고,
3 3 3 3 3 3
f Ú`â` â` â`(a)와 f Û`â` â` â`(b)의 값은 모두 공역 X={1, 2, 3, 4, 5}의
4 4 4 4 4 4
원소이므로 합이 4가 되는 경우는 다음과 같다.
Ú f Ý`(a)=1, f Û`(b)=3일 때 a=1, b=4 Ü f(a)=b, f(b)=a`(a+b)를 만족시키는 두 수 a, b가 두 쌍
Û f Ý`(a)=2, f Û`(b)=2일 때 a=2, b=2 존재하는 경우
Ü f Ý`(a)=3, f Û`(b)=1일 때 a=5, b=1 1, 2, 3, 4의 네 수를 두 원소씩 두 쌍으로 묶는 경우는
따라서 구하는 모든 순서쌍 (a, b)는 {1, 2}, {3, 4} 또는 {1, 3}, {2, 4} 또는 {1, 4}, {2, 3}의
(1, 4), (2, 2), (5, 1)이다. 3가지이므로 함수 f 의 개수는 3이다.
채점 요소 배점 f f f
X X X X X X
⑴ 합성함수의 함숫값의 규칙성을 찾아 m의 최솟값 구하기 50% 1 1 1 1 1 1
⑵ 조건을 만족시키는 순서쌍 (a, b) 구하기 50% 2 2 2 2 2 2

3 3 3 3 3 3

4 4 4 4 4 4 Ⅴ

Ú ~ Ü에서 구하는 함수 f의 개수는 1+6+3=10이다. 수

278 ⑤ TIP 그

( f ç f)(x)=x 를 만족시키는 함수 f 는 정의역의 모든 원소 프
( f ç f)(x)=x이면 집합 X의 임의의 두 원소 a, b에 대하여
f(a)=b일 때 f(b)=a를 만족시켜야 한다. a에 대하여
즉, f(a)=a이거나 (a=b인 경우) a는 자기 자신에 대응되거나 ( f(a)=a)
a가 자기 자신에 대응되지 않을 경우에 b(b+a)로 대응된다고
f(a)=b, f(b)=a이어야 한다. (a+b인 경우) yy TIP
하면 b는 a로 대응된다.`( f(a)=b, f(b)=a)
f(5)=6에서 f(6)=5이므로 5, 6에 대한 대응은 생각하지 않고
집합 X의 나머지 원소 1, 2, 3, 4에 대해서 이와 같은 조건을
만족시키는 함수 f의 개수를 구하면 다음과 같다.
Ú f(a)=b, f(b)=a`(a+b)를 만족시키는 두 수 a, b가 존재하지 279 ④
않는 경우
함수 y= f(x)의 그래프는 다음과 같다.
f(1)=1, f(2)=2, f(3)=3, f(4)=4로 함수 f의 개수는
y
1이다. y=f{x}
f
X X
3
1 1

2 2
2
3 3 O x
-1
4 4
( f ç f)(k)= f( f(k))=8에서 f(k)=t라 하면 f(t)=8
Û f(a)=b, f(b)=a`(a+b)를 만족시키는 두 수 a, b가 한 쌍 5
t<0일 때, -2t+3=8에서 t=-
존재하는 경우 2
두 수 a, b가 될 수 있는 값은 t¾0일 때, tÛ`-4t+3=8에서
{1, 2}, {1, 3}, {1, 4}, {2, 3}, {2, 4}, {3, 4}로 tÛ`-4t-5=(t-5)(t+1)=0, t=5 (∵ t¾0)
6가지이고, 이때 나머지 2개의 원소는 모두 자기 자신에게 대응되면 5 5
t=- 또는 t=5이므로 f(k)=- 또는 f(k)=5이다.
2 2
되므로 함수 f 의 개수는 6이다.
5
f f f f(k)=- 를 만족시키는 k의 값은 존재하지 않는다.
X X X X X X 2
1 1 1 1 1 1 f(k)=5에서
2 2 2 2 2 2 k<0일 때, -2k+3=5에서 k=-1
k¾0일 때, kÛ`-4k+3=5에서
3 3 3 3 3 3
kÛ`-4k-2=0, k=2+'6 (∵ k¾0)
4 4 4 4 4 4
따라서 조건을 만족시키는 실수 k의 값의 합은

Ⅴ. 함수와 그래프 61
(-1)+(2+'6)=1+'6이다. 함수식을 통하여 답을 구할 수도 있으나 식을 직접 구하지 않고
본풀이와 같이 답을 구하는 것이 훨씬 편리하다.

280 ③

이차함수 y= f(x)의 그래프의 축이 x=-2이므로 그래프가 x축과 281 12


만나는 두 점은 직선 x=-2에 대하여 대칭이다.
그림과 같이 함수 y= f(x)의 그래프와 직선 y=x의 교점의
따라서 3이 아닌 x절편은 -7이다.
x좌표를 각각 a, b, c, d`(a<b<c<d)라 하자.
y
y y=x
6

y=f{x} y=d
y=c
ab
Oc d x
-7 -2 O 3 x y=b
y=a

ㄱ. ( f ç g)(x)= f(g(x))=0에서 g(x)=k라 하면


y=f{x}
방정식 f(k)=0에서 k=-7 또는 k=3이므로
g(x)=-7 또는 g(x)=3
g(x)가 일차함수이므로 두 방정식을 만족시키는 실수 x가 각각 ( f ç f)(x)= f( f(x))= f(x)에서 f(x)=k라 하면

하나씩 존재한다. 방정식 f(k)=k에서 k=a 또는 k=b 또는 k=c 또는 k=d이므로

그러므로 방정식 ( f ç g)(x)=0의 서로 다른 실근의 개수는 f(x)=a 또는 f(x)=b 또는 f(x)=c 또는 f(x)=d이다.

2이다. (참) Ú f(x)=a인 경우

ㄴ. { f(x)}Û`={g(x)}Û`에서 { f(x)}Û`-{g(x)}Û`=0 함수 y= f(x)의 그래프와 직선 y=a가 만나는 점의 개수가

{ f(x)+g(x)}{ f(x)-g(x)}=0이므로 4이므로 방정식 f(x)=a를 만족시키는 서로 다른 x의 값의

f(x)=-g(x) 또는 f(x)=g(x) 개수는 4이다.

함수 y=-g(x)의 그래프는 함수 y=g(x)의 그래프를 x축에 Û f(x)=b인 경우

대하여 대칭이동한 것이므로 두 함수 y= f(x), y=g(x)의 함수 y= f(x)의 그래프와 직선 y=b가 만나는 점의 개수가

그래프의 교점 또는 두 함수 y= f(x), y=-g(x)의 그래프의 4이므로 방정식 f(x)=b를 만족시키는 서로 다른 x의 값의

교점은 다음과 같이 세 개 존재한다. 개수는 4이다.


y Ü f(x)=c인 경우
6 함수 y= f(x)의 그래프와 직선 y=c가 만나는 점의 개수가
y=g{x} y=f{x} 2이므로 방정식 f(x)=c를 만족시키는 서로 다른 x의 값의
개수는 2이다.
Ý f(x)=d인 경우
-7 -2 O 3 x
함수 y= f(x)의 그래프와 직선 y=d가 만나는 점의 개수가
2이므로 방정식 f(x)=d를 만족시키는 서로 다른 x의 값의
y=-g{x} 개수는 2이다.
그러므로 방정식 { f(x)}Û`={g(x)}Û`의 서로 다른 실근의 개수는 Ú ~ Ý에서 방정식 ( f ç f)(x)= f(x)를 만족시키는 서로 다른
3이다. (거짓) x의 값의 개수는 4+4+2+2=12이다.
ㄷ. ( f ç f)(x)= f( f(x))=0에서 f(x)=t라 하면 ∴ n(X)=12
방정식 f(t)=0에서 t=-7 또는 t=3이므로
f(x)=-7 또는 f(x)=3
방정식 f(x)=-7의 두 근의 합이 2_(-2)=-4이고, 282 ①
마찬가지로 방정식 f(x)=3의 두 근의 합이 -4이다.
방정식 f( f(x+2))=4에서 f(x+2)=t라 하면
그러므로 방정식 ( f ç f)(x)=0의 모든 실근의 합은
f(t)=4
(-4)+(-4)=-8이다. (참)
y
따라서 옳은 것은 ㄱ, ㄷ이다. y=f{x}
6
참고 4
주어진 그래프에서 함수 f(x), g(x)의 식을 모두 구할 수 있다. 2
19
6 1 O 2 5 6 8 11 13 15161718 x
f(x)=- (x+2)Û`+6, g(x)=- x+1
25 3

62 수학 <하>
주어진 그래프에 의하여 방정식 f(t)=4에서
t=6 또는 t=11 또는 t=15 또는 t=17이므로
284 ②

f(x+2)=6 또는 f(x+2)=11 또는 f(x+2)=15 또는 함수 y= f(x)의 그래프가 x=1일 때를 기준으로 함수식이


f(x+2)=17이다. 1
바뀌므로 방정식 g(x)=1의 두 근 x= , x=3과 y=g(x)의
2
주어진 그래프에서 모든 실수 x에 대하여 f(x)É6이므로
함수식이 바뀌는 x=1, x=2, x=3을 기준으로 x의 범위를 나누어
방정식 f(x+2)=11 또는 f(x+2)=15 또는 f(x+2)=17의
해는 존재하지 않는다. 함수 y=( f ç g)(x)의 그래프를 그리면 다음과 같다. yy TIP
방정식 f(x+2)=6의 해는 주어진 그래프에서 1
Ú 0ÉxÉ 일 때
2
f(8)= f(16)=6이므로 x+2=8 또는 x+2=16에서
y y=g{x} y y=f{x} y y={ f`©g}{x}
x=6 또는 x=14이다. 4 4 4
따라서 방정식 f( f(x+2))=4의 서로 다른 실근의 개수는 2이고, 3 3 3
2 2 2
서로 다른 두 실근의 합은
1 1 1
6+14=20이다.
O 11 2 3 4 x O 1 2 3 4 x O 11 2 3 4 x
2 2

1
Û <xÉ1일 때
2
y y=g{x} y y=f{x} y y={ f`©g}{x}

4 4 4 함
283 ② 3
2
3
2
3
2



주어진 함수 f(x)는 1 1 1

á x+2 (0ÉxÉ2) O 11 2 3 4 x O 1 2 3 4 x O 11 2 3 4 x 프
f(x)=Ò 이다. 2 2
» -2x+8 (2<xÉ4)
방정식 f( f(x))=5- f(x)에서 f(x)=t라 하면 Ü 1<xÉ2일 때
방정식 f(t)=5-t에서 y y=g{x} y y=f{x} y y={ f`©g}{x}
4 4 4
3 3 3 3
0ÉtÉ2일 때, t+2=5-t이므로 t=
2 2 2 2
2<tÉ4일 때, -2t+8=5-t이므로 t=3 1 1 1
3 3 O 1 2 3 4 x O 1 2 3 4 x O 11 2 3 4 x
즉, t= 또는 t=3이므로 f(x)= 또는 f(x)=3이다.
2 2 2
3 Ý 2<xÉ3일 때
Ú 방정식 f(x)= 의 해는
2
y y=g{x} y y=f{x} y y={ f`©g}{x}
3 13 4 4 4
2<xÉ4일 때, -2x+8= 에서 x=
2 4 3 3 3
y 2 2 2
4 1 1 1
y=f{x}
O 1 2 3 4 x O 1 2 3 4 x O 11 2 3 4 x
2 2
3 Þ 3<xÉ4일 때
2
y y=g{x} y y=f{x} y y={ f`©g}{x}
O 2 4 x
4 4 4
Û 방정식 f(x)=3의 해는 3 3 3
2 2 2
0ÉxÉ2일 때, x+2=3에서 x=1 1 1 1
5
2<xÉ4일 때, -2x+8=3에서 x=
2 O 1 2 3 4 x O 1 2 3 4 x O 11 2 3 4 x
2
y
4 y=f{x} Ú ~ Þ에 의하여 함수 y=( f ç g)(x)의 그래프는 다음과 같다.
y
3 y={ f`©g}{x}
2 4
3

O 2 4 x 2
1
Ú, Û에서 구하는 서로 다른 실근의 합은
13 5 27 O 1 1 2 3 4 x
+1+ = 2
4 2 4

Ⅴ. 함수와 그래프 63
따라서 함수 y=( f ç g)(x)의 그래프와 x축, y축 및 직선 x=4로 y
y=f{x}
둘러싸인 도형의 넓이는 1
1 1 1 3 15
+ +1+ + = 이다.
2 4 2 2 4
TIP
<합성함수의 그래프 그리기> O 1 1 x
2
다음 그림과 같이 두 함수 y= f(x)와 y=g(x)의 그래프가
1
주어졌을 때, 합성함수 y=(g ç f)(x)의 그래프는 다음과 같은 함수 f(x)에서 x= 일 때를 기준으로 함수식이 바뀌므로 방정식
2
방법으로 빨리 그릴 수 있다. 1 1 3
y y f(x)= 의 두 근 x= , x= 과 f(x)의 함수식이 바뀌는
y=f{x} y=g{x} 2 4 4
3 3
1
2 2 x= 을 기준으로 x의 범위를 나누어 함수 y=( f ç f)(x)의
2
1 1 그래프를 그리면 다음과 같다.
O 1 2 3 x O 1 2 3 x 1
Ú 0ÉxÉ 일 때
4
❶ y=g(x)의 식이 바뀌는 지점이 x=2이므로 방정식 y y y
f(x)=2의 해인 x=2를 기준으로 x의 값의 범위를 나눈다. y=f{x} y=f{x} y={ f`©f}{x}
1 1 1
❷ y= f(x)에서 x의 값이 1에서 2로 커짐에 따라 y의 값이
1
3에서 2로 작아짐을 [그림1]과 같이 화살표로 표시한다. 2
❸ f의 치역이 g의 정의역이 되므로, y=g(x)에서 x의 값이
O 1 1 1 x O 1 1 x O 1 x
3에서 2로 작아짐에 따라 g의 함숫값 y가 1에서 3으로 4 2 2 4
커짐을 [그림2]와 같이 화살표로 나타낸다. 1 1
Û <xÉ 일 때
❹ 합성함수 (g ç f)(x)의 정의역은 f(x)의 정의역과 같고, 4 2
치역은 g(x)의 치역과 같음을 고려하여 [그림3]과 같이 y y y
y=f{x} y=f{x} y={ f`©f}{x}
[그림1]의 x축과 [그림2]의 y축에 표시해 놓은 화살표를 1 1 1

그리면 1ÉxÉ2에서 합성함수 y=(g ç f)(x)의 그래프를 1


2
그릴 수 있다.
y y=f{x} y y=g{x} y y={ g`©f}{x} O 1 1 1 x O 1 1 x O 1 1 x
3 3 3 4 2 2 4 2
2 2 2 1 3
Ü <xÉ 일 때
1 1 1 2 4
y y y
O 1 2 3 x O 1 2 3 x O 1 2 3 x y=f{x} y=f{x} y={ f`©f}{x}
1 1 1
[그림1] [그림2] [그림3]
1
마찬가지로 2ÉxÉ3일 때, 앞의 ❷, ❸, ❹의 과정을 반복하면 2
다음과 같다.
O 1 3 1 x O 1 1 x O 1 1 3 x
y y y
y=f{x} y=g{x} y={ g`©f}{x} 2 4 2 4 2 4
3 3 3
3
2 2 2 Ý <xÉ1일 때
4
1 1 1 y y y
y=f{x} y=f{x} y={ f`©f}{x}
O 1 2 3 x O 1 2 3 x O 1 2 3 x 1 1 1

따라서 1ÉxÉ3에서 합성함수 y=(g ç f)(x)의 그래프는 1


2
다음과 같다.
y y={ g`©f}{x} O 1 3 1 x O 1 1 x O 1 1 3 1 x
3 2 4 2 4 2 4
2 Ú ~ Ý에 의하여 함수 y=( f ç f)(x)의 그래프는 다음과 같다.
1 y
y={ f`©f}{x}
1
O 1 2 3 x

285 4
1 x
O 1 1 3
함수 y= f(x)의 그래프는 다음과 같다. 4 2 4

64 수학 <하>
마찬가지 방법으로 함수 y=( f ç f ç f)(x)의 그래프를 그리면 다른 풀이
다음과 같다. 방정식 2{h(x)}Ü`-11{h(x)}Û`+19h(x)-10=0에서
y
y={ f©f©f}{x} {h(x)-1}{h(x)-2}{2h(x)-5}=0이므로
1
5
h(x)=1 또는 h(x)=2 또는 h(x)= 이다.
2
1 h(x)=g( f(x))
y=
4 =|2-|xÛ`-2|-1|+1
O 11315371 x =||xÛ`-2|-1|+1
8482848
Ú -'2ÉxÉ'2일 때
1 h(x)=|(-xÛ`+2)-1|+1=|xÛ`-1|+1
방정식 h(x)= 을 만족시키는 x의 값은 함수 y=h(x)의 그래프와
4
Û x<-'2 또는 x>'2일 때
1
직선 y= 의 교점의 x좌표이고, 함수 y=h(x)의 그래프가 h(x)=|xÛ`-3|+1
4
1 Ú, Û에서 함수 y=h(x)의 그래프는 다음과 같다.
직선 x= 에 대하여 대칭이므로 구하는 모든 x의 값의 합은
2 y
5
1 5
8_ =4이다. 2 y=
2 2
y=2
2
y=1 Ⅴ
1

-Â3 -1 O 1 Â3 x 수
286 11
-Â2 Â2


방정식 2{h(x)}Ü`-11{h(x)}Û`+19h(x)-10=0에서 래
따라서 방정식 h(x)=1의 해의 개수는 4, 프
{h(x)-1}{h(x)-2}{2h(x)-5}=0이므로 방정식 h(x)=2의 해의 개수는 5,
5 5
h(x)=1 또는 h(x)=2 또는 h(x)= 이다. 방정식 h(x)= 의 해의 개수는 2이므로
2 2
y y=|x-1|+1 구하는 서로 다른 실근의 개수는 4+5+2=11이다.
5
5
2 y=
2
2 y=2
y=1
1
O 1 x
287 풀이 참조

f ÑÚ`(4)=-5에서 f(-5)=4이다.
함수 g(x)=|x-1|+1의 그래프가 위와 같으므로
f(3x-2)=2x+a에서 yy ㉠
h(x)=g( f(x))=1일 때, f(x)=1 yy ㉠
3x-2=-5일 때 x=-1이므로 ㉠에 x=-1을 대입하면
h(x)=g( f(x))=2일 때, f(x)=0 또는 f(x)=2 yy ㉡
f(-5)=-2+a=4에서 a=6이다.
5 1 5
h(x)=g( f(x))= 일 때, f(x)=- 또는 f(x)= yy ㉢ 따라서 f(3x-2)=2x+6 yy ㉡
2 2 2
한편, f(x)=2-|xÛ`-2|에 대하여 함수 y= f(x)의 그래프는 3x-2=10일 때 x=4이므로 ㉡에 x=4를 대입하면
y=|xÛ`-2|의 그래프를 x축에 대하여 대칭이동한 후 y축의 방향으로 f(10)=14이다.
2만큼 평행이동한 것이므로 다음과 같다. f ÑÚ`(10)=k라 하면 f(k)=10이고,
y ㉡에서 2x+6=10일 때 x=2이므로 ㉡에 x=2를 대입하면
y=2-|x@-2| 5
5 f(4)=10에서 f ÑÚ`(10)=4이다.
2 y=
2
2 y=2 ∴ f(10)+ f ÑÚ`(10)=14+4=18
y=1
1 채점 요소 배점
x
O 1
y=- a의 값 구하기 40%
1 2
-
2 f(10)+ f ÑÚ`(10)의 값 구하기 60%

㉠에서 방정식 f(x)=1의 서로 다른 실근의 개수는 4,


㉡에서 방정식 f(x)=0의 서로 다른 실근의 개수는 3,
㉡에서 방정식 f(x)=2의 서로 다른 실근의 개수는 2,
1
㉢에서 방정식 f(x)=- 의 서로 다른 실근의 개수는 2,
288 ⑤
2
hÑÚ`(2)=k라 하면 h(k)=2이다. yy 참고
5
㉢에서 방정식 f(x)= 의 서로 다른 실근의 개수는 0
2 f(1-x)=h(x)에서 x=k를 대입하면
따라서 구하는 서로 다른 실근의 개수는 4+3+2+2=11이다. f(1-k)=h(k)=2 yy ㉠

Ⅴ. 함수와 그래프 65
이때, f ÑÚ`(2)=-3에서 f(-3)=2 yy ㉡
역함수를 가지는 함수 f 는 일대일대응이므로
290 3

㉠, ㉡에서 1-k=-3, k=4, 즉 h(4)=2 조건 ㈎에서 gÑÚ`( f(2x-1))=5x-3이므로


∴ hÑÚ`(2)=4 g(gÑÚ`( f(2x-1)))=g(5x-3)
다른 풀이 f(2x-1)=g(5x-3)
양변에 x=2를 대입하면
y= f(1-x)에서 x에 대하여 정리하면
f(3)=g(7)=4 (∵ ㈏)
1-x= f ÑÚ`(y), x=1- f ÑÚ`(y)
∴ f ÑÚ`(4)=3
x와 y를 서로 바꾸면 y=1- f ÑÚ`(x)이므로
다른 풀이
h(x)= f(1-x)의 역함수는
hÑÚ`(x)=1- f ÑÚ`(x)이다. 조건 ㈏에 의하여 gÑÚ`(4)=7이다. yy ㉠
∴ hÑÚ`(2)=1- f ÑÚ`(2) 조건 ㈎에서 gÑÚ`( f(2x-1))=5x-3 yy ㉡
=1-(-3)=4 5x-3=7일 때 x=2이므로 ㉡에 x=2를 대입하면
gÑÚ`( f(3))=7이다. yy ㉢
참고
함수 gÑÚ`가 일대일대응이므로 ㉠, ㉢에서 f(3)=4이다.
함수 f(x)의 역함수가 존재하고 모든 실수 x에 대하여
∴ f ÑÚ`(4)=3
f(1-x)=h(x)가 성립하므로 함수 h(x)의 역함수도
존재한다.

291 ③

함수 y= f(x)의 그래프를 나타내 보면 다음과 같다.


289 풀이 참조
y
함수 y= f(x)의 역함수가 y=g(x)이므로 g(x)= f ÑÚ`(x)이다. 7
함수 f(2x-5)의 역함수를 구하면 다음과 같다.
y= f(2x-5)를 x에 대하여 풀면
3
1 5
2x-5= f ÑÚ`(y), x= f ÑÚ`(y)+
2 2
x와 y를 서로 바꾸면 O 2 x
1 5 1 5
y= f ÑÚ`(x)+ , y= g(x)+ 함수 f(x)가 일대일대응이므로 임의의 실수 x에 대하여
2 2 2 2
1 5 ( f ç g)(x)=x일 때, g(x)= f ÑÚ`(x)이다.
따라서 f(2x-5)의 역함수는 g(x)+ 이다.
2 2 (g ç g ç g)(5)=g(g(g(5))) yy ㉠
채점 요소 배점 g(5)=a라 하면 f(a)=5, a<2이므로
y= f(2x-5)를 x에 대하여 풀기 50% -2a+7=5에서 a=1
함수 f(2x-5)의 역함수를 g(x)를 이용하여 나타내기 50% ㉠에서 g(g(g(5)))=g(g(1)) yy ㉡
g(1)=b라 하면 f(b)=1, b>2이므로
다른 풀이 1
- (b-2)Û`+3=1에서 (b-2)Û`=4, b=4 (∵ b>2)
2
h(x)=2x-5라 하면 yy ㉠
㉡에서 g(g(1))=g(4)
f(2x-5)=( f ç h)(x)이므로
g(4)=c라 하면 f(c)=4, c<2이므로
f(2x-5)의 역함수는 ( f ç h)ÑÚ`(x)이다.
3
1 5 -2c+7=4, c=
㉠에서 hÑÚ`(x)= x+ 이고, 2
2 2
3
f ÑÚ`(x)=g(x)이므로 ∴ (g ç g ç g)(5)=
2
( f ç h)ÑÚ`(x)=(hÑÚ` ç f ÑÚ`)(x)=(hÑÚ` ç g)(x)
1 5
=hÑÚ`(g(x))= g(x)+
2 2
1 5
따라서 f(2x-5)의 역함수는
2
g(x)+ 이다.
2 292 ②

채점 요소 배점 f(1)=3이고, 함수 f 가 일대일대응이므로
함수 f(2x-5)를 합성함수로 나타내어 y=2x-5의 역함수 구하기 50% f(2)=1, f(3)=2 또는 f(2)=2, f(3)=1이다.
함수 f(2x-5)의 역함수를 g(x)를 이용하여 나타내기 50% Ú f(2)=1, f(3)=2이면

66 수학 <하>
f(1)=3, f Û`(1)=2, f Ü`(1)=1 ㄴ. 함수 f `:`X`2Ú`Y의 역함수가 존재할 때,
f(2)=1, f Û`(2)=3, f Ü`(2)=2 함수 f ç f ÑÚ`는 Y에서 Y로의 항등함수이고,
f(3)=2, f Û`(3)=1, f Ü`(3)=3 함수 f ÑÚ` ç f 는 X에서 X로의 항등함수이다.
이므로 f Ü`=I가 성립한다. 이때, X+Y이면 f ç f ÑÚ`+ f ÑÚ` ç f 이다. (거짓)
Û f(2)=2, f(3)=1이면 ㄷ. 두 함수 f `:`X`2Ú`Y, g`:`Y`2Ú`Z에 대하여
f(1)=3, f Û`(1)=1, f Ü`(1)=3 함수 f ç g`:`Y`2Ú`Y가 정의되기 위한 필요충분조건은
이므로 f Ü`+I가 되어 성립하지 않는다. Z,X이다. (거짓)
Ú, Û에서 f(1)=3, f(2)=1, f(3)=2이므로 ㄹ. 두 함수 f `:`X`2Ú`Y, g`:`Z`2Ú`W에 대하여
g(1)=2, g(2)=3, g(3)=1이고, f Ü`=I에서 g Ü`=( f ÑÚ`)Ü`=I이다. Y,Z일 때, 합성함수 g ç f `:`X`2Ú`W가 정의되고, 함수 g ç f의
3_3+1 3_3+2
따라서 g Ú`â`=g =g, g Ú`Ú`=g =g Û`이므로 정의역은 함수 f 의 정의역과 같다. (참)
g Ú`â`(2)+g Ú`Ú`(3)=g(2)+g(g(3)) ㅁ. 두 함수 f `:`X`2Ú`Y, g`:`Y`2Ú`Z에 대하여
=3+g(1)=3+2=5 g ç f `:`X`2Ú`Z가 항상 정의된다.
두 함수 f, g 의 역함수가 모두 존재하면 두 함수 f, g 는 모두
일대일대응이므로 함수 g ç f도 일대일대응이다.
293 ③ 즉, 함수 g ç f의 역함수가 존재한다. (참)
따라서 옳은 것은 ㄱ, ㄹ, ㅁ이다.
( f ÑÚ` ç f ÑÚ`)(3)=3에서 ( f ç f)ÑÚ`(3)=3이므로 Ⅴ
( f ç f)(3)=3이다. 함
따라서 ( f ç f)(2)=1, ( f ç f)(3)=3을 만족시켜야 하므로 f(2)의 수

값에 따라 경우를 나누어 생각해 보자. 9 그
Ú f(2)=1인 경우 295 2 래

( f ç f)(2)= f( f(2))= f(1)=1에서 f ÑÚ`(g(1-x))=2x+1에서
f(2)=1, f(1)=1이 되어 f 는 일대일대응이 아니게 되므로 g(1-x)= f(2x+1)
성립하지 않는다. 1-x=t라 하면 x=1-t이므로
Û f(2)=2인 경우 g(t)= f(2(1-t)+1)= f(-2t+3)
( f ç f)(2)= f( f(2))= f(2)=2+1이 되므로 =3(-2t+3)-2=-6t+7
성립하지 않는다. 따라서 두 함수 y=3x-2, y=-6x+7의 그래프와 y축으로
Ü f(2)=3인 경우 둘러싸인 부분은 다음과 같다.
( f ç f)(2)= f( f(2))= f(3)=1이고, y
( f ç f)(3)= f( f(3))= f(1)=3에서 7
f(2)=3, f(1)=3이 되어 f 는 일대일대응이 아니게 되므로
성립하지 않는다.
Ý f(2)=4인 경우
( f ç f)(2)= f( f(2))= f(4)=1이다. 1
이때, ( f ç f)(3)=3이려면 f(3)=3이 되어야 하므로 O 1 x
함수 f 는 다음과 같다.
-2
f f
X X X
따라서 구하는 넓이는
1 1 1
1 9
2 2 2
_{7-(-2)}_1=
2 2
3 3 3

4 4 4

Ú ~ Ý에서 조건을 만족시키는 함수 f 는 Ý와 같으므로 296 ②


f(1)=2, f ÑÚ`(4)=2
f(4x-3-2g(x))=x에서 f ÑÚ`(x)=4x-3-2g(x)이고,
∴ 2 f(1)+ f ÑÚ`(4)=2_2+2=6
f ÑÚ`(x)=g(x)이므로 3g(x)=4x-3
4
g(x)= x-1 yy 참고
3
294 ㄱ, ㄹ, ㅁ
방정식 f(x)g(x)=0에서 f(x)=0 또는 g(x)=0
ㄱ. 일대일대응이면 일대일함수이므로 함수 f가 일대일함수인 것은 Ú 방정식 f(x)=0의 실근을 a라 하면
함수 f 가 일대일대응이기 위한 필요조건이다. (참) f(a)=0이므로 g(0)=a에서 a=-1

Ⅴ. 함수와 그래프 67
4
Û 방정식 g(x)= x-1=0의 실근은 x=
3 TIP
3 4
직선 y=x 위의 점 P가 다음과 같으므로 점 P의 좌표는
따라서 구하는 모든 실근의 합은
(6, 6)이다.
3 1
-1+ =- y
4 4
P
참고
6Â2
4 6
함수 f(x)는 g(x)= x-1의 역함수이므로
3
3 O x
f(x)= (x+1)이다. 6
4

297 ①

f(x)=-xÛ`-6x+b=-(x+3)Û`+b+9이므로 299 ②

함수 y= f(x)의 그래프의 개형은 다음과 같다. 방정식 { f ÑÚ`(3x+1)}Û`= f ÑÚ`(3x+1) f(3x+1)에서


f ÑÚ`(3x+1){ f ÑÚ`(3x+1)- f(3x+1)}=0이므로
f ÑÚ`(3x+1)=0 또는 f ÑÚ`(3x+1)= f(3x+1)이다.
Ú f ÑÚ`(3x+1)=0에서 f(0)=3x+1이고,
주어진 그래프에서 f(0)=2이므로
-3 a 1 x
1
집합 X에서 집합 Y로의 함수 f(x)의 역함수가 존재하려면 3x+1=2에서 x=
3
함수 f(x)는 일대일대응이어야 한다. Û f ÑÚ`(3x+1)= f(3x+1)에서
정의역이 X={x|aÉxÉ1}인 함수 f(x)가 일대일함수이기 3x+1=t라 하면 f ÑÚ`(t)= f(t)이다.
위해서 a¾-3이어야 하고, yy ㉠ 함수 y= f(x)의 그래프가 증가하는 형태이므로
치역이 집합 Y={y|-3ÉyÉ12}와 같으려면 함수 y= f(x)의 그래프가 역함수 y= f ÑÚ`(x)의 그래프와 만나는
aÉxÉ1에서 함숫값이 -3É f(x)É12이어야 하므로 교점은 직선 y=x와의 교점과 같으므로 방정식 f ÑÚ`(t)= f(t)의
f(a)=12, f(1)=-3이다. 해는 방정식 f(t)=t의 해와 같다.
f(1)=b-7=-3에서 b=4 주어진 그래프에서 f(-2)=-2, f(5)=5이고, 함수 y= f(x)의
f(a)=-(a+3)Û`+13=12에서 그래프는 직선 y=x와 두 점에서 만나므로
(a+3)Û`=1, a=-2 (∵ ㉠) 방정식 f(t)=t의 해는 t=-2 또는 t=5이다.
∴ a+b=2 3x+1=-2에서 x=-1
4
3x+1=5에서 x=
3
298 -6 Ú, Û에서 구하는 모든 실근의 합은
1 4 2
x¾2일 때 함수 y=xÛ`-4x+k=(x-2)Û`+k-4의 그래프의 +(-1)+ =
3 3 3
개형은 다음과 같다.

300 ⑤

{2,`k-4} 등식 f ÑÚ`(a)=g(b)에서 ( f ç g)(b)=a이므로


함수의 그래프가 증가하는 형태이므로 함수의 그래프가 역함수의 11
f(g(1))= f { }=5
2
그래프와 만나는 교점은 직선 y=x와의 교점과 같다.
f(g(2))= f(5)=4
따라서 점 P가 직선 y=x 위에 존재하므로
f(g(3))= f(3)=3
OPÓ=6'2일 때, P(6, 6)이다. yy TIP f(g(4))= f(1)=2
함수 y=xÛ`-4x+k`(x¾2)의 그래프와 직선 y=x의 교점의 1
f(g(5))= f { }=1
x좌표는 방정식 xÛ`-4x+k=x, 즉 xÛ`-5x+k=0의 해이므로 2
x=6이 방정식 xÛ`-5x+k=0의 해이다. f(g(6))= f(0)=0
36-30+k=0 따라서 등식 f ÑÚ`(a)=g(b)를 만족시키는 두 자연수 a, b의 순서쌍
∴ k=-6 (a, b)는 (5, 1), (4, 2), (3, 3), (2, 4), (1, 5)로 5개이다.

68 수학 <하>
다른 풀이
302 36
함수 y= f(x)와 그 역함수 y= f ÑÚ`(x)의 그래프는 서로 직선
1
y=x에 대하여 대칭이다. x-3¾0, 즉 x¾6일 때
2
11
g(1)= = f ÑÚ`(5) 1 3
2 f(x)=x-3+{ x-3}= x-6
2 2
g(2)=5= f ÑÚ`(4)
1
g(3)=3= f ÑÚ`(3) x-3<0, 즉 x<6일 때
2
g(4)=1= f ÑÚ`(2) 1 1
f(x)=x-3-{ x-3}= x
1 2 2
g(5)= = f ÑÚ`(1)
2 이므로 함수 y= f(x)의 그래프와 그 역함수 y=g(x)의 그래프는
g(6)=0= f ÑÚ`(0) 다음과 같다.
따라서 등식 f ÑÚ`(a)=g(b)를 만족시키는 두 자연수 a, b의 순서쌍 y y=x
(a, b)는 (5, 1), (4, 2), (3, 3), (2, 4), (1, 5)로 5개이다.
y=g{x}

y=f{x}
301 ③
3
함수 f(x)=xÛ`+1`(x¾0)에 대하여 함수 y= f(x)의 그래프와 두 Ⅴ
O 6 x
직선 x=1, x=2 및 x축으로 둘러싸인 도형은 다음과 같다. 함

y y=f{x} 두 함수 y= f(x), y=g(x)의 그래프가 만나는 한 교점은 두 직선 와
5 그
1
y= x와 y=x의 교점인 (0, 0)이고, 래
2

3
다른 한 교점은 두 직선 y= x-6과 y=x의 교점으로 방정식
2
2
S¡ 3
1 x-6=x에서 x=12이므로 (12, 12)이다.
2
O 1 2 x 두 교점 사이의 거리는 12'2이고, 점 (6, 3)과 직선 y=x 사이의
|6-3| 3'2
함수 y= f(x)의 그래프가 두 점 (1, 2), (2, 5)를 지나므로 거리가 = 이므로
¿¹1Û`+(-1)Û` 2
함수 y= f(x)의 그래프와 직선 y=x에 대하여 대칭인 역함수
y= f ÑÚ`(x)의 그래프는 두 점 (2, 1), (5, 2)를 지난다. 직선 y=x와 함수 y= f(x)의 그래프로 둘러싸인 도형의 넓이는
1 3'2
함수 y= f ÑÚ`(x)의 그래프와 두 직선 x=2, x=5 및 x축으로 _12'2_ =18이다.
2 2
둘러싸인 도형은 다음과 같고, 그림과 같은 두 도형의 넓이가
따라서 두 함수 y= f(x), y=g(x)의 그래프로 둘러싸인 도형의
서로 같다.
넓이는 2_18=36이다.
y y=f{x} y=x
5

303 ⑤
y=f —!{x}
2 주어진 사각형이 사다리꼴이므로 그 넓이는
1 S™
1 1
{g(c)+g(d)}(d-c)= (d+e)(d-c)이다.
2 2
O 1 2 5 x y y=g{x} y=x
따라서 SÁ+Sª는 다음 그림의 색칠된 부분의 넓이와 같다. f
y e
y=f{x}
d
5
c

S™ b
y=f —!{x} a O b c d e f x
2

1 g(d)=e, g(e)= f이므로 h(e)=d, h( f)=e이고,
g(g(d))=g(e)= f, g(g(c))=g(d)=e이므로
O 1 2 5 x
(h ç h)( f)=d, (h ç h)(e)=c이다.
즉, 구하는 넓이는 가로, 세로의 길이가 2, 5인 직사각형의 넓이에서
따라서 주어진 사각형의 넓이는
가로, 세로의 길이가 1, 2인 직사각형의 넓이를 뺀 것과 같다.
1 1
∴ SÁ+Sª=2_5-1_2=8 (d+e)(d-c)= {h(e)+h( f)}{(h ç h)( f)-(h ç h)(e)}
2 2

Ⅴ. 함수와 그래프 69
Ü k²(A'B)일 때
304 32
f(k)=1, fõ(k)=1, fA'B(k)=1이므로
점 A(3, 1)이 함수 y= f(x)의 그래프 위의 점이므로 fA'B(k)= f(k)×fõ(k)=1
f(3)=1이고 f ÑÚ`(1)=3이다. Ú ~ Ü에서 fA'B(x)= f(x)×fõ(x)이다. (참)
2 f ÑÚ`(1)+ f(1)=1에 대입하면 따라서 옳은 것은 ㄴ, ㄷ이다.
f(1)=1-2 f ÑÚ`(1)=1-2_3=-5에서 f ÑÚ`(-5)=1이므로
B(-5, 1)이다.
이때, 두 직선 AC, BD가 직선 y=x에 수직이고, 두 함수 y= f(x),
y= f ÑÚ`(x)의 그래프가 직선 y=x에 대하여 서로 대칭이므로 306 ⑤

두 점 A, B를 직선 y=x에 대하여 대칭이동한 점이 각각 C, D이다. ㄱ. f(2n)= f(n)에 의하여


즉, C(1, 3), D(1, -5)이다. f(100)= f(50)= f(25)
사다리꼴 ACBD에서 ACÓ=2'2, BDÓ=6'2이고, 높이는 선분 f(2n-1)=(-1)Ç` 에 의하여
AC의 중점 (2, 2)와 선분 BD의 중점 (-2, -2) 사이의 거리와 f(25)= f(2_13-1)=(-1)Ú` Ü`=-1 (참)
같으므로 4'2이다. ㄴ. f(2n-1)=(-1)Ç` 에서
따라서 사다리꼴 ACBD의 넓이는 n=1일 때 f(1)=-1,
1
_(2'2+6'2)_4'2=32 yy TIP n=2일 때 f(3)=1,
2
n=3일 때 f(5)=-1,
TIP n=4일 때 f(7)=1,
직선 AB가 x축에 평행하고, 두 직선 AB와 CD가 직선 y=x에 n=5일 때 f(9)=-1이다.
대하여 대칭이므로 직선 CD는 y축에 평행하다. f(2n)= f(n)에 의하여
따라서 두 직선 AB, CD가 서로 수직이므로 f(2)= f(1)=-1,
사각형 ACBD의 넓이는 f(4)= f(2)= f(1)=-1,
1 1 f(6)= f(3)=1,
_ABÓ_CDÓ= _8_8=32로 구할 수도 있다.
2 2 f(8)= f(4)= f(2)= f(1)=-1,
C
f(10)= f(5)=-1
B A ∴ f(1)+ f(2)+ f(3)+y+ f(10)=-4 (참)
ㄷ. n이 홀수이므로 n=2k-1`(k는 자연수)이라 하면
f(n)= f(2k-1)=(-1)û`
f(n+8)= f(2k+7)
D k+4
= f(2(k+4)-1)=(-1)
이때, k가 홀수이면 k+4가 홀수이므로
f(n)= f(n+8)=-1
305 ④
k가 짝수이면 k+4가 짝수이므로
ㄱ. (반례) A=R인 경우 모든 실수 x에 대하여 x<A이므로 f(n)= f(n+8)=1
함수 f의 치역은 {0}이다. (거짓) ∴ f(n)= f(n+8) (참)
ㄴ. 임의의 실수 k에 대하여 따라서 옳은 것은 ㄱ, ㄴ, ㄷ이다.
Ú k<A일 때
f(k)=0, fA‚` (k)=1이므로
f(k)=1- fA‚` (k)=0
Û k²A일 때 307 ④

f(k)=1, fA‚` (k)=0이므로 0ÉxÉ1에서 정의된 함수 f(x)=4x-[4x]는


f(k)=1- fA‚` (k)=1 1
0Éx< 일 때, [4x]=0이므로 f(x)=4x
Ú, Û에서 f(x)=1- fA‚` (x)이다. (참) 4
ㄷ. 임의의 실수 k에 대하여 1 1
Éx< 일 때, [4x]=1이므로 f(x)=4x-1
4 2
Ú k<A일 때
1 3
k<(A'B)이므로 f(k)=0, fA'B(k)=0 Éx< 일 때, [4x]=2이므로 f(x)=4x-2
2 4
fA'B(k)= f(k)×fõ(k)=0 3
Éx<1일 때, [4x]=3이므로 f(x)=4x-3
Û k<B일 때 4
k<(A'B)이므로 fõ(k)=0, fA'B(k)=0 x=1일 때, f(1)=4-[4]=0
fA'B(k)= f(k)×fõ(k)=0 즉, 함수 y= f(x)의 그래프는 다음과 같다.

70 수학 <하>
y 함수 y=( f ç f)(x)의 그래프는 다음과 같다.
y=f{x}
1 y
y={ f©f}{x}

2
y=1
O 2 4 x
O 1 1 3 1 x 함수 y=( f ç f)(x)의 그래프와 직선 y=1의 교점의 x좌표가
4 2 4
각각 -1, 1, 3, 5이므로 방정식 ( f ç f)(x)=1의 서로 다른 실근의
방정식 ( f ç f)(x)= f(x)에서 f(x)=t라 하면 f(t)=t이다.
합은 (-1)+1+3+5=8이다.
y y=x
y=f{x}
1

309 10
O 1 1 3 1 x
4 2 4 다항함수 f(x)가 n차`(n은 자연수)일 때,
nÛ`
함수 y= f(x)의 그래프와 직선 y=x의 교점은 위와 같이 3개이므로 f( f(x))의 최고차항은 (xÇ` )Ç` =x 항이므로 차수가 nÛ`이다.
세 교점의 x좌표를 각각 0, a, b`(0<a<b)라 하면 yy 참고 따라서 조건 ㈎를 만족시키려면 nÛ`=1에서 n=1이므로 Ⅴ
f(x)는 일차함수이다. 함
방정식 f(t)=t의 해는 t=0 또는 t=a 또는 t=b이므로
일차함수인 f(x)는 역함수가 존재하므로 수
f(x)=0 또는 f(x)=a 또는 f(x)=b이다. 와
y
f( f(x))=x에서 f(x)= f ÑÚ`(x)이다. 그
y=f{x} y=x
따라서 조건 ㈏에서 f(-1)=2이면 f ÑÚ`(-1)= f(-1)=2이므로 래
1 프
f(2)=-1이다.
y=b
f(x)=ax+b`(a, b는 실수)라 하면
y=a f(-1)=2, f(2)=-1이므로
-a+b=2, 2a+b=-1에서 a=-1, b=1
O 1 1 3 1 x
4 2 4 따라서 f(x)=-x+1이다.
3 5
위와 같이 방정식 f(x)=0, f(x)=a, f(x)=b의 근의 개수가 각각 g(x)=xÛ`+3x+1={x+ }Û`- 에서
2 4
5, 4, 4이므로 모든 근의 개수는
5 5
5+4+4=13 g( f(x))={-x+ }Û`- 이므로
2 4
참고 5 5
함수 (g ç f)(x)는 x= 일 때 최솟값 - 를 가진다.
2 4
두 직선 y=4x-1, y=x의 교점의 x좌표는 4x-1=x에서
5 5
1 1 m= , n=-
x= 이므로 a= 이고, 2 4
3 3
∴ 6m+4n=15-5=10
두 직선 y=4x-2, y=x의 교점의 x좌표는 4x-2=x에서
2 2 참고
x= 이므로 b= 이다.
3 3 다항함수 f(x)가 모든 실수 x에 대하여 f( f(x))=x를
만족시키면 f(x)는 일차함수이고, 이때 f(x)= f ÑÚ`(x)이므로
함수 y= f(x)의 그래프가 직선 y=x에 대하여 대칭이다.

308 8 따라서 함수 y= f(x)는 y=x 또는 기울기가 -1인 직선이다.


즉, 모든 실수 x에 대하여 f( f(x))=x를 만족시키는 다항함수
방정식 ( f ç f)(x)=( f ç f ç f)(x)에서 f(x)는 다음의 2가지뿐이다.
( f ç f)(x)= f(( f ç f)(x))이므로 ( f ç f)(x)=t라 하면 ❶ f(x)=x
f(t)=t이다. ❷ f(x)=-x+k (k는 상수)
|t-2|=t의 양변을 제곱하면 (t-2)Û`=tÛ`, -4t+4=0
즉, t=1이므로 ( f ç f)(x)=1이다.
f(x)=|x-2|에 대하여 함수 y= f(x)의 그래프는 다음과 같고,
y
y=f{x}

310 8

2
f(x)=g(x)가 되도록 하는 x의 값은
O 2 x
xÛ`+2x=xÜ`-axÛ`+ax+2a에서

Ⅴ. 함수와 그래프 71
xÜ`-(a+1)xÛ`+(a-2)x+2a=0 따라서 f(2), f(3)의 값을 정하는 방법의 수는
(x+1)(x-2)(x-a)=0이므로 5+4+3+2+1=15이고,
x=-1 또는 x=2 또는 x=a이다. f(1), f(4)의 값을 정하는 방법의 수가 각각 5이므로
이때, a=-1 또는 a=2일 경우 집합 X가 될 수 있는 집합은 n(A)=15_5_5=375이다.
X={-1}, X={2}, X={-1, 2}이므로 Û n(B)는 일대일함수인 f 의 개수이므로
각 집합 X의 모든 원소의 합을 모두 더하면 n(B)=5_4_3_2=120이다.
-1+2+(-1+2)=2에서 주어진 조건을 만족시키지 않는다. Ü n(A;B)는 f(2)É f(3)이면서 일대일함수인 f 의 개수이다.
따라서 a+-1, a+2이고, 집합 X는 -1, 2, a 중에서 원소를 yy TIP
갖는 집합이므로 집합 X의 개수는 k=2Ü`-1=7이고, 각 집합은 f(2)=1일 때, f(3)으로 가능한 값은 2, 3, 4, 5의 4가지
{-1}, {2}, {a}, {-1, 2}, {-1, a}, {2, a}, {-1, 2, a} f(2)=2일 때, f(3)으로 가능한 값은 3, 4, 5의 3가지
이므로 각 집합의 모든 원소의 합을 모두 더하면 f(2)=3일 때, f(3)으로 가능한 값은 4, 5의 2가지
4_(-1+2+a)=8에서 a=1 f(2)=4일 때, f(3)으로 가능한 값은 5의 1가지
∴ a+k=1+7=8 f(2)=5일 때, f(3)으로 가능한 값은 존재하지 않는다.
f(2), f(3)의 값을 정하는 방법의 수는
4+3+2+1=10이고,
f(1)로 가능한 값은 1, 2, 3, 4, 5 중 f(2), f(3)의 값을 제외한
3가지,
311 5
f(4)로 가능한 값은 1, 2, 3, 4, 5 중 f(1), f(2), f(3)의 값을
f(4)=2, g(4)=3에서 g(4)> f(4)이므로 함수 h(x)의 정의에 제외한 2가지이므로
의하여 h(4)=g(4)=3 yy ㉠ n(A;B)=10_3_2=60
함수 h(x)는 일대일대응이므로 h(3)+3이고 g(3)=3이므로 Ú ~ Ü에서 n(A'B)=375+120-60=435
h(3)= f(3)이고 f(3)¾g(3)=3이어야 하므로 TIP
h(3)= f(3)=4 yy ㉡
f(2)É f(3)을 만족시키는 함수 f 가 일대일함수이면
㉠, ㉡에서 h(1)+3, h(1)+4이고
f(2)+ f(3)이므로 f(2)< f(3)이다.
g(1)=2에서 h(1)+1이므로 h(1)=2, h(2)=1
따라서 h(2)=g(2)= f(2)=1이다.
∴ f(2)+h(3)=1+4=5

참고1 313 ⑴ 15 ⑵ 50

á f(x) ( f(x)¾g(x)) ⑴ 2Å` 의 일의 자리의 수는 네 수 2, 4, 8, 6이 차례로 반복된다.


함수 h(x)=Ò 는 f(x), g(x)의 함숫값
» g(x) (g(x)> f(x)) 따라서 함수 (g ç f)(x)의 함숫값은
중 작지 않은 값을 함숫값으로 갖는 함수이다. g( f(1))=2k, g( f(2))=4k, g( f(3))=8k, g( f(4))=6k가
차례로 반복되므로
참고2 함수 g ç f의 치역은 {2k, 4k, 6k, 8k}이고,`(k+0)
문제의 조건만으로는 f(1)의 값이 정해지지 않는다. 치역의 모든 원소의 합은 2k+4k+6k+8k=20k이다.
즉, 함수 f(x)는 f(1)=1인 경우와 f(1)=2인 경우의 2가지가 20k=300에서 k=15
존재한다. ⑵ 함수 ( f ç g)(x)=(2û` Å` 의 일의 자리의 수)이므로
자연수 m에 대하여
k=1, 5, 9, y, 4m-3일 때,
함수 f ç g의 치역은 {2, 4, 8, 6},
k=2, 6, 10, y, 4m-2일 때,
함수 f ç g의 치역은 {4, 6},
312 ④
k=3, 7, 11, y, 4m-1일 때,
n(A'B)=n(A)+n(B)-n(A;B)이다. 함수 f ç g의 치역은 {8, 4, 2, 6},
Ú n(A)는 f(2)É f(3)을 만족시키는 함수 f 의 개수이다. k=4, 8, 12, y, 4m일 때,
f(2)=1일 때, f(3)으로 가능한 값은 1, 2, 3, 4, 5의 5가지 함수 f ç g의 치역은 {6}이다.
f(2)=2일 때, f(3)으로 가능한 값은 2, 3, 4, 5의 4가지 따라서 n({( f ç g)(x)|x는 자연수})=2를 만족시키는 자연수
f(2)=3일 때, f(3)으로 가능한 값은 3, 4, 5의 3가지 k는 k=4m-2`(m은 자연수)이므로 조건을 만족시키는 20
f(2)=4일 때, f(3)으로 가능한 값은 4, 5의 2가지 이하의 자연수 k는 2, 6, 10, 14, 18이다.
f(2)=5일 때, f(3)으로 가능한 값은 5의 1가지 ∴ 2+6+10+14+18=50

72 수학 <하>
y=f{x}
314 ②

합성함수 h ç g ç f 가 상수함수가 되는 경우를 살펴보면 다음과 같다.


Ú 함수 h가 상수함수인 경우
{1,`a-2}
f g h
A A A A

1 1 1 1
방정식 f(x)=g(x)가 서로 다른 세 실근을 가지려면 두 함수
2 2 2 2 y= f(x), y=g(x)의 그래프가 서로 다른 세 점에서 만나야 한다.
이때, 함수 y= f(x)의 그래프가 증가하는 형태이므로 함수의
모든 함수 f, g 에 대하여 h ç g ç f 는 상수함수이다. 그래프가 역함수의 그래프와 만나는 교점은 직선 y=x와의 교점과
함수 h가 상수함수인 경우는 h(x)=1 또는 h(x)=2의 같다.
2가지이고, 함수 f 와 g의 개수는 각각 2Û`=4이므로 즉, 함수 y= f(x)의 그래프와 직선 y=x가 서로 다른 세 점에서
구하는 함수의 개수는 만나야 한다.
2_4_4=32 Ú x¾1에서 함수 y= f(x)의 그래프가 직선 y=x와 접할 때
Û 함수 h가 상수함수가 아닌 경우 이차방정식 2xÛ`-4x+a=x, 즉 2xÛ`-5x+a=0의 판별식을
함수 h는 h(1)=1, h(2)=2 또는 h(1)=2, h(2)=1의 25
D라 하면 D=25-8a=0이므로 a= Ⅴ
2가지이다. 8
( - ①) 함수 g 가 상수함수인 경우 Û x<1에서 함수 y= f(x)의 그래프가 직선 y=x와 접할 때 함

f g h 이차방정식 -xÛ`+2x+a-3=x, 즉 xÛ`-x-a+3=0의 와
A A A A
11 그
판별식을 D라 하면 D=1+4a-12=0이므로 a= 래
1 1 1 1 4

y
2 2 2 2 y=x
25
a=
8
모든 함수 f 에 대하여 h ç g ç f 는 상수함수이다.
함수 g 가 상수함수인 경우는 g(x)=1 또는 g(x)=2의 11
a=
2가지이고, 함수 f의 개수는 2Û`=4이므로 구하는 함수의 개수는 4

2_2_4=16 O
x
( - ②) 함수 g 가 상수함수가 아닌 경우
f g h Ú, Û에 의해 함수 y= f(x)의 그래프와 직선 y=x는
A A A A
25 11
1 1 1 1 a> 또는 a< 일 때 한 점에서 만나고,
8 4
2 2 2 2 25 11
a= 또는 a= 일 때 두 점에서 만나고,
8 4
h ç g ç f 가 상수함수이려면 f 가 상수함수이어야 한다. 11 25
<a< 일 때 세 점에서 만난다.
4 8
함수 g가 상수함수가 아닌 경우는
11 25
g(1)=1, g(2)=2 또는 g(1)=2, g(2)=1의 2가지이고, 따라서 구하는 실수 a의 값의 범위는 <a< 이다.
4 8
함수 f 가 상수함수인 경우는 f(x)=1 또는 f(x)=2의
2가지이므로 구하는 함수의 개수는
2_2_2=8 316 172

Ú, Û에서 구하는 함수의 개수는 조건 ㈎에 의하여


32+16+8=56이다. á x-1 (1ÉxÉ2)
f(x)=Ò yy ㉠
» -x+3 (2<xÉ3)
조건 ㈏에 의하여
11 25
315 4
<a<
8 f(2015)= f {3_
2015
3
}

함수 f(x)에서 2015 2015


=3 f { }=3 f {3_ }
3 3Û`
x¾1일 때, y=2(x-1)Û`+a-2의 그래프는 꼭짓점의 좌표가
2015 2015
(1, a-2)이고 아래로 볼록하고, =3Û` f { }=3Û` f {3_ }
3Û` 3Ü`
x<1일 때, y=-(x-1)Û`+a-2의 그래프는 꼭짓점의 좌표가

(1, a-2)이고 위로 볼록하다.
2015
즉, 함수 y= f(x)의 그래프의 개형은 다음과 같다. =3ß` f { }
3ß`

Ⅴ. 함수와 그래프 73
2015 f(5)= f(4+1)= f(4) f(1)- f(4-1)
이때, 2< É3이므로
3ß` = f(4)- f(3)=-1-(-2)=1
2015 2015 f(6)= f(5+1)= f(5) f(1)- f(5-1)
f(2015)=3ß` f { }=3ß`_{- +3} (∵ ㉠)
3ß` 3ß`
= f(5)- f(4)=1-(-1)=2
172
=3ß`_ =172 f(7)= f(6+1)= f(6) f(1)- f(6-1)
3ß`
= f(6)- f(5)=2-1=1

317 ① 이므로 자연수 n에 대하여 f(n)의 값은 1, -1, -2, -1, 1, 2가


반복된다.
등식 f(2x+y)=4 f(x)+ f(y)+2xy+12의 양변에 y 대신 -x를 999=6_166+3이므로
대입하면 f(999)= f(3)=-2
f(x)=4 f(x)+ f(-x)-2xÛ`+12
3 f(x)+ f(-x)=2xÛ`-12 yy ㉠
이 식의 양변에 x 대신 -x를 대입하면
3 f(-x)+ f(x)=2xÛ`-12 yy ㉡ 319 ④

㉠의 양변에 3을 곱한 식에서 ㉡의 양변을 각각 빼면 f(x+y)= f(x) f(y) yy ㉠


8 f(x)=4xÛ`-24이므로 ㄱ. ㉠에 x=1, y=0을 대입하면
1 f(1)= f(1) f(0)에서 2=2 f(0)이므로 f(0)=1이다. (거짓)
f(x)= xÛ`-3
2
ㄴ. f(1)=2이므로
∴ f(4)=5
f(2)= f(1+1)= f(1) f(1)=4
다른 풀이 f(3)= f(2+1)= f(2) f(1)=8
f(2x+y)=4 f(x)+ f(y)+2xy+12 yy ㉢ f(4)= f(3+1)= f(3) f(1)=16
㉢에 x=0, y=0을 대입하면 f(5)= f(4+1)= f(4) f(1)=32 (참)
f(0)=4 f(0)+ f(0)+12에서 f(0)=-3이다. ㄷ. f(x)=a, f(y)=b라 하면
㉢에 y=0을 대입하면 x= f ÑÚ`(a), y= f ÑÚ`(b)이다.
f(2x)=4 f(x)+9 (∵ f(0)=-3) yy ㉣ ㉠에서 f(x+y)= f(x) f(y)=ab이므로
㉢에 x=1, y=-1을 대입하면 f ÑÚ`(ab)=x+y= f ÑÚ`(a)+ f ÑÚ`(b)이다. (참)
f(1)=4 f(1)+ f(-1)+10 따라서 옳은 것은 ㄴ, ㄷ이다.
3 f(1)+ f(-1)+10=0 yy ㉤
㉢에 x=-1, y=1을 대입하면
f(-1)=4 f(-1)+ f(1)+10
3 f(-1)+ f(1)+10=0 yy ㉥
320 2

5 함수 f 가 일대일대응이므로 f Û` â` Ú`¡`(1)+ f Û`â`Ú`á`(2)=5에서


두 식 ㉤, ㉥을 연립하여 풀면 f(-1)= f(1)=-
2 f Û` â` Ú`¡`(1)=2, f Û` â` Ú`á`(2)=3 또는 f Û` â` Ú` ¡`(1)=3, f Û`â`Ú`á`(2)=2이다.
㉣에 x=1을 대입하면 이때, f(1)=1이거나 f(3)=3이면 위의 조건을 만족시키지 않으므로
f(2)=4 f(1)+9=-1 f(1)+1, f(3)+3이다.
다시 ㉣에 x=2를 대입하면 f(1)의 값에 따라 경우을 나누어 생각하면 다음과 같다.
f(4)=4 f(2)+9=5 Ú f(1)=2인 경우 f(3)+3을 만족시키는 함수 f 는 다음과 같다.
f
X X
1 1
318 ①
2 2
등식 f(x+y)= f(x) f(y)- f(x-y)의
3 3
양변에 x=1, y=0을 대입하면
f(1)= f(1) f(0)- f(1)이고, f(1)=1이므로 f(0)=2이다. 이때, f(1)=2, f Û`(1)=3, f Ü`(1)=1이므로
f(2)= f(1+1)= f(1) f(1)- f(1-1) f Ü`=I`(I는 항등함수)이다.
3_672+2 3_673
= f(1)- f(0)=1-2=-1 즉, f Û` â` Ú`¡`= f = f Û`, f Û` â` Ú`á`= f =I이므로
f(3)= f(2+1)= f(2) f(1)- f(2-1) f Û` â` Ú`¡`(1)+ f Û` â` Ú`á`(2)= f Û`(1)+I(2)=3+2=5를 만족시킨다.
= f(2)- f(1)=-1-1=-2 Û f(1)=3인 경우 f(3)의 값에 따라 경우를 나누어 생각하면
f(4)= f(3+1)= f(3) f(1)- f(3-1) 다음과 같다.
= f(3)- f(2)=-2-(-1)=-1 ( - ①) f(3)=1인 경우 함수 f 는 다음과 같다.

74 수학 <하>
f h= f ç g 를 만족시키는 함수 f 가 유일하게 결정되므로
X X
yy TIP
1 1
구하는 순서쌍 ( f, g)의 개수는 3_6=18이다. (거짓)
2 2
yy 참고
3 3
따라서 옳은 것은 ㄱ, ㄴ이다.
이때, f(1)=3, f Û`(1)=1이고, f(2)=2이므로 TIP
f Û`â`Ú`¡`(1)=1, f Û`â`Ú`á`(2)=2에서 f Û`â`Ú`¡`(1)+ f Û` â` Ú`á`(2)=3이므로
함수 g 는 일대일대응이므로 역함수 gÑÚ`가 존재한다.
f Û`â`Ú`¡`(1)+ f Û`â`Ú`á`(2)=5를 만족시키지 않는다.
따라서 h= f ç g 에서 함수 g 가 Ú ~ ß 중의 하나이면 함수 f는
( - ②) f(3)=2인 경우 함수 f는 다음과 같다.
f=h ç gÑÚ`로 유일하게 결정된다.
f
X X
1 1 참고

2 2 ㄷ에서 f ç g=h가 되는 두 함수 f, g의 순서쌍 ( f, g)를 각각


구해 보면 다음과 같다.
3 3
f ç g가 Ú이 되는 경우 두 함수 f, g의 순서쌍은
이때, f(1)=3, f Û`(1)=2, f Ü`(1)=1이므로 f Ü`=I이다. ( , ), ( , ), ( , ), ( , ), ( , ), ( , )으로 6개이고,
즉, f Û`â`Ú` ¡`= f
3_672+2
= f Û`, f Û`â`Ú`á`= f
3_673
=I이므로 f ç g가 Ý가 되는 경우 두 함수 f, g의 순서쌍은


f Û`â`Ú`¡`(1)+ f Û`â`Ú`á`(2)= f Û`(1)+I(2)=2+2=4이므로 ( , ), ( , ), ( , ), ( , ), ( , ), ( , )으로 6개이고, 수

f Û`â`Ú`¡`(1)+ f Û`â`Ú`á`(2)=5를 만족시키지 않는다. f ç g가 Þ가 되는 경우 두 함수 f, g의 순서쌍은

Ú, Û에서 조건을 만족시키는 함수 f 는 Ú과 같다. ( , ), ( , ), ( , ), ( , ), ( , ), ( , )으로 6개이다. 래
3_673+1 3_674 프
f Û`â`Û` â`= f = f, f Û`â`Û` Û`= f =I이므로
f Û`â`Û` â`(3)+ f Û`â`Û` Û`(1)= f(3)+I(1)=1+1=2
322 ③

f(5)=4이므로 ( f ç f ç f)(5)=( f ç f)(4)=3이다.


321 ② 이때, f(4)=4 또는 f(4)=5이면 ( f ç f)(4)=4가 되므로
f(4)=1 또는 f(4)=2 또는 f(4)=3이다.
ㄱ. 집합 A={1, 2, 3}에서 A로의 일대일대응인 함수의 개수는
( f ç f)(1)=1이기 위한 조건으로 경우를 나누어 생각하면 다음과
3_2_1=6이므로 n(S)=6이다. (참)
같다.
ㄴ. (SÁ'Sª'S£)‚``=SÁ‚``;Sª‚``;S£‚``이고,
Ú f(1)=1일 때
집합 SÁ‚``, Sª‚``, S£‚``은 각각 f(1)=1, f(2)=2, f(3)=3인
f
함수의 집합이다. X X
따라서 집합 SÁ‚``;Sª‚``;S£‚``은 f(1)=1, f(2)=2, f(3)=3을 1 1
모두 만족시키는 함수의 집합이므로 원소는 항등함수뿐이다. 2 2
즉, f<(SÁ'Sª'S£)‚``이면 f=I이다. (참) 3 3

ㄷ. A에서 A로의 일대일대응을 모두 나타내면 다음과 같다. 4 4


5 5
Ú A A A A A A
ÛA AA A A A Ü
A A A A A A
1 1 1 1 1 1 1 1 1 1 1 1 1 1 1 1 1 1 ( f ç f)(4)=3을 만족시키기 위해서 f(4)+1이다.
2 2 2 2 2 2 2 2 2 2 2 2 2 2 2 2 2 2 f(4)=2, f(2)=3일 때 f(3)으로 가능한 값은 1, 2, 3, 4, 5의
3 3 3 3 3 3 3 3 3 3 3 3 3 3 3 3 3 3 5가지이고,
f(4)=3, f(3)=3일 때 f(2)로 가능한 값은 1, 2, 3, 4, 5의
Ý A A A A A A
ÞA AA A A A ß
A A A A A A 5가지이므로
1 1 1 1 1 1 1 1 1 1 1 1 1 1 1 1 1 1 함수 f의 개수는 5+5=10이다.
2 2 2 2 2 2 2 2 2 2 2 2 2 2 2 2 2 2 Û f(1)=2, f(2)=1일 때
3 3 3 3 3 3 3 3 3 3 3 3 3 3 3 3 3 3 f
X X
1 1
일대일대응인 두 함수의 합성함수도 일대일대응이므로
2 2
h= f ç g도 A에서 A로의 일대일대응이다.
3 3
h ç h ç h=I를 만족시키는 함수 h를 위의 6개의 일대일대응인
4 4
함수 중에서 찾으면 Ú, Ý, Þ의 3개이다.
5 5
이때, 함수 h에 대하여 함수 g가 Ú ~ ß의 6개 중의 하나이면

Ⅴ. 함수와 그래프 75
( f ç f)(4)=3을 만족시키기 위해서 f(4)+1, f(4)+2이다.
f(4)=3, f(3)=3일 때 1가지이므로
324 풀이 참조

함수 f의 개수는 1이다. ⑴ 0<x<60일 때 y=1000이고, x¾60일 때


Ü f(1)=3, f(3)=1일 때 0Éx-60<20이면 y=1000+500
f 20Éx-60<40이면 y=1000+2_500
X X
40Éx-60<60이면 y=1000+3_500
1 1
2 2

3 3 20(n-1)Éx-60<20n이면 y=1000+500n이다.
4 4 x-60 x-60
n-1É <n일 때, [ ]=n-1이므로
20 20
5 5
x-60
y=1000+500{[ ]+1}
( f ç f)(4)=3을 만족시키기 위해서 f(4)+3이다. 20
f(4)=1일 때 f(2)로 가능한 값은 1, 2, 3, 4, 5의 5가지이고, x-60 x
=1500+500[ ]=1500+500[ -3]
20 20
f(4)=2, f(2)=3일 때 1가지이므로
x x
함수 f 의 개수는 5+1=6이다. =1500+500{[ ]-3}=500[ ]
20 20
Ý f(1)=4, f(4)=1이면 ( f ç f)(4)=4이므로 조건을 ( 1000 (0<x<60)
만족시키지 못하고, 따라서 f(x)={ x 이다.
500[ ] (x¾60)
f(1)=5이면 f(5)=4이므로 ( f ç f)(1)=1을 만족시키지 9 20
못한다. ⑵ 3시간 45분은 225분이므로
Ú ~ Ý에서 구하는 함수 f의 개수는 10+1+6=17이다. 225
f(225)=500[ ]=500_11=5500
20
따라서 3시간 45분 동안 주차했을 때 주차요금은 5500원이다.
323 5

함수 f(x)의 역함수가 존재하므로 함수 f(x)는 일대일대응이다.


1 1
g(x)=(a+b+2){ + }x, h(x)=-(x-c)Û`+4라
3a 3b+6 325 12
하면 함수 y=g(x)의 그래프는 기울기가 양수인 직선`
집합 A는 함수 f(x)의 치역이다.
(∵ a>0, b>0)이고, x=3일 때 함숫값이
Ú x가 정수일 때
1 1
g(3)=(a+b+2){ + }_3 [x]=x이므로
3a 3b+6
1 1 [x] 3x
={a+(b+2)}{ + } f(x)=[ ]+[ ]=[1]+[3]=4
a b+2 x [x]
a b+2 Û x가 정수가 아닐 때
= + +2
b+2 a [x]
x¾1이므로 1É[x]<x에서 [ ]=0이므로
a b+2 x
¾2¾Ð + +2=4 yy ㉠ [x]
b+2 a 3x 3x
f(x)=[ ]+[ ]=[ ]
이므로 x¾3에서 f(x)¾4이다. x [x] [x]
함수 y=h(x)의 그래프는 꼭짓점이 (c, 4)인 위로 볼록한 3x 3x
이때, [x]=n`(n은 자연수)이면 f(x)=[ ]=[ ]이고,
[x] n
이차함수의 그래프이므로 h(x)É4이다.
3x 3
n<x<n+1이므로 3< <3+ 이다.
n n
( - ①) n=1일 때
3x
3< <6이므로
{3,`4} n
3x
f(x)=[ ]의 값은 3 또는 4 또는 5이다.
n
따라서 함수 f(x)가 일대일함수이고, 치역이 실수 전체의 집합
( - ②) n=2일 때
R이기 위해서 g(3)=h(3)=4이어야 한다.
3x 9
이때, g(3)=4이면 ㉠에서 등호가 성립하므로 3< < 이므로
n 2
a b+2 3x
= 에서 aÛ`=(b+2)Û` f(x)=[ ]의 값은 3 또는 4이다.
b+2 a n
a=b+2 (∵ a>0, b>0)이므로 a-b=2 ( - ③) n¾3인 경우
또한 h(3)=4에서 c=3 3 3x
3+ É4이므로 3< <4에서
∴ a-b+c=2+3=5 n n

76 수학 <하>
3x y
f(x)=[ ]의 값은 3이다. y=f™{x}
n 2
따라서 A={ f(x)|x¾1}={3, 4, 5}이므로 1
집합 A의 모든 원소의 합은 3+4+5=12이다. O 2 4 6 8 10 12 x

y=g{x}
따라서 두 함수 y= fª(x)와 y=g(x)의 그래프가 서로 다른 네
1 1
점에서 만나도록 하는 a의 값의 범위가 - Éa<- 이므로
1 2 4
326 ⑴1 ⑵-
2 1
구하는 a의 최솟값은 - 이다.
2
3Ú` `â `â
⑴ fÁ¼(3Ú` â` â`)=3Ú` â` â`-10[ ] yy ㉠
10
3Ú` `â `â k
이때, 3Ú` â` â`은 자연수이므로 =n+ 라 하자.
10 10
(n은 자연수, k는 0ÉkÉ9인 정수) 327 ③

3Ú` `â `â 함수 y= f(x)의 그래프가 점 (a, a)`(a>0)를 지나고 기울기가


[ ]=n이고, 3Ú` â` â`=10n+k이므로 yy ㉡
10
1보다 큰 직선이므로 이와 직선 y=x에 대하여 대칭인 함수
㉠에 대입하면 fÁ¼(3Ú` â` â`)=10n+k-10n=k이고,
y= f ÑÚ`(x)의 그래프는 점 (a, a)를 지나고 기울기가 1보다 작은 Ⅴ
㉡에서 k는 3Ú` â` â`의 일의 자리의 수와 같다. 함
직선이다.
3µ` (m은 자연수)의 일의 자리의 수는 3, 9, 7, 1이 반복되므로 수
á f(x) (x<a) 와
100=4_25에서 3Ú` â` â`의 일의 자리의 수는 1이다. 즉, 함수 g(x)=Ò 의 그래프가 다음과 같다.
» f ÑÚ`(x) (x¾a) 그
∴ fÁ¼(3Ú` â` â`)=1 래
y y=x 프
x
⑵ 함수 fª(x)=x-2[ ]는 y=g{x}
2
0Éx<2일 때 fª(x)=x
2Éx<4일 때 fª(x)=x-2 a
4Éx<6일 때 fª(x)=x-4
⋮ O a x
이므로 0Éx<12에서 함수 y= fª(x)의 그래프는 다음과 같다.
ㄱ. (h ç hÑÚ`)(x)=x이므로 방정식 g(x)=(h ç hÑÚ`)(x)의 해는
y
y=f™{x} 함수 y=g(x)의 그래프와 직선 y=x의 교점의 x좌표와 같다.
2 두 그래프가 점 (a, a)에서만 만나므로 방정식의 해는
x=a 뿐이다. (참)
O 2 4 6 8 10 12 x
ㄴ. g(h(x))=g(x+a), h(g(x))=g(x)+a이다.
한편, 함수 y=g(x)의 그래프는 함수 y=g(x+a)의 그래프는 함수 y=g(x)의 그래프를 x축의
0Éx<6일 때, 조건 ㈎에 의하여 방향으로 -a만큼 평행이동한 것이고,
점 (0, 1)을 지나고 기울기가 a인 직선이고, 함수 y=g(x)+a의 그래프는 함수 y=g(x)의 그래프를 y축의
6Éx<12일 때, 조건 ㈏에 의하여 방향으로 a만큼 평행이동한 것이므로
0Éx<6일 때의 함수 y=g(x)의 그래프를 x축의 방향으로 다음과 같이 한 점에서만 만난다. yy TIP
6만큼 평행이동한 것이므로 y y=h{g{x}}
á ax+1 (0Éx<6)
g(x)=Ò 이다. 2a
» a(x-6)+1 (6Éx<12) y=g{h{x}}
1 a
직선 y=ax+1이 점 (2, 0)을 지날 때 a=- 이고,
2
1
점 (4, 0)을 지날 때 a=- 이므로 a x
4 O
1 1
- Éa<- 일 때, 0Éx<6에서 두 함수 y= fª(x), 그러므로 방정식 g(h(x))=h(g(x))의 해의 개수는 1이다.
2 4
y=g(x)의 그래프가 두 점에서 만난다. (거짓)
조건 ㈏에 의하여 6Éx<12에서 두 함수 y= fª(x), y=g(x)의 ㄷ. h(x)=x+a의 역함수는 hÑÚ`(x)=x-a이므로
그래프는 0Éx<6에서의 그래프와 같으므로 (h ç g ç hÑÚ`)(x)=(h ç g)(x-a)=h(g(x-a))
1 1 =g(x-a)+a
- Éa<- 일 때, 6Éx<12에서 두 함수의 그래프가
2 4 에서 이 함수의 그래프는 함수 y=g(x)의 그래프를 x축의
두 점에서 만난다. 방향으로 a만큼, y축의 방향으로 a만큼 평행이동한 것이다.

Ⅴ. 함수와 그래프 77
h ç gÑÚ` ç hÑÚ`=h ç (h ç g)ÑÚ`={(h ç g) ç hÑÚ`}ÑÚ` y
1
에서 함수 y=(h ç gÑÚ` ç hÑÚ`)(x)가 함수 y=(h ç g ç hÑÚ`)(x)의
역함수이므로 두 함수의 그래프는 직선 y=x에 대하여
대칭이다. 1
2
y 1
y={h©g—!©h—!}{x}
y=x 1 4
8
O 1 2 1 1 2 1 x
1 29 9 3 2 3
2a y={h©g©h—!}{x}
27 27
14 1 1 1 2
조건 ㈎에 의하여 f { }=1- f { }이고, É É 이므로
O 2a x 15 15 27 15 27
1 1
그러므로 모든 실수 x에 대하여 f{ }=
15 8
(h ç g ç hÑÚ`)(x)É(h ç gÑÚ` ç hÑÚ`)(x)이다. (참) 14 1 7
∴ f{ }=1- =
따라서 옳은 것은 ㄱ, ㄷ이다. 15 8 8

TIP
x<0에서 두 함수 y=h(g(x)), y=g(h(x))의 그래프가
평행하므로 만나지 않는다.
0ÉxÉa에서 두 함수 y=h(g(x)), y=g(h(x))의 그래프는
한 점에서만 만난다.
x>a에서 두 함수 y=h(g(x)), y=g(h(x))의 그래프가
평행하므로 만나지 않는다.

7
328 8

조건 ㈎에 의하여 함수 y= f(x)의 그래프는


1 1
점 { , }에 대하여 대칭이다. yy ㉠
2 2
조건 ㈏의 식에 x=0을 대입하면
f(0)
f(0)= 에서 f(0)=0이고, ㉠에 의하여 f(1)=1이다.
2
조건 ㈏의 식에 x=1을 대입하면
1 f(1) 1 2 1
f { }= = 이고, ㉠에 의하여 f { }= 이다.
3 2 2 3 2
즉, 함수 y= f(x)의 그래프가
1 1 1 1 2 1
세 점 { , }, { , }, { , }을 지나므로
3 2 2 2 3 2
1 2 1
조건 ㈐에 의하여 ÉxÉ 일 때 f(x)= 이다.
3 3 2
y
1

1
2

O 1 1 2 1 x
3 2 3

1 2 1
이때, 조건 ㈏에 의하여 ÉxÉ 에서 f(x)= 이고,
9 9 4
1 2 1
다시 조건 ㈏에 의하여 ÉxÉ 에서 f(x)= 이다.
27 27 8

78 수학 <하>
2x-7
유리함수와 무리함수 따라서 유리함수 y= 의 그래프의 점근선의 방정식은
x+3
x=-3, y=2임을 빠르게 구할 수 있다.

329 ②
332 ③
x-1 xÜ`+1 x+1
Ö _ 4x+3 4(x+2)-5 -5
x xÛ`-2x xÛ`-3x+2 함수 y= = = +4의 그래프를 x축의
x+2 x+2 x+2
x-1 x(x-2) x+1
= _ _ 방향으로 2만큼, y축의 방향으로 -4만큼 평행이동하면
x (x+1)(xÛ`-x+1) (x-1)(x-2)
1 -5 -5
= y= +4-4= 이므로
xÛ`-x+1 (x-2)+2 x
a=2, b=-4, c=-5
∴ a+b+c=-7

330 13
333 ④
주어진 등식의 좌변의 분모를 같게 하면
3 3
a b a(x+3)+b(x-2) 함수 y= +1의 그래프는 함수 y= 의 그래프를 x축의
+ = x-2 x Ⅴ
x-2 x+3 (x-2)(x+3)
방향으로 2만큼, y축의 방향으로 1만큼 평행이동한 것이므로 함
(a+b)x+(3a-2b) 수
= 3
xÛ`+x-6 <보기>의 함수의 그래프 중 평행이동하여 함수 y= 의 그래프와 와
x
이므로 x에 대한 항등식 그
겹쳐지는 것을 찾으면 된다. 래
(a+b)x+(3a-2b) 5x 프
= x-3 3 3
xÛ`+x-6 xÛ`+x-6 ㄱ. 함수 y= =1- 의 그래프는 함수 y= 의 그래프를
x x x
에서 a+b=5, 3a-2b=0
y축에 대하여 대칭이동한 후 y축의 방향으로 1만큼 평행이동한
a=2, b=3
것이다.
∴ aÛ`+bÛ`=2Û`+3Û`=13
3
따라서 평행이동만으로 함수 y= 의 그래프와 겹쳐질 수 없다.
x
2x+1 2(x-1)+3 3
ㄴ. 함수 y= = = +2의 그래프는
x-1 x-1 x-1
331 ②
3
함수 y= 의 그래프를 x축의 방향으로 1만큼,
2x-7 2(x+3)-13 -13 x
y= = = +2에서
x+3 x+3 x+3 y축의 방향으로 2만큼 평행이동한 것이다.
점근선의 방정식은 x=-3, y=2이므로 yy TIP ㄷ. 함수 y=
6x+3 3(2x-1)+6
=
2x-1 2x-1
a=-3, b=2이다.
6 3
∴ a+b=-1 = +3= +3
2x-1 1
x-
2
TIP
3 1
의 그래프는 함수 y= 의 그래프를 x축의 방향으로 만큼,
rx+s x 2
<유리함수 y= 의 그래프의 점근선의 방정식>
px+q y축의 방향으로 3만큼 평행이동한 것이다.
rx+s 3
유리함수 y= (ps-qr+0, p+0)의 그래프의 점근선의 따라서 평행이동하여 함수 y= +1의 그래프와 겹쳐지는 것은
px+q x-2
q r ㄴ, ㄷ이다.
방정식은 x=- , y= 이다.
p p
즉, 점근선의 방정식은 다음과 같이 구할 수 있다.
x=(분모를 0으로 만드는 x의 값),
334 ④

y=(분모, 분자의 x항의 계수의 비) 주어진 그래프에서 점근선이 두 직선 x=-1, y=2이므로


[증명] k
유리함수를 y= +2 (k는 0이 아닌 상수)라 하자.
r s x+1
x+
rx+s p p 이 함수의 그래프가 점 (-3, 0)을 지나므로 yy TIP
y= = q
px+q
x+ k
p +2=0, k=4
r q rq s rq s -2
x+ }- + - +
p{ p pÛ` p pÛ` p r 따라서 주어진 그래프를 나타내는 유리함수는
= q = q +
p 4 4+2(x+1) 2x+6
x+ x+ y= +2= = 이므로
p p x+1 x+1 x+1

Ⅴ. 함수와 그래프 79
a=2, b=6, c=1 y=3이므로 y절편이 양수이다. 따라서 그래프가 제1사분면을
∴ a+b+c=9 지남을 알 수 있으므로 그래프는 모든 사분면을 지난다.
다른 풀이

ax+b
유리함수 y= 의 점근선의 방정식은 x=-c, y=a이다.
x+c
주어진 그래프에서 점근선이 x=-1, y=2이므로
336 ②

c=1, a=2이다. kx-2


함수 y= 의 그래프의 두 점근선의 방정식은
x+1
2x+b
또한 함수 y= 의 그래프가 점 (-3, 0)을 지나므로 x=-1, y=k이다.
x+1
-6+b 5x+1
=0, b=6 함수 y= 의 그래프의 두 점근선의 방정식은
-2 x+2
∴ a+b+c=2+6+1=9 x=-2, y=5이다.
네 직선 x=-1, x=-2, y=k, y=5로 둘러싸인 부분은 가로의
TIP
길이가 1, 세로의 길이가 |k-5|인 직사각형이고, 이 넓이가 7이므로
<점근선의 방정식으로부터 유리함수의 식 세우기> |k-5|=7에서 k=12 또는 k=-2
점근선의 방정식이 x=p, y=q인 유리함수는 따라서 모든 실수 k의 값의 곱은 -24이다.
k
y= +q (k는 0이 아닌 상수) 또는
x-p
qx+r
y=
x-p
`(r+-pq)로 놓을 수 있다. 337 ②

이때, 그래프가 지나는 한 점의 좌표를 대입하면 k 또는 r의 값을 함수 f(x)가 x+-3인 모든 실수 x에 대하여


구하여 함수식을 구할 수 있다. f( f(x))=x이므로 f ÑÚ`(x)= f(x)이다.
따라서 f(-2)=1이므로 f ÑÚ`(-2)=1, 즉 f(1)=-2이다.

335 ④ f(-2)=-2a+b=1, f(1)=


a+b
4
=-2

3-2x -2(x+1)+5 5 두 식을 연립하여 풀면 a=-3, b=-5이다.


함수 y= = = -2의 그래프는 다음과
x+1 x+1 x+1 -3x-5 -9-5 -7
따라서 f(x)= 이므로 f(3)= = 이고,
같다. x+3 3+3 3
y f(3)= f ÑÚ`(3)이므로
3-2x
7
y= x+1
f ÑÚ`(3)=-
3
-1 O x
-2

338 19

조건 ㈏의 두 직선 y=x+10, y=-x+2의 교점의 좌표는


① 정의역은 {x|x+-1인 모든 실수}, x+10=-x+2에서 x=-4, y=6이므로 (-4, 6)이다.
치역은 {y|y+-2인 모든 실수}이다. (거짓) 따라서 주어진 유리함수의 그래프는 점 (-4, 6)에 대하여 대칭이다.
② 점근선의 방정식은 x=-1, y=-2이다. (거짓) yy TIP
5 즉, 점근선의 방정식이 x=-4, y=6이므로 유리함수를
③ 함수 y= 의 그래프는 점 (0, 0)에 대하여 대칭이므로
x
6x+k
5 y= (k는 상수)라 하면
y= -2의 그래프는 점 (-1, -2)에 대하여 대칭이다. x+4
x+1
조건 ㈎에서 이 함수의 그래프가 점 (1, 3)을 지나므로
(거짓)
6+k
④ 직선 x=-1이 점근선이므로 제2사분면을 지난다. (참) 3= , k=9
5
yy 참고 6x+9
즉, y= 이므로 p=6, q=9, r=4
5 x+4
⑤ 함수 y= 의 그래프를 x축의 방향으로 -1만큼,
x ∴ p+q+r=19
y축의 방향으로 -2만큼 평행이동한 그래프이다. (거짓)
따라서 선지 중 옳은 것은 ④이다. TIP
k
참고 유리함수 y= (k는 상수)의 그래프는 두 직선
x
점근선이 두 직선 x=-1, y=-2이므로 제2, 3, 4사분면을 y=x, y=-x에 대하여 대칭이고, 두 점근선의 교점인
모두 지남을 알 수 있다. 이때, y절편을 구해 보면 x=0일 때 원점 (0, 0)에 대하여 대칭이다.

80 수학<하>
이 유리함수의 그래프를 평행이동하면 유리함수의 그래프가 2
0ÉxÉ2에서 함수 y=- +3의 그래프가 다음과 같다.
x+1
대칭이 되는 두 직선과 두 점근선의 교점도 평행이동되므로
y
유리함수의 그래프는 두 점근선의 교점에 대하여 대칭이고,
두 점근선의 교점을 지나면서 기울기가 1 또는 -1인 직선에
대해서도 대칭이다. 3
y=-x y y=x y=-{x-a}+b
y y={x-a}+b
k
y= x k
y= x-a +b -1 O 2 x
x
O {a,`b}
따라서 x=0에서 최솟값, x=2에서 최댓값을 갖는다.
O x
2 7
M= f(2)=- +3= ,
2+1 3
2
m= f(0)=- +3=1
0+1
4
∴ M-m=
339 ③ 3

3x-1
함수 y=
x+1
의 역함수를 구하면 다음과 같다. Ⅴ

y=
3x-1
x+1
을 x에 대하여 풀면 341 ④ 수

-y-1 직선 y=mx는 원점을 지나는 직선이므로 직선 y=mx가 유리함수 그
xy+y=3x-1, x(y-3)=-y-1, x= 이고, 래
y-3 5x-12 5(x-2)-2 -2
y= = = +5의 그래프와 한 점에서 프
-x-1 x-2 x-2 x-2
x와 y를 서로 바꾸면 y=
x-3 접하는 경우는 다음과 같이 두 가지이다.
-x-1 y
따라서 역함수는 y= 이므로 yy TIP
x-3
a=-1, b=-1, c=-3
∴ abc=-3
5
TIP
rx+s
<유리함수 y= 의 역함수> O2 x
px+q
rx+s 직선이 유리함수의 그래프에 접할 때,
유리함수 f(x)= (ps-qr+0, p+0)의
px+q
5x-12
-qx+s =mx, 5x-12=mx(x-2)
역함수는 f ÑÚ`(x)= 이다. x-2
px-r
에서 이차방정식 mxÛ`+(-2m-5)x+12=0이 중근을 가져야
(즉, q와 r의 부호를 바꿔서 서로 자리를 바꾼다.)
하므로 판별식을 D라 하면
[증명]
D=(2m+5)Û`-48m=0, 4mÛ`-28m+25=0
rx+s
y= 를 x에 대하여 풀면 이차방정식의 근과 계수의 관계에 의하여 두 실근 m의 값의
px+q
28
(px+q)y=rx+s, (py-r)x=-qy+s 합은 =7이다.
4
-qy+s
x= 이고 x와 y를 서로 바꾸면
py-r
-qx+s -qx+s
y= 이므로 f ÑÚ`(x)= 이다.
px-r px-r
3x-1
342 ②
이를 이용하여 함수 y= 의 역함수는
x+1 8 8
함수 y= 위의 점 P의 좌표를 {a, }이라 하면 점 P와 원점
-x-1 x a
y= 임을 바로 구할 수 있다.
x-3 64
사이의 거리는 ¾ÐaÛ`+ 이다.
aÛ`
64
이때, aÛ`>0, >0이므로 산술평균과 기하평균의 관계에 의하여
aÛ`
64 64
340 ④ aÛ`+ ¾2¾ÐaÛ`_ =16
aÛ` aÛ`
3x+1 3(x+1)-2 -2 64
f(x)= = = +3 (단, 등호는 aÛ`= , 즉 a=Ñ2'2일 때 성립한다.)
x+1 x+1 x+1 aÛ`

Ⅴ. 함수와 그래프 81
64
이므로 aÛ`+
aÛ`
의 최솟값은 16이다. 343 ①

64 3
따라서 ¾ÐaÛ`+ 의 최솟값은 4이다. 유리함수 y= -1의 그래프의 점근선의 방정식은
aÛ` x-a
x=a, y=-1이다.
다른 풀이 Ú a¾0일 때
8 그래프가 다음과 같으므로 항상 제1사분면을 지난다.
함수 y= 의 그래프 위의 점 P와 원점 사이의 거리가 최소가 될 때
x y
8
점 P는 함수 y= 의 그래프와 직선 y=x의 교점이다. yy TIP
x
y a x
y=x
O
-1
8
y=
x

O x

Û a<0일 때
그래프가 제1사분면을 지나지 않기 위해서는 다음과 같이
8 x=0일 때 함숫값이 0 이하이어야 한다.
=x에서 xÛ`=8, x=2'2 또는 x=-2'2이므로 점 P의 좌표가 y
x
(2'2, 2'2 ) 또는 (-2'2, -2'2 )일 때, 점 P와 원점 사이의
거리가 최소이다.
a O x
따라서 구하는 최솟값은 ¿¹(2'2 )Û`+(2'2 )Û`=4이다. -1

TIP
k 3 3
유리함수 y= (k+0)의 그래프 위의 점 P와 원점 사이의 따라서 -1É0, ¾-1
x -a a
k 3É-a (∵ a<0), aÉ-3
거리가 최소일 때 점 P는 함수 y= 의 그래프와 직선 y=x
x
Ú, Û에서 구하는 a의 값의 범위는 aÉ-3이다.
또는 직선 y=-x의 교점이다.
[증명]

유리함수 y=
k
x
(k>0)의 그래프 위의 점 P의 좌표를 344 ⑤

k x x
{p, }라 하면 점 P와 원점 사이의 거리는 +
p '¶x+9+3 '¶x+9-3
kÛ` x('¶x+9-3)+x('¶x+9+3)
¾ÐpÛ`+ 이다. 산술평균과 기하평균의 관계에 의하여 =
pÛ` ('¶x+9+3)('¶x+9-3)
kÛ` kÛ` kÛ` x('¶x+9-3+'¶x+9+3) 2x'¶x+9
pÛ`+ ¾2¾Ðp`Û _ =2k (∵ k>0)이고, 이때 등호는 pÛ`= , = =
pÛ` pÛ` pÛ` '¶x+9 Û`-3Û` x
즉 p=Ñ'k일 때 성립하므로 점 P의 좌표는 ('k, 'k ) 또는 =2'¶x+9 (∵ x+0)

(-'k, -'k )이다.


k
유리함수 y=
x
(k<0)에 대해서도 마찬가지로 구하면 345 ④

점 P의 좌표는 ('¶-k, -'¶-k ) 또는 (-'¶-k, '¶-k )이다. 1


무리식 '¶3x+6+ 의 값이 실수이려면 근호 안의 식이
k '¶2-x
따라서 함수 y= (k+0)의 그래프 위의 점 P와 원점 사이의
x 0 이상이어야 하므로 3x+6¾0, 2-x¾0이다.
거리가 최소일 때 점 P는 직선 y=x 또는 직선 y=-x 위의 1
이때, 에서 분모가 0이면 안되므로 2-x+0이다.
점이다. '¶2-x
y y=x y=-x y 즉, 3x+6≥0이고, 2-x>0이다.
k 따라서 -2≤x<2이므로 정수 x는 -2, -1, 0, 1로 4개이다.
y= {k>0} P
x
P x
O x O P k
P y=
x
{k<0} 346 ⑤

a의 값의 부호에 따른 무리함수 y='¶ax (a+0)의 그래프는

82 수학<하>
다음과 같다. ① 정의역은 {x|x¾-2}이다. (거짓)
y y ② 치역은 {y|y¾3}이다. (거짓)
a<0 a>0 ③ x의 값이 커짐에 따라 y의 값은 커진다. (거짓)
④ 제1, 2사분면을 지나고, 제3, 4사분면을 지나지 않는다. (거짓)
⑤ 함수 y='¶2x의 그래프를 x축의 방향으로 -2만큼,
O x O x y축의 방향으로 3만큼 평행이동한 것이다. (참)
따라서 선지 중 옳은 것은 ⑤이다.
① a>0이면 정의역과 치역은 0 이상의 모든 실수의 집합이다. (참)
② a<0이면 정의역은 {x|xÉ0}이고 치역은 {y|y¾0}이다. (참)
③ a<0이면 지나는 사분면은 제2사분면뿐이다. (참)
④ 두 함수의 그래프가 직선 y=x에 대하여 대칭이면 두 함수는 349 ㄱ, ㄴ, ㄹ, ㅂ, ㅅ
서로 역함수 관계이다. 즉, 무리함수 y='¶ax의 역함수를 구하면
ㄱ. 함수 y=xÛ`(x¾0)의 그래프는 함수 y='x의 그래프를 직선
다음과 같다.
y=x에 대하여 대칭이동한 것이다.
yÛ`
y='¶ax를 x에 대하여 풀면 yÛ`=ax, x= 이고, ㄴ. 함수 y=1+'¶-x의 그래프는 함수 y='x의 그래프를 y축에
a
xÛ` 대하여 대칭이동한 후 y축의 방향으로 1만큼 평행이동한 것이다.
x와 y를 서로 바꾸면 y= 이다. 이때, y='¶ax¾0이므로
a ㄷ. 함수 y='¶2x-1의 그래프는 함수 y='¶2x의 그래프를 x축의
y='¶ax의 역함수는 y=
xÛ`
`(x¾0)이다. 방향으로
1
만큼 평행이동한 것이므로 함수 y='x의 그래프를

a 2 함

xÛ` 평행이동 또는 대칭이동하여 겹쳐질 수 없다.
따라서 y='¶ax의 그래프는 y= `(x¾0)의 그래프와 와
a
ㄹ. 함수 y=-'¶x+4의 그래프는 함수 y='x의 그래프를 x축의 그
직선 y=x에 대하여 대칭이다. (참) 래
방향으로 -4만큼 평행이동한 후 x축에 대하여 대칭이동한 프
⑤ |a|의 값이 커질수록 그래프는 x축에서 멀어지고, y축에
것이다.
가까워진다. (거짓)
ㅁ. 함수 y=3'x+1의 그래프는 함수 y=3'x='¶9x의 그래프를
y
y=Â-3˛xÊ y=Â3xÊ y축의 방향으로 1만큼 평행이동한 것이므로 함수 y='x의
y=Â-2˛xÊ y=Â2xÊ 그래프를 평행이동 또는 대칭이동하여 겹쳐질 수 없다.
y=Â-x˛° y=Âx° ㅂ. 함수 y='¶x+5-2의 그래프는 함수 y='x의 그래프를 x축의
방향으로 -5만큼, y축의 방향으로 -2만큼 평행이동한 것이다.
1 1
O x ㅅ. 함수 y= '¶4x-1=¾Ðx- 의 그래프는 함수 y='x의
2 4
따라서 선지 중 옳지 않은 것은 ⑤이다. 1
그래프를 x축의 방향으로 만큼 평행이동한 것이다.
4
따라서 겹쳐질 수 있는 그래프의 식은 ㄱ, ㄴ, ㄹ, ㅂ, ㅅ이다.
347 ④

y='¶-4x+12+5=2'¶-(x-3)+5이므로 이 함수의 그래프는


함수 y=2'x의 그래프를 y축에 대하여 대칭이동한 후 x축의 350 ④

방향으로 3만큼, y축의 방향으로 5만큼 주어진 그래프에서 정의역은 {x|xÉ4}, 치역은 {y|y¾-2}이므로
평행이동한 것이므로 a=3, b=5이다.
y='¶a(x-4)-2 (a<0) yy TIP
∴ a+b=8
라 하면 이 함수의 그래프가 점 (3, 0)을 지나므로
0='¶a(3-4)-2, 2='¶-a, a=-4
348 ⑤ 즉, y='¶-4(x-4)-2='¶-4x+16-2
이므로 b=16, c=-2
함수 y='¶2x+4+3='¶2(x+2)+3의 그래프는
∴ a+b+c=-4+16+(-2)=10
함수 y='¶2x의 그래프를 x축의 방향으로 -2만큼,
y축의 방향으로 3만큼 평행이동한 것이므로 다음과 같다. TIP
y 함수 y='¶ax+b+c의 정의역이 {x|xÉ4},
y=Â2xÊ+˛4˛+3 치역이 {y|y¾-2}이므로 정의역이 {x|xÉ0},
치역이 {y|y¾0}인 무리함수 y='¶ax (a<0)의 그래프를
3 x축의 방향으로 4만큼, y축의 방향으로 -2만큼 평행이동한
것이라 할 수 있다. 따라서 주어진 무리함수의 식을

-2 O x y='¶a(x-4)-2 (a<0)이라 놓을 수 있다.

Ⅴ. 함수와 그래프 83
참고 y
y=Â-x˛+˛3˛-2
양의 실수 a에 대하여
O 3
정의역이 {x|x¾0}, 치역이 {y|y¾0}인 무리함수는 x

y='¶ax
정의역이 {x|xÉ0}, 치역이 {y|y¾0}인 무리함수는 -2 {3,`-2}
y='¶-ax
ㄷ. 함수 y=-'¶2-x+3=-'¶-(x-2)+3의 그래프는
정의역이 {x|x¾0}, 치역이 {y|yÉ0}인 무리함수는
함수 y=-'¶-x의 그래프를 x축의 방향으로 2만큼, y축의
y=-'¶ax
방향으로 3만큼 평행이동한 것이므로 점 (2, 3)을 지나면서
정의역이 {x|xÉ0}, 치역이 {y|yÉ0}인 무리함수는
왼쪽 아래로 그려진다.
y=-'¶-ax
이때, y절편이 -'¶2-0+3=-'2+3>0이므로 이 함수의
라 놓을 수 있다.
그래프는 다음과 같이 제2사분면을 지난다.
y
y=Â-a˛x˛ y=ÂaxÊ y {2,`3}
3

x
O y=-Â2-˛x˛+3
y=-Â-a˛x˛ y=-ÂaxÊ O 2 x

ㄹ. 함수 y=-'¶x-1+4의 그래프는 함수 y=-'x의 그래프를


x축의 방향으로 1만큼, y축의 방향으로 4만큼 평행이동한

351 ③ 것이므로 점 (1, 4)를 지나면서 오른쪽 아래로 그려진다. 따라서


이 함수의 그래프는 다음과 같이 제2사분면을 지나지 않는다.
7 7 7 y
( f ç gÑÚ`)ÑÚ` { }=(g ç f ÑÚ`) { }=g{ f ÑÚ` { }} yy㉠
6 6 6 {1,`4}
4
7 7
이때, f ÑÚ` { }=k라 하면 f(k)= 이므로
6 6 y=-Âx-˛1˛+4
x
k+1 7
= 에서 7k-14=6k+6, k=20 O 1
k-2 6
7 따라서 그래프가 제2사분면을 지나는 것은 ㄴ, ㄷ이다.
㉠에서 g{ f ÑÚ` { }}=g(20)='¶2_20-4=6
6
7
∴ ( f ç gÑÚ`)ÑÚ` { }=6
6

352 ③
353 ④
ㄱ. 함수 y='¶x+4-2='¶x-(-4)-2의 그래프는 함수 y='x의
주어진 이차함수의 그래프가 위로 볼록하므로 a<0이고, 축이
그래프를 x축의 방향으로 -4만큼, y축의 방향으로 -2만큼
b
평행이동한 것이므로 점 (-4, -2)를 지나면서 오른쪽 위로 y축보다 오른쪽에 있으므로 - >0에서 b>0, y절편이
2a
그려진다. 음수이므로 c<0이다.
이때, y절편이 '¶0+4-2=0이므로 이 함수의 그래프는 다음과 한편, 무리함수 y='¶ax-c+b의 그래프는 y='¶ax의 그래프를
같이 제2사분면을 지나지 않는다. c
y
x축의 방향으로 만큼, y축의 방향으로 b만큼 평행이동한 것이다.
a
y=Âx+˛4˛-2 c
-4
이때, a<0이므로 이 무리함수의 그래프는 점 { , b}를 지나면서
a
O x
c
왼쪽 위로 그려지고, >0, b>0이므로 그래프는 다음과 같다.
a
{-4,`-2} -2 y
ㄴ. 함수 y='¶-x+3-2='¶-(x-3)-2의 그래프는 함수 y=ÂaxÊ-˛c˛+b
y='¶ -x의 그래프를 x축의 방향으로 3만큼, y축의 방향으로
-2만큼 평행이동한 것이므로 점 (3, -2)를 지나면서 왼쪽 위로 b

그려진다. c x
O
이때, y절편이 '¶-0+3-2<0이므로 이 함수의 그래프는 다음과 a

같이 제2사분면을 지난다. 따라서 구하는 답은 ④이다.

84 수학<하>
이때, 함수 f(x), g(x)의 정의역의 범위에 의하여 x¾3이므로
354 ④
x=5
함수 y='¶a-x+2='¶-(x-a)+2의 그래프는 함수 y='¶-x의 따라서 두 함수의 그래프의 교점의 좌표는 (5, 5)이다.
그래프를 x축의 방향으로 a만큼, y축의 방향으로 2만큼 평행이동한
채점 요소 배점
것이므로 -5ÉxÉ3에서 그래프의 개형은 다음과 같다.
무리함수 y= f(x)의 그래프 그리기 30%
무리함수의 역함수와 정의역, 치역 구하기 30%
{a,`2} 함수와 역함수의 그래프의 교점의 좌표 구하기 40%

-5 3 x

따라서 x=3일 때 최솟값 3을 갖고, x=-5일 때 최댓값을 갖는다.


x=3일 때 y='¶a-3+2=3이므로 '¶a-3=1
양변을 각각 제곱하면 a-3=1, a=4
356 ⑤

따라서 x=-5일 때 y='¶4-(-5)+2=5이므로 f(x)='¶-2x+a+b라 하면 함수 y= f(x)와 그 역함수


구하는 최댓값은 5이다. y= f ÑÚ`(x)의 그래프가 모두 점 (1, 3)을 지나므로
f(1)=3, f(3)=1이다.
f(1)='¶a-2+b=3, f(3)='¶a-6+b=1에서
'¶a-2=3-b, '¶a-6=1-b이다. yy ㉠ Ⅴ
두 식의 양변을 각각 제곱하면 함

a-2=bÛ`-6b+9, a-6=bÛ`-2b+1
355 풀이 참조
두 식의 양변을 각각 빼면 4=-4b+8에서 b=1



⑴ 함수 f(x)='¶x-1+3의 그래프는 함수 y='x의 그래프를 ㉠에 대입하면 a=6 프
x축의 방향으로 1만큼, y축의 방향으로 3만큼 평행이동한 ∴ a+b=7
것이므로 다음과 같다.
y

357 ④
3
함수 y='¶x-1의 그래프와 직선 y=mx가 접하므로 다음과 같이
m>0이고, x에 대한 방정식 '¶x-1=mx의 실근이 오직 하나이다.
y y=mx
O 1 x

⑵ 함수 y='¶x-1+3의 정의역은 {x|x¾1}, 치역은 {y|y¾3}이다. y=Âx-˛1˛


{a,`b}
y='¶x-1+3을 x에 대하여 풀면
y-3='¶x-1, (y-3)Û`=x-1, x=(y-3)Û`+1
O 1 x
x와 y를 서로 바꾸면 y=(x-3)Û`+1
이때, 역함수의 정의역, 치역은 각각 원래 함수의 치역, 정의역과 '¶x-1=mx의 양변을 각각 제곱하면 x-1=mÛ`xÛ`이고, 이차방정식
같으므로 함수 g(x)의 정의역은 {x|x¾3}, 치역은 mÛ`xÛ`-x+1=0 yy ㉠
{y|y¾1}이다. 의 판별식을 D라 하면 D=1-4mÛ`=0이어야 하므로
∴ g(x)=(x-3)Û`+1 (x¾3) 1 1
m=- 또는 m=
2 2
⑶ y y=g{x} y=x 1
이때, m>0이므로 m= yy TIP
2
1
㉠에 대입하면 xÛ`-x+1=0
y=f{x} 4
xÛ`-4x+4=0, (x-2)Û`=0
즉, x=2에서 접하므로
O x a=2, b='¶2-1=1
함수 y= f(x)의 그래프가 증가하는 형태이므로 함수의 그래프가 1
∴ m+a+b= +2+1=
7
역함수 y=g(x)의 그래프와 만나는 교점은 직선 y=x와의 2 2

교점과 같다. TIP


방정식 '¶x-1+3=x에서 '¶x-1=x-3의 양변을 각각 1
m=- 일 때, 함수 y='¶x-1의 그래프와 직선 y=mx의
제곱하면 x-1=(x-3)Û`, xÛ`-7x+10=0 2
(x-2)(x-5)=0, x=2 또는 x=5 위치 관계는 다음과 같다.

Ⅴ. 함수와 그래프 85
y
1
y=- x
359 ③
2 y=Âx-˛1˛
1 2 3
+ +
x(x+1) (x+1)(x+3) (x+3)(x+6)
O 1 x
1 1 2 1 1 3 1 1
={ - }+ { - }+ { - } yy TIP
x x+1 2 x+1 x+3 3 x+3 x+6
1 1 1 (x+6)-x 6
참고로 직선 y=- x는 함수 y=-'¶x-1의 그래프와 접한다. = - = =
2 x x+6 x(x+6) x(x+6)
방정식 '¶x-1=mx의 양변을 제곱하여 방정식 x-1=mÛ`xÛ`의 6 3
이므로 = 에서
해를 구할 때, 방정식 -'¶x-1=mx의 해도 함께 구해진다. x(x+6) 4
x(x+6)=8, xÛ`+6x-8=0
1 1
따라서 m=- 또는 m= 의 두 해가 구해질 때, 각각의 이 이차방정식은 두 실근을 가지므로 근과 계수의 관계에 의하여
2 2
경우가 조건에 맞는지 직접 확인해 보아야 한다. 모든 실수 x의 값의 곱은 -8이다.

TIP
<부분분수>
1 1 1 1
= { - } (단, A+B)
AB B-A A B
[증명]
1 1 1 1 B-A 1
{ - }= _ =
B-A A B B-A AB AB

358 ④

함수 y=-'¶3x+6-6=-'¶3(x+2)-6의 그래프는
함수 y=-'¶3x의 그래프를 x축의 방향으로 -2만큼, 360 ③

y축의 방향으로 -6만큼 평행이동한 것이다. x-1 x-1 (x-1)Û`


= =
직선 y=kx는 원점을 지나고 기울기가 k인 직선이고, 이 직선이 2 (x-1)-2 x-3
1-
점 (-2, -6)을 지날 때, -6=-2k에서 k=3이다. x-1 x-1
Ú k=3일 때, 다음과 같이 두 그래프는 한 점에서 만난다. xÛ`-2x+1 (x-3)(x+1)+4
= =
y x-3 x-3
k=3
-2 4
=x+1+
O x x-3
이므로 x에 대한 항등식
-6 4 c
y=-Â3xÊ+˛6˛-6
x+1+ =ax+b+ 에서
x-3 x-3
a=1, b=1, c=4
Û 0Ék<3일 때, 다음과 같이 두 그래프는 만나지 않는다. ∴ a+b+c=6
y 0 k<3
-2
O x

-6
y=-Â3xÊ+˛6˛-6 361 ③

3aÛ`+5a+2 (a+6)(3a-13)+80
Ü k<0 또는 k>3일 때, 다음과 같이 두 그래프는 한 점에서 만난다. =
a+6 a+6
y y 80
k<0 k>3 = +3a-13
a+6
-2 O -2 자연수 a에 대하여 이 값이 정수가 되려면 a+6이 80의 약수이어야
x O x
한다.
-6 -6
y=-Â3xÊ+˛6˛-6 y=-Â3xÊ+˛6˛-6 자연수 a에 대하여 a+6>6이고, 6보다 큰 80의 약수 중 최솟값은
8, 최댓값은 80이므로 자연수 a의 최솟값은 a+6=8에서 2,
Ú ~ Ü에서 0Ék<3일 때 두 그래프가 만나지 않으므로 구하는 최댓값은 a+6=80에서 74이다.
정수 k는 0, 1, 2로 3개이다. 따라서 구하는 값은 2+74=76이다.

86 수학<하>
3k 4k k(3b+4a)
362 ② v+võ=
a
+ =20에서
b ab
=20이므로

2b+c a+c a+2b 20ab


= = =k라 하면 k=
a 2b c 4a+3b
( -ak+2b+c=0 따라서 물질 A의 부피는
Ò a-2bk+c=0 yy ㉠ 3k 3 20ab 60b
v= = × =
a a 4a+3b 4a+3b
9 a+2b-ck=0
세 식의 양변을 각각 더하면
(2-k)a+2(2-k)b+(2-k)c=0
(2-k)(a+2b+c)=0이므로 k=2 (∵ a+2b+c+0) 364 ⑤

㉠에 대입하여 풀면 a=2b=c이므로 yy TIP1


2x+k k+6
y= = +2 (k>0)의 그래프의 개형은 다음과 같다.
a`:`b`:`c=2`:`1`:`2 x-3 x-3
따라서 a=2t, b=t, c=2t (t+0)라 하면 y 2x+k
y=
x-3
aÜ`+bÜ`+cÜ` 8tÜ`+tÜ`+8tÜ` 17
= =
abc 4tÜ` 4
다른 풀이 2

a+2b+c+0이므로 O 3 x Ⅴ
k 함
2b+c a+c a+2b 2(a+2b+c) -
= = = =2에서 yy TIP2 3 수
a 2b c a+2b+c 와
2b+c=2a, a+c=4b, a+2b=2c 그
ㄱ. 점근선의 방정식이 x=3, y=2이므로 유리함수의 그래프는 두 래
세 식을 연립하여 풀면 a=2b=c이므로 yy TIP1 직선 x=3, y=2와 만나지 않는다. (참) 프

a`:`b`:`c=2`:`1`:`2 ㄴ. 두 점근선의 교점이 (3, 2)이므로 이 유리함수의 그래프는 직선


따라서 a=2t, b=t, c=2t (t+0)라 하면 y=(x-3)+2=x-1과 직선 y=-(x-3)+2=-x+5에
aÜ`+bÜ`+cÜ` 8tÜ`+tÜ`+8tÜ` 17 대하여 대칭이다. (참)
= =
abc 4tÜ` 4 k
ㄷ. y절편이 - <0이므로 이 함수의 그래프는 모든 사분면을
3
TIP1
지난다. (참)
2a-2b-c=0 yy ㉠ 따라서 옳은 것은 ㄱ, ㄴ, ㄷ이다.
a-4b+c=0 yy ㉡
a+2b-2c=0 yy ㉢
㉠+㉡을 계산하면 3a-6b=0에서 a=2b
㉡-㉢을 계산하면 -6b+3c=0에서 c=2b 365 ②

따라서 a=2b=c이다. 1
fÁ(1)= f(1)=
2
TIP2 1
1 2 1
a c e fª(1)=( f ç fÁ)(1)= f { }= =
= = 일 때, b+d+ f+0이면 2 1 3
b d f +1
2
a c e a+c+e 1
= = = 가 성립한다.
b d f b+d+ f 1 3 1
f£(1)=( f ç fª)(1)= f { }= =
3 1 4
+1
3
1
1 4 1
f¢(1)=( f ç f£)(1)= f { }= =
4 1 5
+1
4

363 ⑤
이므로 fÇ(1)=
1
(n은 자연수)
n+1
두 물질 A, B의 부피를 각각 v, võ라 하자.
1
m m ∴ fÁ¼¼¼(1)=
d= 에서 v= 이므로 두 물질 A, B의 질량을 각각 1001
v d
다른 풀이
3k, 4k (k+0)라 하면
3k 4k x
v= , võ= 이다. fÁ(x)= f(x)=
a b x+1

Ⅴ. 함수와 그래프 87
x
fª(x)=( f ç fÁ)(x)=
x+1
=
x 367 11
x 2x+1
+1 1
x+1 A{a, } (a>0)이라 하면
a
x
2x+1 x k
f£(x)=( f ç fª)(x)= = 점 C는 함수 y= 의 그래프 위의 점 중에서 x좌표가 a인 점이므로
x 3x+1 x
+1
2x+1 k
C{a, }이고,
⋮ a
x k 1
∴ fÇ(x)= (n은 자연수) 점 B는 함수 y= 의 그래프 위의 점 중에서 y좌표가 인 점이므로
nx+1 x a
x 1
따라서 fÁ¼¼¼(x)= 이므로 B{ka, }이다.
1000x+1 a
1 ABÓ=|ka-a|=|a(k-1)|,
fÁ¼¼¼(1)= 이다.
1001 k 1 k-1
ACÓ=| - |=| |이므로
a a a
1 k-1
삼각형 ABC의 넓이는 _|a(k-1)|_| |=50에서
2 a
(k-1)Û`=100, k-1=Ñ10
366 6
∴ k=11 (∵ k>0)
4 2x-4
f(x)=- +2=
x x
2x-4
2_ -4
x -4
f Û`(x)= f( f(x))= =
2x-4 x-2
x
-4
f Ü`(x)= f Û`( f(x))= =x
2x-4
-2
x
이므로 f Ü`=I이다. (I는 항등함수) 368 ④
3_166+2
f Þ`â` â`= f = f Û`이고, f Ü`= f Û` ç f=I에서 함수 y= f ÑÚ`(x)의 그래프가 x축, y축과 만나는 점을 각각
f Û`의 역함수는 f 이므로 g= f이다. A(a, 0), B(0, b)라 할 때, 두 점 (0, a), (b, 0)이 함수
∴ g(-1)= f(-1)=6 y= f(x)의 그래프 위의 점이므로 f(0)=a, f(b)=0이다.
다른 풀이 Ú f(0)=a인 a의 값을 구하면
x+1 x+1
g=( f Þ`â` â`)ÑÚ`=( f ÑÚ`)Þ`â` â`이므로 yy TIP f{
x-1
}=2x+1에서
x-1
=0일 때, x=-1이므로
g(-1)=( f ÑÚ`)Þ`â` â`(-1)이다. f(0)=2_(-1)+1=-1에서 a=-1
4 4 Û f(b)=0인 b의 값을 구하면
f(x)=- +2=-1에서 x= 이므로
x 3
x+1 1
4 f{ }=2x+1에서 2x+1=0일 때, x=- 이므로
f ÑÚ`(-1)= x-1 2
3
1
4 4 - +1
f(x)=- +2= 에서 x=6이므로 2 1
x 3 b= =-
1 3
- -1
4 2
( f ÑÚ`)Û`(-1)= f ÑÚ` { }=6
3 1
따라서 A(-1, 0), B{0, - }이므로 삼각형 OAB의 넓이는
4 3
f(x)=- +2=6에서 x=-1이므로
x 1 1 1
_1_ = 이다.
( f ÑÚ`)Ü`(-1)= f ÑÚ`(6)=-1 2 3 6
4
따라서 , 6, -1의 세 수가 반복되고, 참고
3
500=3_166+2이므로 x+1 x+1
f{ }=2x+1에서 t= 이라 하자.
x-1 x-1
g(-1)=( f ÑÚ`)Þ`â` â`(-1)=( f ÑÚ`)Û`(-1)=6
t+1
x에 대하여 정리하면 x= 이다.
TIP t-1
t+1 3t+1
( f Þ`â` â`)ÑÚ`=( f ç f ç y ç f)ÑÚ` 즉, f(t)=2{ }+1= 이므로
t-1 t-1
= f ÑÚ` ç f ÑÚ`ç y ç f ÑÚ` 3x+1
f(x)= 이다.
=( f ÑÚ`)Þ`â` â` x-1

88 수학<하>
Ú k<0일 때
369 ②
3x-2
함수 y=| |의 그래프와 직선 y=k가 만나지 않으므로
k x+1
함수 |y|= (k>0)의 그래프는 다음과 같다. yy TIP
|x| f(k)=0
y Û k=0 또는 k=3일 때
D
3x-2
함수 y=| |의 그래프와 직선 y=k가 한 점에서 만나므로
x+1
f(k)=1
A C x
O Ü 0<k<3 또는 k>3일 때
3x-2
함수 y=| |의 그래프와 직선 y=k가 두 점에서 만나므로
B x+1
f(k)=2
이 함수의 그래프에 접하는 정사각형 ABCD는 Ú ~ Ü에서 구하는 값은
k f(0)+ f(1)+ f(2)+y+ f(10)=1_2+2_9=20
함수 |y|= 의 그래프와 직선 y=x, y=-x의
|x|
교점에서 접하므로 네 교점의 좌표를 각각
(a, a), (a, -a), (-a, a), (-a, -a)라 하자. (단, a>0)

|y|=
k k
의 그래프가 점 (a, a)를 지나므로 a= 에서 k=aÛ`이고, Ⅴ
|x| a 함
정사각형 ABCD의 한 변의 길이는 2'2a이므로 수

넓이는 (2'2a)Û`=8aÛ`=48에서 aÛ`=6이다. 371 풀이 참조 와



∴ k=aÛ`=6 -2x+k k-6 래
함수 y= = -2의 그래프의 점근선의 방정식은 프
x-3 x-3
TIP
x=3, y=-2이다.
k
x>0, y>0일 때 y= , 이때, k-6=0, 즉 k=6이면 주어진 함수는 y=-2이므로
x
제1, 2사분면을 지나지 않는다. 따라서 조건을 만족시키지 않으므로
k
x<0, y>0일 때 y=- , k-6+0이다.
x
k Ú k-6>0, 즉 k>6일 때
x<0, y<0일 때 y= ,
x 함수의 그래프는 다음과 같이 항상 제1, 3, 4사분면을 지나고,
k 제2사분면은 지나지 않으므로 k>6일 때 항상 조건을 만족시킨다.
x>0, y<0일 때 y=- 이므로
x
y -2x+k
k y=
제1, 3사분면에 함수 y= 의 그래프를 그리고, x-3
x
3
k O x
제2, 4사분면에 함수 y=- 의 그래프를 그린다. -2
x

370 20

3x-2 3x-2 Û k-6<0, 즉 k<6일 때


함수 y=| |의 그래프는 함수 y= 의 그래프에서
x+1 x+1 함수의 그래프는 제1, 3, 4사분면을 항상 지나고, 이때
y<0인 부분을 x축에 대하여 대칭이동한 그래프이다. 제2사분면을 지나지 않으려면 다음과 같이 y절편이 0보다 작거나
3x-2 -5 같아야 한다.
함수 y= = +3의 그래프의 점근선의 방정식은
x+1 x+1 y
x=-1, y=3이므로
3x-2
함수 y=| |의 그래프는 그림과 같고, k의 값에 따라
x+1 O 3 x
f(k)의 값을 구하면 다음과 같다. -2
y
-2x+k
y=
x-3
k>3
k
즉, É0에서 k¾0이다. 따라서 조건을 만족시키는 k의 값의
-3
3
k=3
0<k<3
y= |3x-2
x+1 | 범위는 0Ék<6이다.
k=0 Ú, Û에서 구하는 실수 k의 값의 범위는
-1 O x
k<0 0Ék<6 또는 k>6이다.

Ⅴ. 함수와 그래프 89
채점 요소 배점 이때 치역의 원소 중 정수의 개수가 6이려면 정수는
k-6+0임을 구하기 20% 4, 3, 2, 1, 0, -1이 포함되어야 하므로
k-6>0일 때 조건을 만족시키는 k의 값의 범위 구하기 30% -2É f(1)<-1이어야 한다.
k-6<0일 때 조건을 만족시키는 k의 값의 범위 구하기 30% 5+k
-2É <-1에서 -3<kÉ-1이므로
-2
구하는 실수 k의 값의 범위 구하기 20%
조건을 만족시키는 정수 k는 -2, -1이다.
Ú, Û에서 조건을 만족시키는 모든 정수 k의 값의 합은
372 ② (-29)+(-28)+(-2)+(-1)=-60이다.

2x+2 4
함수 y= = +2의 그래프가 다음과 같다.
x-1 x-1
y
374 풀이 참조
2x+2
y=
x-1 2x+2 -4
유리함수 y= = +2의 그래프의 점근선의 방정식은
x+3 x+3
1
- 3 x=-3, y=2이다.
3
2 2x+2 y
O y=
x x+3
1 5
-1 P R

2
2x+2 1 Q
y=-1일 때 =-1에서 2x+2=-x+1, x=-
x-1 3
-3 O x
2x+2
y=3일 때 =3에서 2x+2=3x-3, x=5이므로
x-1
1 -4 -4
yÉ-1일 때 - Éx<1이고, P{a, +2}라 하면 Q(a, 2), R{-3, +2}이므로
3 a+3 a+3
y¾3일 때 1<xÉ5이다. -4 Û
QRÓ Û`=(a+3)Û`+{ }`이고,
따라서 치역이 {y|yÉ-1 또는 y¾3}일 때 a+3
1 산술평균과 기하평균의 관계에 의하여
정의역은 [x|- Éx<1 또는 1<xÉ5]이므로
3 -4 Û -4 Û
(a+3)Û`+{ }`¾2¾Ð(a+3)Û`_{ }`=8
정의역에 속하는 정수는 0, 2, 3, 4, 5로 5개이다. a+3 a+3
(단, 등호는 (a+3)Û`=4, 즉 a=-5 또는 a=-1일 때 성립한다.)
따라서 구하는 QRÓ Û`의 최솟값은 8이다.
373 -60

채점 요소 배점
5x+k k+15
유리함수 y= = +5의 그래프의 점근선의 방정식이
x-3 x-3 유리함수의 그래프의 점근선의 방정식 구하기 20%
x=3, y=5이므로 x<1에서 함수의 그래프의 개형은 두 점 Q, R의 좌표 구하기 30%
다음과 같은 두 가지이다. QRÓ Û`의 최솟값 구하기 50%

Ú y y Û y y
다른 풀이

2x+2 -4
유리함수 y= = +2의 그래프를 평행이동하여도
x+3 x+3
5 5 5 5
4
QRÓ Û`의 값은 변하지 않으므로 함수의 그래프를 유리함수 y=- 의
x
O 1 O3 1 3 x x 1 3 1 3 x x 그래프로 평행이동시키자.
점 P, Q, R가 이동된 점을 각각 P', Q', R'이라 하면 함수
Ú k+15<0, 즉 k<-15일 때 4
y=- 의 그래프 위의 점 P'에서 x축, y축에 내린 수선의 발이 각각
x
x<1에서 치역은 {y|5<y< f(1)}이고,
이때 치역의 원소 중 정수의 개수가 6이려면 정수는 Q', R'이고, QRÓ Û`=Q'R'Ó Û`이다.
y
6, 7, 8, 9, 10, 11이 포함되어야 하므로
11< f(1)É12이어야 한다.
P' R'
5+k
11< É12에서 -29Ék<-27이므로
-2 x
조건을 만족시키는 정수 k는 -29, -28이다. Q' O 4
y=-
x
Û k+15>0, 즉 k>-15일 때
x<1에서 치역은 {y| f(1)<y<5}이고,

90 수학<하>
직사각형 OR'P'Q'에서 Q'R'Ó=OP'Ó이고, 함수 y=-
4
의 그래프와 x=-1 y
x y=mx-2m+1

직선 y=-x의 교점이 P'일 때 OP'Ó이 최소이다.


4 A C y=2
- =-x에서 x=Ñ2이므로 P'(2, -2) 또는 P'(-2, 2)일 때
x O {2,`1}
x
OP'Ó Û`=8로 최소이다. B

따라서 구하는 QRÓ Û`의 최솟값은 8이다.

채점 요소 배점 직선 y=mx-2m+1과 직선 x=-1의 교점은


a B(-1, -3m+1)이고, 직선 y=mx-2m+1과
y= (a는 상수) 꼴로 평행이동시키기 20%
x
직선 y=2의 교점은 mx-2m+1=2에서
a
QRÓ Û`이 최소가 되는 함수 y= 의 그래프 위의 점의 좌표 구하기 40% 2m+1 1 1
x x= = +2이므로 C{ +2, 2}이다.
m m m
QRÓ Û`의 최솟값 구하기 40%
이때, 삼각형 ABC의 넓이는
1 1 1
_ABÓ_ACÓ= _{2-(-3m+1)}_[ +2-(-1)]
2 2 m
1 1
= (3m+1){ +3}
2 m
375 ①
1 1 Ⅴ
= {9m+ +6} 함
2 2 m
함수 y= (x>0)의 그래프를 x축의 방향으로 1만큼, 수
x 이고, 산술평균과 기하평균의 관계에 의하여 와
2 1 1 1 1 그
y축의 방향으로 2만큼 평행이동하면 y= +2 (x>1)이므로 {9m+ +6}¾ {2¾Ð9m_ +6}=6
x-1 2 m 2 m 래

2 1 1
점 P의 좌표를 {a, +2} (a>1)라 하자. (단, 등호는 9m= , 즉 m= 일 때 성립한다.)
a-1 m 3
y 따라서 삼각형 ABC의 넓이의 최솟값은 6이다.

R { 2
P a, a-1 +2 }
2
y= +2`{x>1}
x-1

O Q x 377 8

직사각형 ROQP의 넓이는 x+-b인 모든 실수 x에 대하여 ( f ç f)(x)=x이므로


2 2a 2(a-1)+2 f(x)= f ÑÚ`(x)이고, 두 점근선의 교점이 직선 y=x 위에 존재한다.
a_{ +2}= +2a= +2a
a-1 a-1 a-1 함수 y= f(x)의 그래프의 점근선의 방정식이
2 x=-b, y=-2이므로 b=2이다.
= +2a+2
a-1
-2x+a a+4
2 ∴ f(x)= = -2 yy㉠
= +2(a-1)+4 x+2 x+2
a-1
x=-2 y
이고, 산술평균과 기하평균의 관계에 의하여
2 2
+2(a-1)+4¾2¾Ð _2(a-1)+4=8
a-1 a-1 B A
2 O x
(단, 등호는 =2(a-1), 즉 a=2일 때 성립한다.) D C y=-2
a-1
따라서 직사각형 ROQP의 넓이의 최솟값은 8이다.

점 A가 함수 y= f(x)의 그래프 위의 점이므로 A(t, f(t))라 하면


1
(삼각형 ABC의 넓이)= _| f(t)+2|_|t+2|=5 yy㉡
376 6 2
a+4
2x-3 -5 이때, ㉠에서 f(t)+2=
유리함수 y= = +2의 그래프의 두 점근선의 방정식은 t+2
x+1 x+1
( f(t)+2)(t+2)=a+4이므로 ㉡에 대입하면
x=-1, y=2이므로 A(-1, 2)이다.
1
직선 y=mx-2m+1=m(x-2)+1은 점 (2, 1)을 지나고, |a+4|=5, a=6 (∵ a>0) yy TIP
2
m>0이므로 삼각형 ABC는 다음과 같다. ∴ a+b=6+2=8

Ⅴ. 함수와 그래프 91
TIP 참고

k xÁ<xª<-b 또는 -b<xÁ<xª이면 f(xÁ)> f(xª)이다.


유리함수 y= +b (a, b, k는 상수)의 그래프 위의
x-a
점 P에서 두 점근선 x=a, y=b에 내린 수선의 발을 각각
Q, R라고 할 때, 삼각형 PQR의 넓이는 항상 일정하다.
P(p, q)라 하면 Q(a, q), R(p, b)이므로
PQÓ=|a-p|, PRÓ=|b-q|에서 삼각형 PQR의 넓이는
1 1
_PQÓ_PRÓ= |(a-p)(b-q)|이고, yy㉢
379 ⑤
2 2
4x 4(x-4)+16 16
k 함수 f(x)= = = +4의 그래프는
이때, 점 P(p, q)가 y= +b의 그래프 위의 점이므로 x-4 x-4 x-4
x-a
다음과 같다.
k
q= +b에서 (q-b)(p-a)=k이다. y
p-a y=f{x}
1
따라서 ㉢에서 삼각형 PQR의 넓이는 |k|로 일정하다.
2
4

O 4 x

ㄱ. 함수 f의 역함수가 존재하므로 ( f ç f)(x)=x일


필요충분조건은 f(x)= f ÑÚ`(x)이다.
4x 4x
378 ② f(x)=
x-4
에서 f ÑÚ`(x)=
x-4
이므로

8 f(x)= f ÑÚ`(x)이다. 따라서 ( f ç f)(x)=x이다. (참)


ㄱ. (반례) 함수 f(x)= +2의 그래프는 점 (4, 4)를 지나므로
x ㄴ. 직선 mx-y+4-4m=0은 y=m(x-4)+4이므로
f(4)= f ÑÚ`(4)=4이지만 b=0, c=2이므로 b+c+0이다. 점 (4, 4)를 지나고 기울기가 m인 직선이다.
(거짓) 이때, 점 (4, 4)는 함수 y= f(x)의 그래프의 두 점근선의
ㄴ. 유리함수 y= f(x)의 그래프가 직선 y=x에 대하여 대칭이면 교점이므로 m<0일 때 직선과 함수 y= f(x)의 그래프는
두 점근선의 교점 (-b, c)가 직선 y=x 위에 있으므로
만나지 않는다. (참) yy TIP
c=-b이다. yy TIP
ㄷ. 함수의 그래프가 직선 y=-(x-4)+4에 대하여 대칭이므로
a 곡선 위의 두 점 A, B 사이의 거리가 최소가 될 때는
f(0)= +c=0에서 a=-bc=cÛ`이다. (참)
b
점 (4, 4)를 지나고 기울기가 1인 직선 y=x와 곡선이 만나는
ㄷ. a>0일 때 함수 y= f(x)의 그래프의 개형은 다음과 같다.
두 점이 A, B일 때이다.
4x
=x에서 x(x-4)=4x, xÛ`-8x=0, x(x-8)=0
Q x-4
∴ x=0 또는 x=8
y=c
따라서 A(0, 0), B(8, 8)일 때 ABÓ=8'2로 최소이다. (참)
P 따라서 옳은 것은 ㄱ, ㄴ, ㄷ이다.

x=-b TIP
두 점 P(xÁ, f(xÁ)), Q(xª, f(xª))에 대하여 두 점근선의 교점 (4, 4)를 지나고 기울기가 m인 직선
xÁ<-b<xª이면 f(xÁ)< f(xª)이다. (거짓) yy 참고 4x
y=m(x-4)+4와 유리함수 y= 의 그래프의 위치 관계는
따라서 옳은 것은 ㄴ이다. x-4
다음과 같다.
TIP
❶ m<0일 때 ❷ m=0일 때
k y y y y
<‘유리함수 f(x)= +b가 b=a를 만족시킨다.’의
x-a
다른 표현>
❶ 두 점근선의 교점 (a, b)가 직선 y=x 위에 있다. 4 4 4 4

❷ 함수 y= f(x)의 그래프가 직선 y=x에 대하여 대칭이다. O O4 4 x x O O4 4 x x


❸ x+a인 모든 실수 x에 대하여 f(x)= f ÑÚ`(x)이다.
❹ x+a인 모든 실수 x에 대하여 ( f ç f)(x)=x이다.
만나지 않는다. 만나지 않는다.
y y

92 수학<하> 4 4

O O4 4 x x
y y

4 4

O 4 x O 4 x

❸ m>0일 때
y
381 ④

4
함수 y= 의 그래프는 직선 y=x에 대하여 대칭이므로
x
4 그래프 위의 두 점 A, B에 대하여 직선 AB의 기울기가 -1일 때,
O 4 x 두 점 A, B는 직선 y=x에 대하여 대칭이다.
y
y=x
서로 다른 두 점에서 만난다. A

O x

따라서 A(a, b)라 하면 B(b, a)이고,


4
두 점이 곡선 y= 위의 점이므로 ab=4이다.
380 ⑤ x
OAÓ=OBÓ=¿¹aÛ`+bÛ`이므로 OAÓ_OBÓ=aÛ`+bÛ`=16
두 집합 A, B에 대하여 A;B+∆이려면 함수
-2x-1 이때, ABÓ=¿¹2(a-b)Û`이고,

y= 의 그래프와 직선 y=kx-k-6이 적어도 함
x+1 (a-b)Û`=aÛ`+bÛ`-2ab=16-8=8이므로 수
한 점에서 만나야 한다. 와
ABÓ='¶2_8=4

-2x-1 1
이때, 함수 y= = -2의 그래프는 다음과 같다. 래
x+1 x+1 프
y
382 36

xy-2x-2y=k를 y에 대하여 정리하면


O (x-2)y=2x+k이고,
-1 -2x-1 x
y= 2x+k
x+1 x+2일 때 y= 이다. yy TIP
-2 x-2
2x+k k+4
유리함수 y= = +2의 그래프의 점근선의 방정식은
x-2 x-2
{1,`-6} x=2, y=2이므로 유리함수의 그래프는 직선 y=x에 대하여 대칭이다.

y=kx-k-6=k(x-1)-6은 기울기가 k이고, 이때, 직선 x+y=8의 기울기가 -1이므로 직선이 유리함수의

점 (1, -6)을 지나는 직선이므로 그림과 같이 두 그래프가 그래프와 만나는 두 점 P, Q는 직선 y=x에 대하여 대칭이다.

만나는 교점의 개수는 직선이 곡선에 접할 때를 기준으로 달라진다. 점 P의 좌표를 (a, b)라 하면 점 Q의 좌표는 (b, a)이므로

-2x-1 ab=14이다.
곡선 y= 과 직선 y=kx-k-6이 접할 때,
x+1 또한 두 점 P, Q가 직선 x+y=8 위의 점이므로 a+b=8이다.
-2x-1 OPÓ=OQÓ=¿¹aÛ`+bÛ`이므로
=kx-k-6에서 kxÛ`-4x-k-5=0이고,
x+1
OPÓ_OQÓ=aÛ`+bÛ`=(a+b)Û`-2ab
이 이차방정식의 판별식을 D라 하면
=8Û`-2_14=36
D
=(-2)Û`-k(-k-5)=0이므로
4 TIP
kÛ`+5k+4=0, (k+4)(k+1)=0
(x-2)y=2x+k에서 x=2일 때 함숫값이 존재하면
∴ k=-4 또는 k=-1
k=-4이다. 이때, (x-2)y=2x-4에서 x=2일 때 항상
따라서 두 그래프가 적어도 한 점에서 만나도록 하는
등식을 만족시키고, x+2일 때 y=2이므로 이 도형은 두 직선
실수 k의 값의 범위는 kÉ-4 또는 k¾-1이다. yy TIP x=2, y=2를 나타낸다. 이 도형이 직선 x+y=8과 만나는
두 점은 각각 (2, 6), (6, 2)이고, 두 점의 x좌표의 곱은
TIP
12이므로 조건을 만족시키지 않는다. 따라서 k+-4이다.
-2x-1
곡선 y= 과 직선 y=kx-k-6의 위치 관계는
x+1
다음과 같다.
❶ -4<k<-1일 때 만나지 않는다. 383 ④
❷ k=-4 또는 k=-1 또는 k=0일 때 한 점에서 만난다.
조건 ㈎에 의하여 점 B(a, b)와 점 C는 직선 y=x에 대하여
❸ k<-4 또는 -1<k<0 또는 k>0일 때 두 점에서 만난다.
대칭이므로 C(b, a)이고, 직선 BC는 점 B(a, b)를 지나고

Ⅴ. 함수와 그래프 93
기울기가 -1이므로 x+1+2¿¹xÛ`-1+x-1
=
직선의 방정식은 x+y-a-b=0 x+1-(x-1)
한편, 점 A(-2, 2)에서 직선 BC에 내린 수선의 발을 H라 하면 2x+2¿¹xÛ`-1
= =x+¿¹xÛ`-1
2
BCÓ=¿¹2(a-b)Û`='2(a-b)
따라서 x='2를 대입하면 구하는 값은
|-2+2-a-b| a+b
AHÓ= =
¿¹1Û`+1Û` '2 '2+¿¹'2 Û`-1='2+1이다.
조건 ㈏에 의하여 삼각형 ABC의 넓이는 2'3이므로
1 1 a+b
_BCÓ_AHÓ= _'2(a-b)_ =2'3에서
2
aÛ`-bÛ`=4'3
2 '2 386 ③

2 2 ¿¹nÛ`<¿¹nÛ`+2n<¿¹nÛ`+2n+1이므로
또한 점 B는 곡선 y= 위의 점이므로 b= , 즉 ab=2
x a n<¿¹nÛ`+2n<n+1에서
(aÛ`+bÛ`)Û`=(aÛ`-bÛ`)Û`+4aÛ`bÛ`
¿¹nÛ`+2n의 정수부분은 n이고, yy ㉠
=(4'3)Û`+4_2Û`=64
소수부분은 f(n)=¿¹nÛ`+2n-n이다.
∴ aÛ`+bÛ`=8
2n 2n 2n(¿¹nÛ`+2n+n)
= =
f(n) ¿¹nÛ`+2n-n (nÛ`+2n)-nÛ`
=¿¹nÛ`+2n+n
384 ③
2n
이므로 ㉠에 의하여 의 정수부분은 2n이다.
제1사분면 위의 두 점 C, D가 직선 y=x에 대하여 대칭이고, f(n)
두 점 A, C가 원점에 대하여 대칭이므로 yy TIP
C(a, b) (a>b)라 하면 D(b, a), A(-a, -b)이다.
ADÓ='2(a+b), CDÓ='2(a-b)이고,
387 12

이때, ABÓ`:`BCÓ=CDÓ`:`ADÓ=1`:`2에서 ADÓ=2CDÓ이므로 B의 장력을 Tõ, 밀도를 qõ, 주파수를 wõ라 하면


'2(a+b)=2'2(a-b), a=3b yy ㉠ 1 Tõ
wõ= ¾Ð yy ㉠
6 2L qõ
또한 점 C(a, b)가 함수 y= 의 그래프 위의 점이므로
x 1
A의 장력은 3Tõ, 밀도는 nqõ, 주파수는 wõ이므로
ab=6 yy ㉡ 2
㉠, ㉡을 연립하여 풀면 bÛ`=2, aÛ`=18 1 1 3Tõ
wõ= ¾Ð yy ㉡
2 2L nqõ
또한 점 C(a, b)가 원 xÛ`+yÛ`=k 위의 점이므로
㉠을 ㉡으로 양변을 각각 나누면
k=aÛ`+bÛ`=20
n n
2=¾Ð , =4
TIP 3 3
∴ n=12
6
함수 y= 의 그래프와 원 xÛ`+yÛ`=k가 모두 직선 y=x에
x
대하여 대칭이므로 두 그래프의 교점도 2개씩 서로 직선 y=x에
대하여 대칭이다. 388 ②

즉, 두 점 C, D가 직선 y=x에 대하여 대칭이고, 두 점 A, B가 ax+b


유리함수 y= 의 그래프의 점근선의 방정식은
직선 y=x에 대하여 대칭이다. cx+2
마찬가지로 두 그래프가 모두 원점에 대하여 대칭이므로 두 2 a
x=- , y= 이고, 주어진 그래프에 의하여
그래프의 교점도 2개씩 서로 원점에 대하여 대칭이다. c c

즉, 두 점 A, C가 원점에 대하여 대칭이고, 두 점 B, D가 2 a


- <0, <0에서 c>0, a<0이다.
c c
원점에 대하여 대칭이다.
ax+b b
이때, 함수 y= 의 y절편이 음수이므로 <0에서 b<0이다.
cx+2 2
a
한편, 무리함수 y=-'¶a-bx+c=-¾Ð-b{x- }+c의
b
385 ④
그래프는 y=-'¶-bx의 그래프를 x축의 방향으로
a
b
만큼,
x='2일 때 x+1>0, x-1>0이므로 y축의 방향으로 c만큼 평행이동한 것이다.
'¶x+1+'¶x-1
a
'¶x+1-'¶x-1 이때, -b>0, >0, c>0이므로 무리함수의 그래프는
b
('¶x+1+'¶x-1)Û` a
= 점 { , c}를 지나면서 오른쪽 아래로 그려진다.
('¶x+1-'¶x-1)('¶x+1+'¶x-1) b

94 수학<하>
y
c 391 7

함수 y='¶a(x+3)의 그래프는 점 (-3, 0)을 지난다.


O a x
b y
y=-Âa-˛bx˛+c {ii}a<0 A

B D
따라서 지나는 사분면은 제1, 4사분면이다. {i}a>0
C

-3 O x

389 ③
사각형 ABCD와 만나지 않으려면
1 Ú a>0일 때, 함수 y='¶a(x+3)의 그래프가
함수 y=4-'¶kx+1=-¾Ðk{x+ }+4의 그래프는
k 점 C(1, 2)보다 아래에 위치해야 하므로
1
함수 y=-'¶kx의 그래프를 x축의 방향으로 - 만큼, f(1)='¶4a<2에서 4a<4, a<1
k
따라서 0<a<1이다.
y축의 방향으로 4만큼 평행이동한 것이다.
Û a<0일 때, 함수 y='¶a(x+3)의 그래프가
ㄱ. 4-'¶kx+1É4이므로 k의 값에 관계없이 치역은
점 B(-5, 4)보다 아래에 위치해야 하므로
{y|yÉ4}이다. (참) Ⅴ
1 f(-5)='¶-2a<4에서 -2a<16, a>-8 함
ㄴ. k>0일 때, kx+1¾0에서 x¾- 이므로 정의역은 수
k 따라서 -8<a<0이다.

1 Ú, Û에서 -8<a<0 또는 0<a<1이므로 그
[x|x¾- ]이다. (거짓)
k 정수 a는 -7, -6, -5, -4, -3, -2, -1로 7개이다. 래
y 프

4
3
y=4-ÂkxÊ+˛1˛
392 ⑤

무리함수 y=a'¶bx+c의 치역이 {y|yÉ0}이므로


1 O x
- a<0 yy ㉠
k
c
ㄷ. 함수 y=4-'¶kx+1의 그래프의 y절편은 3이고 k<0일 때, y=a'¶bx+c=a®Éb{x+ }의 그래프는 y=a'¶bx의 그래프를
b
1 c c
- >0이므로 그래프는 제4사분면을 지나지 않는다. (참) x축의 방향으로 - 만큼 평행이동시킨 그래프이므로 - >0,
k b b
y
c
즉 <0
4 b
y=4-ÂkxÊ+˛1˛ 3 한편, y=a'¶bx의 그래프에서 정의역은 {x|xÉ0}이므로
b<0 yy ㉡
O -1 x
따라서 c>0 yy ㉢
k
b b
따라서 옳은 것은 ㄱ, ㄷ이다. 유리함수 y= +c의 그래프는 함수 y= 의 그래프를 x축의
x+a x
방향으로 -a>0만큼, y축의 방향으로 c>0만큼 평행이동시킨
그래프이므로
390 ③
㉠, ㉡, ㉢에 의하여 유리함수 y=
b
+c의 그래프로 알맞은 것은
x+a
A(a, 0)이므로 B(a, 3'a)이다.
⑤이다.
점 C의 x좌표를 t라 하면 '¶2t=3'a이므로
9
2t=9a, t= a
2
9 1
따라서 D{ a, 0}이다.
2 393 6
9 7 두 점 P, Q의 좌표를 각각 (a, '¶2a), (b, '¶2b)라 하면
ABÓ=3'a, ADÓ= a-a= a에서 ABÓ=ADÓ이므로
2 2
'¶2a+'¶2b
7 선분 PQ의 중점의 y좌표는 =6에서
3'a= a, 7a=6'a, 49aÛ`=36a, a(49a-36)=0 2
2
'¶2a+'¶2b=12이다. yy ㉠
36
∴ a= (∵ a>0) 직선 PQ의 기울기는
49

Ⅴ. 함수와 그래프 95
'¶2a-'¶2b '2('a-'b) ¿¹(a-b)Û`+(-a+b)Û`=¿¹2(a-b)Û`이고, yy ㉡
=
a-b ('a+'b)('a-'b) ㉠에서 이차방정식의 근과 계수의 관계에 의하여
'2 a+b=-1, ab=-k이므로
=
'a+'b
(a-b)Û`=(a+b)Û`-4ab=1+4k
12
이때, ㉠에서 'a+'b= =6'2이므로 1
'2 ㉡에 대입하면 f(k)='¶2(1+4k) {k>- }이고,
4
'2 1
구하는 직선 PQ의 기울기는 = 이다. 함수 y= f(x)의 그래프의 개형은 다음과 같다.
6'2 6
y

y=f{x}
394 ③

두 곡선 y='¶-2x-4, y='¶-2x+4에서 1 O x
-
4
곡선 y='¶-2x+4를 x축의 방향으로 -4만큼 평행이동하면
따라서 구하는 답은 ②이다.
곡선 y='¶-2x-4가 되므로 yy TIP
다음 그림에서 색칠된 두 부분의 넓이는 서로 같다.
y 396 ②
y=Â-2˛x˛-˛4˛ y=Â-2˛x˛+˛4˛
2 함수 f(x)='¶x+3-1에 대하여 함수 y= f(x)의 그래프는
y=2
함수 y='x의 그래프를 x축의 방향으로 -3만큼,
x y축의 방향으로 -1만큼 평행이동한 것이므로
-4 -2 O 2
정의역이 {x|x>-3}일 때, 치역은 {y|y>-1}이다.
즉, 구하는 도형의 넓이(빗금친 부분)는 다음 색칠된 부분의 넓이와
y
같다. y=Âx+˛3˛-1
y
y=Â-2˛x˛-˛4˛ y=Â-2˛x˛+˛4˛ -3
2 O x
y=2
-1

x 2x+4 -2
-4 -2 O 2 g(x)= = +2이므로 함수 y=g(x)의 그래프는
x+3 x+3
따라서 구하는 넓이는 가로의 길이가 4, 세로의 길이가 2인 다음과 같다.
직사각형의 넓이와 같으므로 4_2=8이다. 2x+4 y
y=
x+3
TIP
두 함수 y='¶-2x-4, y='¶-2x+4의 그래프는 모두 함수
2
y='¶-2x의 그래프를 평행이동한 것이므로 평행이동하면 서로
겹쳐진다. 즉, 점 (-2, 0)이 점 (2, 0)으로 평행이동된 것으로
-3 -1 O x
볼 수 있으므로 x축의 방향으로 4만큼 평행이동한 것이다.

x>-1에서 g(-1)<g(x)<2이고, g(-1)=1이므로


x>-3에서 정의된 함수 y=(g ç f)(x)의 치역은
395 ②
{y|1<y<2}이다.
곡선 y=xÛ`과 직선 y=-x+k가 두 점에서 만나므로
이차방정식 xÛ`=-x+k, 즉 xÛ`+x-k=0의 판별식을 D라 하면 397 -9
1
D=1+4k>0에서 k>- 이다. 함수 f(x)의 역함수를 구하면 다음과 같다.
4
y 1 1
y=x@ y= xÛ`+x+ (xÉ-1)에서 2y=(x+1)Û`
2 2
y=-x+k A x+1=-'¶2y (∵ xÉ-1), x=-'¶2y-1
x와 y를 서로 바꾸면 y=-'¶2x-1
B ∴ f ÑÚ`(x)=-'¶2x-1
O x
함수 y= f ÑÚ`(x)는 x의 값이 커질 때 y의 값은 작아지므로
방정식 xÛ`+x-k=0의 두 실근을 a, b라 하면 yy ㉠ 2ÉxÉ8에서 f ÑÚ`(2)¾ f ÑÚ`(x)¾ f ÑÚ`(8),
두 점 A, B의 좌표가 (a, -a+k), (b, -b+k)이므로 즉 -5É f ÑÚ`(x)É-3이므로
선분 AB의 길이는 { f ÑÚ`(x)|2ÉxÉ8}={x|-5ÉxÉ-3}

96 수학<하>
a 따라서 a=9이고, 이때 함수 y= f(x)의 그래프는 다음과 같다.
한편, g(x)= +b에서
x+1 y=f{x} y
Ú a>0이면 1ÉxÉ3에서 g(3)Ég(x)Ég(1)이므로
y=Â3-˛x˛+9
a a
+bÉg(x)É +b {x 3} 9
4 2 2x+3
y= x-2 {x>3}
a a
따라서 +b=-5, +b=-3을 만족시켜야 하므로 2
4 2
두 식을 연립하여 풀면 a=8, b=-7 O 3 x
Û a<0이면 1ÉxÉ3에서 g(1)Ég(x)Ég(3)이므로
a a f(2)=1+9=10이므로
+bÉg(x)É +b
2 4 f(2) f(k)=40에서 f(k)=4이다.
a a f(k)=4일 때, k>3이므로
따라서 +b=-5, +b=-3을 만족시켜야 하므로
2 4 2k+3
f(k)= =4에서 2k+3=4k-8
두 식을 연립하여 풀면 a=-8, b=-1 k-2
Ü a=0이면 모든 실수 x에 대하여 g(x)=b이므로 11
∴ k=
조건을 만족시키지 못한다. 2
Ú ~ Ü에서 a+b의 최솟값은 (-8)+(-1)=-9이고,
a+b의 최댓값은 8+(-7)=1이다. 399 48 Ⅴ
따라서 구하는 값은 (-9)_1=-9이다. 함
함수 y='¶x+4-3의 그래프는 함수 y='x의 그래프를 x축의 수
다른 풀이 와
방향으로 -4만큼, y축의 방향으로 -3만큼 평행이동한 것이고,

{ f ÑÚ`(x)|2ÉxÉ8}를 다음과 같이 구할 수 있다. 함수 y='¶-x+4+3의 그래프는 함수 y='¶-x의 그래프를 x축의 래
f ÑÚ`(x)=t라 하면 x= f(t)이므로 프
방향으로 4만큼, y축의 방향으로 3만큼 평행이동한 것이다.
{ f ÑÚ`(x)|2ÉxÉ8}={x|2É f(x)É8}이다. 두 함수 y= f(x), y=g(x)의 그래프와 두 직선 x=-4, x=4를
1 1 나타내면 다음과 같다.
2É xÛ`+x+ É8에서 4ÉxÛ`+2x+1É16
2 2
y
4É(x+1)Û`É16, -4Éx+1É-2 (∵ xÉ-1) y=Â-x˛+˛4˛+3
-5ÉxÉ-3 A
∴ { f ÑÚ`(x)|2ÉxÉ8}={x|-5ÉxÉ-3} 3
B
4
398 ⑤ -4 O x
y=Âx+˛4˛-3
C
2x+3 7
Ú x>3일 때 f(x)= = +2이므로 -3
x-2 x-2
x>3에서 함수 y= f(x)의 그래프가 다음과 같다. 이때, 그림의 A, C 부분의 넓이가 서로 같으므로
y (구하는 도형의 넓이)=(A 부분의 넓이)+(B 부분의 넓이)
9 =(C 부분의 넓이)+(B 부분의 넓이)
2x+3 =8_6=48
y= x-2 {x>3}
2

O 3 x 400 ②

5
Û xÉ3일 때 f(x)='¶-(x-3)+a이므로 함수 y=-'¶2x-5-2=-¾Ð2{x- }-2의 그래프는
2
xÉ3에서 y= f(x)의 그래프의 개형은 다음과 같다. 5
함수 y=-'¶2x의 그래프를 x축의 방향으로 만큼,
y 2
y축의 방향으로 -2만큼 평행이동한 것이므로 다음과 같다.
y 5
{3,`a} 2
O x
x=3 -2
조건 ㈎에서 함수 f의 치역이 {y|y>2}이어야 하므로
Ú, Û에서 함수 y= f(x)의 그래프에 의하여
2<aÉ9이어야 한다. {i} {ii}
또한 조건 ㈏에 의하여 함수 f가 일대일함수이어야 하므로 직선 kx+y=0, y=-kx는 기울기가 -k이고 원점을 지나고,
a¾9이어야 한다. n(A;B)=2이므로 두 그래프가 두 점에서 만나야 한다.

Ⅴ. 함수와 그래프 97
이때, 두 그래프는 다음과 같은 경우를 기준으로 교점의 개수가 달라 Û 함수 y=|x+a|의 그래프가 점 (0, 0)을 지날 때
진다. a=0
Ú 두 그래프가 접할 때 Ú, Û에 의하여 g(a)=3인 a의 값의 범위는
-kx=-'¶2x-5-2에서 '¶2x-5=kx-2 1
0<a< 이다. yy TIP
양변을 각각 제곱하면 2
2x-5=(kx-2)Û` 1
m=0, n=
2
이차방정식 kÛ`xÛ`-2(2k+1)x+9=0의 판별식을 D라 하면
∴ mÛ`+4nÛ`=1
D
=(2k+1)Û`-9kÛ`=0이므로 -5kÛ`+4k+1=0
4 TIP
1 y=|x+a|의 그래프를 이동시켜보면서 y='¶|x|+x의
(5k+1)(k-1)=0, k=- 또는 k=1
5
그래프와 만나는 교점의 개수를 살펴보면 다음과 같다.
이때, 두 그래프가 접할 때 직선의 기울기는 음수이므로
-k<0, 즉 k>0에서 k=1 1 1
❶ a> 일 때 ❷ a= 일 때
2 2
5 y y y y
Û 직선 y=-kx가 점 { , -2}를 지날 때
2
y=|x+a|
y=|x+a| y=|x+a|
y=|x+a|
5 4
-2=- k에서 k= y y y y
2 5
y=|x+a| y=´|x¨ |¨+|¨x¨+
y=´|x¨ ¨ ¨xy=|x+a|
¨ y=´|x¨
y=´|x¨ |¨+|¨x¨+
¨ ¨x¨
Ú, Û에서 함수의 그래프와 직선이 두 점에서 만나도록 하는 y=|x+a| y=|x+a|
O O x x O O x x
4
k의 값의 범위는 Ék<1이다. yy TIP y=´|x¨ |¨+|¨x¨+
¨ ¨x¨
5 한 점에서 만난다.y=´|x¨ 두 점에서 y=´|x¨ |¨+|¨x¨+
y=´|x¨
만난다. ¨ ¨x¨
OyO y x x OyO y x x
TIP 1
y=|x+a|
y=|x+a|
❸ 0<a< 일때 ❹ y=|x+a|
y=|x+a|
aÉ0일 때
2
곡선 y=-'¶2x-5-2와 직선 kx+y=0의 위치 관계는 y y y y
다음과 같다. y=|x+a| y=´|x¨
y=|x+a| y=´|x¨
|¨+|¨x¨+
¨ ¨x¨ y=|x+a|
y=|x+a| y=´|x¨
y=´|x¨
|¨+|¨x¨+
¨ ¨x¨
❶ kÉ0 또는 k>1일 때 만나지 않는다. O O x x O O x x

4 y=´|x¨ |¨+|¨x¨+
y=´|x¨ ¨ ¨x¨ y=´|x¨ |¨+|¨x¨+
y=´|x¨ ¨ ¨x¨
❷ 0<k< 또는 k=1일 때 한 점에서 만난다.
5 O O x x O O x x
4
❸ Ék<1일 때 두 점에서 만난다. 세 점에서 만난다. 두 점에서 만난다.
5

402 ⑤

401 ①
집합 Aû={x|'¶3|x|+x=2x+k}는 함수 y='¶3|x|+x의
함수 y='¶|x|+x는 x≥0일 때 y='¶2x, x<0일 때 y=0이므로 그래프와 직선 y=2x+k의 교점의 x좌표를 원소로 하는 집합이다.
그래프가 다음과 같다. 따라서 n(Aû)=3이면 두 함수의 그래프의 서로 다른 교점의 개수가
y 3이므로 집합 B={k|n(Aû)=3}은 함수 y='¶3|x|+x의 그래프
{i} {ii}
y=|x+a| 와 직선 y=2x+k가 서로 다른 세 점에서 만나도록 하는 모든 k의
y=´|x¨|¨+¨x¨
값의 집합이다.
á 2'x (x¾0)
함수 y='¶3|x|+x=Ò 이므로 그래프가 다음과 같다.
» '¶-2x (x<0)
-a O x {i}{ii}
y
이때, 두 함수 y=|x+a|와 y='¶|x|+x의 그래프는 다음과 같은 y=Â-2˛xÊ y=2Âx°
경우를 기준으로 교점의 개수가 달라진다.
Ú 두 함수 y='¶|x|+x, y=|x+a|의 그래프가 접할 때
O x
즉, 직선 y=x+a가 곡선 y='¶2x와 접하면
'¶2x=x+a 이때, 직선 y=2x+k는 기울기가 2이고, y절편이 k이므로
양변을 각각 제곱하면 두 그래프는 다음과 같은 경우를 기준으로 교점의 개수가 달라진다.
2x=(x+a)Û`에서 이차방정식 xÛ`+2(a-1)x+aÛ`=0의 Ú 직선 y=2x+k가 함수 y=2'x의 그래프에 접할 때
D 2x+k=2'x의 양변을 각각 제곱하면
판별식을 D라 하면 =(a-1)Û`-aÛ`=0이므로
4
(2x+k)Û`=4x에서 이차방정식 4xÛ`+2(2k-2)x+kÛ`=0의
1
-2a+1=0, a= 판별식을 D라 하면
2

98 수학<하>
D y
=(2k-2)Û`-4kÛ`=0이므로
4 2 {ii}
1 {i}
-8k+4=0에서 k=
2
Û 직선 y=2x+k가 점 (0, 0)을 지날 때
k=0
Ú, Û에 의하여 두 그래프가 세 점에서 만나도록 하는 -2 O x

1 Ú 함수 y='¶x-k의 그래프가 직선 y=x+2와 접할 때


k의 값의 범위는 0<k< yy TIP
2
x+2='¶x-k의 양변을 각각 제곱하면
1
B=[k|0<k< ], C={k|a<k<b}이고, (x+2)Û`=x-k에서 이차방정식 xÛ`+3x+k+4=0의
2
7
1 판별식을 D라 하면 D=9-4(k+4)=0이므로 k=-
B=C이므로 a=0, b= 4
2
Û 함수 y='¶x-k의 그래프가 직선 y=x+2가 x축과 만나는
1
∴ a+b= 교점인 점 (-2, 0)을 지날 때
2
0='¶-2-k, k=-2
TIP
Ú, Û에 의하여 두 그래프가 서로 다른 두 점에서 만나도록 하는
곡선 y='¶3|x|+x와 직선 y=2x+k의 위치 관계는
k의 값의 범위는
다음과 같다.
7

1 -2Ék<- 이다. yy㉡ 함
❶ k<0 또는 k> 일 때 한 점에서 만난다. 4 수
2 와
따라서 ㉠, ㉡을 모두 만족시키는 k의 값의 범위는
1 그
❷ k=0 또는 k= 일 때 두 점에서 만난다. 15 7 15 7
2 - <k<- 이므로 m=- , n=- 래
8 4 8 4 프
1
❸ 0<k< 일 때 세 점에서 만난다. n 14
2 ∴ =
m 15

TIP
함수 y='¶x-k의 그래프는 x의 값이 커질 때 y의 값이 무한히
8x+15
커지므로 함수 y= 의 그래프와 x>-1에서는 항상 한
x+1
점에서 만난다.
403 ④
이때, 두 함수 y='¶x-k, y=
8x+15
의 그래프는
x+1
함수 y='¶x-k의 그래프는 함수 y='x의 그래프를
15
x축의 방향으로 k만큼 평행이동한 것이다. ❶ kÉ- 이면
8
8x+15 7 x<-1에서도 한 점에서 만나므로 두 그래프는 두 점에서
함수 y= = +8의 그래프는 다음과 같다.
x+1 x+1
만난다.
y
15
15 ❷ k>- 이면
8
8x+15 x<-1에서 만나지 않으므로 두 그래프는 한 점에서 만난다.
y=
8 x+1

-1
15 O x
-
8

8x+15
이때, 함수 y= 의 그래프가 x축과 만나는 점의 x좌표는
x+1 404 ④
8x+15 15
0= 에서 x=- 이므로 함수 y=4'¶x+2+k의 그래프는 함수 y=4'x의 그래프를 x축의
x+1 8
8x+15 방향으로 -2만큼, y축의 방향으로 k만큼 평행이동한 것이므로
함수 y='¶x-k의 그래프가 함수 y= 의 그래프와
x+1 점 (-2, k)를 지나면서 오른쪽 위로 증가하는 형태이다. 따라서
15 함수의 그래프가 역함수의 그래프와 만나는 교점은 직선 y=x와의
한 점에서 만나려면 k>- yy㉠
8 교점과 같다.
이어야 한다. yy TIP 따라서 함수 y= f(x)의 그래프가 직선 y=x와 서로 다른 두 점에서
한편, 함수 y='¶x-k의 그래프와 직선 y=x+2는 만나도록 하는 정수 k의 개수를 구하면 된다. 이때, 두 그래프는
다음과 같은 경우를 기준으로 교점의 개수가 달라진다. 다음과 같은 경우를 기준으로 교점의 개수가 달라진다.

Ⅴ. 함수와 그래프 99
y y=x {ii} '2
ABÓ=2, CHÓ='2{2- }=2'2-1이므로
{i}
2
1
삼각형 ABC의 넓이는 _2_(2'2-1)=2'2-1이다.
2

O x
{-2,`k}
406 ⑤

함수 y='¶1-x (xÉ1)의 역함수는


Ú 직선 y=x가 함수 f(x)=4'¶x+2+k의 그래프에 접할 때
y=1-xÛ` (x¾0)이다.
4'¶x+2+k=x에서 4'¶x+2=x-k의 양변을 각각 제곱하면
16(x+2)=(x-k)Û`에서 이차방정식 두 함수의 그래프의 교점은 yy TIP
xÛ`-2(k+8)x+kÛ`-32=0의 판별식을 D라 하면 방정식 '¶1-x=1-xÛ`에서 양변을 각각 제곱하면
D 1-x=(1-xÛ`)Û`, 1-x=(1-x)Û`(1+x)Û`
=(k+8)Û`-(kÛ`-32)=0이므로
4 (1-x){(1-x)(x+1)Û`-1}=0
16k+96=0, k=-6 (1-x)(-xÜ`-xÛ`+x)=0
Û 직선 y=x가 점 (-2, k)를 지날 때 x(x-1)(xÛ`+x-1)=0에서
k=-2 -1Ñ'5
x=0 또는 x=1 또는 x=
Ú, Û에 의하여 함수 y= f(x)의 그래프와 직선 y=x가 서로 다른 2
두 점에서 만나도록 하는 k의 값의 범위는 -6<kÉ-2이므로 이때, 두 함수의 정의역의 공통 범위가 0ÉxÉ1이므로
정수 k는 -5, -4, -3, -2로 4개이다. -1+'5
교점의 x좌표는 0, 1, 로 3개이다.
2
-1+'5 7+'5
∴ m+n=3+{0+1+ }=
2 2
405 ③
TIP
함수 f(x)='¶x+2의 그래프는 다음과 같다. 함수 y='¶1-x의 그래프는 점 (1, 0)을 지나면서 왼쪽 위로
y y=Âx+˛2˛ 그려지는 형태이므로 감소하는 형태이다. 따라서 역함수의
그래프와의 교점이 반드시 직선 y=x 위에 있다고 볼 수
Â2
없으므로 위의 풀이와 같이 직접 함수와 역함수의 교점의 좌표를
구하는 방정식을 세워 교점을 구한다.
-2 O x

x=0일 때 함숫값이 '2이므로 A(0, '2 )이다. 121


함수 y= f ÑÚ`(x)의 그래프는 함수 y= f(x)의 그래프와 y=x에 407 16
대하여 대칭이므로 점 ('2, 0)을 지난다. 정사각형 ADBC의 넓이가 최소이려면 대각선 AB의 길이가
즉, B('2, 0)이다. 최소이면 된다.
함수 y= f(x)의 그래프가 증가하는 형태이므로 함수의 그래프가 즉, 점 A와 직선 y=x 사이의 거리가 최소이면 되므로 그림과 같이
역함수의 그래프와 만나는 교점은 직선 y=x와의 교점과 같다. 기울기가 1인 직선이 함수 y= f(x)의 그래프에 접할 때의 접점이
'¶x+2=x에서 양변을 각각 제곱하면 x+2=xÛ` A이면 된다.
xÛ`-x-2=0, (x+1)(x-2)=0, x=-1 또는 x=2 y
y=f —!{x}
이때, 점 (-1, -1)은 y= f(x)의 그래프 위의 점이 아니므로 y=x
C(2, 2)이다.
y y=f—!{x}
y=x B D
C y=f{x}
A y=f{x}
C A
x O x
O B
기울기가 1인 접선을 y=x+a (a는 상수)라 하면
'¶x-3=x+a에서 양변을 각각 제곱하면
x-3=(x+a)Û`에서 이차방정식 xÛ`+(2a-1)x+aÛ`+3=0의
한편, 삼각형 ABC는 ACÓ=BCÓ인 이등변삼각형이므로 점 C에서 판별식을 D라 하면
선분 AB에 내린 수선의 발을 H라 하면 D=(2a-1)Û`-4(aÛ`+3)=0이므로
'2 '2 11
점 H는 선분 AB의 중점으로 H{ , }이다. -4a-11=0, a=-
2 2 4

100 수학<하>
11 11 함수 y=xÛ`-4 (x¾0)의 그래프와 접하는 접점이 B일 때이다.
직선 y=x와 직선 y=x- 사이의 거리가 '2이므로
4 8 기울기가 1인 접선을 y=x+b (b는 상수)라 하면
11 이차방정식 xÛ`-4=x+b, 즉 xÛ`-x-4-b=0의
정사각형 ADBC의 대각선의 길이가 '2이다.
4
판별식을 D라 하면 D=1-4(-4-b)=0이므로
따라서 정사각형 ADBC의 넓이의 최솟값은
17
1 11 121 17+4b=0, b=-
_{ '2}Û`= 이다. 4
2 4 16
17 17'2
다른 풀이 두 직선 y=x- 과 y=x 사이의 거리가 이므로
4 8
점 A와 직선 y=x 사이의 거리의 최솟값을 다음과 같이 구할 수 있다. 17'2
구하는 선분 AB의 길이의 최댓값은 이다.
4
y='¶x-3에서 yÛ`=x-3, x=yÛ`+3이므로
점 A의 좌표를 (aÛ`+3, a)(a¾0)이라 하면 점 A와 직선 y=x 다른 풀이 2

사이의 거리는 ㉠에서 선분 AB의 길이는 점 B와 직선 y=x 사이의 거리의 2배와


1 11 같으므로 다음과 같이 구할 수 있다.
|{a- }Û`+ |
|aÛ`+3-a| 2 4
= 이므로 점 B가 함수 y=xÛ`-4(x¾0)의 그래프 위의 점이므로
¿¹1Û`+(-1)Û` '2
점 B의 좌표를 (a, aÛ`-4)(a¾0)이라 하면 점 B와 직선 y=x
1 11 11'2
a= 일 때 최솟값 = 를 갖는다. 사이의 거리는
2 4'2 8
1
|-{a- }Û`+ |
17 Ⅴ
|a-aÛ`+4| 2 4 함
= 이므로
¿¹1Û`+(-1)Û` '2 수

408 ②
1
a= 일 때 최댓값
17
=
17'2
를 갖는다.


2 4'2 8 래
두 함수 y='¶x+4, y=xÛ`-4 (x¾0)가 서로 역함수 관계이므로 프
17'2
두 함수의 그래프는 직선 y=x에 대하여 대칭이다. 따라서 구하는 선분 AB의 길이의 최댓값은 이다.
4
이때, 직선 AB의 기울기가 -1이므로 각각 두 함수의 그래프 위에
있는 점 A, B는 직선 y=x에 대하여 대칭이다. 따라서 선분 AB의
길이는 점 A와 직선 y=x 사이의 거리의 2배이다. yy ㉠
y
409 풀이 참조

y=x@-4
y=x ⑴ 곡선 y='¶4-x가 x축, y축과 만나는 점은 각각
2 y=Âx+˛4˛ A(4, 0), B(0, 2)이다.
A y

-4 O 2 x
y=Â4-˛x˛ B
P
-4 B
y=-x+k
O A x
-4<k<2일 때 직선 y=-x+k와 함수 y='¶x+4의 그래프가
만나는 점 A는 제2사분면 위에 존재하고, 점 A와 직선 y=x (사각형 OAPB의 넓이)
사이의 거리가 최대일 때는 기울기가 1인 직선이 함수 y='¶x+4의 =(삼각형 OAB의 넓이)+(삼각형 PAB의 넓이)
그래프와 접하는 접점이 A일 때이다. 1
이때, 삼각형 OAB의 넓이가 _4_2=4로 일정하므로 삼각형
2
기울기가 1인 접선을 y=x+a (a는 상수)라 하면
PAB의 넓이가 최대일 때, 사각형 OAPB의 넓이가 최대이다.
'¶x+4=x+a에서 양변을 각각 제곱하면
삼각형 PAB의 밑변을 선분 AB라 하면 ABÓ=2'5이므로 점
x+4=xÛ`+2ax+aÛ`에서 이차방정식
P와 직선 AB 사이의 거리가 최대일 때, 삼각형 PAB의 넓이가
xÛ`+(2a-1)x+aÛ`-4=0의 판별식을 D라 하면
최대이다.
17
D=(2a-1)Û`-4(aÛ`-4)=0이므로 -4a+17=0, a= x y
4 직선 AB의 방정식은 + =1, x+2y-4=0이고,
4 2
17 17'2
두 직선 y=x+ , y=x 사이의 거리가 이므로 y='¶4-x에서 yÛ`=4-x, x=4-yÛ`이므로
4 8
17'2 점 P의 좌표를 P(4-aÛ`, a)라 하면 yy㉠
구하는 선분 AB의 길이의 최댓값은 이다.
4 점 P와 직선 AB 사이의 거리는
다른 풀이 1 |4-aÛ`+2a-4| |-aÛ`+2a| |-(a-1)Û`+1|
= =
¿¹1Û`+2Û` '5 '5
㉠에서 선분 AB의 길이는 점 B와 직선 y=x 사이의 거리의 1
이므로 a=1일 때 최대이고, 최댓값 을 갖는다. yy㉡
2배와도 같으므로 다음과 같이 구할 수 있다. '5
점 B와 직선 y=x 사이의 거리가 최대일 때는 기울기가 1인 직선이 따라서 삼각형 PAB의 넓이의 최댓값이

Ⅴ. 함수와 그래프 101


1 1
2
_2'5_ =1이므로
'5 410 ④

사각형 OAPB의 넓이의 최댓값은 4+1=5이다. 1 (x-1)+1


1+
⑵ 사각형 OAPB의 넓이가 최대일 때, ㉡에서 a=1이므로 ㉠에서 x-1 x-1 x 2
f(x)= = = = +1이
1 (x-1)-1 x-2 x-2
점 P의 좌표는 (3, 1)이다. 1-
x-1 x-1
채점 요소 배점 x+1, x+2에서 정의되므로 함수 y= f(x)의 그래프는
사각형 OAPB의 넓이를 두 삼각형 OAB, PAB의 넓이의 합으로 다음과 같다.
80%
나타내어 최댓값 구하기 y
y=f{x}
사각형 OAPB의 넓이가 최대일 때, 점 P의 좌표 구하기 20%

1
1 2
O x
다른 풀이 -1

⑴ 점 P와 직선 AB 사이의 거리의 최댓값을 다음과 같이 직선


AB와 평행하고 곡선 y='¶4-x에 접하는 직선과 직선 AB ㄱ. 정의역은 {x|x는 x+1, x+2인 실수}이다. (거짓)
사이의 거리로 구할 수 있다. ㄴ. f(0)=0이므로 제3사분면을 지나지 않는다. (참)
y ㄷ. 직선 y=-x+3은 두 점근선의 교점 (2, 1)을 지나는 직선이다.
이때 f(1)이 정의되지 않으므로 직선 y=-x+3에 대하여 대칭
y=Â4-˛x˛ B
이 아니다. (거짓)
P
ㄹ. 직선 y=k가 함수 y= f(x)의 그래프와 만나지 않도록 하는 k의
값은 1 또는 -1이므로 모든 실수 k의 값의 합은 0이다. (참)
O A x
따라서 옳은 것은 ㄴ, ㄹ이다.
1
직선 AB의 기울기가 - 이므로 직선 AB에 평행하고 곡선
2
1
y='¶4-x에 접하는 접선을 y=- x+k (k는 상수)라 하자.
2 411 ②
1
'¶4-x=- x+k의 양변을 각각 제곱하면 3
2 함수 y= (x>0)의 그래프를 x축의 방향으로 4만큼, y축의 방향
x
1
4-x={- x+k}Û` 에서 이차방정식 3
2 으로 m만큼 평행이동하면 y= +m (x>4)이다.
x-4
xÛ`+2(2-2k)x+4kÛ`-16=0 yy㉢
y
의 판별식을 D라 하면
D B
=(2-2k)Û`-(4kÛ`-16)=0이므로
4 A
5 B
-8k+20=0, k= yy㉣ B
2 A
A
1 5 1
직선 y=- x+ 와 직선 y=- x+2 사이의 거리는 O x
2 2 2
직선 AB 위의 점 (0, 2)와 직선 x+2y-5=0 사이의 1 1
한편, 직선 y= x+k는 기울기가 인 직선이고, k의 값에
거리와 같으므로 2 2
|0+4-5| 1 3
= 관계없이 ABÓ가 일정하려면 함수 y= (x>0)의 그래프 위의
x
¿¹1Û`+2Û` '5
1 점 A를 x축의 방향으로 4만큼, y축의 방향으로 m만큼 평행이동한
따라서 점 P와 직선 AB 사이의 거리의 최댓값은 이다.
'5 점이 B가 되어야 한다.
(이후 풀이는 본풀이와 같다.) m 1
즉, 직선 AB의 기울기가 = 을 만족시켜야 하므로 m=2이다.
5 4 2
⑵ 사각형 OAPB의 넓이가 최대일 때, ㉣에서 k= 이므로
2 이때, ABÓ=¿¹4Û`+mÛ`=¿¹4Û`+2Û`='¶20=2'5이므로 n=2'5
㉢에 대입하면 xÛ`-6x+9=0, (x-3)Û`=0 ∴ mn=4'5
x=3, y='¶4-3=1에서 점 P의 좌표는 (3, 1)이다.

채점 요소 배점
사각형 OAPB의 넓이를 두 삼각형 OAB, PAB의 넓이의 합으로
80%
412 ②
나타내어 최댓값 구하기
3x+8
사각형 OAPB의 넓이가 최대일 때, 점 P의 좌표 구하기 20% 함수 y= 은
|x+1|-1

102 수학<하>
3x+8 -2 y 2
x<-1일 때 y= = -3 y=
-x-2 x+2 x
A
3x+8 8
x¾-1일 때 y= = +3
x x D
이므로 그래프가 다음과 같다. B
y O C x

y=3
또한 ㄴ에서 두 직선 AB, OD의 기울기의 합이 0이므로
-1 ∠DOC=∠DCO
O x
∠ADO=∠DOC+∠DCO=2∠DOC yy ㉡
y=-3
-5 ∴ ∠AOC=∠AOD+∠DOC
x=-2 1
=∠AOD+ ∠ADO (∵ ㉡)
2
3x+8
따라서 함수 y= 의 그래프와 직선 y=k가 한 점에서 3
|x+1|-1 = ∠AOD (∵ ㉠) (참)
2
만나도록 하는 실수 k의 값은 -3<kÉ3 또는 k=-5이므로
따라서 옳은 것은 ㄱ, ㄴ, ㄷ이다.
정수 k는 -5, -2, -1, 0, 1, 2, 3으로 7개이다.

참고
414 ③ Ⅴ
3x+8 함
함수 y= 의 그래프와 직선 y=k의 교점의
|x+1|-1 수
조건 ㈎에서 y=¿¹1-xÛ`에서 y¾0이고 양변을 각각 제곱하면 와
위치 관계는 다음과 같다.
yÛ`=1-xÛ`에서 xÛ`+yÛ`=1이므로 함수 y=¿¹1-xÛ`의 그래프는 그
❶ k<-5 또는 k>3일 때 두 점에서 만난다. 래
중심이 원점이고 반지름의 길이가 1이면서 y¾0인 범위에 그려지는
❷ k=-5 또는 -3<kÉ3일 때 한 점에서 만난다. 프
반원이다.
❸ -5<kÉ-3일 때 만나지 않는다.
y

-1 O 1 x

조건 ㈏에서 함수 y= f(x)의 그래프가 직선 x=1에 대하여 대칭이


므로 다음과 같다.
413 ⑤
y

2 2 1
-
2 b a
ㄱ. 직선 AB의 방정식은 y- = (x-a),
a b-a -1 O 1 3 x
2 2
y=- (x-a)+ 이다. 조건 ㈐에서 함수 y= f(x)의 그래프가 y축에 대하여 대칭이므로
ab a
2 2 다음과 같다.
y=0일 때, (x-a)= 에서 x=a+b이므로
ab a y
점 C의 x좌표는 a+b이다. (참) 1
2 2
-
b a 2 -3 -1 O 1 3 x
ㄴ. 직선 AB의 기울기는 =- 이다.
b-a ab
점 D는 선분 AB의 중점이므로 점 D의 좌표는 이때 다시 조건 ㈏에 의하여 함수 y= f(x)의 그래프가 직선 x=1에
2 2 대하여 대칭이고, 이와 같이 그래프를 그려나가면 함수 y= f(x)의
» a+b
+
a b ¼ a+b a+b } 그래프는 -1ÉxÉ1에서 f(x)=¿¹1-xÛ`이고 2 간격으로 모양이
, ,즉{ ,
2 2 2 ab
반복된다.
직선 OD의 기울기는
y
a+b
ab 2 1 y=f{x}
= 이다.
a+b ab
2
따라서 두 직선 AB와 OD의 기울기의 합은 O 1 3 5 x
y=kx+1
2 2
- + =0이다. (참) 직선 y=kx+1은 기울기가 k<0이고 점 (0, 1)을 지나므로 함수
ab ab
ㄷ. ABÓ=2OAÓ이면 AOÓ=ADÓ이므로 y= f(x)의 그래프와 서로 다른 6개의 점에서 만나려면 그림과 같이
∠AOD=∠ADO이다. yy ㉠ 직선 y=kx+1이 점 (5, 0)을 지나야 한다.

Ⅴ. 함수와 그래프 103


0=5k+1 y
y=Â2xÊ+˛3˛
1
∴ k=-
5

O x

13
415 -
4
<aÉ-3 또는 0Éa<3

함수 y=x+|x-a|는
a='¶2a+3의 양변을 각각 제곱하면
x¾a일 때, y=x+(x-a)=2x-a
aÛ`-2a-3=0, (a+1)(a-3)=0
x<a일 때, y=x-(x-a)=a이므로 그래프의 개형이 다음과 같다.
a=-1 또는 a=3에서 a=3 (∵ a>0)
Ú ~ Ý에서 두 그래프가 두 점에서 만나도록 하는
13
실수 a의 값의 범위는 - <aÉ-3 또는 0Éa<3이다.
{a,`a} 4
점 (a, a)가 직선 y=x 위의 점이므로 함수 y=x+|x-a|의 참고
그래프가 곡선 y='¶2x+3과 만나는 점의 개수는 다음과 같은 두 함수 y=x+|x-a|, y='¶2x+3의 그래프의 위치 관계는
경우를 기준으로 달라진다. 다음과 같다.
Ú 직선 y=2x-a가 곡선 y='¶2x+3에 접하는 경우 13
❶ a<- 또는 a>3일 때, 만나지 않는다.
y 4
y=Â2xÊ+˛3˛
13
❷ a=- 또는 -3<a<0 또는 a=3일 때,
4
한 점에서 만난다.
O x 13
❸- <aÉ-3 또는 0Éa<3일 때, 두 점에서 만난다.
4

2x-a='¶2x+3의 양변을 각각 제곱하면


(2x-a)Û`=2x+3
4xÛ`-2(2a+1)x+aÛ`-3=0의 판별식을 D라 할 때,
416 ④

D 13 조건 ㈎, ㈏를 만족시키는 함수 y= f(x)의 그래프는 다음과 같다.


=(2a+1)Û`-4(aÛ`-3)=0이므로 a=- y
4 4 y=f{x} 6
3
Û 직선 y=2x-a가 점 {- , 0}을 지나는 경우
2
y 2
y=Â2xÊ+˛3˛ -10 -6 -2 O 2 6 10 14 x

3 ax 2a
- 함수 y= =- +a의 그래프의 점근선의 방정식은
2 x+2 x+2
O x x=-2, y=a이다.
a의 값에 따라 두 함수의 그래프를 나타내면 다음과 같다.
Ú a<0일 때
ax
y y=
3 y=f{x} x+2
0=2_{- }-a에서 a=-3
2
Ü 직선 y=a가 점 (0, 0)을 지나는 경우
y 2 2
y=Â2xÊ+˛3˛ -10 -6 -2 O 6 10 14 x
a

O x 그림과 같이 -2<x<0에서 한 점에서만 만난다.


Û a=0일 때
ax
y= =0이므로 두 그래프는 만나지 않는다.
x+2
a=0 Ü a>0일 때
Ý 점 (a, a)가 곡선 y='¶2x+3 위에 있는 경우 ( - ①) 0<a<2 또는 a>6일 때

104 수학<하>
ax 5 5 5
y y= x+2 y='¶x+5에서 x= 일 때 함숫값이 각각 { }Û`-5= ,
y=f{x} 6 2 2 4
5 15
¾Ð +5=¾Ð 이므로
2 2 2

x
5 15 5
-10 -6 -2 O2 6 10 14 <¾Ð 에서 xª> 이다. (거짓)
4 2 2
ㄴ. 점 P는 두 함수 y=-xÛ`+5 (0ÉxÉ'5), y='¶x+5의
0<a<2일 때 그림과 같이 x<-2인 범위 내에서
그래프의 교점이고, 점 R는 두 함수 y=xÛ`-5 (x>'5),
유한개의 점에서만 만난다.
y='¶-x+5의 그래프의 교점이다.
y ax
y= 이때, 두 함수 y='¶-x+5와 y=-xÛ`+5 (x¾0)가 서로
x+2
y=f{x} 6 역함수이고, y='¶x+5와 y=xÛ`-5 (x¾0)가 서로
역함수이므로 각각 두 함수가 직선 y=x에 대하여 대칭이다.
2
따라서 두 점 P, R가 직선 y=x에 대하여 대칭이므로
-10 -6 -2 O2 6 10 14 x (x£, y£)=(yÁ, xÁ)에서 x£=yÁ, y£=xÁ이다.
a>6일 때 그림과 같이 x>-2인 범위 내에서 유한개의 ∴ xÁ yÁ=x£ y£ (참)
점에서만 만난다. ㄷ. x£(xÁ-5)=yÁ(y£-5)의 양변을 x£ yÁ로 나누면
( - ②) a=2 또는 a=6일 때 xÁ-5 y£-5

=

이고, 이때 좌변은 두 점 (xÁ, yÁ), (5, 0)을 Ⅴ
y ax 함
y=
x+2
지나는 직선의 기울기의 역수이고, 우변은 두 점 (x£, y£), (0, 5) 수
y=f{x} 를 지나는 직선의 기울기이다. 와
6
두 점 P, R와 두 점 (5, 0), (0, 5)가 각각 직선 y=x에 대하여 그

대칭이므로 두 점 (xÁ, yÁ), (5, 0)을 지나는 직선과 두 점 프
2
-10 -6 -2 O 2 6 10 14 x (x£, y£), (0, 5)를 지나는 직선은 서로 직선 y=x에 대하여
대칭이다.
따라서 두 직선의 기울기가 서로 역수 관계에 있으므로
a=2일 때 그림과 같이 x<-2인 범위 내에서 무수히
yy TIP
많은 점에서 만난다.
y xÁ-5 y£-5
ax = 이다. (참)
y= yÁ x£
x+2
y=f{x} 6 따라서 옳은 것은 ㄴ, ㄷ이다.

2
다른 풀이
-10 -6 -2 O2 6 10 14 x
ㄱ. 점 Q는 두 함수 y=xÛ`-5 (x>'5), y='¶x+5의 그래프의
a=6일 때 그림과 같이 x>-2인 범위 내에서 무수히
교점이다. 두 함수 y=xÛ`-5 (x¾0), y='¶x+5가 서로 역함수
많은 점에서 만난다.
관계이고, 그래프의 개형이 증가하는 형태이므로 두 그래프의
( - ③) 2<a<6일 때
y
교점은 직선 y=x 위에 있다. 따라서 함수 y=xÛ`-5 (x¾0)의
ax
y= 그래프와 직선 y=x의 교점이 점 Q이다.
x+2
y=f{x} xÛ`-5=x에서 xÛ`-x-5=0
6

a 1+'¶21
xª= (∵ x>'5)
2
2
-10 -6 -2 O2 6 10 14 x 1+'¶21 5 5
이때, > 이므로 '5<xª< 를 만족시키지 않는다.
2 2 2
(거짓)
그림과 같이 무수히 많은 점에서 만난다.
Ú ~ Ü에서 구하는 a의 값의 범위는 2ÉaÉ6이므로
정수 a의 값의 합은 2+3+4+5+6=20이다.
TIP
직선 y=ax+b를 직선 y=x에 대하여 대칭이동한 직선은
x-b
이 함수와 역함수 관계에 있으므로 y= 이다.
a
417 ⑤
두 직선의 기울기가 a,
1
이므로 두 직선이 직선 y=x에 대하여
a
ㄱ. 점 Q는 두 함수 y=xÛ`-5 (x>'5), y='¶x+5의 그래프의
대칭이면 두 직선의 기울기는 서로 역수 관계에 있다.
교점이므로 xª>'5이고, 두 함수 y=xÛ`-5 (x>'5),

Ⅴ. 함수와 그래프 105


1
418 -1<kÉ-
3
또는 k>2+2'2 419 ②

á -1 (x<1) -6 -6
f(x-1)=Ò 이고, 함수 y=| +3|의 그래프는 함수 y= +3의 그래프의
» 1 (x¾1) x-1 x-1
x축 아래 부분을 x축에 대하여 대칭이동시킨 그래프이므로 다음과
á -1 (x<-2)
f(x+2)=Ò 이므로 같다.
» 1 (x¾-2)
y
( -2 (x<-2) y= |x-1
-6
+3|
f(x-1)+ f(x+2)=Ò 0 (-2Éx<1) 이다.
9 2 (x¾1)
( - 2 (x<-2) 3
È x+2
따라서 g(x)=Ò 0 (-2<x<1) 이므로 O 1 x
È 2
9 x+2 (x¾1) 무리함수 y=-'¶k-x+3의 그래프는 점 (k, 3)을 지나면서 왼쪽
함수 y=g(x)의 그래프는 다음과 같다. 아래로 그래프가 그려진다.
y
y
y= |x-1
-6
+3|

y=g{x} 3
O
-2 1 x y=-Âk-˛x˛+3
{ii} O1 23 x
{i}
{iii}
방정식 g(x)=x+k의 실근의 개수는 함수 y=g(x)의 그래프와 -6
함수 y=| +3|의 그래프가 직선 y=3과 만나는 점의 x좌표는
x-1
직선 y=x+k의 교점의 개수와 같고,
6
두 그래프는 다음과 같은 경우를 기준으로 교점의 개수가 달라진다. -3=3에서 x=2이다.
x-1
2 k<2일 때 두 그래프는 서로 만나지 않고, k¾2일 때 두 그래프는
Ú 함수 y=- (x<-2)의 그래프와 직선 y=x+k가 접할 때
x+2
항상 만난다.
2
- =x+k에서 이차방정식 xÛ`+(k+2)x+2k+2=0의 ∴ a=2
x+2
판별식을 D라 하면 D=(k+2)Û`-4(2k+2)=0이므로 직선 y=m(x-4)-1은 점 (4, -1)을 지나는 직선이다.
y
kÛ`-4k-4=0, k=2+2'2 (∵ k>0) y=m{x-4}-1 |x-1
y=
-6
+3|
2
Û 직선 y=x+k가 점 {1, }를 지날 때
3
1 3
k=-
3
Ü 직선 y=x+k가 점 (1, 0)을 지날 때 O 1 4
-1 3 x
k=-1
Ú ~ Ü에 의하여 두 그래프가 두 점에서 만나도록 하는 -6 -6
+3=0에서 x=3이므로 함수 y=| +3|의 그래프가
x-1 x-1
실수 k의 값의 범위는
x축과 만나는 점의 좌표는 (3, 0)이다.
1
-1<kÉ- 또는 k>2+2'2이다. yy 참고 직선 y=m(x-4)-1이 점 (3, 0)을 지날 때, m=-1이다.
3
이때, m<-1일 때 두 그래프는 세 점에서 만난다.
yy TIP1 , TIP2
참고
∴ b=-1
직선 y=x+k가 점 (-2, 0)을 지날 때 k=2이다. ∴ a+b=2+(-1)=1
따라서 함수 y=g(x)의 그래프와 직선 y=x+k의 위치 관계는
다음과 같다.
1
❶ kÉ-1 또는 - <k<2 또는 k=2+2'2일 때, TIP1
3
직선 y=m(x-4)-1이 점 (3, 0)을 지날 때 m=-1이므로
한 점에서 만난다.
y=-x+3이다.
1
❷ -1<kÉ- 또는 k>2+2'2일 때, 두 점에서 만난다. 6
3 이때, 1<x<3에서 곡선 y= -3과 직선 y=-x+3이
x-1
❸ 2Ék<2+2'2일 때, 만나지 않는다.
만나는지의 여부에 따라 교점의 개수가 달라지므로 이를 다음과

106 수학<하>
같이 확인할 수 있다. 1 1
_PRÓ Û`= _'6 Û`=3이다.
2 2
6
방정식 -3=-x+3에서 6=(x-1)(-x+6)
x-1
TIP
xÛ`-7x+12=0, (x-3)(x-4)=0
직선 y=x+2는 기울기가 1이므로 x축의 양의 방향과 이루는
x=3 또는 x=4이므로
각의 크기가 45ù이다. 따라서 ∠PQR=45ù이므로 직각삼각형
두 그래프는 1<x<3에서 만나지 않는다.
PRQ는 PRÓ=QRÓ인 직각이등변삼각형이다.
-6
따라서 함수 y=| +3|의 그래프와
x-1
직선 y=m(x-4)-1은 m=-1일 때 두 점에서 만난다.

TIP2 421 13

-6 함수 y='¶x-[x]-[x]는
함수 y=| +3|의 그래프와 직선 y=m(x-4)-1의
x-1
0Éx<1일 때 y='x
위치 관계는 다음과 같다.
1Éx<2일 때 y='¶x-1-1
❶ m=0일 때, 만나지 않는다.
2Éx<3일 때 y='¶x-2-2
❷ m>0 또는 -1<m<0일 때, 한 점에서 만난다.
⋮ ⋮
❸ m=-1일 때, 두 점에서 만난다. Ⅴ
이므로 함수 y='¶x-[x]-[x]의 그래프는 다음과 같다.
❹ m<-1일 때, 세 점에서 만난다. 함
y 수

2 그
420 3 1
1 2 3 4


O x
삼각형 PQR는 PRÓ=QRÓ인 직각이등변삼각형이다. yy TIP -1
-2 y=´x-¨ù[x]¨ù-[x]
3 y
y=- y=x+2 -3


x+2

P 직선 y=kx+2는 점 (0, 2)를 지나고 기울기가 k인 직선이고,


Q
R x n(A;B)=2를 만족시키려면 함수 y='¶x-[x]-[x]의 그래프와
O
-2 서로 다른 두 점에서 만나야 한다. 이를 만족시키려면 직선
y=kx+2의 기울기가 점 (1, -1)을 지날 때보다 크거나 같고,
점 (2, -2)를 지날 때보다는 작아야 한다.
따라서 PQÓ='2_PRÓ이므로 PRÓ가 최소일 때, PQÓ가 최소이다. 직선 y=kx+2가 점 (1, -1)을 지날 때
함수의 그래프를 평행이동하여도 선분 PR의 길이는 변하지 -1=k+2에서 k=-3
3 직선 y=kx+2가 점 (2, -2)를 지날 때
않으므로 함수 y=- 의 그래프와 직선 y=x+2를 x축의
x+2
-2=2k+2에서 k=-2
3
방향으로 2만큼 평행이동하면 각각 y=- 과 y=x이고, 이때 따라서 실수 k의 값의 범위는 -3Ék<-2이므로
x
3 a=-3, b=-2
PRÓ의 최솟값은 함수 y=- 의 그래프 위의 점에서 직선
x ∴ aÛ`+bÛ`=13
3
y=x까지의 거리의 최솟값과 같다. 이는 함수 y=- 의 그래프와
x
직선 y=-x의 교점과 원점 사이의 거리와 같다.
y y=x 422 ①

모든 실수 k에 대하여 집합 Aû={x| f(x)=k}가


x n(Aû)=1을 만족시키려면 함수 y= f(x)의 그래프가 모든
O 3 실수 k에 대하여 직선 y=k와 한 점에서 만나야 하므로
y=-
x 함수 f(x)는 일대일대응이어야 한다.
y=-x -2x+9 -1
[ 1 ] ax+b<0일 때, 함수 f(x)= = -2의
x-5 x-5
3
- =-x, 즉 xÛ`=3에서 x=Ñ'3이므로 교점의 좌표가 그래프는 다음과 같다.
x
(-'3, '3 ) 또는 ('3, -'3 )이다. b
Ú a>0일 때, ax+b<0에서 ax<-b, x<-
a
따라서 PRÓ의 최솟값은 ¿¹(-'3)Û`+'3 Û`='6이고, b
- 의 값에 따라 그래프를 나타내면 다음과 같다.
이때, 삼각형 PQR의 넓이는 a

Ⅴ. 함수와 그래프 107


b b
( -①) - <5일 때
a
( -②) - =5일 때
a 423 k>4

y y y y 3x-5 3x-5
함수 y=| +k|의 그래프는 함수 y= +k의 그래프의
y y x-1 x-1
O O 5 5 x x O O 5 5 x x x축 아래 부분을 x축에 대하여 대칭이동시킨 것이다.
-2
O-2 5 x -2
O-2 5 x
3x-5 -2
-2 -2 함수 y= +k= +k+3의 그래프의 점근선의 방정식이
x-1 x-1
b x=1, y=k+3이므로
( -③) - >5일 때
y y a
3x-5
y
k+3의 값에 따라 x>1에서 함수 y=| +k|의 그래프의
O O 5 5 x x x-1
O-2
-2 5 x 개형은 다음과 같다.
-2 Ú k+3É0일 때, 즉 kÉ-3일 때
y y
O 1 x
y= | 3x-5
x-1
+k|
b
Û a<0일 때, ax+b<0에서 ax<-b, x>-
a
b k+3 -{k+3}
- 의 값에 따라 그래프를 나타내면 다음과 같다.
a 3x-5
y= +k O 1 x
b b x-1
( -①) - >5일 때 ( -②) - =5일 때
a a 1<xÁ<xª<2인 모든 실수 xÁ, xª에 대하여 f(xÁ)> f(xª)이므로
y y y y f(xÁ)<5< f(xª)를 만족시키는 xÁ, xª가 존재하지 않는다.
y y
Û k+3>0일 때, 즉 k>-3일 때
O O 5 5 x x O O 5 5 x x
O-2 5 x O-2 5 x y y
-2 -2
-2 -2

k+3 k+3
b
( -③) - <5일 때
y y
a
y=
3x-5
x-1
+k y= | 3x-5
x-1
+k|

y O 1 x O 1 x
O O 5 5 x x
O 5 x 3x-5
-2-2 x=2일 때 함수 y= +k의 함숫값이 k+1이므로
-2 x-1
3x-5
k+1의 값에 따라 1<x<2에서 함수 y=| +k|의
x-1
[ 2 ] 또한 ax+b¾0에서 함수 f(x)='¶ax+b+c의 그래프는 그래프의 개형은 다음과 같다.
다음과 같다.
( -①) k+1É0일 때, 즉 -3<kÉ-1일 때
Ú a>0일 때 Û a<0일 때 y

{- ab , c}{- a , c} {- ab , c}{- a , c}
b b

[ 1 ], [ 2 ]에서 함수 f(x)가 일대일대응이 되는 경우를 찾아보면


O 1 2 x
a>0일 때 함수 f(x)가 일대일대응이 되는 경우는 존재하지 않고,
a<0일 때 함수 f(x)가 일대일대응이 되는 경우는 Û-②의 경우에 1<xÁ<xª<2인 모든 실수 xÁ, xª에 대하여
b f(xÁ)> f(xª)이므로 f(xÁ)<5< f(xª)를 만족시키는
다음과 같이 - =5이고, c=-2가 될 때뿐이다.
a xÁ, xª가 존재하지 않는다.
y ( -②) k+1>0일 때, 즉 k>-1일 때
y

O 5 x

-2

O 1 2 x
b
∴ +c=(-5)+(-2)=-7 함수 y= f(x)의 그래프가 x축과 만나는 점을
a

108 수학<하>
a(1<a<2)라 하면 a<xª<2일 때, f(xÁ)< f(xª), y
xÁ<xª를 만족시키는 xÁ이 항상 존재한다. 6
y=Âx+˛3˛6˛
이때, f(xÁ)<5< f(xª)를 만족시키려면 a<xª<2이고
-36 O y=-Âx°+6 36 x
f(xª)>5인 xª가 존재해야 하므로 f(2)>5이면 된다.
즉, f(2)=|1+k|=k+1>5에서 k>4이다. 이때, 함수 y='¶x+36의 그래프를 x축에 대하여 대칭이동한 후
Ú, Û에서 구하는 실수 k의 값의 범위는 k>4이다. x축의 방향으로 36만큼, y축의 방향으로 6만큼 평행이동하면
함수 y=-'¶x+6의 그래프와 일치하므로 색칠된 두 도형 A와 C가
서로 합동이다.
y
6
y=Âx+˛3˛6˛
424 ①
A l B m C
-36 O y=-Âx°+6 36 x
두 함수 y='¶x+2, y='¶2-x의 그래프를 나타내면 다음과 같다.
y 선분 x=0 (0ÉyÉ6)을 l이라 하고,
y=Â2-˛x˛ y=Âx+˛2˛
곡선 y=-'¶x+6(0ÉxÉ36)을 m이라 하면
Â2
구하는 점의 개수는 x축, y축, x=36, y=6으로 둘러싸인
직사각형의 둘레 및 내부에 포함되는 x좌표와 y좌표가 모두 정수인
점의 개수에서 선분 l 위에 있는 x좌표와 y좌표가 모두 정수인 점의 Ⅴ
-2 O 2 x 함
개수를 빼고, 곡선 m 위에 있는 x좌표와 y좌표가 모두 정수인 점의

구하는 직사각형의 세로의 길이를 a (0<a<'2)라 하면 개수를 더한 것과 같다. yy TIP1 와

yy TIP 이때, 선분 l 위에 있는 x좌표, y좌표가 모두 정수인 점의 개수와




'¶2-x=a에서 x=2-aÛ`이므로 직사각형의 가로의 길이는 곡선 m 위에 있는 x좌표, y좌표가 모두 정수인 점의 개수가 7로 프

4-2aÛ`이다. 서로 같다. yy TIP2


따라서 직사각형의 둘레의 길이는 따라서 구하는 점의 개수는 x축, y축, x=36, y=6으로 둘러싸인
1 직사각형의 둘레 및 내부에 포함되는 x좌표, y좌표가 모두 정수인
2(4-2aÛ`+a)=-4{aÛ`- a}+8
2
점의 개수와 같다.
1 33
=-4{a- }Û`+ 0ÉxÉ36에서 정수인 x는 37개, 0ÉyÉ6에서 정수인 y는 7개이므로
4 4
1 33 구하는 값은 37_7=259이다.
이므로 a= 일 때, 최댓값 l= 을 갖는다.
4 4
1 TIP1
이때, 직사각형의 세로의 길이는 a= ,
4 구하는 점의 개수를 두 도형 A, B의 내부와 경계에 있는 점의
31 개수의 합으로 구할 때, 선분 l 위의 점의 개수를 중복해서 세게
가로의 길이는 4-2aÛ`= 이므로
8
된다.
1 31 31
직사각형의 넓이는 S= _ = 이다. 또한 도형 A와 도형 C가 합동이므로 두 도형 A, B를 두 도형 B,
4 8 32
31 C로 바꾸어 생각할 수 있고, 이때 두 도형 B, C의 내부와
8_
8S 32 31 경계에 있는 점의 개수를 B, C를 합친 직사각형의 내부와
∴ = =
l 33 33 경계에 있는 점의 개수로 구할 때, 곡선 m 위의 점의 개수를
4
한 번 빼서 세게 된다.
TIP
만약 직사각형의 가로의 길이를 2k라 놓고, (0<k<2) TIP2
세로의 길이는 '¶2-k라 하면 곡선 y=-'¶x+6 위의 점의 y좌표가 n이면 x좌표는
직사각형의 둘레의 길이는 2(2k+'¶2-k)이다. (n-6)Û`이므로 y좌표가 정수일 때 x좌표도 정수이다. 따라서
이와 같은 무리식이 포함된 식의 최댓값을 구하는 것은 어려우므로 곡선 m 위에 있는 x좌표, y좌표가 모두 정수인 점의 개수는
본풀이와 같이 세로의 길이를 기준으로 풀이해야 한다. y좌표가 정수인 점의 개수와 같다. 또한 선분 l 위에 있는
x좌표와 y좌표가 모두 정수인 점의 개수도 y좌표가 정수인 점의
개수와 같으므로 y좌표의 범위 0ÉyÉ6에서 두 부분 위에 있는
x좌표와 y좌표가 모두 정수인 점의 개수는 7로 같다.

425 259
다른 풀이

두 함수 y='¶x+36, y=-'¶x+6의 그래프와 두 함수 y='¶x+36, y=-'¶x+6의 그래프와 x축으로 둘러싸인


x축으로 둘러싸인 도형을 나타내면 다음과 같다. 도형을 나타내면 다음과 같다.

Ⅴ. 함수와 그래프 109


y 지나야 하므로 1=b-1에서 b=2
6 따라서 b의 최솟값은 2이다.
y=Âx+˛3˛6˛
y=n
13 13
-36 O 36 x ∴ _2=
y=-Âx°+6 24 12
도형의 내부 또는 경계에 포함되는 점 중에서 y좌표가 n인 점의
개수를 f(n)이라 하자. (단, n은 0ÉnÉ6인 정수)
'¶x+36=n에서 x=nÛ`-36
-'¶x+6=n에서 x=(n-6)Û`이고, nÛ`-36, (n-6)Û`은 모두 427 9

정수이므로 nÛ`-36ÉxÉ(n-6)Û`인 정수 x의 개수는 1 1 -x+3


함수 f(x)는 함수 y= { -1}= (x>2)의
(n-6)Û`-(nÛ`-36)+1=-12n+73에서 f(n)=73-12n이다. 2 x-2 2x-4
따라서 구하는 점의 개수는 4x+3 1
역함수이므로 f(x)= {x>- }이다.
2x+1 2
f(0)+ f(1)+ f(2)+y+ f(6)
1 3
=73+(73-12)+(73-24)+(73-36) 함수 y= xÛ`+ (x¾0)의 역함수 g(x)를 구하면 다음과 같다.
2 2
+(73-48)+(73-60)+(73-72) 2y-3=xÛ` (x¾0), x='¶2y-3
=73_7-12_(1+2+3+4+5+6)=259 x와 y를 서로 바꾸면 y='¶2x-3이므로
g(x)='¶2x-3이다.
4x+3 4x+3
h(x)=g( f(x))=g{ }=¾Ð2_ -3
2x+1 2x+1
426 ①
=¾Ð
2x+3
2x+1
3x ∴ h(1)_h(2)_y_h(120)
Ú 1ÉxÉ6일 때 부등식 ax-1É 가 성립하려면 y=ax-1
x+2
5 7 9 243
3x =¾Ð _¾Ð _¾Ð _y_¾Ð
의 함숫값이 y= 의 함숫값보다 항상 작거나 같아야 하므로 3 5 7 241
x+2
243
3x =¾Ð ='¶81=9
1ÉxÉ6인 범위에서 직선 y=ax-1이 함수 y= 의 3
x+2
그래프보다 아래쪽 또는 같게 위치해야 한다.
3x
Û 1ÉxÉ6일 때 부등식 Ébx-1이 성립하려면 y=bx-1의
x+2
3x 428 ④
함숫값이 y= 의 함숫값보다 항상 크거나 같아야 하므로
x+2 4
집합 A^는 유리함수 y= +tÛ`의 그래프를 나타내고,
3x x-t
1ÉxÉ6인 범위에서 직선 y=bx-1이 함수 y= 의
x+2 집합 B^는 중심이 (t, t)이고, 반지름의 길이가 2'2인 원을 나타낸다.
그래프보다 위쪽 또는 같게 위치해야 한다. ㄱ. t<0일 때 집합 A^가 나타내는 유리함수의 그래프의 두
3x -6 점근선의 교점 (t, tÛ`)이 제2사분면 위에 존재하므로 그래프는
1ÉxÉ6에서 함수 y= = +3의 그래프는 다음과 같고,
x+2 x+2
다음과 같다.
두 직선 y=ax-1, y=bx-1은 y절편이 -1이고, 기울기가 각각 a,
y
b인 직선이다.
y y=bx-1
{t,`t@}
y=ax-1
3
O 1
-2 -1 6 x x
O
따라서 제4사분면을 지나지 않는다. (거짓)
4
Ú에 의하여 a의 값이 최대이려면 ㄴ. t=1일 때 집합 A^;B^는 함수 y= +1의 그래프와
x-1
3x 9
직선 y=ax-1이 함수 y= 의 그래프 위의 점 {6, }를 원 (x-1)Û`+(y-1)Û`=8의 교점의 집합이다.
x+2 4
4
9 13 두 식을 연립하여 풀면 (x-1)Û`+{ }Û`=8에서
지나야 하므로 =6a-1에서 a= x-1
4 24
16
13 x-1=k로 치환하면 kÛ`+ =8
따라서 a의 최댓값은 이다. kÛ`
24
kÝ`-8kÛ`+16=0, (kÛ`-4)Û`=0에서 k=Ñ2이므로
Û에 의하여 b의 값이 최소이려면
x=3 또는 x=-1
3x
직선 y=bx-1이 함수 y= 의 그래프 위의 점 (1, 1)을 두 그래프는 두 점 (-1, -1), (3, 3)에서 만난다.
x+2

110 수학<하>
y 41
430 m=10 또는
4
Ém<13

k의 값에 따른 곡선 y='¶5-2x와 직선 y=-x+k의 교점의 개수


1
는 다음 경우를 기준으로 변한다.
O 1 x
y

y=Â5-˛2˛x˛
따라서 두 점 사이의 거리는 4'2이다. (참)
ㄷ. ㄴ에서 유리함수 그래프의 두 점근선의 교점 (t, tÛ`)과 원의 중심
(t, t)가 일치할 때, 두 그래프가 두 점에서 만나므로 두
O 5 x
점근선의 교점과 원의 중심의 위치에 따라 두 그래프의 2 {ii}{i}
위치 관계가 다음과 같다. Ú 곡선 y='¶5-2x와 직선 y=-x+k가 접할 때
Ú t=tÛ` Û t+tÛ` '¶5-2x=-x+k의 양변을 각각 제곱하면
5-2x=(-x+k)Û`에서 이차방정식
{t,`t} {t,`t} xÛ`-2(k-1)x+kÛ`-5=0의 판별식을 D라 하면
D
=(k-1)Û`-(kÛ`-5)=0이므로 -2k+6=0, k=3
{t,`t@} {t,`t@}
4 Ⅴ
{t,`t}={t,`t@}
{t,`t}={t,`t@} 5 함
Û 직선 y=-x+k가 점 { , 0}을 지날 때
2 수

5 5
두 그래프가 항상 두 점에서 만나므로 모든 실수 t에 대하여 0=- +k에서 k= 그
2 2

n(A^;B^)=2이다. (참) (0 (k>3) 프
따라서 옳은 것은 ㄴ, ㄷ이다. È 5
Ú, Û에서 f(k)=Ò 1 {k=3 또는 k< }
2
È 5
92 { Ék<3}
2
이므로 함수 y= f(x)의 그래프는 다음과 같다.
y
D E
2

429 ③
1
B
C

f(x)='¶3+x+'¶3-x라 할 때, A
O 5 3 x
3+x¾0, 3-x¾0이어야 하므로 -3ÉxÉ3 2
이때, f(x)¾0이므로 { f(x)}Û`이 최댓값과 최솟값을 가질 때 5 5
A(3, 0), B{ , 1}, C(3, 1), D{ , 2}, E(3, 2)라 하면
f(x)도 각각 최댓값과 최솟값을 갖는다. 2 2
{ f(x)}Û`=('¶3+x+'¶3-x)Û` xÛ`+yÛ`=m은 중심이 원점이고 반지름의 길이의 제곱이 m인
=6+2'¶(3+x)(3-x) 원이므로 이 원이 임의의 점 P를 지날 때 m=OPÓ Û`이다.
5 29
=6+2¿¹9-xÛ` (-3ÉxÉ3) OAÓ Û`=9, OBÓ Û`={ }Û`+1Û`= , OCÓ Û`=3Û`+1Û`=10,
2 4
-3ÉxÉ3에서 함수 y=9-xÛ`은 x=0일 때 최댓값을 갖고,
5 41
x=-3 또는 x=3일 때 최솟값을 가지므로 ODÓ Û`={ }Û`+2Û`= , OEÓ Û`=3Û`+2Û`=13이므로
2 4
f(x)는 x=0일 때 최댓값 f(0)='3+'3=2'3을 갖고, OBÓ<OAÓ<OCÓ<ODÓ<OEÓ이다.
x=-3 또는 x=3일 때 최솟값 f(-3)= f(3)='6을 갖는다. 따라서 함수 y= f(x)의 그래프와 원 xÛ`+yÛ`=m의 교점의 개수가
∴ ab=2'3_'6=6'2 3이 되도록 하는 m의 값의 범위는
41
참고 m=10 또는 Ém<13이다. yy TIP
4
함수 y='¶3+x+'¶3-x의 그래프는 다음과 같다.
y TIP
2Â3 y=Â3+˛x˛+Â3-˛x˛ m>1일 때 원 xÛ`+yÛ`=m은 함수 y= f(x)의 그래프와
y=Â3-˛x˛
Â3 y=Â3+˛x˛ x<0에서 항상 한 점에서 만난다.
따라서 함수 y= f(x)의 그래프와 원 xÛ`+yÛ`=m의 위치 관계는
다음과 같다.
-3 O 3 x
❶ 0<m<1일 때, 만나지 않는다.

Ⅴ. 함수와 그래프 111


29
❷ m=1 또는 ÉmÉ9일 때, 한 점에서 만난다.
4
29 41
❸ 1<m< 또는 9<m<10 또는 10<m< 또는
4 4
m¾13일 때, 두 점에서 만난다.
41
❹ m=10 또는 Ém<13일 때, 세 점에서 만난다.
4
y

D E

C
B

O A x

112 수학<하>
Ⅴ 경우의 수 구하는 경우의 수는 합의 법칙에 의하여
9+8=17이다.

경우의 수
435 ⑴ 15 ⑵ 13

나오는 두 눈의 수의 합의 최솟값은 1+1=2이고,

431 74 최댓값은 6+6=12이다.


⑴ 2에서 12까지의 자연수 중 소수는 2, 3, 5, 7, 11이다.
주어진 차림표에서 한 가지 음식을 택할 때 두 주사위의 눈의 수를 각각 a, b라 하면 합이 소수인
한식 5가지 메뉴 중에서 1가지를 고르는 방법의 수는 5이고, 순서쌍 (a, b)는 다음과 같다.
한식을 제외한 중식 3가지, 일식 4가지 메뉴 중에서 1가지를 고르는 Ú 합이 2인 경우 (1, 1)
방법의 수는 7이므로 Û 합이 3인 경우 (1, 2), (2, 1)
a=5, b=7 Ü 합이 5인 경우 (1, 4), (2, 3), (3, 2), (4, 1)
∴ aÛ`+bÛ`=25+49=74 Ý 합이 7인 경우
(1, 6), (2, 5), (3, 4), (4, 3), (5, 2), (6, 1)

432 ④ Þ 합이 11인 경우 (5, 6), (6, 5)


Ú ~ Þ에서 구하는 경우의 수는
나오는 두 눈의 수의 합의
1+2+4+6+2=15이다.
최솟값은 1+1=2, 최댓값은 6+6=12이므로
⑵ 8 이상의 합성수는 8, 9, 10, 12이다.
2에서 12까지의 자연수 중 5의 배수는 5, 10이다.
두 주사위의 눈의 수를 각각 a, b라 하면 합이 8 이상의 합성수인
두 주사위의 눈의 수를 각각 a, b라 하면
순서쌍 (a, b)는 다음과 같다.
두 눈의 수의 합이 5 또는 10인 순서쌍 (a, b)는 다음과 같다.
Ú 합이 8인 경우 (2, 6), (3, 5), (4, 4), (5, 3), (6, 2)
Ú a+b=5인 경우
Û 합이 9인 경우 (3, 6), (4, 5), (5, 4), (6, 3)
(1, 4), (2, 3), (3, 2), (4, 1)
Ü 합이 10인 경우 (4, 6), (5, 5), (6, 4)
Û a+b=10인 경우
Ý 합이 12인 경우 (6, 6)
(4, 6), (5, 5), (6, 4)
Ú ~ Ý에서 구하는 경우의 수는
Ú, Û의 경우는 동시에 일어날 수 없으므로
5+4+3+1=13이다.
구하는 경우의 수는 합의 법칙에 의하여

4+3=7이다. 우
436 6 의

433 ② 0부터 9까지의 정수 중 곱해서 9가 되는 세 수는 1, 1, 9 또는
1, 3, 3이다.
n번째 자리에는 숫자 n이 적힌 카드가 오지 않도록 나열하는 방법은
따라서 세 자리 자연수의 백의 자리의 수를 a,
다음과 같다.
십의 자리의 수를 b, 일의 자리의 수를 c라 하면
1번째 2번째 3번째 순서쌍 (a, b, c)는 다음과 같다.
2 3 1 Ú 세 수가 1, 1, 9인 경우

3 1 2 (1, 1, 9), (1, 9, 1), (9, 1, 1)
따라서 구하는 방법의 수는 2이다. Û 세 수가 1, 3, 3인 경우
(1, 3, 3), (3, 1, 3), (3, 3, 1)
Ú, Û의 경우는 동시에 일어날 수 없으므로
434 17
구하는 자연수의 개수는 합의 법칙에 의하여
주머니에서 꺼낸 두 공에 적힌 숫자를 각각 a, b (a>b)라 하면 3+3=6이다.
두 수의 차가 3 미만인 경우의 순서쌍 (a, b)는 다음과 같다.
Ú a-b=1인 경우
(2, 1), (3, 2), (4, 3), (5, 4), (6, 5), 437 ②

(7, 6), (8, 7), (9, 8), (10, 9) 40=2Ü`_5이므로 40과 서로소인 수는
Û a-b=2인 경우 2의 배수가 아니면서 5의 배수가 아니어야 한다.
(3, 1), (4, 2), (5, 3), (6, 4), 100 이하의 자연수 중에서 2의 배수의 개수는 50이고
(7, 5), (8, 6), (9, 7), (10, 8) 5의 배수의 개수는 20이다.
Ú, Û의 경우는 동시에 일어날 수 없으므로 이때, 2와 5의 최소공배수인 10의 배수의 개수는 10이므로

Ⅵ. 경우의 수 113
구하는 40과 서로소인 100 이하의 자연수의 개수는 Ú 중간에 Q지점을 지나는 경우
100-(50+20-10)=40이다. P → Q로 이동하는 방법의 수는 3
Q → R로 이동하는 방법의 수는 2
이므로 P → Q → R의 순서로 이동하는 방법의 수는
438 ③
3_2=6이다.
A지점에서 B지점까지 도로를 따라 최단거리로 이동하는 방법의 Û 중간에 Q지점을 지나지 않는 경우
수를 일일이 세면 다음과 같다. P → R로 바로 이동하는 방법의 수는 3이다.
4 9 14 Ú, Û에서 구하는 방법의 수는 6+3=9이다.
1 B
3 5
1 5

1
2
2 443 ②

A 1 백의 자리의 숫자는 1, 2, 3, 4 중 하나이므로 4가지


따라서 구하는 방법의 수는 14이다. 십의 자리의 숫자는 백의 자리의 숫자를 제외하고 4가지
일의 자리의 숫자는 백의 자리와 십의 자리의 숫자를 제외하고 3가지
따라서 구하는 세 자리 자연수의 개수는 4_4_3=48이다.
439 ②

집합 {1, 2, 3, 4, 5}의 부분집합 중 원소의 개수가 2 이상이면서


모든 원소의 합이 4 또는 10인 부분집합은 다음과 같다. 444 ⑴ 100 ⑵ 90

Ú 모든 원소의 합이 4인 경우 ⑴ 백의 자리의 숫자는 2, 4, 6, 8 중에 하나이고, 십의 자리와 일의


{1, 3} 자리의 숫자는 각각 1, 3, 5, 7, 9 중에 하나이다.
Û 모든 원소의 합이 10인 경우 따라서 구하는 자연수의 개수는 4_5_5=100이다.
{1, 4, 5}, {2, 3, 5}, {1, 2, 3, 4} ⑵ 짝수가 되려면 일의 자리의 숫자가 0, 2, 4 중에 하나가 되어야 하고
Ú, Û의 경우는 동시에 일어날 수 없으므로 구하는 부분집합의 백의 자리의 숫자는 0이 될 수 없다.
개수는 합의 법칙에 의하여 따라서 구하는 짝수의 개수는 5_6_3=90이다.
1+3=4이다.

445 ②
440 ③
주사위의 눈의 수가 3 이상인 경우의 수는 3, 4, 5, 6에서 4이고
조건 ㈏에서 f(3)¾2이고 f(1)=a, f(2)=b라 하면
동전의 앞면이 나오는 경우의 수는 1이므로 구하는 경우의 수는
순서쌍 (a, b)는 다음과 같다.
곱의 법칙에 의하여 4_1=4이다.
Ú f(3)=2인 경우, (1, 1)로 1개이다.
Û f(3)=3인 경우, (1, 2)로 1개이다.
Ü f(3)=4인 경우, (1, 3), (2, 2)로 2개이다.
Ý f(3)=5인 경우, (1, 4), (2, 3)으로 2개이다.
446 27

Ú ~ Ý는 동시에 일어날 수 없으므로 서로 다른 두 주사위를 던져서 나오는 두 눈의 수를 각각 a, b라 하면


합의 법칙에 의하여 구하는 경우의 수는 ab가 짝수인 경우의 수는 순서쌍 (a, b)의 개수에서 ab가 홀수인
1+1+2+2=6이다. 경우의 수를 제외하면 된다.
순서쌍 (a, b)의 개수는 6_6=36이고, ab가 홀수이기 위해선 a와
b가 모두 홀수이어야 하므로
441 36 3_3=9이다.
따라서 구하는 경우의 수는
치즈의 유무에 따라 2가지, 매운 정도에 따라 6가지,
36-9=27이다.
양에 따라 3가지 종류가 있으므로 떡볶이를 주문하는 방법의 수는
곱의 법칙에 의하여 다른 풀이

2_6_3=36이다. 나오는 눈의 수의 곱이 짝수인 경우는 다음과 같다.


Ú (홀수, 짝수)가 나오는 경우
3_3=9(가지)
442 ③
Û (짝수, 홀수)가 나오는 경우
P지점에서 출발해서 R지점에 도착하는 경로를 케이스 분류하면 3_3=9(가지)
다음과 같다. Ü (짝수, 짝수)가 나오는 경우

114 수학 <하>
3_3=9(가지) (n-9)(2n+21)=0
Ú ~ Ü에서 구하는 경우의 수는 ∴ n=9 (∵ n은 자연수)
9+9+9=27이다.

451 ③

447 ⑴ 24 ⑵ 15
사용할 수 있는 색은 4가지이므로
⑴ (x+y+z)(a+b+c+d)(p+q)(r+s)의 전개식에서 A에 칠할 수 있는 색은 4가지
s를 포함하는 항은 (r+s)에서 s를 선택하여 곱하는 경우이다. B에 칠할 수 있는 색은 A에 칠한 색을 제외한 3가지
즉, (x+y+z)(a+b+c+d)(p+q)_s이므로 C에 칠할 수 있는 색은 A, B에 칠한 색을 제외한 2가지
구하는 항의 개수는 D에 칠할 수 있는 색은 A, C에 칠한 색을 제외한 2가지
3_4_2_1=24이다. 이므로 구하는 방법의 수는
⑵ (x+y)(p+q+r)를 전개할 때 4_3_2_2=48이다.
(x+y)와 (p+q+r)에서 각각 하나씩 항을 선택하여 곱하므로
구하는 항의 개수는 2_3=6이다.
마찬가지 방법으로 452 59

(x+y+z)(a+b+c)의 전개식에서 항의 개수는 500원짜리 동전을 지불하는 방법은


3_3=9이다. 0, 1, 2, 3개로 4가지
이때, (x+y)(p+q+r)와 (x+y+z)(a+b+c)의 전개식에서 100원짜리 동전을 지불하는 방법은
동류항이 없으므로 구하는 항의 개수는 0, 1, 2, 3, 4개로 5가지
6+9=15이다. 50원짜리 동전을 지불하는 방법은
0, 1, 2개로 3가지
이때, 0원을 지불하는 방법의 수는 1가지이므로
448 32
구하는 지불하는 방법의 수는
c, u, s, t, o, m에서 자음은 c, s, t, m으로 4개이고 모음은 u, o로 4_5_3-1=59이다.
2개이다.
Ú (자음) → (모음) → (자음)순으로 나열하는 경우
4_2_3=24
453 ④

Û (모음) → (자음) → (모음)순으로 나열하는 경우 다음과 같이 전체집합의 각각의 원소 1, 2, 3, y, 10은 경



2_4_1=8 세 집합 ㉠ A, ㉡ B-A, ㉢ U-B 중에 한 곳에 속하게 된다. 의
Ú, Û에서 구하는 경우의 수는 U 수
㉢ B
24+8=32이다. A

449 ②
따라서 구하는 순서쌍 (A, B)의 개수는
720을 소인수분해하면 3_3_3_y_3=3Ú` â`
720=2Ý`_3Û`_5Ú`
이므로 양의 약수의 개수는
(4+1)_(2+1)_(1+1)=30 454 81

참고
f(1)의 값은 2, 3, 4로 3가지
f(2)의 값은 1, 3, 4로 3가지
약수와 배수는 일반적으로 자연수 범위에서 다루지만 그 의미가
f(3)의 값은 1, 2, 4로 3가지
좀 더 명확하도록 ‘양의 약수’라는 표현을 사용하였다.
f(4)의 값은 1, 2, 3으로 3가지
이므로 구하는 함수 f의 개수는
3_3_3_3=81이다.
450 9

18=2_3Û`이므로
18Ç` =2Ç` _3Û` Ç` 의 양의 약수의 개수는
455 ②

(n+1)(2n+1)=190이다. 조건 ㈎에서 함수 f는 일대일함수이고,


2nÛ`+3n-189=0 조건 ㈏에서 함수 f의 치역은 {2, k, 6} (2<k<6)이다.

Ⅵ. 경우의 수 115
k는 3, 4, 5 중에서 하나이므로 3가지이고 D라 하면
치역의 원소 3개를 정의역의 원소 3개에 짝지어 주는 방법의 수는 D=aÛ`-8b¾0, aÛ`¾8b yy ㉠
3_2_1=6이다. a, b의 값 중에서 ㉠을 만족시키는 경우는 다음과 같다.
따라서 구하는 함수의 개수는 aÛ`
1 4 9 16 25 36
3_6=18이다. 8b
8 ◯ ◯ ◯ ◯
16 ◯ ◯ ◯
456 ③
24 ◯ ◯
x+3yÉ10에서 y의 값에 따라 케이스 분류하면 다음과 같다. 32 ◯
Ú y=1인 경우 40
x+3É10, xÉ7이므로 순서쌍 (x, y)는 48
(1, 1), (2, 1), (3, 1), (4, 1), (5, 1), (6, 1)로 6개이다.
따라서 순서쌍 (a, b)는 (3, 1), (4, 1), (4, 2), (5, 1), (5, 2)
Û y=2인 경우
(5, 3), (6, 1), (6, 2), (6, 3), (6, 4)로 10개이다.
x+6É10, xÉ4이므로 순서쌍 (x, y)는
(1, 2), (2, 2), (3, 2), (4, 2)로 4개이다.
Ü y=3인 경우
x+9É10, xÉ1이므로 순서쌍 (x, y)는
(1, 3)으로 1개이다. 459 ⑤

한편, y¾4인 경우 조건을 만족시키는 x의 값이 존재하지 않는다. x+3y+2z=18에서 계수가 가장 큰 y의 값에 따라 케이스 분류하면
Ú ~ Ü에서 구하는 순서쌍의 개수는
다음과 같다. yy TIP
6+4+1=11이다.
Ú y=1일 때
TIP x+3+2z=18, x+2z=15이므로 순서쌍 (x, z)는
케이스 분류를 할 때에는 계수가 가장 큰 항의 값을 기준으로 (13, 1), (11, 2), (9, 3), (7, 4), (5, 5), (3, 6), (1, 7)로
경우를 나누는 것이 계산에 편리하다. 7개이다.
Û y=2일 때
x+6+2z=18, x+2z=12이므로 순서쌍 (x, z)는
457 ④ (10, 1), (8, 2), (6, 3), (4, 4), (2, 5)로 5개이다.
Ü y=3일 때
몇 자리 자연수를 만드는지에 따라 케이스 분류하면 다음과 같다.
x+9+2z=18, x+2z=9이므로 순서쌍 (x, z)는
Ú 한 자리 자연수인 경우
(7, 1), (5, 2), (3, 3), (1, 4)로 4개이다.
7로 하나뿐이므로 1개
Ý y=4일 때
Û 두 자리 자연수인 경우
x+12+2z=18, x+2z=6이므로 순서쌍 (x, z)는
16, 25, 34, 43, 52, 61, 70으로 7개
(4, 1), (2, 2)로 2개이다.
Ü 세 자리 자연수인 경우
Þ y=5일 때
300 이하의 세 자리 자연수이므로 백의 자리의 숫자는
x+15+2z=18, x+2z=3이므로 순서쌍 (x, z)는
1 또는 2이다.
(1, 1)로 1개이다.
백의 자리의 숫자가 1인 경우 십의 자리의 숫자와 일의 자리의
Ú~~ ~ Þ에서 구하는 순서쌍의 개수는
숫자의 합이 6이므로
7+5+4+2+1=19이다.
106, 115, 124, 133, 142, 151, 160으로 7개
백의 자리의 숫자가 2인 경우 십의 자리의 숫자와 일의 자리의
숫자의 합이 5이므로
205, 214, 223, 232, 241, 250으로 6개
그러므로 세 자리 자연수의 개수는 13이다. 460 ②

Ú ~ Ü에서 구하는 자연수의 개수는 48xÛ`-14nx+nÛ`=0에서 (6x-n)(8x-n)=0이므로


1+7+13=21이다. n n
x= 또는 x= 이다.
6 8
그러므로 이차방정식이 정수해를 갖기 위해서는
458 ②
n이 6의 배수이거나 8의 배수가 되어야 한다.
이차함수 y=xÛ`+ax+2b의 그래프가 x축과 적어도 한 점에서 200 이하의 자연수 중에서 6의 배수의 개수는 33이고
만나려면 방정식 xÛ`+ax+2b=0이 실근을 가져야 하므로 판별식을 8의 배수의 개수는 25이다.

116 수학 <하>
이때, 6과 8의 최소공배수인 24의 배수의 개수는 8이므로 이때, 삼각형이 정의되기 위해서는 a+b>c가 성립해야 하므로
구하는 자연수 n의 개수는 23
a+b+c>2c에서 c< 이다.
33+25-8=50 2
23 23
그러므로 Éc< 에서 8ÉcÉ11 (∵ c는 자연수)이고,
3 2
c의 값에 따라서 케이스 분류하면 다음과 같다.
461 ⑤
Ú c=11인 경우
100원짜리 동전 x개, 500원짜리 동전 y개, 1000원짜리 지폐 z장을 a+b+11=23, a+b=12이므로 순서쌍 (a, b)는
합하여 3500원이 되도록 하면 (1, 11), (2, 10), (3, 9), (4, 8), (5, 7), (6, 6)으로 6개이다.
100x+500y+1000z=3500 Û c=10인 경우
x+5y+10z=35 a+b+10=23, a+b=13이므로 순서쌍 (a, b)는
이고, z의 값에 따라 케이스 분류하면 다음과 같다. (3, 10), (4, 9), (5, 8), (6, 7)로 4개이다.
Ú z=0일 때 Ü c=9인 경우
x+5y+0=35, x+5y=35이므로 순서쌍 (x, y)는 a+b+9=23, a+b=14이므로 순서쌍 (a, b)는
(35, 0), (30, 1), (25, 2), (20, 3), (15, 4), (10, 5), (5, 6), (5, 9), (6, 8), (7, 7)로 3개이다.
(0, 7)로 8개이다. Ý c=8인 경우
Û z=1일 때 a+b+8=23, a+b=15이므로 순서쌍 (a, b)는
x+5y+10=35, x+5y=25이므로 순서쌍 (x, y)는 (7, 8)로 1개이다.
(25, 0), (20, 1), (15, 2), (10, 3), (5, 4), (0, 5)로 Ú ~ Ý에서 구하는 순서쌍의 개수는
6개이다. 6+4+3+1=14이다.
Ü z=2일 때
x+5y+20=35, x+5y=15이므로 순서쌍 (x, y)는
(15, 0), (10, 1), (5, 2), (0, 3)으로 4개이다.
Ý z=3일 때
x+5y+30=35, x+5y=5이므로 순서쌍 (x, y)는
(5, 0), (0, 1)로 2개이다.
464 ④

Ú ~ Ý에서 구하는 방법의 수는 a+c-2b=8에서 bÉ3이고,

8+6+4+2=20이다. 5
a+c¾-1+4=3이므로 -2bÉ5, b¾- 에서
2
5 경
- ÉbÉ3이다. 우
2
462 9
정수 b의 값에 따라 케이스 분류하면 다음과 같다.


5`g, 10`g, 20`g짜리의 세 종류의 저울추의 개수를 각각 Ú b=3인 경우
x, y, z (x, y, z는 자연수)라 하자. a+c-6=8, a+c=14이므로 순서쌍 (a, c)는
5x+10y+20z=80 (-1, 15), (0, 14), (1, 13), y, (10, 4)로 12개이다.
x+2y+4z=16 Û b=2인 경우
Ú z=1인 경우 a+c-4=8, a+c=12이므로 순서쌍 (a, c)는
x+2y+4=16, x+2y=12이므로 순서쌍 (x, y)는 (-1, 13), (0, 12), (1, 11), y, (8, 4)로 10개이다.
(2, 5), (4, 4), (6, 3), (8, 2), (10, 1)로 5개이다. Ü b=1인 경우
Û z=2인 경우 a+c-2=8, a+c=10이므로 순서쌍 (a, c)는
x+2y+8=16, x+2y=8이므로 순서쌍 (x, y)는 (-1, 11), (0, 10), (1, 9), y, (6, 4)로 8개이다.
(2, 3), (4, 2), (6, 1)로 3개이다. Ý b=0인 경우
Ü z=3인 경우 a+c-0=8, a+c=8이므로 순서쌍 (a, c)는
x+2y+12=16, x+2y=4이므로 순서쌍 (x, y)는 (-1, 9), (0, 8), (1, 7), y, (4, 4)로 6개이다.
(2, 1)로 1개이다. Þ b=-1인 경우
Ú ~ Ü에서 구하는 경우의 수는 a+c+2=8, a+c=6이므로 순서쌍 (a, c)는
5+3+1=9이다. (-1, 7), (0, 6), (1, 5), (2, 4)로 4개이다.
ß b=-2인 경우
a+c+4=8, a+c=4이므로 순서쌍 (a, c)는
463 14
(-1, 5), (0, 4)로 2개이다.
23 Ú ~ ß에서 구하는 순서쌍의 개수는
a+b+c=23이고, aÉbÉc이므로 c¾ 이다.
3 12+10+8+6+4+2=42이다.

Ⅵ. 경우의 수 117
A A A A B B B
465 11
B A B B
밑면의 반지름의 길이가 4이고 높이가 5이므로 수조 전체의 부피는 B A B
p_4Û`_5=80p B A
B A A B B
3 3
이 수조에 물이 만큼 차있으므로 80p_ =60p에서 B A B
4 4
B A
수조의 남은 부피는 20p이다.
B A A B
수조에 넣는 부피가 각각 p, 3p, 4p인 돌의 개수를 B A
각각 x, y, z (x, y, z는 자연수)라 할 때, B A A A B B
x+3y+4z=20 B A B
Ú z=1인 경우 B A
B A A B
x+3y+4=20, x+3y=16이므로 순서쌍 (x, y)는
B A
(13, 1), (10, 2), (7, 3), (4, 4), (1, 5)로 5개이다.
따라서 구하는 경우의 수는 14이다.
Û z=2인 경우
x+3y+8=20, x+3y=12이므로 순서쌍 (x, y)는
(9, 1), (6, 2), (3, 3)으로 3개이다.
Ü z=3인 경우 468 24

x+3y+12=20, x+3y=8이므로 순서쌍 (x, y)는 회장, 부회장, 회원 3명이 가져온 책을 각각 a, b, c, d, e라 하자.
(5, 1), (2, 2)로 2개이다. 각자가 가져오지 않은 책으로 나누어 가지려면 각자 스스로 가져온
Ý z=4인 경우 책을 제외하고, 회장과 부회장은 서로의 책이 아니어야 하므로
x+3y+16=20, x+3y=4이므로 순서쌍 (x, y)는 조건을 만족시키는 경우를 수형도로 나타내면 다음과 같다.
(1, 1)로 1개이다.
a b c d e
Ú ~ Ý에서 구하는 방법의 수는 c d a e b
5+3+2+1=11이다. b e a
e a b
b a
e a b d
b a d
466 11
d a b
카드 뒷면에 적을 수 있는 문자를 수형도로 나타내면 다음과 같다. b a
A B C D E d c a e b
b c a e d b e a
d a e c e a b
e a c d b a
e a b c
c d a e b c b
e a b d b a c
c a
d c a e b e c a b d
e a b c b a d
c b d a b
b a
d a b c
e c a b d
c b
d a b c
b a c
c b
c a
따라서 구하는 방법의 수는 11이다.
따라서 구하는 방법의 수는 8+8+8=24이다.

467 14 469 ⑤

첫 경기부터 A가 이긴 횟수가 B가 이긴 횟수보다 항상 많거나 공사로 인하여 진입이 제한된 길을 제외한 도로망에서 A지점에서
같으므로 첫 경기는 무조건 A가 이긴다. 출발하여 B지점까지 도로를 따라 최단거리로 이동하는 방법을
주어진 상황을 수형도로 나타내면 다음과 같다. 일일이 세면 다음과 같다.

118 수학 <하>
15 25 46 5_5_4_3 yy ㉡
1 B
5
㉠, ㉡에서 구하는 자연수의 개수는
1 10 21
4 10 k=9_9_8_7-5_5_4_3
1 11
3 6 10 10 k 9_9_8_7-5_5_4_3
∴ =
1 12 12
2 3 4
A =378-25=353
1 1 1 1
따라서 구하는 방법의 수는 46이다.
473 9

470 24 0, 1, 2, y, k의 (k+1)개의 숫자를 이용해서


중복을 허용하지 않고 만들어지는 세 자리 자연수의 개수는
(x-y)Ü`=xÜ`-3xÛ`y+3xyÛ`-yÜ`
a=k_k_(k-1) yy ㉠
(a+b+c)Û`=aÛ`+bÛ`+cÛ`+2ab+2bc+2ca
중복을 허용하여 만들어지는 세 자리 자연수의 개수는
이므로 (x-y)Ü`(a+b+c)Û`의 전개식에서 서로 다른 항의 개수는
b=k_(k+1)_(k+1) yy ㉡
4_6=24이다.
㉠, ㉡에서
b k(k+1)Û` (k+1)Û` 25
= = =
a kÛ`(k-1) k(k-1) 18
471 30
25k(k-1)=18(k+1)Û`
A지점에서 출발해서 D지점에 도착하는 경로를 케이스 분류하면 25kÛ`-25k=18kÛ`+36k+18
다음과 같다. 7kÛ`-61k-18=0, (7k+2)(k-9)=0
Ú 중간에 B지점만 지나는 경우 ∴ k=9 (∵ k>0)
A → B로 이동하는 방법의 수는 3
B → D로 이동하는 방법의 수는 2
이므로 A → B → D로 이동하는 방법의 수는 474 ②

3_2=6 백의 자리 숫자에 따라 케이스 분류하면 다음과 같다.


Û 중간에 C지점만 지나는 경우 Ú 백의 자리 숫자가 1인 경우
A → C로 이동하는 방법의 수는 2 일의 자리 숫자는 3, 5, 7, 9로 4가지이고
C → D로 이동하는 방법의 수는 4 이때, 십의 자리 숫자는 0부터 9까지 10개의 정수 중 백의 자리
이므로 A → C → D로 이동하는 방법의 수는 숫자와 일의 자리 숫자를 제외한 8가지이므로 경
2_4=8 우
4_8=32 의
Ü 중간에 B, C지점을 모두 지나는 경우 Û 백의 자리 숫자가 2인 경우 수
A → B → C → D의 순서로 이동하는 방법의 수는 일의 자리 숫자는 1, 3, 5, 7, 9로 5가지이고
3_1_4=12 이때, 십의 자리 숫자는 0부터 9까지 10개의 정수 중 백의 자리
A → C → B → D의 순서로 이동하는 방법의 수는 숫자와 일의 자리 숫자를 제외한 8가지이므로
2_1_2=4 5_8=40
그러므로 중간에 B, C지점을 모두 지나는 방법의 수는 Ü 백의 자리 숫자가 3인 경우
12+4=16 일의 자리 숫자는 1, 5, 7, 9로 4가지이고
Ú ~ Ü에서 구하는 방법의 수는 6+8+16=30이다. 이때, 십의 자리 숫자는 0부터 9까지 10개의 정수 중 백의 자리
숫자와 일의 자리 숫자를 제외한 8가지이므로
4_8=32
472 353
Ú ~ Ü에서 구하는 자연수의 개수는
네 자리 자연수에서 각 자릿수가 모두 다르기 위해서는 32+40+32=104이다.
천의 자리의 숫자는 0을 제외하고 9가지
백의 자리의 숫자는 천의 자리의 숫자를 제외하고 9가지
십의 자리의 숫자는 천의 자리와 백의 자리의 숫자를 제외하고 8가지 475 72

일의 자리의 숫자는 천의 자리와 백의 자리와 십의 자리의 숫자를 만들 수 있는 자연수는 1, 1, 2, 2, 3, 3, 5, 7, 7, 7 중


제외하고 7가지이므로 2개 이상의 숫자를 곱한 수이므로
만들 수 있는 각 자리의 숫자가 모두 다른 네 자리 자연수의 개수는 구하는 자연수의 개수는 1, 1, 2, 2, 3, 3, 5, 7, 7, 7을 모두 곱한 수
9_9_8_7 yy ㉠ 2Û`_3Û`_5_7Ü`의 양의 약수의 개수와 같다.
이때, 위에서 구한 네 자리 자연수 중에서 각 자리의 숫자에 6의 따라서 구하는 자연수의 개수는
약수가 하나도 없는 자연수를 위와 같은 방법으로 구하면 그 개수는 3_3_2_4=72이다.

Ⅵ. 경우의 수 119
476 72 479 41

주사위를 3번 던졌을 때 전체 경우의 수는 두 점 A(a, 2), B(3, b)에 대하여 선분 AB를 1`:`2로 내분하는
6_6_6=216 점의 좌표는
abc가 10의 배수가 되려면 주사위의 눈의 수는 5와 짝수가 모두 3+2a b+4 2a b+1
{ , }, 즉 { +1, +1}
적어도 한 번씩 나와야 한다. 3 3 3 3
이때, abc가 10의 배수가 되지 않는 경우를 케이스 분류하면 다음과 이때, 이 점의 x좌표와 y좌표가 모두 정수이므로
같다. a는 3의 배수이고, b는 3으로 나눈 나머지가 2인 수이다.
Ú 눈의 수에 5가 한 번도 포함되지 않을 때 20 이하의 자연수 중에서 3의 배수는
5를 제외한 1, 2, 3, 4, 6 중에서만 나와야 하므로 경우의 수는 3, 6, 9, 12, 15, 18로 6개이고,
5_5_5=125 3으로 나눈 나머지가 2인 수는
Û 눈의 수에 짝수가 한 번도 포함되지 않을 때 2, 5, 8, 11, 14, 17, 20으로 7개이다.
2, 4, 6을 제외한 1, 3, 5 중에서만 나와야 하므로 경우의 수는 그런데 a=3, b=2이면 두 점 A, B가 서로 같으므로 제외해야 한다.
3_3_3=27 따라서 구하는 순서쌍 (a, b)의 개수는
Ú, Û에서 눈의 수에 5와 짝수가 모두 포함되지 않을 때가 6_7-1=41이다.
중복되므로 1, 3 중에서만 나오는 경우의 수는 2_2_2=8
따라서 구하는 경우의 수는 480 풀이 참조
216-(125+27-8)=72이다.
Ú 지불할 수 있는 방법의 수
10000원짜리 지폐 2장을 지불할 수 있는 방법은 3가지
5000원짜리 지폐 3장을 지불할 수 있는 방법은 4가지
477 ④
1000원짜리 지폐 6장을 지불할 수 있는 방법은 7가지
540을 소인수분해하면 540=2Û`_3Ü`_5 이고 0원을 지불하는 경우를 제외해야 하므로
짝수인 양의 약수는 반드시 2를 인수로 가져야 한다. 지불할 수 있는 방법의 수는
그러므로 구하는 약수의 개수는 전체 양의 약수의 개수에서 a=3_4_7-1=84-1=83이다.
2를 인수로 갖지 않는 홀수인 양의 약수의 개수를 빼면 된다. Û 지불할 수 있는 금액의 수
2Û`_3Ü`_5의 양의 약수 중 홀수의 개수는 3Ü`_5의 양의 약수의 10000원짜리 지폐 2장으로 지불할 수 있는 금액은
개수와 같으므로 0원, 10000원, 20000원
(2+1)_(3+1)_(1+1)-(3+1)_(1+1)=16 5000원짜리 지폐 3장으로 지불할 수 있는 금액은
0원, 5000원, 10000원, 15000원
1000원짜리 지폐 6장으로 지불할 수 있는 금액은
0원, 1000원, 2000원, 3000원, 4000원, 5000원, 6000원
478 풀이 참조
이때, 10000원짜리 지폐 1장으로 지불할 수 있는 금액과
24=2Ü`_3이므로 소수 p를 다음의 경우로 케이스 분류할 수 있다. 5000원짜리 지폐 2장으로 지불할 수 있는 금액은 서로 같고,
Ú p=2인 경우 5000원짜리 지폐 1장으로 지불할 수 있는 금액과
24p=2Ý`_3이므로 모든 양의 약수의 개수는 1000원짜리 지폐 5장으로 지불할 수 있는 금액이 서로 같으므로
(4+1)_(1+1)=10 10000원짜리 지폐 2장과 5000원짜리 지폐 3장을
Û p=3인 경우 모두 1000원짜리 지폐로 각각 20장, 15장으로 바꾸어 생각하면
24p=2Ü`_3Û`이므로 모든 양의 약수의 개수는 구하는 지불할 수 있는 금액의 수는 1000원짜리 지폐
(3+1)_(2+1)=12 20+15+6=41장을 지불하는 방법의 수와 같다.
Ü p+2, p+3인 경우 따라서 1000원짜리 지폐 41장을 지불하는 방법의 수는
24p=2Ü`_3_p이므로 모든 양의 약수의 개수는 42가지이고 0원을 지불하는 경우를 제외해야 하므로
(3+1)_(1+1)_(1+1)=16 지불할 수 있는 금액의 수는
Ú ~ Ü에서 구하는 값들의 합은 b=42-1=41이다.
10+12+16=38이다. Ú, Û에서 구하는 값은
a+b=83+41=124
채점 요소 배점
24를 소인수분해하기 10% 채점 요소 배점

p의 값이 2일 때, 3일 때, 2, 3이 아닐 때로 케이스를 나누어서 약수의 지불할 수 있는 방법의 수 구하기 40%


80%
개수 구하기 지불할 수 있는 금액의 수 구하기 40%
각각의 케이스에서의 약수의 개수의 합 구하기 10% a+b의 값 구하기 20%

120 수학 <하>
다음과 같다.
481 ④
Ú 백의 자리의 숫자가 1인 경우
다음과 같이 가로 방향으로 평행한 직선과 세로 방향으로 평행한 십의 자리의 숫자와 일의 자리의 숫자에 들어갈 수 있는 숫자는
직선에 각각 번호를 붙이자. 1을 제외한 0부터 9까지의 음이 아닌 정수이므로 십의 자리와
1 2 3 4 5 6 7 8 일의 자리의 숫자를 결정하는 방법의 수는
1 9_8=72
2 Û 백의 자리의 숫자가 2인 경우
3
4
Ú과 마찬가지이므로 이때의 방법의 수는
5 9_8=72
6 Ü 백의 자리의 숫자가 3인 경우
7
십의 자리의 숫자가 0, 1, 2, 4, 5, 6, 7인 수가 각각 8개씩
색칠된 사각형을 포함하도록 하려면 세로 방향으로 평행한 존재하므로 십의 자리의 숫자가 7이면서 가장 큰 수인 379가
직선에서는 1, 2, 3, 4 중에서 한 개, 6, 7, 8 중에서 한 개를 200번째 수이다.
선택하고 가로 방향으로 평행한 직선에서는 1, 2 중에서 한 개, Ú ~ Ü에서 각 자리의 숫자가 모두 다른 세 자리 자연수를 작은
5, 6, 7 중에서 한 개를 선택해야 한다. 수부터 나열했을 때 379가 200번째 수이므로 구하고자 하는
따라서 색칠된 사각형을 포함하는 사각형의 개수는 당첨번호는 380이다.
4_3_2_3=72이다.

482 ③ 484 ③

외심이 삼각형의 외부에 존재하기 위해서는 만들어지는 삼각형이 숫자 1이 백의 자리 또는 십의 자리 또는 일의 자리에 있을 때


둔각삼각형이 되어야 한다. 만들 수 있는 자연수의 개수는 다음과 같다.
만들어질 수 있는 둔각삼각형은 다음과 같다. Ú 숫자 1이 백의 자리의 숫자인 경우
Ú 이웃한 3개의 점으로 만들어지는 경우 십의 자리의 숫자와 일의 자리의 숫자는 각각 0, 1, 2, y, 9 중에서
A 하나씩 선택하면 되므로 자연수의 개수는 10_10=100
B H Û 숫자 1이 십의 자리의 숫자인 경우
백의 자리의 숫자는 0, 1, 2, 3, 4 중에 하나이고
C G 일의 자리의 숫자는 0, 1, 2, y, 9 중에 하나이므로 자연수의

개수는 5_10=50 우

D F Ü 숫자 1이 일의 자리의 숫자인 경우

E 백의 자리의 숫자는 0, 1, 2, 3, 4 중에 하나이고
이러한 둔각삼각형은 원의 둘레의 8개의 점에 대해서 각각 1개씩 십의 자리의 숫자는 0, 1, 2, y, 9 중에 하나이므로 자연수의
만들어지므로 개수는 8이다. 개수는 5_10=50
Û 이웃한 2개의 점과 이웃하지 않은 한 점으로 만들어지는 경우 Ú ~ Ü에서 숫자 1을 쓴 횟수는
A 100+50+50=200이다.
B H
TIP
Û에서 백의 자리의 숫자가 0인 경우 두 자리 자연수가
C G
만들어지고 Ü에서 백의 자리의 숫자와 십의 자리의 숫자가 모두
0인 경우 한 자리 자연수가 만들어진다.
D F
예를 들어, Û에서 백의 자리의 숫자가 0, 일의 자리의 숫자가
E
4인 경우 014이고 이를 14로 생각할 수 있다.
이러한 둔각삼각형은 원의 둘레의 이웃한 2개의 점에
또한 Ü에서 백의 자리의 숫자와 십의 자리의 숫자가 모두 0이고
대해서 각각 2개씩 만들어지므로 개수는
일의 자리의 숫자만 1이면 001이고 이는 1이다.
2_8=16이다.
Ú, Û에서 구하는 삼각형의 개수는
8+16=24이다.

485 ②
483 380
전체집합 U와 두 부분집합 A, B에 대하여 다음과 같이 각각의
조건을 만족시키는 세 자리 자연수를 작은 수부터 개수를 세어 보면 원소가 포함되는 부분은 ㉠, ㉡, ㉢, ㉣ 중에 하나이다.

Ⅵ. 경우의 수 121
U 따라서 구하는 함수 f의 개수는
㉠ A B
2_2_2_1_3_2_1=48이다.
㉡ ㉢ ㉣

488 84

조건 ㈎에서 1, 3, 5는 모두 ㉡에 속하게 되고 서로 인접하지 않은 두 영역 A, C를 칠하는 방법에 따라 케이스


조건 ㈏에서 7, 8은 모두 ㉠에 속한다. 분류하면 다음과 같다. yy 참고
그러므로 나머지 2, 4, 6이 ㉠, ㉢, ㉣ 중에 하나에 속하면 되므로 Ú A, C에 같은 색을 칠하는 경우
구하는 순서쌍 (A, B)의 개수는 A, C에 칠할 수 있는 색은 4가지
3_3_3=27 B, D에 칠할 수 있는 색은 각각 A, C에 칠한 색을 제외한 3가지
이므로 이때의 방법의 수는
4_3_3=36이다.
486 ②
Û A, C에 서로 다른 색을 칠하는 경우
전체집합의 부분집합 중 서로 다른 두 집합을 각각 A, B라 하고 A에 칠할 수 있는 색은 4가지
A,B를 만족시킨다고 하자. C에 칠할 수 있는 색은 A에 칠한 색을 제외한 3가지
다음과 같이 전체집합의 각각의 원소 1, 2, 3, 4, 5는 B, D에 칠할 수 있는 색은 각각 A, C에 칠한 색을 제외한 2가지
세 집합 ㉠ A, ㉡ B-A, ㉢ U-B 중에 한 곳에 속하게 된다. 이므로 이때의 방법의 수는
U 4_3_2_2=48이다.
㉢ B Ú, Û에서 구하는 방법의 수는
A
㉡ 36+48=84이다.

참고

이웃하지 않은 두 영역 B, D를 칠하는 방법에 따라 케이스 분류


두 집합 A, B를 만드는 모든 방법의 수는
해도 결과는 동일하다.
3_3_3_3_3=243
이때, 두 집합 A, B가 서로 같은 집합이 되는 경우를 제외해야 한다.
A=B가 되는 경우는 다음과 같이 전체집합의 각각의 원소
1, 2, 3, 4, 5가 두 집합 ㉣ A(=B), ㉤ U-A 중에 한 곳에 속할
489 ⑤

때이다. 침수로 인해서 통행이 제한된 도로를 지우고


A지점에서 B지점까지 최단거리로 이동하는 방법을 일일이 세면
U
B=A 다음과 같다.

276
㉣ A 1 84 B
84
1 2 24 192
3 60
1 3 24 108
이를 만족시키는 두 집합 A, B를 만드는 방법의 수는 24
4 7 36
1 12 48
2_2_2_2_2=32
따라서 구하는 두 부분집합을 선택하는 방법의 수는 1
5 12 12 12 12 12
243-32=211이다. 따라서 구하는 방법의 수는 276이다.
다른 풀이

487 48 침수로 인해서 통행이 제한된 도로를 지우고 다음과 같이


C, D지점을 정하자.
조건 ㈎에 의하여 함수 f는 일대일대응이다.
23
조건 ㈐에 의하여 A 1 7 B
f(2)의 값으로 가능한 값은 1, 2로 2가지, 7
1 2 2 16
f(3)의 값으로 가능한 값은 1, 2, 3 중 f(2)의 값을 제외한 2가지, 3 5
1 3 2 9
f(4)의 값으로 가능한 값은 1, 2, 3, 4 중 f(2), f(3)의 값을 제외한 4 7 2 3 4
1 1
2가지, C D
1
f(5)의 값으로 가능한 값은 1, 2, 3, 4, 5 중 5 12 1 1 1
f(1), f(2), f(3), f(4)의 값을 제외한 1가지이다. (∵ 조건 ㈏) A지점에서 C지점까지 이동하는 방법의 수는 12이고,
일대일대응이 되기 위해서 남은 정의역의 원소 6, 7, 8과 공역의 C지점에서 D지점까지 이동하는 방법의 수는 1이며
원소 6, 7, 8이 대응하는 방법의 수는 3_2_1이다. D지점에서 B지점까지 이동하는 방법의 수는 23이다.

122 수학 <하>
그러므로 구하는 방법의 수는 A a A a a A a A
12_1_23=276이다. b a
a A b A
a B
490 65 b A a a
b a A a
도시 A에서 도시 D로 갈 때 또는 B A b
도시 D에서 도시 A로 갈 때 걸리는 시간에 따라 a a A
경우의 수를 구해 보면 다음과 같다. b a
2시간 30분인 경우는 도시 C만 거치는 경우 1가지, a A
두 도시 B, C를 모두 거치지 않는 경우 1가지로 총 2가지이다. a

3시간인 경우는 도시 B만 거치는 경우 2가지, B A a A b A a A


도시 C만 거치는 경우 3가지로 총 5가지이다. a a
b A b A
3시간 30분인 경우는 도시 B를 거치는 경우 2가지,
B B
도시 C를 거치는 경우 2가지로 총 4가지이다.
a a
따라서 왕복 평균 소요시간이 6시간 이하가 되는 경우를 A → D로 a A a B A a
이동할 때와 D → A로 이동할 때 걸리는 시간으로 케이스 분류하면 b b
다음과 같다. a A a A
Ú 둘 다 2시간 30분인 경우: 2_2=4(가지) a a
b A a b A
Û 하나가 2시간 30분이고 다른 하나가 3시간인 경우
b B
2_2_5=20(가지) B A a
Ü 하나가 2시간 30분이고 다른 하나가 3시간 30분인 경우 a a A a
2_2_4=16(가지) b b
Ý 둘 다 3시간인 경우 a A a A
5_5=25(가지) a a

Ú ~ Ý에서 구하는 경로의 수는 따라서 구하는 경우의 수는


4+20+16+25=65이다. 11+9+16+16=52이다.

491 ③ 경

주어진 규칙에 따라 만들 수 있는 문자열을 3자리까지만 써 보면 의

다음과 같다. 492 200 수

A a A a A a 1부터 49까지의 수를 6으로 나눈 나머지는


a b
1, 2, 3, 4, 5, 0 중의 하나이다.
b A a A
B a 이 중 홀수는 6으로 나눈 나머지가 1, 3, 5이고,
a 두 수의 합이 6의 배수가 되는 경우는 선택한 두 수를 6으로 나눈
나머지가 1, 5인 경우와 모두 3인 경우이다.
B A a b A a
b b Ú a, b를 6으로 나눈 나머지가 1, 5인 경우
a A B A 6으로 나눈 나머지가 1인 수는
a a 1, 7, 13, y, 49로 9개
b A b 6으로 나눈 나머지가 5인 수는
B a A 5, 11, 17, y, 47로 8개
a a
이때, 서로 다른 두 수 a, b의 순서쌍 (a, b)의 개수는
이때, A, a 뒤에 올 수 있는 문자는 각각 2가지이고,
9_8_2=144
B, b 뒤에 올 수 있는 문자는 각각 3가지이다.
Û a, b를 6으로 나눈 나머지가 모두 3인 경우
문자열의 세 번째 자리에서 A 또는 a의 개수는 17이고, B 또는 b의
6으로 나눈 나머지가 3인 수는
개수는 6이므로 구하는 경우의 수는
3, 9, 15, y, 45로 8개
17_2+6_3=52이다.
서로 다른 두 수 a, b의 순서쌍 (a, b)의 개수는
다른 풀이 8_7=56
주어진 규칙에 따라서 4개의 문자열을 만드는 경우를 일일이 세면 Ú, Û에서 구하는 순서쌍 (a, b)의 개수는
다음과 같다. 144+56=200이다.

Ⅵ. 경우의 수 123
Û 카드에 적힌 숫자의 합이 13인 학생이 있는 경우
493 19
한 명의 학생이 1, 3, 4, 5를 가지고
조건 ㈎, ㈏에 의하여 2를 나머지 2명의 학생 중 한 명이 갖는 방법의 수는
1교시부터 4교시에 국어, 영어를 연달아서 넣어야 하므로 3_2=6
다음과 같이 케이스를 분류할 수 있다. Ü 카드에 적힌 숫자의 합이 14인 학생이 있는 경우
Ú 국어 영어 한 명의 학생이 2, 3, 4, 5를 가지고
1을 나머지 2명의 학생 중 한 명이 갖는 방법의 수는
조건 ㈐에 의하여 과학은 3, 5교시 또는 3, 6교시 또는
3_2=6
4, 6교시에 한다.
Ý 카드에 적힌 숫자의 합이 15인 학생이 있는 경우
이 중 과학이 3, 6교시일 때, 조건 ㈑에 의하여
한 명의 학생이 1, 2, 3, 4, 5를 모두 갖는 방법의 수는 3
나머지 교시에 수학, 체육 순서로 넣어야 하고
Ú ~ Ý에서 구하는 방법의 수는
과학이 3, 5교시 또는 4, 6교시일 때,
243-(18+6+6+3)=210이다.
나머지에 수학, 체육을 넣는 경우의 수는 2이다.
그러므로 경우의 수는
1+2_2=5
495 16
Û 국어 영어
하나의 가방에 축구공을 최대 4개 담을 수 있으므로 10개의
조건 ㈐에 의하여 과학은 1, 4교시 또는 1, 5교시 또는 1, 6교시
축구공을 모두 나누어 담으려면 적어도 3개 이상의 가방을 사용해야
또는 4, 6교시에 한다.
한다.
과학이 1, 4교시 또는 1, 6교시일 때, 나머지 교시에 수학, 체육
사용하는 가방의 개수에 따라 케이스 분류하면 다음과 같다.
순서로 와야 하고,
Ú 가방을 3개 사용하는 경우
과학이 1, 5교시 또는 4, 6교시일 때, 나머지 교시에 수학, 체육을
10=4+4+2
넣는 경우의 수는 2이다.
=4+3+3
그러므로 경우의 수는
에서 2가지이다.
2+2_2=6
Û 가방을 4개 사용하는 경우
Ü 국어 영어 10=4+4+1+1
조건 ㈐에 의하여 1, 2교시 중 하나에 과학을 넣고 =4+3+2+1
5, 6교시 중 하나에 과학을 넣으면 된다. =4+2+2+2
과학을 배치하는 경우의 수는 2_2 =3+3+3+1
나머지 교시에 수학, 체육을 넣는 경우의 수는 2이다. =3+3+2+2
그러므로 경우의 수는 에서 5가지이다.
2_2_2=8 Ü 가방을 5개 사용하는 경우
Ú ~ Ü에 의하여 구하는 방법의 수는 10=4+3+1+1+1
5+6+8=19이다. =4+2+2+1+1
=3+3+2+1+1
=3+2+2+2+1
494 ①
=2+2+2+2+2
1, 2, 3, 4, 5가 하나씩 적힌 5장의 카드를 3명의 학생이 나누어 갖는 에서 5가지이다.
전체 경우의 수는 Ý 가방을 6개 사용하는 경우
3_3_3_3_3=243 10=4+2+1+1+1+1
이고, 이때 1, 2, 3, 4, 5를 모두 합하면 15이므로 =3+3+1+1+1+1
나누어 가진 카드에 적힌 숫자의 합이 12 이상인 학생이 있는 경우를 =3+2+2+1+1+1
케이스 분류하면 다음과 같다. =2+2+2+2+1+1
Ú 카드에 적힌 숫자의 합이 12인 학생이 있는 경우 에서 4가지이다.
( -①) 한 명의 학생이 3, 4, 5를 가지고 Ú ~ Ý에서 구하는 방법의 수는
1, 2를 나머지 2명의 학생이 나누어 갖는 방법의 수는 2+5+5+4=16이다.
3_2_2=12
( -②) 한 명의 학생이 1, 2, 4, 5를 가지고
3을 나머지 2명의 학생 중 한 명이 갖는 방법의 수는 496 ③

3_2=6 먼저 동그란 접시와 네모난 접시에 나누어 담을 쿠키의 개수를


방법의 수는 12+6=18 기준으로 경우를 케이스 분류하면 다음과 같다.

124 수학 <하>
이때, 동그란 접시의 개수는 2이므로 적어도 2개의 쿠키를 배정해야 TIP
한다. 천의 자리의 수가 1인 경우는 나머지 세 자리의 수가 모두
Ú 동그란 접시에 2개, 네모난 접시에 8개 담는 경우 0이어도 1000이 되므로 자연수이다.
동그란 접시 2개에 쿠키 2개를 나누어 담는
방법의 수는 2=1+1로 1이다
Û 동그란 접시에 3개, 네모난 접시에 7개 담는 경우
동그란 접시 2개에 쿠키 3개를 나누어 담는
방법의 수는 3=2+1로 1이다. 498 288

Ü 동그란 접시에 4개, 네모난 접시에 6개 담는 경우 왼쪽으로는 이동할 수 없고 위쪽 또는 아래쪽 또는 오른쪽으로만


동그란 접시 2개에 쿠키 4개를 나누어 담는 이동해야 하므로 가로 방향의 도로 중에서 하나씩을 선택해서
방법의 수는 4=3+1=2+2로 2이다. 지나가면 된다.
Ý 동그란 접시에 5개, 네모난 접시에 5개 담는 경우
① ①① ①① ①① ①① ① ① ①① ①① ①① ①① ①
동그란 접시 2개에 쿠키 5개를 나누어 담는 ② ② ② ②
② ② ② ② ② ② ② ②
방법의 수는 5=4+1=3+2로 2이다. ② ② ② ②
Þ 동그란 접시에 6개, 네모난 접시에 4개 담는 경우 ③ ③ ③ ③
③ ③ ③ ③
동그란 접시 2개에 쿠키 6개를 나누어 담는 ③ ③ ②④ ④② ③④ ③ ③ ④③ ②④ ④② ③④ ③
A A④ B A B A B B
방법의 수는 6=5+1=4+2=3+3으로 3이다.
[그림1] [그림2]
ß 동그란 접시에 7개, 네모난 접시에 3개 담는 경우
예를 들어, [그림1]에서 왼쪽에서부터 ② → ③ → ① → ④ → ③을
동그란 접시 2개에 쿠키 7개를 나누어 담는
선택하면 [그림2]와 같이 이동하는 경우이다.
방법의 수는 7=6+1=5+2=4+3으로 3이다.
그러므로 구하는 방법의 수는
à 동그란 접시에 8개, 네모난 접시에 2개 담는 경우
3_4_2_4_3=288이다.
동그란 접시 2개에 쿠키 8개를 나누어 담는
방법의 수는 8=7+1=6+2=5+3=4+4로 4이다.
¡ 동그란 접시에 9개, 네모난 접시에 1개 담는 경우
동그란 접시 2개에 쿠키 9개를 나누어 담는
방법의 수는 9=8+1=7+2=6+3=5+4로 4이다.
499 88

Ú ~ ¡에서 구하는 방법의 수는 다음과 같이 가로 방향으로 평행한 직선과 세로 방향으로 평행한


1+1+2+2+3+3+4+4=20이다. 직선에 각각 번호를 붙이자.

가로선 중 2, 3 사이에 있는 별을 X, 4, 5 사이에 있는 별을 Y라 우
하자. 의

1 2 3 4 5 6

1
497 ②
2
X
각 자리의 숫자는 3과 6을 제외한 0, 1, 2, 4, 5, 7, 8, 9의 8개의 3
숫자만 사용할 수 있고, 규칙대로 말했을 때 2000이 크기 순서대로 4
Y
나열했을 때 몇 번째 자연수인지를 알아야 한다. 5

이때, 조건을 만족시키는 8개의 숫자로 만들 수 있는 0부터 6

999까지의 수는 Ú X를 포함하는 직사각형의 개수는


8_8_8=512 두 가로선 중 하나를 1, 2 중에서 뽑고,
이고, 이때 모든 자리에 0이 오는 경우는 자연수가 아니므로 세 자리 다른 하나를 3, 4, 5, 6 중에서 뽑고,
이하의 자연수의 개수는 512-1=511이다. 두 세로선 중 하나를 1, 2, 3 중에서 뽑고,
또한 천의 자리의 수가 1인 네 자리 자연수의 개수도 같은 다른 하나를 4, 5, 6 중에서 뽑으면 되므로
아이디어로 2_4_3_3=72
8_8_8=512 yy TIP Û Y를 포함하는 직사각형의 개수도 마찬가지로 구하면
그러므로 2000보다 작은 자연수의 개수가 511+512=1023이므로 4_2_2_4=64
2000은 규칙대로 나열했을 때, 1024번째 자연수이다. X와 Y를 모두 포함하는 직사각형의 개수도 마찬가지로 구하면
1번을 부여받은 학생부터 번호를 말하여 10명씩 돌아가면서 숫자를 2_2_2_3=24이고,
말하게 되고, Ú, Û에 이 경우가 모두 포함되어 있으므로 각각의 경우에서
1024는 10으로 나눈 나머지가 4이므로 2000을 말하는 학생의 이 경우를 빼주면 구하는 직사각형의 개수는
번호는 4이다. (72-24)+(64-24)=88

Ⅵ. 경우의 수 125
500 32 502 ④

조건 ㈎의 부등식 15Éa+3b+5cÉ20에서 c의 값에 따라 케이스 조건 ㈎에서 방정식 f(x)=0을 만족시키는 음의 실수가 존재하지


분류하면 다음과 같다. 않으므로 조건 ㈏에서 방정식 f(x)=0의 근은 모두 음이 아닌
Ú c=1일 때 정수이다.
10Éa+3bÉ15이므로 순서쌍 (a, b, c)의 개수는 방정식 xÜ`-9xÛ`+mx+n=0의 세 근을 각각 a, b, c라 할 때
b=1일 때 7ÉaÉ12에서 6개 삼차방정식의 근과 계수의 관계에 의하여
b=2일 때 4ÉaÉ9에서 6개 a+b+c=9
b=3일 때 1ÉaÉ6에서 6개 ab+bc+ca=m
b=4일 때 1ÉaÉ3에서 3개 abc=-n
이므로 6+6+6+3=21(개) 이므로 a+b+c=9를 만족시키는 음이 아닌 세 정수를 구해 보면
Û c=2일 때 다음과 같다.
5Éa+3bÉ10이므로 순서쌍 (a, b, c)의 개수는 9=9+0+0
b=1일 때 2ÉaÉ7에서 6개 =8+1+0=7+2+0=6+3+0=5+4+0
b=2일 때 1ÉaÉ4에서 4개 =7+1+1=6+2+1=5+3+1=5+2+2
b=3일 때 a=1에서 1개 =4+4+1=4+3+2=3+3+3
이므로 6+4+1=11(개) 위의 각각의 경우마다 순서쌍 (m, n)이 하나씩 정해지므로
Ü c=3일 때 yy 참고
0Éa+3bÉ5이므로 순서쌍 (a, b, c)의 개수는 구하는 순서쌍의 개수는 12이다.
b=1일 때 1ÉaÉ2이므로 2개
참고
Ú ~ Ü에서 부등식 15Éa+3b+5cÉ20을 만족시키는
각각의 경우에서 (m, n)을 구해 보면 다음과 같다.
자연수 a, b, c의 순서쌍 (a, b, c)의 개수는
a+b+c=9+0+0인 경우 (m, n)=(0, 0)
21+11+2=34이다.
a+b+c=8+1+0인 경우 (m, n)=(8, 0)
이때, 조건 ㈏에서 세 점 (0, a), (1, b), (2, c)가 한 직선 위에
a+b+c=7+2+0인 경우 (m, n)=(14, 0)
있지 않으므로
a+b+c=6+3+0인 경우 (m, n)=(18, 0)
b-a c-b
= 에서 a+c=2b a+b+c=5+4+0인 경우 (m, n)=(20, 0)
1-0 2-1
를 만족시키는 경우는 제외해야 한다. a+b+c=7+1+1인 경우 (m, n)=(15, -7)
a+c=2b를 만족시키는 경우는 (2, 2, 2), (5, 3, 1)로 2가지이므로 a+b+c=6+2+1인 경우 (m, n)=(20, -12)
구하는 순서쌍의 개수는 34-2=32이다. a+b+c=5+3+1인 경우 (m, n)=(23, -15)
a+b+c=5+2+2인 경우 (m, n)=(24, -20)
a+b+c=4+4+1인 경우 (m, n)=(24, -16)
a+b+c=4+3+2인 경우 (m, n)=(26, -24)
501 ④
a+b+c=3+3+3인 경우 (m, n)=(27, -27)
그림과 같이 위의 왼쪽 방부터 차례대로 1, 2, 3, 4, 5, 6번 방이라
하자.

1 2 3
503 63
4 5 6
문자판에서 G는 오직 하나뿐이므로 반드시 칠해야 하고,
Ú 1번 방에서 시작하는 경우 G부터 거꾸로 N, I, R, P, S를 칠할 칸을 선택하면 된다.
1 → 2 → 3 → 6 → 5 → 4, 1 → 4 → 5 → 6 → 3 → 2, 이때, 문자판이 좌우대칭의 모양으로 쓰여 있으므로 G부터 세로
1 → 4 → 5 → 2 → 3 → 6의 세 가지 방법이 있고, 방향의 칸을 몇 칸 선택해서 색칠할 지에 따라 케이스 분류하면
3, 4, 6번 방에서 시작하는 경우에도 마찬가지로 각각 세 가지 다음과 같다.
방법이 있으므로 모두 3_4=12(가지) Ú 세로 방향으로 G만 칠하는 경우
Û 2번 방에서 시작하는 경우 S P R I N G N I R P S
2 → 1 → 4 → 5 → 6 → 3, 2 → 3 → 6 → 5 → 4 → 1의 두 가지 S P R I N I R P S
방법이 있고, S P R I R P S
5번 방에서 시작하는 경우에도 마찬가지로 두 가지 방법이 S P R P S
있으므로 모두 2_2=4(가지) S P S
Ú, Û에서 구하는 방법의 수는 12+4=16이다. S

126 수학 <하>
N부터 순차적으로 색칠할 N, I, R, P, S가 적힌 칸을 (1+2+3+4+5+6)+a+b+c=21+a+b+c
결정하는 경우가 각각 2가지씩이므로 따라서 삼각형의 한 변에 있는 수의 합은
2_2_2_2_2=32(가지) 21+a+b+c a+b+c
=7+ 이므로
Û 세로 방향으로 G, N까지 칠하는 경우 3 3
a+b+c의 값은 3의 배수가 되어야 한다.
S P R I N G N I R P S
집합 {a, b, c}로 가능한 것은
S P R I N I R P S
R P S
{1, 2, 3}, {1, 2, 6}, {1, 3, 5}, {1, 5, 6},
S P R I
S P R P S
{2, 3, 4}, {4, 5, 6}, {2, 4, 6}, {3, 4, 5}이고
S P S 각각의 경우에 삼각형의 한 변에 있는 수의 합은
S 9, 10, 10, 11, 10, 12, 11, 11이 된다.

I부터 순차적으로 색칠할 I, R, P, S가 적힌 칸을 결정하는 이 중에서 1부터 6까지의 자연수로 삼각형을 완성할 수 있는 경우는

경우가 각각 2가지씩이므로 {1, 2, 3}, {1, 3, 5}, {4, 5, 6}, {2, 4, 6}이고

2_2_2_2=16(가지) 각각의 경우에 꼭짓점의 위치를 정해주는 방법의 수는

Ü 세로 방향으로 G, N, I까지 칠하는 경우 3_2_1=6이므로 구하는 방법의 수는 4_6=24이다.

S P R I N G N I R P S
S P R I N I R P S
S P R I R P S
S P R P S
S P S 505 20

S
선택한 4개의 점을 꼭짓점으로 하는 사각형은 사다리꼴이고 두 직선
R부터 순차적으로 색칠할 R, P, S가 적힌 칸을 결정하는 l¡ A B C D E
lÁ, lª 사이의 거리가 1이므로 높이가 1이다.
경우가 각각 2가지씩이므로 l™
그러므로 넓이가 3이 되려면 밑변의 길이와 윗변의 F 길이의
G H I합이 6이
J K
2_2_2=8(가지) 되어야 하므로
A B C D E
l¡ A B C D E
Ý 세로 방향으로 G, N, I, R까지 칠하는 경우 l¡
두 직선 lÁ, lª에서 각각 선택한 두 점 사이의 거리가
l™ l™
S P R I N G N I R P S 3, 3 또는 2, 4 또는 1, 5이고
F 다음
G H 그림과 같다. F G H I J K
I J K
S P R I N I R P S
l¡ A B C D E l¡ A B C D E l¡
A B C D E
S P R I R P S
l™ l™ l™
S P R P S F G H I J K F G H I J K F G H I J K

S P S [그림1] [그림2] [그림3] 우
l¡ A B C D E l¡
A B C D E

S
Ú
l™ 평행한 두 변의 길이가
l™ 모두 3인 경우 수
P부터 순차적으로 색칠할 P, S가 적힌 칸을 결정하는 F G H I J K F G H I J K
직선 lÁ에서 거리가 3인 두 점을 선택하는 경우는
경우가 각각 2가지씩이므로 A B C D E
l¡ {A, D}, {B, E}에서 2가지
2_2=4(가지) l™ 직선 lª에서 거리가 3인 두 점을 선택하는 경우는
F G H I J K
Þ 세로 방향으로 G, N, I, R, P까지 칠하는 경우 {F, I}, {G, J}, {H, K}에서 3가지
S P R I N G N I R P S 즉, 사각형의 개수는 2_3=6
S P R I N I R P S Û 평행한 두 변의 길이가 2, 4인 경우
S P R I R P S 직선 lÁ에서 거리가 2인 두 점을 선택하는 경우는
S P R P S {A, C}, {B, D}, {C, E}에서 3가지
S P S 직선 lª에서 거리가 4인 두 점을 선택하는 경우는
S
{F, J}, {G, K}에서 2가지
남은 S가 적힌 칸을 결정하는 경우의 수는 3이다. 이므로 3_2=6(가지)
Ú ~ Þ에서 구하는 방법의 수는 직선 lÁ에서 거리가 4인 두 점을 선택하는 경우는
32+16+8+4+3=63이다. {A, E}에서 1가지
직선 lª에서 거리가 2인 두 점을 선택하는 경우는
{F, H}, {G, I}, {H, J}, {I, K}에서 4가지
이므로 1_4=4(가지)
504 24
즉, 사각형의 개수는 6+4=10
삼각형의 세 꼭짓점에 들어가는 수를 각각 a, b, c라 하자. Ü 평행한 두 변의 길이가 1, 5인 경우
삼각형의 각 변에 있는 수의 합을 모두 더하면 6개의 수를 한 번씩 직선 lÁ에서 거리가 1인 두 점을 선택하는 경우는
더하고 꼭짓점의 수는 한 번씩 더 더해지므로 {A, B}, {B, C}, {C, D}, {D, E}에서 4가지

Ⅵ. 경우의 수 127
직선 lª에서 거리가 5인 두 점을 선택하는 경우는
순열과 조합
{F, K}에서 1가지
즉, 사각형의 개수는 4_1=4
Ú ~ Ü에서 넓이가 3인 사각형의 개수는
6+10+4=20이다. 506 ⑴3 ⑵6

⑴ 60=5_4_3이므로 °P¨=60을 만족시키는 r의 값은 r=3이다.


⑵ 12_ÇPª=ÇP¢에서
12_n(n-1)=n(n-1)(n-2)(n-3)
12=(n-2)(n-3)에서
12=4_3이므로 n-2=4 yy TIP
∴ n=6

TIP
12=(n-2)(n-3)에서
nÛ`-5n-6=(n-6)(n+1)=0
으로 식을 정리해도 n=6을 구할 수 있다.

참고

ÇP¨의 정의에 의하여 n, r는 모두 자연수이고 n¾r이다.

507 ③

① ¢P¼=1 (참)
n!
② ÇP¨= (참)
(n-r)!
③ ¢PÁ=4, ¢P£=4_3_2=24이므로 ¢PÁ+¢P£ (거짓)
④ 0!=1이므로 3!_0!=3!=3_2_1=6 (참)
n!
⑤ ÇP¨=
(n-r)!
=n(n-1)(n-2)_y_(n-r+1) (참)
따라서 선지 중 옳지 않은 것은 ③이다.

508 ⑤

서로 다른 n개에서


첫 번째 두 번째 세 번째 r번째
자리 자리 자리 자리
[그림1]
[그림1]의 첫 번째 자리에 올 대상을 정하는 방법은
n 가지이다.


첫 번째 두 번째 세 번째 r번째
자리 자리 자리 자리
[그림2]
이때, [그림2]의 나머지 자리에 남은 것을 일렬로 배열하는 경우의
수는 ÇÐÁP¨ÐÁ 이다.
따라서 서로 다른 n개에서 r개를 택하여 일렬로 배열하는 순열의
수 ÇP¨는 곱의 법칙에 의하여
ÇP¨= n _ ÇÐÁP¨ÐÁ 이다.

128 수학 <하>
∴ ㈎: n, ㈏: ÇÐÁP¨ÐÁ 경우의 수는
¢Pª=12
참고
o a를 한 문자로 생각하여 3개의 문자를 일렬로 나열하는 경우의
등식 ÇP¨=n_ÇÐÁP¨ÐÁ의 증명은 다음과 같다.
수는 3!=6
(n-1)!
n_ÇÐÁP¨ÐÁ=n_ o와 a의 자리를 바꾸는 경우의 수는 2!=2
{n-1-(r-1)}!
n! 따라서 구하는 경우의 수는
=
(n-r)! 12_6_2=144이다.
=ÇP¨

509 120 514 72

노래 3팀과 춤 3팀이 각각 공연할 순서를 정하는 방법의 수는


서로 다른 5개의 인형을 일렬로 나열하는 방법의 수는
3!_3!=6_6=36
5!=5_4_3_2_1=120
가장 먼저 공연하는 팀이 노래 팀인지 춤 팀인지를 선택하는 경우의
수는 2이므로 구하는 방법의 수는

510 10 36_2=72

ÇP£=n(n-1)(n-2)이므로
n(n-1)(n-2)=720에서
720=10_9_8 yy TIP 515 ③

∴ n=10 힙합 4곡의 순서를 정하는 방법의 수는


4!=24
TIP 힙합 힙합 힙합 힙합
힙합 힙합 힙합 힙합
n(n-1)(n-2)=720에서
nÜ`-3nÛ`+2n-720=0, (n-10)(nÛ`+7n+72)=0 이때, 의 5개의 자리 중 3개를 택하여 발라드 3곡의 순서를 정하는
으로 식을 정리해도 n=10을 구할 수 있다. 방법의 수는
°P£=60
따라서 구하는 경우의 수는
24_60=1440이다.
511 ④ 경


양 끝 적어도 한 자리에 남학생이 서는 경우의 수는 전체 6명이

일렬로 서는 경우의 수에서 양 끝에 모두 여학생이 서는 경우의
수를 뺀 것과 같다. 516 96

전체 6명의 학생이 일렬로 서는 경우의 수는 6!이다. 조건을 만족시키려면 양 끝의 두 수가 4, 5 또는 3, 5이어야 한다.


양 끝에 모두 여학생이 서는 경우의 수는 Ú 양 끝의 두 수가 4, 5인 경우
여학생 4명 중 2명을 뽑아 양 끝에 배열하는 경우의 수가 ¢Pª이고, 나머지 네 수 0, 1, 2, 3을 배열하는 방법의 수는
이를 제외한 나머지 4명을 가운데 배열하는 경우의 수가 4!이므로 4!=24이고,
¢Pª_4! 양 끝의 4, 5가 서로 자리를 바꿀 수 있으므로 숫자열의 개수는
따라서 구하는 경우의 수는 24_2!=48
6!-¢Pª_4!=4!(30-12)=432 Û 양 끝의 두 수가 3, 5인 경우
Ú과 같은 아이디어로 숫자열의 개수는 48
Ú, Û에서 구하는 경우의 수는
512 ④
48_2=96이다.
1등, 2등, 3등 중에서 2개의 순위에 A, B가 포함되어야 하므로
두 사람의 순위를 정하는 방법의 수는 £Pª=6
이고 나머지 4명을 나머지 등수로 정하면 되므로 구하는 경우의 수는
6_4!=144 517 ⑤

부모가 같은 열에 앉는 경우를 케이스 분류하면 다음과 같다.


Ú 부모가 1열에 앉는 경우
513 144
부모가 1열에 앉는 경우의 수는 2!=2
o와 a 사이에 나머지 4개의 문자 중 2개의 문자를 선택하여 나열하는 자녀 3명이 2열에 앉는 경우의 수는 3!=6

Ⅵ. 경우의 수 129
따라서 경우의 수는 2_6=12이다.
Û 부모가 2열에 앉는 경우
522 5

부모가 2열에 앉는 경우의 수는 £Pª=6 등식 Á¼C¨Ðª=Á¼Cª¨Ð£을 만족시키려면


자녀 3명이 나머지 자리에 앉는 경우의 수는 3!=6 r-2=2r-3 또는 (r-2)+(2r-3)=10이어야 한다.
따라서 경우의 수는 6_6=36이다. Ú r-2=2r-3인 경우
Ú, Û에서 구하는 경우의 수는 r=1이므로
12+36=48이다. r-2<0, 2r-3<0이다.
따라서 이 경우는 성립하지 않는다.
Û (r-2)+(2r-3)=10인 경우
3r=15, r=5
518 ⑤
주어진 등식은 Á¼C£=Á¼C¦이므로 성립한다.
X에서 Y로의 함수의 개수는 정의역 X의 각각의 원소가 공역 Y의 Ú, Û에서 r=5이다.
원소 중에서 하나를 선택하면 되므로
a=5_5_5=125
X에서 Y로의 일대일함수의 개수는 공역 Y의 원소 중에서 서로 523 ③

다른 3개를 선택해서 정의역 X의 원소와 대응시켜 주면 되므로 집합 X의 원소 중 홀수는 1, 3, 5, 7, 9로 5개이므로


b=°P£=5_4_3=60 이 중 3개를 선택하는 방법의 수는 °C£=10
∴ a+b=125+60=185 또한 2개 이상의 짝수를 원소로 가지므로 구하는 부분집합의 개수는
10_(¢Cª+¢C£+¢C¢)=10_(6+4+1)=110

519 ②
524 ③
조건 ㈏에 의하여 f(1), f(3)은 3, 5이므로
7개의 문자 중에서 a, c를 반드시 포함해야 하므로
f(1), f(3)을 3, 5에 대응시키는 방법의 수는
b, d, e, f, g 중에서 3개를 선택하는 방법의 수는 °C£=10이다.
2!=2이고, f(2), f(4), f(5)와 1, 2, 4를
선택한 5개의 문자를 일렬로 나열하는 전체 경우의 수는 5!이고,
대응시키는 경우의 수는 3!=6이다.
이 중에서 a와 c가 서로 이웃한 경우의 수는 2!_4!이므로
따라서 함수의 개수는 2_6=12이다.
구하는 문자열의 개수는
10_(5!-2!_4!)=10_4!_(5-2)=720
다른 풀이
520 ③
7개의 문자 중에서 a, c를 반드시 포함해야 하므로
① ÇC¼=1 (거짓) b, d, e, f, g 중에서 3개를 선택하여 나열하는 방법의 수는
¦P¢ °P£=60
② ¦C¢= (거짓)
4!
이때, 이 세 문자 사이와 양 끝을 포함하여 총 4자리 중에서
③ ¤Cª=¤C¢ (참)
2자리를 선택하여 a, c를 배열하는 방법의 수는
④ £Pª=6, £Cª=3이므로 £Pª=£Cª+3 (거짓)
¢Pª=12이므로 구하는 문자열의 개수는
⑤ ¥P¦=8! (거짓)
60_12=720이다.
따라서 선지 중 옳은 것은 ③이다.

525 128

521 11
원소의 개수가 1인 부분집합의 개수는
n(n-1)(n-2) ¥CÁ=8
ÇPª=n(n-1), ÇC£= 이므로
3_2_1 원소의 개수가 3인 부분집합의 개수는
3_ÇPª=2_ÇC£에서 8_7_6
¥C£= =56
n(n-1)(n-2) 3!
3_n(n-1)=2_
3_2_1 원소의 개수가 5인 부분집합의 개수는
n-2=9 (∵ n¾3) ¥C°=¥C£=56
∴ n=11 원소의 개수가 7인 부분집합의 개수는
참고 ¥C¦=¥CÁ=8
따라서 구하는 원소의 개수가 홀수인 부분집합의 개수는
ÇC¨의 정의에 의하여 n, r는 자연수이고 n¾r이다.
8+56+56+8=128이다.

130 수학 <하>
Û 모두 2학년인 경우
526 ④
°C¢=°CÁ=5
7개의 점 중 서로 다른 3개의 점을 선택하는 경우의 수는 Ü 모두 3학년인 경우
7_6_5 6_5
¦C£= =35 ¤C¢=¤Cª= =15
3! 2!
이때, 지름 위의 3개의 점을 꼭짓점으로 하는 삼각형은 존재하지 Ú ~ Ü에서 구하는 경우의 수는
않는다. 1+5+15=21이다.
지름 위의 4개의 점 중 3개의 점을 선택하는 경우의 수는
¢C£=¢CÁ=4
따라서 구하는 삼각형의 개수는 531 44
35-4=31이다.
1부터 9까지의 자연수 중에는 홀수가 5개, 짝수가 4개 포함되어
있다.
선택한 6장의 카드에 적힌 수의 합이 홀수가 되려면
527 ④ 6개의 숫자 중에서 홀수의 개수가 3 또는 5가 되어야 한다.
Ú 홀수의 개수가 3인 경우
가로 방향의 4개의 평행선 중 2개를 선택하고 세로 방향의 5개의
홀수 3개, 짝수 3개를 선택하는 방법의 수는
평행선 중 2개를 선택하면 선택한 4개의 평행선으로 둘러싸인
°C£_¢C£=40
평행사변형이 만들어진다.
Û 홀수의 개수가 5인 경우
따라서 구하는 평행사변형의 개수는
홀수 5개, 짝수 1개를 선택하는 방법의 수는
4_3 5_4
¢Cª_°Cª= _ =60이다. °C°_¢CÁ=4
2! 2!
Ú, Û에서 구하는 경우의 수는
40+4=44이다.

528 ④

9명 중 회장 1명을 뽑는 경우의 수는 532 19


»CÁ=9
회원 n명이 서로 한 번씩 악수하는 방법의 수는 회원 n명 중 2명을
이때, 나머지 8명 중 부회장 2명을 뽑는 경우의 수는
뽑는 방법의 수와 같으므로
8_7 경
¥Cª= =28 n(n-1)
2! ÇCª= =171 우
2! 의
따라서 구하는 경우의 수는
n(n-1)=342=19_18 수
9_28=252이다.
∴ n=19

529 350
533 ②

주재료의 5종류에서 2종류를 선택하는 경우의 수는 10개의 팀 중 경기를 할 서로 다른 두 팀을 선택하는 방법의 수는


5_4 10_9
°Cª= =10 Á¼Cª= =45
2! 2!
계란은 반드시 선택하므로 나머지 부재료 7종류에서 3종류를 각 팀이 6회씩 경기를 하므로 구하는 총 경기의 수는
선택하는 경우의 수는 6_45=270이다.
7_6_5
¦C£= =35
3!
따라서 구하는 방법의 수는
10_35=350이다.
534 ⑤

티셔츠 5벌 중에서 3벌을 선택하는 방법의 수는


°C£=°Cª=10
원피스 4벌 중에서 2벌을 선택하는 방법의 수는
530 21
¢Cª=6
선택한 4명의 학년에 따라 경우를 나누면 다음과 같다. 선택된 티셔츠 3벌과 원피스 2벌을 합쳐 총 5벌을 일렬로 진열하는
Ú 모두 1학년인 경우 방법의 수는 5!이므로 구하는 방법의 수는
¢C¢=1 10_6_5!=7200이다.

Ⅵ. 경우의 수 131
Ú 4인실과 6인실에 각각 4명/4명 배정하는 경우
535 ②
1
¥C¢_¢C¢_ _2!=70
구하는 경우의 수는 전체 사탕 중에서 2개를 뽑는 경우의 수에서 2!
같은 맛 사탕으로 2개를 꺼내는 경우를 제외시키면 된다. Û 4인실과 6인실에 각각 3명/5명 배정하는 경우
전체 사탕 15개 중에서 2개의 사탕을 꺼내는 방법의 수는 ¥C£_°C°=56
15_14 Ü 4인실과 6인실에 각각 2명/6명 배정하는 경우
Á°Cª= =105
2! ¥Cª_¤C¤=28
같은 맛 사탕으로 2개를 뽑는 방법의 수는 Ú ~ Ü에서 구하는 방법의 수는
4_3 5_4 3_2 3_2 70+56+28=154이다.
¢Cª+°Cª+£Cª+£Cª= + + +
2! 2! 2! 2!
=6+10+3+3=22
따라서 구하는 경우의 수는 540 ②

105-22=83이다.
다음과 같이 주어진 대진표에서 배정받을 수 있는 6개의 자리를
왼쪽에서부터 각각 a, b, c, d, e, f 라 하자.

536 64

사용할 수 있는 쿠키는 7개이므로 세트상품 안의 초콜릿의 개수는 a b c d e f


3 이상 9 이하의 홀수이다. 이와 같은 대진표에서 a, b끼리, c, d끼리, e, f 끼리는 서로 구분이
세트상품에 초콜릿을 각각 3개, 5개, 7개, 9개 넣을 때 없고, a와 b, c와 d의 두 조도 서로 구분이 없다.
쿠키를 각각 7개, 5개, 3개, 1개 넣어야 한다. e, f에 들어갈 반을 정하는 경우의 수가 ¤Cª=15
이때, 초콜릿은 모두 서로 같은 제품이므로 쿠키를 먼저 고르고 나머지 4개의 반을 a와 b, c와 d의 구분이 없는 두 조로 나누는
나머지 개수를 초콜릿으로 채우면 된다. 1
경우의 수가 ¢Cª_ªCª_ =3
따라서 만들 수 있는 세트상품의 종류는 2!
¦C¦+¦C°+¦C£+¦CÁ=1+21+35+7=64(가지)이다. 따라서 구하는 방법의 수는
15_3=45이다.

537 ④
541 풀이 참조
볼펜 8개를 4개씩 두 묶음으로 나누는 방법의 수는
(n-1)! r(n-1)!
1 8_7_6_5 1 ÇÐÁP¨+r_ÇÐÁP¨ÐÁ= +
¥C¢_¢C¢_ = _1_ =35 (n-r-1)! (n-r)!
2! 4! 2_1
이때, 두 묶음을 두 사람에게 나누어주는 방법의 수는 (n-1)!
= {(n-r)+r}
(n-r)!
2!=2
n!
따라서 구하는 방법의 수는 =
(n-r)!
35_2=70이다. =ÇP¨
따라서 등식 ÇÐÁP¨+r_ÇÐÁP¨ÐÁ=ÇP¨가 성립한다.

538 630
순열의 수의 정의를 이용하여
채점 요소 배점

7명을 2명, 2명, 3명의 조로 나누는 방법의 수는 (n-1)! r(n-1)!


ÇÐÁP¨= , r_ÇÐÁP¨ÐÁ= 50%
1 7_6 5_4 1 (n-r-1)! (n-r)!
¦Cª_°Cª_£C£_ = _ _1_
2! 2! 2! 2_1 로 나타내기
=105 좌변을 정리하여 등식이 성립함을 보이기 50%
이때, 세 개의 조를 서로 다른 3개의 방에 배정하는 방법의 수는
3!=6
따라서 구하는 방법의 수는 542 ④
105_6=630이다.
성인이 앉을 수 있는 자리는
A1, B1 또는 A1, B4 또는 A3, B1 또는 A3, B4로 4가지,
이 두 자리에 성인 2명이 앉는 방법의 수가 2!,
539 ③
나머지 5개의 자리에 어린이가 앉는 방법의 수는 5!
4인실과 6인실에 배정하는 사람 수는 각각 따라서 구하는 방법의 수는
4명/4명 또는 3명/5명 또는 2명/6명이다. 4_2!_5!=960이다.

132 수학 <하>
543 ⑤ 546 ④

그림과 같이 앉을 수 있는 좌석에 각각 1부터 7까지의 자연수를 300보다 크고 4000보다 작은 자연수 중 세 자리 자연수와 네 자리


번호로 지정하자. 자연수로 나누어서 구해 보면 다음과 같다.
무대
Ú 세 자리 자연수
백의 자리의 수는 3 또는 4로 2가지,
1열 1 2 3
2열
십의 자리, 일의 자리의 수는 1, 2, 3, 4 중 백의 자리의 수를
3열 4 5 6 제외한 3개 중 2개를 뽑아 나열하면 되므로 £Pª=6(가지)
4열 7 그러므로 자연수의 개수는 2_6=12
A, B가 이웃하게 앉으려면 1열에서 1, 2 또는 2, 3에 앉거나 Û 네 자리 자연수
3열에서 4, 5에 앉아야 하고 각각의 경우에서 A, B가 서로 자리를 천의 자리의 수가 1 또는 2 또는 3으로 3가지,
바꿀 수 있으므로 백의 자리, 십의 자리, 일의 자리의 수는 1, 2, 3, 4 중 천의
A, B가 자리를 정하는 방법의 수는 3_2!=6 자리의 수를 제외한 3개를 나열하면 되므로 £P£=6(가지)
나머지 5개의 자리에 3명이 자리를 정하는 방법의 수는 °P£=60 그러므로 자연수의 개수는 3_6=18
따라서 구하는 방법의 수는 Ú, Û에서 구하는 자연수의 개수는
6_60=360이다. 12+18=30이다.

544 ④

여학생 두 명을 한 묶음으로 보고 이 한 묶음과 빈 의자 2개를


547 ④

여 여 , ○, ○과 같이 나타내면 이를 나열하는 방법은 다음과 조건 ㈎에 의해 이웃한 자리에 홀수와 짝수가 교대로 나타나야 하고
같이 3가지이다. 조건 ㈏에 의해 각 자리의 수 중에서 홀수는 홀수 개 있어야 한다.
그러므로 만의 자리의 수부터 일의 자리의 수까지 다음과 같은
여 여 ○○, ○ 여 여 ○, ○○ 여 여
순서로 나타내야 한다.
이때, 남학생은 이웃하지 않아야 하므로 위에서 두 여학생의 묶음과
홀수 짝수 홀수 짝수 홀수
빈 의자 사이 또는 양 끝 자리에 앉아야 한다.
그러므로 남학생 2명이 앉을 자리를 정하는 방법의 수는 ¢Pª=12 9 이하의 자연수 중에서 홀수, 짝수는 각각 5개, 4개이므로 홀수
두 여학생이 서로 자리를 바꾸는 방법의 수가 2 중에서 3개, 짝수 중에서 2개를 선택하여 나열하면 된다.
따라서 구하는 방법의 수는 따라서 구하는 방법의 수는 경

3_12_2=72이다. °P£_¢Pª=60_12=720이다. 의

545 ③

구하는 경우의 수는 4명이 임의로 핸드폰을 되돌려 받는 경우의


548 ④

수에서 모두 다른 사람의 핸드폰을 받는 경우의 수를 뺀 것과 같다. 조건 ㈎에 의하여 2, 4, 6이 서로 이웃하지 않아야 하고,


4명이 임의로 핸드폰을 되돌려 받는 경우의 수는 4!=24이고, 조건 ㈏에 의하여 3, 5가 서로 이웃하지 않아야 한다.
4명의 응시생을 각각 a, b, c, d라 할 때, 모두 다른 사람의 핸드폰을 6개의 수를 나열할 칸에 짝수가 들어갈 칸을 로 표현하면 서로
받는 경우를 수형도를 통해서 일일이 세면 다음과 같다. 이웃하지 않는 경우는 다음과 같다.
a b c d Ú 또는
b a d c 에 2, 4, 6을 배열하는 경우의 수는 3!
c d a a ab1,b 3, 5를 배열할
이때, 나머지 칸에 a a때
b 3,
b 5는 항상 이웃하지
a b a b
d a c 않게 되므로 나머지 수를 배열하는 경우의 수는 3!
c a d b ∴ 2_3!_3!=72
d a b
Û a ab b 또는 a ab b
b a
d a b c 에 2, 4, 6을 배열하는 경우의 수는 3!
c a b 나머지 칸에 1, 3, 5를 배열하는 경우의 수는 3!이고, 이 중
b a a, b자리에 각각 3, 5 또는 5, 3이 오는 경우를 제외하면 되므로
즉, 3_3=9(가지) 2_3!_(3!-2)=48
따라서 구하는 경우의 수는 Ú, Û에서 구하는 방법의 수는
24-9=15이다. 72+48=120이다.

Ⅵ. 경우의 수 133
549 ④ A조 B조 C조
교실 화단 계단 계단
3의 배수이려면 각 자리의 숫자의 합이 3의 배수이어야 한다.
계단 계단 교실 화단
1, 2, 3, 4, 5, 6의 여섯 개의 수 중 네 수의 합이 3의 배수가 되는 교실 화단
화단 계단 계단 교실
경우는 1, 2, 3, 6 또는 1, 2, 4, 5 또는 1, 3, 5, 6 또는 2, 3, 4, 6
계단 교실 화단 계단
또는 3, 4, 5, 6의 5가지이고, 이때 각각 네 수가 나오는 순서를
정하는 방법의 수가 4!이므로 A조가 화단 청소, 계단 청소를 맡거나 교실 청소, 계단 청소를

a=5_4!=120 맡는 경우도 동일하게 셀 수 있으므로 방법의 수는 3_4=12

4의 배수이려면 일의 자리와 십의 자리까지 보았을 때 4의 Ú, Û에서 구하는 방법의 수는

배수이어야 하므로 가능한 경우는 12, 16, 24, 32, 36, 52, 56, 64로 9+12=21이다.

8가지이고, 이때 각각 천의 자리의 수와 백의 자리의 수를 정하는


경우의 수가 ¢Pª이므로 552 32
b=8_¢Pª=96
두 자연수 a, b의 최대공약수가 5!이므로 a, b를 각각
∴ a+b=120+96=216
5!_p, 5!_q (p와 q는 서로소)라 하자.
두 자연수 a, b의 최소공배수가 15!이므로
15!
p_q= =6_7_8_y_15
5!
550 ④
를 만족시켜야 한다.
사전식 배열로 순서대로 배열했을 때, 430번째에 가까워지도록 이때, 6_7_8_y_15를 소인수분해하면
찾으면 다음과 같다. 6_7_8_y_15=2¡`_3Þ`_5Û`_7Û`_11_13이므로
A 인 문자열의 개수는 5!=120 p, q가 서로소가 되려면 2¡`, 3Þ`, 5Û`, 7Û`, 11, 13이 각각 p 또는 q
B 인 문자열의 개수는 5!=120 중에서 어느 한쪽만의 약수가 되어야 하므로 p, q를 정하는 경우의
C 인 문자열의 개수는 5!=120 수는
DA 인 문자열의 개수는 4!=24 2_2_2_2_2_2=64
DB 인 문자열의 개수는 4!=24 이때, a+b이므로 a>b를 만족시키는 순서쌍 (a, b)의 개수는
DCA 인 문자열의 개수는 3!=6 64
=32이다.
2
DCB 인 문자열의 개수는 3!=6
DCE 인 문자열의 개수는 3!=6
DCFA 인 문자열의 개수는 2!=2 553 ④
DCFB 인 문자열의 개수는 2!=2
ㄱ. ÇPÇ=n!, ÇCÇ=1 ∴ ÇPÇ+ÇCÇ (거짓)
이때, 120_3+24_2+6_3+2_2=430이므로
n! (n+1)!
DCFBEA가 430번째 문자열이다. ㄴ. (n+1)ÇP¨=(n+1)_ = =Ç*ÁP¨*Á (참)
(n-r)! (n-r)!
ㄷ. ÇP¨=ÇC¨_r! (거짓)
ㄹ. Ç*ÁC¨*Á=Ç*ÁC(n+1)-(r+1)=Ç*ÁCÇШ (참)
ㅁ. ÇP¼=1, ÇC¼=1, 0!=1 ∴ ÇP¼=ÇC¼=0! (참)
551 21
ㅂ. ÇC¨ÐÁ+ÇC¨=
n!
+
n!
A조를 기준으로 케이스 분류를 하면 다음과 같다. (r-1)!(n-r+1)! r!(n-r)!
n! 1 1
Ú A조가 같은 종류의 청소를 하는 경우 = _{ + }
(r-1)!(n-r)! n-r+1 r
A조가 교실 청소를 한다고 할 때 B, C조가 청소를 나누어 맡는
n! n+1
경우는 다음과 같다. = _
(r-1)!(n-r)! (n-r+1)_r
A조 B조 C조 (n+1)!
= =Ç*ÁC¨ (참)
화단 화단 계단 계단 r!(n-r+1)!
교실 교실 계단 계단 화단 화단 따라서 옳은 것은 ㄴ, ㄹ, ㅁ, ㅂ으로 4개이다.
화단 계단 화단 계단
A조가 화단 청소, 계단 청소를 맡는 경우도 동일하게 셀 수
있으므로 방법의 수는 3_3=9 554 풀이 참조

Û A조가 서로 다른 종류의 청소를 하는 경우 ÇÐÁC¨ÐÁ+ÇÐÁC¨


A조가 교실 청소와 화단 청소를 맡는 경우 B, C조가 청소를 (n-1)! (n-1)!
= +
나누어 맡는 경우는 다음과 같다. (r-1)!(n-r)! r!(n-r-1)!

134 수학 <하>
(n-1)! 1 1 n(n-1)(n+1)=6_165
= _{ + }
(r-1)!(n-r-1)! n-r r n(n-1)(n+1)=2_3Û`_5_11
(n-1)! n n(n-1)(n+1)=9_10_11
= _
(r-1)!(n-r-1)! r(n-r)
∴ n=10
n!
=
r!(n-r)! 다른 풀이
=ÇC¨ f(n)=ÇCª+ÇC£=Ç*ÁC£이므로
따라서 등식 ÇÐÁC¨ÐÁ+ÇÐÁC¨=ÇC¨ 가 성립한다. (n+1)_n_(n-1) n(n-1)(n+1)
f(n)= = 이다.
채점 요소 배점
3! 6
조합의 수의 정의를 이용하여
f(n)=165에서
(n-1)! (n-1)! n(n-1)(n+1)
ÇÐÁC¨ÐÁ= , ÇÐÁC¨= 50% =165
(r-1)!(n-r)! r!(n-r-1)! 6
로 나타내기 n(n-1)(n+1)=6_165
좌변을 정리하여 등식이 성립함을 보이기 50% n(n-1)(n+1)=2_3Û`_5_11
n(n-1)(n+1)=9_10_11
∴ n=10

555 ①

함수 f의 치역이 1, 2는 반드시 원소로 가지고 3은 원소로 갖지 557 ③

않으므로 치역은 {1, 2} 또는 {1, 2, 4}이다. 주어진 삼차방정식의 세 근을 각각 a, 2, -2라 하면


Ú 치역이 {1, 2}인 경우 근과 계수의 관계에 의하여 세 근의 곱은
정의역의 원소 4개를 1개/3개 또는 2개/2개로 나누어서 치역의 144
a_2_(-2)=- =-12
원소에 하나씩 대응시키면 된다. 12
정의역의 원소를 1개/3개로 나누는 방법의 수는 이므로 a=3
¢CÁ_£C£=4 세 근의 합은
정의역의 원소를 2개/2개로 나누는 방법의 수는 9 3
3+2+(-2)= _ÇC¨, 3= _ÇC¨
12 4
1
¢Cª_ªCª_ =3 이므로 ÇC¨=4 yy ㉠
2!
이고, 치역의 원소에 대응시키는 방법의 수는 2!이므로 함수 f 의 2
3_2+2_(-2)+(-2)_3=- _ÇP¨
12 경
개수는 우
1
(4+3)_2!=14 -4=- _ÇP¨이므로 ÇP¨=24 yy ㉡ 의
6

Û 치역이 {1, 2, 4}인 경우 ÇP¨ 24
㉠에서 ÇC¨= =4, =4 (∵ ㉡)
정의역의 원소를 2개/1개/1개로 나누어서 치역의 원소에 하나씩 r! r!
대응시키면 된다. r!=6이므로 r=3이다.
정의역의 원소를 2개/1개/1개로 나누는 방법의 수는 ㉡에 대입하면
1 ÇP£=24=4_3_2이므로 n=4
¢Cª_ªCÁ_ÁCÁ_ =6
2! ∴ n+r=4+3=7
이고, 치역의 원소에 대응시키는 방법의 수는 3!이므로 함수 f 의
개수는
6_3!=36 558 ②

Ú, Û에서 구하는 함수 f의 개수는 2310을 소인수분해하면 2310=2_3_5_7_11이다.


14+36=50이다. Ú 1보다 큰 세 자연수의 곱으로 나타내는 경우
2, 3, 5, 7, 11이 각각 모두 소수이므로
이 5개의 소수를 3개의 조로 나누어 각각의 조에 속한 수를
556 10 곱하여 세 자연수를 만들면 된다.
2, 3, 5, 7, 11을 3개/1개/1개로 나누는 경우
f(n)=ÇCª+ÇC£
n(n-1) n(n-1)(n-2) 1
°C£_ªCÁ_ÁCÁ_ =10
= + 2!
2! 3!
n(n-1) 2, 3, 5, 7, 11을 2개/2개/1개로 나누는 경우
n-2
= {1+ } 1
2! 3 °Cª_£Cª_ÁCÁ_ =15
n(n-1)(n+1) 2!
= =165 이므로 방법의 수는 a=10+15=25
6

Ⅵ. 경우의 수 135
Û 합성수의 곱으로 나타내는 경우 나머지 1개의 원소는 ㉠, ㉣ 중 하나에 속하므로
합성수가 되려면 적어도 2개 이상의 소인수의 곱이어야 하므로 5_2=10
2, 3, 5, 7, 11을 3개/2개로 나누어서 각각을 곱하여 합성수를 Û n(A)=6인 경우
만들어야 한다. 6을 제외한 5개의 원소가 모두 ㉡에 포함되므로 경우의 수는 1이다.
방법의 수는 b=°C£_ªCª=10 Ú, Û에서 n(A)=5 또는 n(A)=6인 경우의 수는
Ú, Û에서 a+b=25+10=35이다. 10+1=11
따라서 두 집합 A, B의 순서쌍 (A, B)의 개수는
243-11=232이다.
559 ③

각 자리의 수가 모두 다르면서 a<b<c<d를 만족시켜야 하므로


천의 자리의 수 a에 따라서 자연수의 개수를 차례로 세보면 다음과
같다. 561 430

Ú a=1인 경우 전체집합 U와 두 부분집합 A, B에 대하여 다음과 같이 각각의


b, c, d는 1을 제외한 8개의 자연수 중에서 3개를 선택하면 작은 원소가 포함되는 부분은 ㉠, ㉡, ㉢, ㉣ 중에서 하나이고
순서대로 b, c, d가 되어야 하므로 자연수의 개수는 2²(A-B)에서 2는 ㉡에 속하지 않고,
¥C£=56 n(B-A)=3에서 ㉣에는 3개의 원소가 포함되어야 한다.
Û a=2인 경우 U
b, c, d는 1, 2를 제외한 7개의 자연수 중에서 3개를 선택하면 ㉠ A B
작은 순서대로 b, c, d가 되어야 하므로 자연수의 개수는 ㉡ ㉢ ㉣
¦C£=35
Ü a=3인 경우
Ú 2<(B-A)인 경우
b, c, d는 1, 2, 3을 제외한 6개의 자연수 중에서 3개를 선택하면
2 이외에 집합 B-A에 포함될 2개의 원소를 선택하는 방법의
작은 순서대로 b, c, d가 되어야 하므로 자연수의 개수는
수는 °Cª=10이고
¤C£=20
나머지 3개의 원소는 ㉠, ㉡, ㉢ 중에서 하나에 포함되면 된다.
Ú ~ Ü에서 천의 자리의 수가 1 또는 2 또는 3인 자연수의 개수는
이때, 나머지 3개의 원소가 모두 ㉠에 포함되면 A=∆이 되므로
56+35+20=111이다.
10_(3_3_3-1)=260(가지)
천의 자리의 수가 4인 수 중에서 작은 순서대로 나열했을 때 4번째
Û 2²(B-A)인 경우
수가 115번째 자연수이다.
B-A에 포함될 3개의 원소를 선택하는 방법의 수는
천의 자리의 수가 4, 백의 자리의 수가 5인 자연수를 작은 순서로
°C£=10이고
나열하면
2는 ㉠, ㉢ 중에서 하나에 포함되어야 하므로 2가지,
4567, 4568, 4569, 4578, y
나머지 2개의 원소는 ㉠, ㉡, ㉢ 중에서 하나에 포함되면 된다.
이므로 115번째 자연수는 4578이다.
이때, 2와 나머지 2개의 원소가 모두 ㉠에 포함되면 A=∆이
되므로
10_(2_3_3-1)=170(가지)
560 ②
Ú, Û에서 두 집합 A, B의 순서쌍 (A, B)의 개수는
전체집합 U와 두 부분집합 A, B에 대하여 다음과 같이 각각의 260+170=430이다.
원소가 포함되는 부분은 ㉠, ㉡, ㉢, ㉣ 중에서 하나이다.
U
㉠ A B

㉡ ㉢ ㉣
562 ③

치역이 {1, 2, 3}이어야 하므로 이에 대응하는 정의역의 원소는


원소 6은 반드시 ㉢에 속하고
3개/1개/1개 또는 2개/2개/1개로 나누어 공역에
나머지 원소 1, 2, 3, 4, 5는 각각 ㉠, ㉡, ㉣ 중 하나에 속하므로
대응되어야 한다.
3_3_3_3_3=243
Ú 3개/1개/1개로 나누는 경우
이때, n(A)<5이므로 n(A)=5 또는 n(A)=6인 경우를
1
제외시켜야 한다. °C£_ªCÁ_ÁCÁ_ =10
2!
Ú n(A)=5인 경우
Û 2개/2개/1개로 나누는 경우
6을 제외한 5개의 원소 중에서 ㉡에 포함될 4개의 원소를
1
선택하는 방법의 수는 °C¢=5이고, °Cª_£Cª_ÁCÁ_ =15
2!

136 수학 <하>
Ú, Û에서 정의역의 원소를 3묶음으로 나누는 방법의 수는
10+15=25이고,
564 31

이 세 묶음을 치역의 원소 1, 2, 3에 하나씩 대응시키는 방법의 수가 네 수를 더해서 12가 되는 경우는


3!=6이므로 구하는 함수 f의 개수는 4+4+3+1, 4+3+3+2, 4+4+2+2, 3+3+3+3
25_6=150이다. 으로 4가지이다.
Ú f(1), f(2), f(3), f(4)의 값이 1, 3, 4, 4인 경우
정의역의 원소 중에서 함숫값 1, 3에 대응되는 원소를 결정하는
방법의 수는 ¢Pª=12이고
나머지 원소는 모두 4에 대응되면 되므로 방법의 수는 1이다.
563 95 그러므로 조건을 만족시키는 함수의 개수는
12_1=12이다.
f(3)¾7이고, f(1)> f(2)> f(3)이므로
Û f(1), f(2), f(3), f(4)의 값이 2, 3, 3, 4인 경우
f(3)É8에서 f(3)의 값은 7 또는 8이다.
Ú인 경우와 같은 아이디어로 함수의 개수는 12이다.
Ú f(3)=7인 경우
f
Ü f(1), f(2), f(3), f(4)의 값이 2, 2, 4, 4인 경우
X Y 정의역의 원소 중에서 함숫값 2에 대응되는 2개의 원소를 결정하는
1 1
2 방법의 수는 ¢Cª=6이고
§C£
2 3 나머지 원소는 모두 4에 대응되면 되므로 방법의 수는 1이다.
4
3
5 그러므로 조건을 만족시키는 함수의 개수는
4 6
7 6_1=6이다.
5 8 Ý f(1), f(2), f(3), f(4)의 값이 3, 3, 3, 3인 경우
6 9 £C™
10 정의역의 모든 원소가 3에 대응되므로 함수의 개수는 1이다.

정의역의 원소 중 1, 2는 공역의 원소 8, 9, 10 중에서 대응되어야 Ú ~ Ý에서 구하는 함수의 개수는

하고, 정의역의 원소 중 4, 5, 6은 공역의 원소 1, 2, 3, 4, 5, 6 12+12+6+1=31이다.

중에서 대응되어야 한다.


f(1), f(2)의 값을 결정하는 방법의 수는 £Cª=3이고, 선택된
두 수는 큰 순서대로 f(1), f(2)에 각각 대응되므로 1가지이다.
f(4), f(5), f(6)의 값을 결정하는 방법의 수는 ¤C£=20이고,
선택된 세 수는 큰 순서대로 f(4), f(5), f(6)에 각각 대응되므로
565 525

1가지이다. 조건 ㈎에서 집합 A의 원소 중에서 f(1), f(2)가 될 값을 선택하는 우

따라서 함수 f의 개수는 3_20=60 방법의 수는 ¤Cª=15이고, 수
Û f(3)=8인 경우 선택된 두 수는 작은 순서대로 f(1), f(2)에 각각 대응되어야
f 하므로 1가지이다.
X Y
1 즉, 15_1=15
1
2 조건 ㈏에서 f(3)< f(4)< f(5)É f(6)을 만족시키려면
2 3 ßC£
4 f(3)< f(4)< f(5)< f(6) 또는 f(3)< f(4)< f(5)= f(6)이다.
3
5
4 6 Ú f(3)< f(4)< f(5)< f(6)인 경우
7 f(3), f(4), f(5), f(6)이 될 값을 선택하는 방법의 수는
5 8
6 9 ™C™ ¤C¢=15이고,
10
선택된 네 수는 작은 순서대로 f(3), f(4), f(5), f(6)에 각각
정의역의 원소 중 1, 2는 공역의 원소 9, 10 중에서 대응되어야 대응되어야 하므로 1가지이다.
하고, 정의역의 원소 중 4, 5, 6은 공역의 원소 1, 2, 3, 4, 5, 6, 7 즉, 15_1=15
중에서 대응되어야 한다. Û f(3)< f(4)< f(5)= f(6)인 경우
f(1), f(2)의 값을 결정하는 방법의 수는 ªCª=1이고, 선택된 f(3), f(4), f(5)가 될 값을 선택하는 방법의 수는
두 수는 큰 순서대로 f(1), f(2)에 각각 대응되므로 1가지이다. ¤C£=20이고,
f(4), f(5), f(6)의 값을 결정하는 방법의 수는 ¦C£=35이고, 선택된 세 수는 작은 순서대로 f(3), f(4), f(5)에 각각
선택된 세 수는 큰 순서대로 f(4), f(5), f(6)에 각각 대응되므로 대응되어야 하고 f(5)= f(6)이므로 1가지이다.
1가지이다. 즉, 20_1=20
따라서 함수 f의 개수는 1_35=35 Ú, Û에서 f(3), f(4), f(5), f(6)의 값을 정하는 방법의 수는
Ú, Û에서 구하는 함수 f의 개수는 15+20=35이므로 구하는 함수 f의 개수는
60+35=95이다. 15_35=525이다.

Ⅵ. 경우의 수 137
12_11_10
566 ② ÁªC£=
3!
=220

이 도형의 선분으로 둘러싸인 직사각형의 개수는 Ú 십이각형과 두 변을 공유하는 경우 각 꼭짓점마다 한 개씩 생기므


가로 선 4개 중에서 2개, 세로 선 5개 중에서 2개를 선택하면 선택한 로 12(개)이다.
4개의 선분으로 둘러싸인 직사각형이 만들어지므로
¢Cª_°Cª=60
이 중 정사각형인 것의 개수를 구하면
한 변의 길이가 1인 정사각형은 12개,
한 변의 길이가 2인 정사각형은 6개,
한 변의 길이가 3인 정사각형은 2개이다.
Û 십이각형과 한 변만 공유하는 경우 각 변마다 8개씩 생기므로
따라서 구하는 정사각형이 아닌 직사각형의 개수는
12_8=96(개)이다.
60-(12+6+2)=40이다.


567 ④ …

선분 PQ와 호 PQ 위에 P, Q가 아닌 점이 각각 a개, (12-a)개


있으므로 P, Q가 아닌 반원 위의 점의 개수는 a+12-a=12이다.
Ú, Û에서 구하는 삼각형의 개수는
12개의 점 중에서 2개를 선택하면 선택한 2개의 점을 지나는 직선이
220-(12+96)=112이다.
만들어지므로
ÁªCª=66
이때, 선분 PQ 위의 점 중에서 2개의 점을 고르면 만들어지는
직선이 모두 동일하므로 한 가지로 세야 한다.
570 ②

선분 PQ 위의 a개의 점 중에서 2개를 선택하는 방법의 수는 ŒCª 두 사람이 동시에 수강하는 수업의 개수에 따라서 경우를 나누면
따라서 만들어지는 직선의 개수는 다음과 같다.
66-ŒCª+1=46, ŒCª=21 Ú 동시에 수강하는 수업이 1개인 경우
a(a-1) 두 사람이 동시에 수강하는 수업을 고르는 경우의 수가
=21, a(a-1)=42=7_6
2! °CÁ=5이고
∴ a=7 나머지 4개의 수업 중에서 A, B가 들을 수업을 한 가지씩 더
고르는 경우의 수가
¢CÁ_£CÁ=12이므로 방법의 수는
568 ②
5_12=60
원 위의 점 중에서 4개의 점 A, C, F, I를 선택했다고 하자. 네 점 Û 동시에 수강하는 수업이 없는 경우
A, C, F, I로 만들어지는 두 선분이 원의 내부에서 교점을 가지는 5개의 수업 중에서 A가 들을 수업을 고르는 경우의 수가
경우는 다음과 같이 1가지뿐이다. °Cª=10이고
A 나머지 3개의 수업 중에서 B가 들을 수업을 고르는 경우의 수가
J B
£Cª=3이므로 방법의 수는
C 10_3=30
I
Ú, Û에서 구하는 경우의 수는
H D 60+30=90이다.

G E
F
즉, 원 위의 10개의 점 중에서 4개를 선택하면 원의 내부에서 교점을 571 ③

갖도록 두 선분을 그리는 방법은 오직 1가지이므로 구하는 방법의 카메라를 진열할 6개의 자리 중에서 2개의 자리를 골라 왼쪽에
수는 크기가 가장 큰 A사 카메라를, 오른쪽에 크기가 가장 작은 B사
10_9_8_7 카메라를 나열하면 되므로 경우의 수는
Á¼C¢= =210
4! ¤Cª=15
나머지 4개의 자리에 나머지 4대의 카메라를 나열하는 방법의 수는
4!=24
569 ⑤
따라서 구하는 방법의 수는
12개의 꼭짓점 중 세 점을 연결하여 만들 수 있는 삼각형의 개수는 15_24=360이다.

138 수학 <하>
나머지 4명의 학생은 모두 자신이 제출하지 않은 과제를 받아야
572 ②
하므로 4명의 학생을 각각 a, b, c, d라 하면 자기 자신과 대응되지
A, B가 같은 날 같은 식당에 2번 가야 하므로 5개의 식당 중에서 않도록 배열하는 방법의 수를 수형도를 이용해서 구해 보면 다음과
A, B가 모두 가는 식당 2개를 고르는 방법의 수는 °Cª=10이고, 같다.
이 2개의 식당을 갈 날을 3일 중에서 2일 정하는 방법의 수는 a b c d
£Pª=6이다. b a d c
이때, 나머지 하루에 A, B가 저녁식사를 할 식당을 하나씩 더 골라야 c d a
하므로 방법의 수는 £CÁ_ªCÁ=6이다. d a c
따라서 구하는 방법의 수는 c a d b
10_6_6=360이다. d a b
b a
d a b c
c a b
573 풀이 참조
b a
남학생이 적어도 1명 포함되도록 선출하는 방법의 수는 즉, 3_3=9
전체 학생회 14명 중에서 3명을 뽑는 경우의 수에서 여학생만 3명을 따라서 구하는 경우의 수는
뽑는 경우의 수를 빼주면 된다. 35_9=315이다.
학생회 14명에서 3명을 선출하는 전체 방법의 수는
14_13_12
Á¢C£= =364
3!
학생회의 여학생의 수를 n이라 할 때, 여학생 중에서 3명을
576 ③

선출하는 방법의 수는 ÇC£이므로 뽑은 카드에 적힌 수 중 가장 큰 수와 가장 작은 수의 합이 9 또는


364-ÇC£=308, ÇC£=56이다. 10이므로 케이스를 나누면 다음과 같다.
n(n-1)(n-2) Ú 가장 큰 수와 가장 작은 수의 합이 9인 경우
ÇC£= =56
3! 가장 큰 수와 가장 작은 수 사이에 적어도 2개의 자연수가 있어야
n(n-1)(n-2)=8_7_6이므로 n=8 하므로 가장 큰 수와 가장 작은 수가 각각 6, 3 또는 7, 2 또는
여학생의 수가 8이므로 구하는 남학생의 수는 14-8=6이다. 8, 1이다.
채점 요소 배점 가장 큰 수와 가장 작은 수가 각각 6, 3인 경우,
여학생 또는 남학생의 수를 미지수로 놓기 20% 4, 5가 나머지 2개의 수가 되어야 하므로 1가지

남학생 중에서 적어도 1명을 선출하는 방법의 수에 대한 식 세우기 40% 가장 큰 수와 가장 작은 수가 각각 7, 2인 경우, 우
남학생의 수 구하기 40% 3, 4, 5, 6 중에서 2개의 수를 더 뽑으면 되므로 ¢Cª=6 의

가장 큰 수와 가장 작은 수가 각각 8, 1인 경우,
2, 3, 4, 5, 6, 7 중에서 2개의 수를 더 뽑으면 되므로 ¤Cª=15

574 8 그러므로 경우의 수는 1+6+15=22


Û 가장 큰 수와 가장 작은 수의 합이 10인 경우
여자선수를 n명이라 하면 남자선수는 (n-2)명이다. 가장 큰 수와 가장 작은 수 사이에 적어도 2개의 자연수가 있어야
이때, 여자 복식팀을 만드는 방법의 수는 하므로 가장 큰 수와 가장 작은 수가 각각 7, 3 또는 8, 2이다.
n(n-1)
ÇCª= 가장 큰 수와 가장 작은 수가 각각 7, 3인 경우,
2!
4, 5, 6 중에서 2개의 수를 더 뽑으면 되므로 £Cª=3
혼성 복식팀을 만드는 방법의 수는
가장 큰 수와 가장 작은 수가 각각 8, 2인 경우,
ÇCÁ_ÇЪCÁ=n(n-2)
3, 4, 5, 6, 7 중에서 2개의 수를 더 뽑으면 되므로 °Cª=10
가능한 여자 복식팀의 수가 혼성 복식팀의 수보다 20만큼 작으므로
n(n-1) 그러므로 경우의 수는 3+10=13
=n(n-2)-20 Ú, Û에서 구하는 경우의 수는
2!
nÛ`-3n-40=0, (n+5)(n-8)=0 22+13=35이다.
n=8 (∵ n>0)
따라서 구하는 여자선수의 수는 8이다.
577 ③

1에서 20까지의 자연수 중 3으로 나눈 나머지가 0, 1, 2인 수는 각각


575 315
6개, 7개, 7개이다. 이 중 세 수를 더해서 3의 배수가 되는 경우는
자신이 제출한 과제를 다시 받을 학생을 정하는 경우의 수는 다음과 같다.
¦C£=35 Ú 세 수가 모두 3으로 나눈 나머지가 0인 경우

Ⅵ. 경우의 수 139
6_5_4
¤C£=
3!
=20 580 ⑤

Û 세 수가 모두 3으로 나눈 나머지가 1인 경우 다음과 같이 주어진 대진표에서 배정받을 수 있는 7개의 자리를


7_6_5 왼쪽에서부터 각각 a, b, c, d, e, f, g라 하자.
¦C£= =35
3!
Ü 세 수가 모두 3으로 나눈 나머지가 2인 경우
¦C£=35
a b c d e f g
Ý 세 수를 3으로 나눈 나머지가 각각 0, 1, 2인 경우
¤CÁ_¦CÁ_¦CÁ=294 1반과 2반이 결승전 이전에 서로 대결하지 않도록 하려면
Ú ~ Ý에서 구하는 경우의 수는 a, b, c, d와 e, f, g에 각각 한 반씩 배정되어야 한다.
20+35+35+294=384이다. 1반과 2반을 a, b, c, d와 e, f, g 중에 한 반씩 배정하는
경우의 수가 2가지
1, 2반을 제외하고 a, b, c, d에 배정될 3개의 반과 e, f, g에
배정될 2개의 반을 고르는 방법의 수는
578 ①
°C£_ªCª=10
처음 꺼내는 1개의 공의 색에 따라서 케이스 분류를 하면 다음과 같다. 이 중 a, b, c, d에 속한 3개의 반 중 1반 또는 2반과 대결할 반을
Ú 처음 꺼낸 공이 빨간색 공인 경우 고르는 방법의 수가
빨간색 공 중에서 1개를 꺼내는 방법의 수는 £CÁ이고, £CÁ=3
남은 공 중에서 파란색 공을 2개, 빨간색 공을 1개 꺼내거나 e, f, g에 배정된 반 중 g에 들어갈 반을 고르는 방법의 수가
파란색 공 3개를 꺼내야 하므로 방법의 수는 £CÁ=3
£CÁ_(£Cª_ªCÁ+£C£)=21 따라서 구하는 방법의 수는
Û 처음 꺼낸 공이 파란색 공인 경우 2_10_3_3=180이다.
파란색 공 중에서 1개를 꺼내는 방법의 수는 £CÁ이고,
남은 공 중에서 파란색 공을 2개, 빨간색 공을 1개 꺼내야 하므로
581 90
방법의 수는
조건을 만족시키도록 구슬을 담는 경우는 각 상자에 2개/2개/2개,
£CÁ_ªCª_£CÁ=9
3개/2개/1개, 4개/1개/1개씩 나누어 넣는 경우이다.
Ú, Û에서 구하는 방법의 수는
Ú 2개/2개/2개로 나누어 넣는 경우
21+9=30이다.
6개의 구슬을 2개, 2개, 2개로 분할한 다음
3개의 상자에 넣어 주면 되므로
1
¤Cª_¢Cª_ªCª_ =15(가지)
579 ③ 3!
Û 3개/2개/1개로 나누어 넣는 경우
벨이 4초 또는 1초 울리고 벨과 벨 사이에는 1초의 간격이 있으므로
6개의 구슬을 3개, 2개, 1개로 분할한 다음
하나의 벨이 울릴 때마다 끝날 때 1초가 더 있다고 생각하면 5초
3개의 상자에 넣어 주면 되므로
또는 2초 울리는 두 가지 종류의 벨을 눌러서 총 19초의 신호가
¤C£_£Cª_ÁCÁ=60(가지)
만들어진다고 생각할 수 있다.
Ü 4개/1개/1개로 나누어 넣는 경우
5초짜리 벨과 2초짜리 벨을 누른 횟수를 각각 x, y라 하면
6개의 구슬을 4개, 1개, 1개로 분할한 다음
5x+2y=19
3개의 상자에 넣어 주면 되므로
등식을 만족시키는 x, y의 값은 x=1, y=7 또는 x=3, y=2이다.
1
Ú x=1, y=7인 경우 ¤C¢_ªCÁ_ÁCÁ_ =15(가지)
2!
5초짜리 벨이 1번, 2초짜리 벨이 7번 울리게 되므로 Ú ~ Ü에서 구하는 방법의 수는
총 8번 중에서 5초짜리 벨이 울릴 순서를 1번 정하면 나머지 모두 15+60+15=90이다.
2초짜리 벨이 울리면 되므로 경우의 수는
¥CÁ=8
Û x=3, y=2인 경우
582 ③

5초짜리 벨이 3번, 2초짜리 벨이 2번 울리게 되므로 가위바위보 게임에서 비기려면 모두 같은 것을 내거나 가위, 바위,
총 5번 중에서 5초짜리 벨이 울릴 순서를 3번을 정하면 나머지 보를 적어도 하나씩 내야 한다.
모두 2초짜리 벨이 울리면 되므로 경우의 수는 Ú 모두 같은 것을 낸 경우
°C£=10 가위, 바위, 보 3가지 중에서 하나를 똑같이 내야 하므로 3가지
Ú, Û에서 구하는 신호의 개수는 Û 가위, 바위, 보를 적어도 하나씩 낸 경우
8+10=18이다. 4명을 2명/1명/1명으로 나누는 경우의 수는

140 수학 <하>
1 ¤C£_£Cª_ÁCÁ=60
¢Cª_ªCÁ_ÁCÁ_ =6
2! Ü 2명/2명/2명으로 나누어 내리는 경우
3개의 조에 가위, 바위, 보를 배분하는 방법의 수가 6명을 2명/2명/2명으로 나누는 방법의 수는
3!=6이므로 1
¤Cª_¢Cª_ªCª_ =15
가위, 바위, 보를 적어도 하나씩 낸 경우의 수는 3!
6_6=36 Ú ~ Ü에서 6층, 5층, 3층에 나누어 내리는 방법의 수는
Ú, Û에서 구하는 경우의 수는 3!=6이므로 구하는 방법의 수는
3+36=39이다. (15+60+15)_6=540이다.

583 ④ 586 ②

4개의 수목원 A, B, C, D에 사전답사를 갈 인원수에 따라 케이스 5개의 파일의 크기의 합은


분류하면 다음과 같다. 100+150+200+250+400=1100(MB)
Ú 3명/1명/1명/1명으로 나누어서 답사하는 경우 이므로 하나의 USB에 5개의 파일을 모두 저장할 수는 없다. 파일을
6명의 학생을 3명/1명/1명/1명으로 나누는 방법의 수는 나누어 저장하는 경우를 케이스 분류하면 다음과 같다.
1 Ú 4개/1개/0개로 나누어 저장하는 경우
¤C£_£CÁ_ªCÁ_ÁCÁ_ =20
3! °C¢_ÁCÁ=5
Û 2명/2명/1명/1명으로 나누어서 답사하는 경우 Û 3개/2개/0개로 나누어 저장하는 경우
6명의 학생을 2명/2명/1명/1명으로 나누는 방법의 수는 °C£_ªCª=10
1 Ü 3개/1개/1개로 나누어 저장하는 경우
¤Cª_¢Cª_ªCÁ_ÁCÁ_ =45
2!2!
1
Ú, Û에서 4개의 수목원에 배정하는 방법의 수는 4!=24이므로 °C£_ªCÁ_ÁCÁ_ =10
2!
구하는 방법의 수는 Ý 2개/2개/1개로 나누어 저장하는 경우
(20+45)_24=1560이다. 1
°Cª_£Cª_ÁCÁ_ =15
2!
Ú ~ Ý에서 구하는 방법의 수는
584 132 5+10+10+15=40이다.

운전면허를 소지하고 있는 사람이 3명이므로 2대의 차에 운전자를


정하는 방법의 수는 £Pª=6 yy ㉠ 587 ③ 경

A, B가 탈 차를 정하는 방법의 수는 ªPª=2 yy ㉡ 의
주어진 자연수가 5의 배수이므로 e=5
운전자와 A, B를 제외한 나머지 4명이 차에 나누어 타는 방법을 수
Ú c=3일 때
케이스 분류하면 다음과 같다.
d=4이고, a와 b의 값으로 가능한 것은 6, 7, 8, 9의 4개이므로
Ú 4인승, 7인승에 각각 2명, 2명 나누어 타는 경우
¢Cª=6
1
¢Cª_ªCª_ _2!=6(가지) Û c=2일 때
2!
Û 4인승, 7인승에 각각 1명, 3명 나누어 타는 경우 d의 값으로 가능한 것은 3, 4의 2개이고, a와 b의 값으로 가능한
¢CÁ_£C£=4(가지) 것은 3, 4, 6, 7, 8, 9 중 d의 값을 제외한 5개이므로
Ü 7인승에 4명이 모두 타는 경우 1가지 2_°Cª=20
㉠, ㉡과 Ú ~ Ü에서 구하는 방법의 수는 Ü c=1일 때
6_2_(6+4+1)=132 d의 값으로 가능한 것은 2, 3, 4의 3개이고, a와 b의 값으로
가능한 것은 2, 3, 4, 6, 7, 8, 9 중 d의 값을 제외한 6개이므로
3_¤Cª=45
585 ③ Ú ~ Ü에서 구하는 자연수의 개수는
6+20+45=71이다.
7층에서 탔던 6명이 3번에 나누어서 내려야 하므로
6명을 3개의 조로 나누어야 한다.
Ú 4명/1명/1명으로 나누어 내리는 경우
6명을 4명/1명/1명으로 나누는 방법의 수는
588 119

1 선택한 3개의 사물함에 적혀 있는 수를 각각 a, b, c`(a<b<c)라


¤C¢_ªCÁ_ÁCÁ_ =15
2! 하자. 선택한 사물함 사이에 각각 n개 이상의 사물함이 있으려면
Û 3명/2명/1명으로 나누어 내리는 경우 b-a>n, c-b>n을 만족시켜야 한다.
6명을 3명/2명/1명으로 나누는 방법의 수는 이때, a에는 +0, b에는 +n, c에는 +2n을 하면 각각

Ⅵ. 경우의 수 141
a, b+n, c+2n이 되므로 조건을 항상 만족시킨다. 다음과 같이 5개의 점이 일직선 위에 존재하는 경우가 2가지이고 이
c+2n이 15 이하의 자연수가 되어야 하므로 5개의 점 중에서 3개를 선택하는 경우 삼각형이 만들어지지 않는다.
c+2nÉ15에서 cÉ15-2n이다. y
즉, 선택한 사물함 사이에 빈 사물함의 개수가 모두 n 이상이 되도록 2

하는 경우의 수는 1부터 15-2n까지의 자연수 중에서 서로 다른


3개의 수를 선택하는 경우의 수와 같으므로
-2 O 2 x
f(n)=15-2nC£이다.
∴ f(3)+ f(4)=»C£+¦C£
-2
=84+35=119
즉, 2_°C£=20 yy ㉡
다음과 같이 3개의 점만 한 직선 위에 존재하는 경우가 10가지이고
589 ②
한 직선 위의 3개의 점을 선택하는 경우 삼각형이 만들어지지
먼저 1행에 1, 2, 3, 4가 적힌 4장의 카드를 배열하는 방법의 수는 않는다.
4!=24이고, y

2행에 각 열마다 1행과 다른 숫자가 적힌 카드를 배열하는 방법의 2

수를 수형도로 일일이 세주면 다음과 같다.


1 2 3 4 x
-2 O 2
2 1 4 3
3 4 1 -2
4 1 3
3 1 4 2 즉, 10_£C£=10 yy ㉢
4 1 2 ㉠, ㉡, ㉢에서 구하는 삼각형의 개수는
2 1 286-(20+10)=256
4 1 2 3
3 1 2
2 1
591 ⑤
즉, 3_3=9
8 이하의 자연수 중에서 짝수의 개수는 4이므로
이때, 2행에 적은 숫자를 기준으로 3행에서 각 열마다 다른 숫자가
적어도 4장의 카드를 뽑아야 짝수가 적힌 카드를 모두 뽑을 수 있다.
적힌 카드를 배열하는 방법의 수도 9이므로 구하는 방법의 수는
그러므로 nÉ3인 경우 f(n)=0이고, f(3)=0
24_9_9=1944
Ú n=4인 경우
4번째 뽑은 카드가 짝수가 적힌 마지막 카드여야 하므로 짝수가
590 256 적힌 카드 4장이 연달아 뽑혀야 한다.

부등식 |x|+|y|É2를 만족시키는 두 정수 x, y에 대하여 순서쌍 f(4)=4!

(x, y)는 다음과 같다. Û n=5인 경우

(0, 0) 5번째 뽑은 카드가 짝수가 적힌 마지막 카드여야 하므로

(1, 0), (0, 1), (-1, 0), (0, -1) 4번째까지 짝수가 적힌 카드 3장과 홀수가 적힌 카드 1장이
뽑혀야 한다.
(2, 0), (0, 2), (-2, 0), (0, -2)
짝수 4개 중에서 3개를 선택하는 방법의 수는 ¢C£=4
(1, 1), (1, -1), (-1, 1), (-1, -1)
이를 좌표평면에 나타내면 다음과 같다. 홀수 4개 중에서 1개를 선택하는 방법의 수는 ¢CÁ=4
y 남은 짝수가 적힌 카드를 마지막에 뽑으면 되므로
2 f(5)=4_4_4!=16_4!
Ü n=6인 경우
6번째 뽑은 카드가 짝수가 적힌 마지막 카드여야 하므로
-2 O 2 x 5번째까지 짝수가 적힌 카드 3장과 홀수가 적힌 카드 2장이
뽑혀야 한다.
-2 짝수 4개 중에서 3개를 선택하는 방법의 수는 ¢C£=4
이 13개의 점 중 3개의 점을 고르는 방법의 수는 홀수 4개 중에서 2개를 선택하는 방법의 수는 ¢Cª=6
13_12_11 남은 짝수가 적힌 카드를 마지막에 뽑으면 되므로
Á£C£= =286 yy ㉠
3! f(6)=4_6_5!=24_5!
이 중 한 직선 위에 있는 세 점을 선택하는 경우의 수를 빼주어야 한다. Ú ~ Ü에서 구하는 값은

142 수학 <하>
f(3)+ f(4)+ f(5)+ f(6) 4!+16_4!+24_5! Ý Y를 4개 쓰는 경우
=
4! 4! X, Y를 각각 4개, 4개 써야 하므로
=1+16+120=137 Y가 서로 이웃하지 앉게 쓰는 방법은 다음과 같이 1가지이다.
X Y X Y X Y X Y
592 306
그러므로 문자열의 개수는 1이다.
1, 2, y, 9 중 세 숫자로 만들 수 있는 세 자리 자연수의 개수는 Ú ~ Ý에서 구하는 문자열의 개수는
»P£=9_8_7=504 7+15+10+1=33이다.
9 이하의 자연수 중 2개의 수를 더해서 7의 배수가 되려면 7로 나눈
나머지가 각각 1, 6 또는 2, 5 또는 3, 4인 두 수를 더해야 한다.
Ú 7로 나눈 나머지가 1, 6인 두 수가 포함되는 경우
594 ④

7로 나눈 나머지가 1, 6인 두 수는 1, 6 또는 6, 8이다. 천의 자리, 백의 자리, 십의 자리, 일의 자리에 대하여 각각 합을


1, 6이 들어 있는 세 자리 자연수의 개수는 구하면 다음과 같다.
1, 6을 제외한 7개의 수 중에서 1개를 선택해서 나열하면 되므로 Ú 천의 자리의 경우
방법의 수는 7_3!=42 천의 자리의 숫자가 1인 네 자리 자연수의 개수는 ¢P£=24이므로
6, 8이 들어 있는 세 자리 자연수의 개수는 1000은 24번 더해진다.
6, 8을 제외한 7개의 수 중에서 1개를 선택해서 나열하면 되므로 천의 자리의 숫자가 2, 3, 4일 때도 마찬가지이므로
방법의 수는 7_3!=42 2000, 3000, 4000도 각각 24번씩 더해진다.
이 중 1, 6, 8이 동시에 들어 있는 세 자리 자연수의 개수 24_(1000+2000+3000+4000)=240000
3!=6만큼이 겹치므로 Û 백의 자리의 경우
1, 6 또는 6, 8이 들어 있는 세 자리 자연수의 개수는 백의 자리의 숫자가 1인 네 자리 자연수의 개수는 3_3_2=18
42+42-6=78 이므로 100은 18번 더해진다.
Û 7로 나눈 나머지가 2, 5인 두 수가 포함되는 경우 백의 자리의 숫자가 2, 3, 4일 때도 마찬가지이므로
7로 나눈 나머지가 2, 5인 두 수는 2, 5 또는 5, 9이다. 200, 300, 400도 각각 18번씩 더해진다.
Ú과 마찬가지로 구하면 자연수의 개수는 78 18_(100+200+300+400)=18000
Ü 3, 4가 들어 있는 세 자리 자연수의 개수는 Ü 십의 자리의 경우
7_3!=42 십의 자리의 숫자가 1인 네 자리 자연수의 개수는 3_3_2=18
Ú ~ Ü에서 구하는 자연수의 개수는 이므로 10은 18번 더해진다.
504-(78_2+42)=504-198=306이다. 십의 자리의 숫자가 2, 3, 4일 때도 마찬가지이므로 경

20, 30, 40도 각각 18번씩 더해진다.

18_(10+20+30+40)=1800 수
593 ④
Ý 일의 자리의 경우
조건 ㈎, ㈏에서 X를 첫 번째에 쓰고 Y는 1개 이상 4개 이하로 쓸 일의 자리의 숫자가 1인 네 자리 자연수의 개수는 3_3_2=18
수 있다. 이므로 1은 18번 더해진다.
Ú Y를 1개 쓰는 경우 일의 자리의 숫자가 2, 3, 4일 때도 마찬가지이므로
첫 번째엔 X가 오고 나머지 7자리 중에서 Y를 쓸 자리를 2, 3, 4도 각각 18번씩 더해진다.
선택하는 방법의 수가 ¦CÁ=7이고 18_(1+2+3+4)=180
나머지 자리는 모두 X를 쓰면 되므로 문자열의 개수는 7이다. Ú ~ Ý에서 구하는 모든 네 자리 자연수의 합은
Û Y를 2개 쓰는 경우 240000+18000+1800+180=259980이다.
X, Y를 각각 6개, 2개 써야 하므로
먼저 X를 6개 나열하고 다음과 같이 ㉠~㉥ 중에서 2개를 골라
그 자리에 Y를 쓰면 된다.
595 ⑤

X ㉠ X ㉡ X ㉢ X ㉣ X ㉤ X ㉥
B F
그러므로 문자열의 개수는 ¤Cª=15이다.
Ü Y를 3개 쓰는 경우 A
X, Y를 각각 5개, 3개 써야 하므로 C E
먼저 X를 5개 나열하고 다음과 같이 ㉠~㉤ 중에서 3개를 골라 D
그 자리에 Y를 쓰면 된다.
인접한 영역을 서로 다른 색으로 칠하기 위해 필요한 서로 다른 색의
X ㉠ X ㉡ X ㉢ X ㉣ X ㉤
개수는 최소 4이다.
그러므로 문자열의 개수는 °C£=10이다. 따라서 사용하는 색의 개수에 따라 케이스를 나누면 다음과 같다.

Ⅵ. 경우의 수 143
Ú 5가지 색을 사용하는 경우 쌓아야 하므로
A에 칠할 수 있는 색은 5가지이고, 3_2=6(가지)이고,
B, C, D, E, F에 네 가지 색을 칠해야 하므로 두 영역에는 같은 나머지 3개는 3행의 1열, 3열에 2개, 1개씩 쌓아야 하므로
색을 칠하고 나머지 세 영역에는 서로 다른 색을 하나씩 2가지이다.
칠해야 한다. 그러므로 경우의 수는 6_2=12
같은 색을 칠할 두 영역을 고르는 방법은 Ü 1행, 3행에 각각 블록을 3개, 4개 나누어 쌓는 경우
B, D 또는 B, E 또는 C, E 또는 C, F 또는 D, F로 5가지이다. 1행의 각 열에 블록을 2개, 1개, 0개씩 쌓아야 하므로
각각 4가지 색을 하나씩 칠하는 경우의 수가 4!=24이므로 3!=6(가지)이고,
구하는 경우의 수는 나머지 4개는 3행의 1열, 3열에 모두 2개씩 쌓아야 하므로
5_5_24=600 1가지이다.
Û 4가지 색을 사용하는 경우 그러므로 경우의 수는 6_1=6
사용할 4가지 색을 고르는 경우의 수는 °C¢=5이고 A에 칠할 수 Ú ~ Ü에서 구하는 방법의 수는
있는 색은 4가지이다. 6+12+6=24이다.
B, C, D, E, F에 세 가지 색을 칠해야 하므로 이 중 서로 같은
색을 칠할 영역이 2개, 2개, 1개이어야 한다.
이 중 한 영역만 한 가지 색으로 칠할 곳을 정하는 경우의 수가 597 450

5이고, 이때 나머지 네 영역 중 2개씩 같은 곳을 칠할 영역을 조건 ㈎에서 함수 f 의 치역의 원소의 개수가 2이므로 치역의 원소를
정하는 방법의 수는 1이다. 정하는 방법의 수는 ¤Cª=15
각 영역에 세 가지 색을 하나씩 칠하는 경우의 수가 이때, 함수 f ç f 가 상수함수이므로 치역의 원소의 개수가 1이다.
3!=6이므로 구하는 경우의 수는 f f
A A A
5_4_5_6=600
1 1 1
Ú, Û에서 구하는 방법의 수는 2 2 2
600+600=1200이다. 3 3 3
4 4 4
5 5 5
6 6 6
596 ③
일반성을 잃지 않고 위와 같이 만약 함수 f 의 치역의 원소로 2, 3을
다음과 같이 [그림1]에서 정면을 기준으로 왼쪽에서부터 각각 1열,
선택한다면 2, 3이 모두 2에 대응되거나 모두 3에 대응되어야 한다.
2열, 3열, 정면에 가까운 앞에서부터 각각 1행, 2행, 3행이라 하자.
즉, 함수 f 의 치역의 원소 2개 중 하나가 함수 f ç f 의 치역의 원소가
3행 되어야 하므로 함수 f ç f 의 치역의 원소를 정하는 방법의 수는 2
이때, 함수 f 에서 2, 3을 제외한 1, 4, 5, 6은 2 또는 3에 대응되어야
2행
하고, 함수 f 의 치역의 원소의 개수가 1이 되는 경우는 제외시켜야
1행 하므로
2Ý`-1=15(가지)
1열 2열 3열 따라서 구하는 함수 f 의 개수는
(정면)
15_2_15=450이다.
[그림1]과 [그림2]에서 1행에는 1열, 2열, 3열에 각각 1개 이상 3개
이하씩 블록을 놓아야 하고, 적어도 한 열에 블록을 3개 쌓아야 한다.
2행에는 2열, 3열에 각각 블록을 1개씩만 놓아야 한다.
598 231
3행에는 1열, 3열에 각각 1개 이상 3개 이하씩 블록을 놓아야 하고,
적어도 한 열에 블록을 3개 쌓아야 한다. 조건 ㈎에서 정의역의 원소 중 자기 자신에게 대응되는 원소의

그러므로 14개의 블록 중에서 가장 아래에 놓이는 블록의 수는 개수가 5 이하이고, 집합 X의 원소의 개수가 7이므로 조건 ㈏를
만족시키려면 자기 자신에게 대응되는 원소의 개수가 홀수가 되어야
7이고, 나머지 7개의 블록을 1행과 3행에 쌓아야 한다.
Ú 1행, 3행에 각각 블록을 5개, 2개 나누어 쌓는 경우 한다. yy TIP
1행의 각 열에 블록을 2개, 2개, 1개씩 쌓아야 하므로 Ú 집합 {x|f(x)=x, x<X}의 원소의 개수가 5인 경우
3가지이고, 집합 X의 원소 중에서 자기 자신에게 대응되는 원소 5개를
나머지 2개는 3행의 1열 또는 3열 중 하나에 쌓아야 하므로 선택하는 경우의 수는
2가지이다. ¦C°=21
그러므로 경우의 수는 3_2=6 나머지 2개의 원소는 서로 엇갈려 대응되어야 하므로
Û 1행, 3행에 각각 블록을 4개, 3개 나누어 쌓는 경우 1가지
1행의 각 열에 블록을 (2개, 1개, 1개) 또는 (2개, 2개, 0개) 그러므로 함수의 개수는 21_1=21이다.

144 수학 <하>
Û 집합 {x|f(x)=x, x<X}의 원소의 개수가 3인 경우
집합 X의 원소 중에서 자기 자신에게 대응되는 원소 3개를
선택하는 경우의 수는


¦C£=35
나머지 4개의 원소는 2개씩 엇갈려 대응되어야 하므로
1
¢Cª_ªCª_ =3
2! 원의 중심과 원 위의 점 2개를 지나는 직선이 3번씩 중복되므로
그러므로 함수의 개수는 35_3=105이다. ㉠에서 빼주면 직선의 개수는
Ü 집합 {x|f(x)=x, x<X}의 원소의 개수가 1인 경우 n(n+1) n n(n-1)
-2_ =
집합 X의 원소 중에서 자기 자신에게 대응되는 원소 1개를 2 2 2
선택하는 경우의 수는 10_9
즉, f(10)= =45
2
¦CÁ=7
Û n이 홀수인 경우
나머지 6개의 원소는 2개씩 엇갈려 대응되어야 하므로
원의 중심과 원 위의 점 2개를 동시에 지나는 직선이 존재하지
1
¤Cª_¢Cª_ªCª_ =15 않으므로 직선의 개수는
3!
n(n+1)
그러므로 함수의 개수는 7_15=105이다. n+1C2=
2
Ú ~ Ü에서 구하는 함수의 개수는
13_14
21+105+105=231이다. 즉, f(13)= =91
2
Ú, Û에서 구하는 값은
TIP
f(10)+ f(13)=45+91=136이다.
x<X인 모든 x에 대하여 ( f ç f)(x)=x를 만족시키려면 자기
자신에게 대응되는 원소를 제외하고 나머지 원소들은 두 개씩
짝지어 서로 엇갈려 대응되어야 한다.
그러므로 엇갈려 대응되는 원소들의 개수가 짝수이고, 전체 600 16

정의역의 원소의 개수는 홀수이므로 자기 자신과 대응되는 시계반대방향으로 1만큼 이동하는 것은 시계방향으로 3만큼
원소의 개수는 홀수이다. 움직이는 것과 같다.
f f 그러므로 말은 한 번의 이동에서 정사각형의 변을 따라
X X X
1 1 1 시계방향으로 1 또는 3만큼 움직이게 된다.
2 2 2 동전을 5번 던져서 말을 움직일 때, 말의 이동거리는 시계방향을 경
3 3 3 우
4 4 4
기준으로 5 이상 15 이하이다.

5 5 5 이때, 말이 점 A에서 출발하여 점 B에 도착하기 위해서는 수
6 6 6 이동거리가 7 또는 11 또는 15가 되어야 한다.
7 7 7
시계방향으로 1만큼 움직인 횟수를 x, 3만큼 움직인 횟수를 y라
예를 들어 위와 같이 자기 자신과 대응되는 원소가 1, 2로 할 때, 이동거리가 7 또는 11 또는 15가 되는 경우를 케이스 분류하면
짝수 개이면 조건을 만족시키지 못하고 남는 원소가 생긴다. 다음과 같다.
Ú 이동거리가 7인 경우
x+y=5, x+3y=7이므로 두 식을 연립하면
x=4, y=1
즉, 앞면이 4번 나오고 뒷면이 1번 나온 것이므로
5번 중 뒷면이 나오는 순서를 결정하는 방법의 수가 °CÁ=5이고
나머지 모두 앞면이 나오면 된다.
599 136
Û 이동거리가 11인 경우
점의 총 개수가 n+1이고 직선이 결정되려면 이 중에서 2개의 x+y=5, x+3y=11이므로 두 식을 연립하면
점을 선택하면 되므로 방법의 수는 x=2, y=3
n(n+1) 즉, 앞면이 2번 나오고 뒷면이 3번 나온 것이므로
n+1Cª= yy ㉠
2 5번 중 뒷면이 나오는 순서를 결정하는 방법의 수가 °C£=10이고
이때, n이 짝수인 경우와 홀수인 경우에 대하여 케이스 분류하면 나머지 모두 앞면이 나오면 된다.
다음과 같다. Ü 이동거리가 15인 경우
Ú n이 짝수인 경우 x+y=5, x+3y=15이므로 두 식을 연립하면
다음과 같이 원 위의 점이 2개씩 원의 중심에 대하여 서로 x=0, y=5
대칭되어 있다. 즉, 모두 뒷면이 나온 것이므로 방법의 수는 1이다.

Ⅵ. 경우의 수 145
Ú ~ Ü에서 구하는 방법의 수는 2가지
5+10+1=16이다. 3송이/1송이/0송이로 나누어 꽂는 경우
장미가 있는 화병에 백합을 몇 송이 넣을지 정해주면 되므로
3가지
601 50
2송이/2송이/0송이로 나누어 꽂는 경우
반지름의 길이가 가장 긴 원판을 A라 하면 A원판 위에 쌓는 원판 장미가 있는 화병에 백합을 몇 송이 넣을지 정해주면 되므로
중에서 반지름의 길이가 가장 긴 원판을 가장 위에 쌓아야 한다. 2가지
Ú A를 가장 위에 쌓는 경우는 문제의 조건을 만족시키지 않는다. 2송이/1송이/1송이로 나누어 꽂는 경우
Û A를 위에서 두 번째에 쌓는 경우 장미가 있는 화병에 백합을 몇 송이 넣을지 정해주면 되므로
A원판 위에 쌓을 원판 1개를 선택하는 방법의 수는 2가지
¢CÁ=4 그러므로 방법의 수는 1_(2+3+2+2)=9
나머지 3개의 원판을 A의 아래에 쌓는 방법의 수는 Û ㉡의 경우
3!=6 3개의 화병이 모두 구분되므로
따라서 이 경우 원판 5개를 쌓는 방법의 수는 같은 종류의 백합 4송이를 화병에 꽂는 경우의 수는
4_6=24 4송이를 모두 한 화병에 넣는 경우 3가지
Ü A를 위에서 세 번째에 쌓는 경우 3송이/1송이/0송이로 나누어 꽂는 경우 3!=6(가지)
A원판 위에 쌓을 원판 2개를 선택하는 방법의 수는 2송이/2송이/0송이로 나누어 꽂는 경우
¢Cª=6 백합을 꽂지 않을 화병만 정해주면 되므로 3가지
이때, 선택한 2개의 원판을 쌓는 방법의 수는 1 2송이/1송이/1송이로 나누어 꽂는 경우
한편 나머지 2개의 원판을 A의 아래에 쌓는 방법의 수는 백합을 2송이 꽂을 화병만 정해주면 되므로 3가지
2!=2 그러므로 방법의 수는 3_(3+6+3+3)=45
따라서 이 경우 원판 5개를 쌓는 방법의 수는 Ü ㉢의 경우
6_1_2=12 3개의 화병이 모두 구분되므로 Û와 같은 아이디어로 방법의
Ý A를 위에서 네 번째에 쌓는 경우 수는 1_(3+6+3+3)=15
A원판 위에 쌓을 원판 3개를 선택하는 방법의 수는 Ú ~ Ü에서 구하는 방법의 수는
¢C£=4 9+45+15=69이다.
이때, 선택한 3개의 원판을 쌓는 방법의 수는 2!=2
한편 나머지 1개의 원판을 A의 아래에 쌓는 방법의 수는 1
따라서 이 경우 원판 5개를 쌓는 방법의 수는
4_2_1=8
Þ A를 위에서 다섯 번째에 쌓는 경우
4개의 원판 중 반지름의 길이가 긴 것을 가장 위로 쌓고 나머지
3개의 원판을 쌓는 방법의 수는
3!=6
Ú ~ Þ에서 구하는 방법의 수는
24+12+8+6=50이다.

602 ②

서로 다른 종류의 장미 3송이를 3개의 화병에 나누어 꽂는 방법의


수는
3송이를 한 화병에 모두 꽂는 경우 1가지 yy ㉠
2송이/1송이/0송이로 나누어 꽂는 경우
£Cª_ÁCÁ=3(가지) yy ㉡
1송이/1송이/1송이로 나누어 꽂는 경우 1가지 yy ㉢
Ú ㉠의 경우
3개의 화병 중 장미가 있는 화병만 구별되고 장미가 없는
화병 2개는 서로 구분되지 않는다.
같은 종류의 백합 4송이를 화병에 꽂는 경우의 수는
4송이를 모두 한 화병에 넣는 경우

146 수학 <하>
MEMO
MEMO
MEMO
MEMO
MEMO
MEMO

You might also like